[{"image": "VQAonline_00000044.png", "question": "What does this Elsevier revision report mean?", "context": "This is a review report of an article on the EVISE revision status website:\n\nDoes this mean my article was rejected? Why there isn't any \"Complete and send review\" for the second reviewer? And why is the status of the article \"Decision Pending\"?\n", "answer": "Thank you for all of your answers and comments. This was some bug in EVISE system. Supporting section of EVISE is trying to fix it in next updates.\n", "topic": "academia", "url": "https://academia.stackexchange.com/questions/61584"}, {"image": "VQAonline_00000006.png", "question": "What will happen to my IEEE conference paper if the status of the accepted paper is \"AAR\"", "context": "I have been notified by the IEEE organizing committee that my paper has been accepted for their conference and requested to register. and the status of the paper is AAR. Please see the quotation below.\n\n[AAR]This paper need thorough revision to be accepted as a full paper\nfor the conference.\n\nI have attached an image of their review process.\n\nWhat will happen to my paper after the submission of the camera-ready paper? Is there any possibility for my paper not to be published in the proceedings and IEEE Xplore? Or is it guaranteed to be published after the submission of camera-ready paper?\n", "answer": "If the Journal/Conference editor/chair has accepted your paper, it is guaranteed to be published, given that you make the changes. That is the reason they emphasize the \"review\" part.\nSome papers have only minor revisions, so if the changes are not made, it won't affect that much the quality of the conference. But if the changes are major, it usually indicates that you have to step up the level of the paper following the suggestion of the reviewers.\nIn conclusion, as long as you make the changes, your paper should be accepted in the conference, but if you neglect to do them, probably it wont.\n", "topic": "academia", "url": "https://academia.stackexchange.com/questions/7365"}, {"image": "VQAonline_00000009.png", "question": "Rediscovery of calculus in 1994: what should have happened to that paper?", "context": "There's a well-known paper in academic circles that features a rediscovery of the trapezoidal rule for numerical integration by a medical researcher:\n\n\u201cA Mathematical Model for the Determination of Total Area Under Glucose Tolerance and Other Metabolic Curves\u201d, Mary M. Tai, Diabetes Care, 1994, 17, 152\u2013154.\n\nI think the paper's only figure says it all:\n\u00a0\u00a0\u00a0\u00a0\u00a0\u00a0\u00a0\u00a0\u00a0\u00a0\u00a0\u00a0\u00a0\u00a0\u00a0\nYou can find comments on many blogs about it, most of them along the lines of \u201cturns out calculus was invented in 1994\u201d. I think it truly is a bit sad that the paper made it past the researcher, her immediate colleagues and friends, a Yale professor of electrical engineering who is thanked for \u201chis expert review\u201d, and most importantly reviewers, without someone giving the author a hint.\nIn my opinion, the most optimistic view is to see it as an educational paper: the method is not new, but that particular medical community didn't know about it, so it was worth publishing. However, that is not what the paper states: the author clearly presents the method as new and names it after her.\n\nOkay, I think I have explained the context. My question is: after that peer-review failure was exposed, what should have happened to the paper? What happened is that the same journal published a series of comments on the paper, and a reply by the author to the comments. Was that the correct/ethical way of handling the issue, as an editor? Or should the paper have been retracted?\n", "answer": "The answer to this question depends on what we should term a peer-review failure. There are tons of journals in the academic market: some journals are happy to publish results as long as they are correct even if they do not push the boundary of innovation. There are some journals that are happy to publish unsound experimental work, and on the other hand, some that encourage theoretical models that are severely impractical and are essentially mathematical mumbo-jumbo. \nPublishing something blindingly obvious even to a high-school student is ridiculous, but we could infer two things: one, the editor and the reviewers had not found this so obvious, and two, the journal did not (or does not?) have high standards on innovation anyway. Given it is a field that potentially could have practitioners who know not a thing about calculus, the publication should not come as a big surprise.\nThe reality is that publication of known facts is not all that uncommon in academia. The hoo-ha over this paper is basically because of the elementariness of the concept. One could expect apologies from journals for wrong results but not necessarily for stale ones. \n", "topic": "academia", "url": "https://academia.stackexchange.com/questions/9602"}, {"image": "VQAonline_00000317.png", "question": "Why aren't artificial derivatives used more often to solve the vanishing gradient problem?", "context": "While looking into the vanishing gradient problem, I came across a paper (https://ieeexplore.ieee.org/abstract/document/9336631) that used artificial derivatives in lieu of the real derivatives. For a visualization, see the attached image: \nAs you can see, the artificial derivative of the ReLU is the sigmoid function, which smoothes out the derivative to circumvent the dying ReLU problem. The authors apply similar smoothing operations for other activation functions such as the sigmoid. This idea seems pretty convincing, and it surprises me that it has not been used thus far, especially considering there is no additional computational cost in replacing the derivatives, as the authors claim. So why is it that vanishing gradients have not been addressed in this way in conventional deep learning frameworks such as Pytorch or Tensorflow?\n", "answer": "\nThis idea seems pretty convincing\n\nIndeed, you don't have to use the exact gradient of the activation function during the backward step.\nThe gradient of the activation function is a multiplicative part of the gradient of the loss. However, the exact gradient of the loss is almost never used. It is used neither in stochastic gradient descent, mini-batch, training with momentum, Adam, Newton nor in most other methods.\nAs only the approximate gradient of the loss is used in practice, we don't have to keep any part of it exact, so there is no need to keep the gradient of the activation function exact. It can be taken approximately, and any approximation is fine (including from the paper you referenced) as long as the loss decreases during training.\n\nSo why is it that vanishing gradients have not been addressed in this way in conventional deep learning frameworks such as Pytorch or Tensorflow?\n\nThis can be done in PyTorch and TensorFlow. You can modify the gradient by specifying the backward() method in PyTorch or grad() method in TensorFlow:\nExtending PyTorch with Custom Activation Functions\ntf.custom_gradient\n", "topic": "ai", "url": "https://ai.stackexchange.com/questions/37353"}, {"image": "VQAonline_00000134.png", "question": "What fast loss convergence indicates on a CNN?", "context": "I'm training two CNNs (AlexNet e GoogLeNet) in two differents DL libraries (Caffe e Tensorflow). The networks was implemented by dev teams of each libraries (here and here)\nI reduced the original Imagenet dataset to 1024 images of 1 category -- but setted 1000 categories to classify on the networks.\nSo I trained the CNNs, varying processing unit (CPU/GPU) and batches sizes, and I observed that the losses converges fastly to near zero (in mostly times before 1 epoch be completed), like in this graph (Alexnet on Tensorflow):\n\nIn portuguese, '\u00c9pocas' is epochs and 'Perda' is loss.\nThe weight decays and initial learning rate are the same as used on models that I downloaded, I only changed the dataset and the batch sizes.\nWhy my networks are converging this way, and not like this way?\n", "answer": "Ok, so the fact that you always give the same category means the network only has to output the same value for any given input. This is exceedingly easy to learn (all the weights always shift in the same directions) and this make this graph. \nOnce you have more categories (try 10 for example), you'll see the graph will be closer to expected.\n", "topic": "ai", "url": "https://ai.stackexchange.com/questions/4666"}, {"image": "VQAonline_00000151.png", "question": "SEIF motion update algorithm doubt", "context": "I want to implement Sparse Extended information slam. There is four step to implement it. The algorithm is available in Probabilistic Robotics Book at page 310, Table 12.3.\n\nIn this algorithm line no:13 is not very clear to me. I have 15 landmarks. So $\\mu_t$ will be a vector of (48*1) dimension where (3*1) for pose. Now $H_t^i$ is a matrix whose columns are dynamic as per the algorithm it is (3j-3) and 3j. J is the values of landmarks 1 to 15. Now how could I multiply a dynamic quantity with a static one. There must be a error that matrix dimension mismatch when implement in matlab.\nPlease help me to understand the algorithm better. \n", "answer": "You are right, that pseudocode is not correct. In particular, the definition of $H_t^i$ in line $11$ should be changed; all the way on the right-hand side, it should have $3N - 3j$ columns of $0$s, rather than $3j$ columns of $0$s.\nWith that change, every matrix $H_t^i$ will have the same number of columns: \n$$6 + 3j - 3 + 3N - 3j = 3 + 3N,$$\nwhich evaluates to a total of $48$ in your case (because you have $N = 15$ landmarks). That's precisely the correct dimensionality required for matrix multiplication with your $\\mu_t$ vector.\n\nThe version of the book that you linked to appears to be a fairly old draft. This webpage contains errata for the third edition for the book, in which page 393 corresponds to what was page 310 in your version of the book. The errata for that third edition of the book can be downloaded at the following URL: http://probabilistic-robotics.informatik.uni-freiburg.de/corrections/pg393.pdf\nThere you'll find the fix that I described above, but also some other fixes (most of them are just notational, adding bars over the $\\mu$ vectors, but it looks like a more serious issue was additionally fixed in line 13, where a minus was changed to a plus).\n", "topic": "ai", "url": "https://ai.stackexchange.com/questions/8048"}, {"image": "VQAonline_00000819.png", "question": "What does Yoshimura mean by \"sugar\" in chapter 92?", "context": "On chapter 92, there's a conversation between Yoshimura and Roma inside Anteiku that goes like this:\n\nYoshimura: What do you have to be careful about in both \"the coffee you served to ghouls\" and 'prostitutes on street corner'?\n Roma: ??\n Yoshimura: \"Sugar\"\n Roma: ...... what? (confused)\n\n\nThis joke went right over my head when I read this chapter. If someone could explain it to me that would be great.\n", "answer": "As stated on Wikia, the original joke is about \"serving coffee for ghouls\" and \"man on a (baseball) base\". The original script goes like this:\n\n\u300c\"\u55b0\u7a2e\"\u3078\u6df9\u308c\u308b\u30b3\u30fc\u30d2\u30fc\u300d\u3068\u304b\u3051\u3066\u300c\u920d\u8db3\u306a\u51fa\u5841\u8005\u300d\u3068\u3068\u304f\u2015\u2015\u300c\u30c8\u30a6\u30eb\u30a4\u306f\u63a7\u3048\u308b\u3088\u3046\u306b\u300d\u3002\nSource (Japanese) \n\nRough translation:\n\n\"Serving coffee for ghouls\" and \"slow-footed man on a (baseball) base\"... is \"to refrain from tourui\".\n\nHere, tourui is either \u7cd6\u985e (saccharides; sweeteners) or \u76d7\u5841 (base stealing; steal; stolen base), which basically means:\n\nNot consuming sugar on coffee for ghouls\nResist stealing a base for slow-footed man\n\nSince this Japanese wordplay is almost impossible to be translated properly (and the joke itself is quite technical), I believe the translator changed it so that joke can be understood easier while trying to preserve the nuance for \"sweetener/sugar\" and \"serving coffee for ghouls\".\n\n... as for the meaning of \"sugar\" in the translated version, it probably means:\n\nSugar, a prostitute that appears on The Crimson Petal and the White (or alternatively, a general/famous codename used by a prostitute)\nSugar baby (Wikipedia)\nBrown sugar, or heroin\n\n", "topic": "anime", "url": "https://anime.stackexchange.com/questions/43600"}, {"image": "VQAonline_00000878.png", "question": "Which chapter depicts Doraemon and Nobita on Easter Island?", "context": "This one is from a Doraemon manga and the story appears to be called 'Battle on Easter Island'.\n\nWhich Chapter is it from and how can I purchase/acquire a copy?\n", "answer": "The \"Battle on Easter Island\" appears in the 3rd chapter of \"The Devil of Time and Space\" arc (\u6642\u7a7a\u306e\u9b54\u795e), in the 2nd volume of The Doraemons Special (\u30b6\u30fb\u30c9\u30e9\u3048\u3082\u3093\u30ba \u30b9\u30da\u30b7\u30e3\u30eb) manga.\nThe arc's storyline revolves around Nostradamus' \"1999 prophecy\".\n\nNostradamus Arc\nNostradamus the prophet, who was sealed, escapes from the space and time dimension plot to make his prediction come true about the world's end in 1999. It is up to Doraemon, Nobita and the gang to stop him.\n\nThe panel on the question depicts Doraemon and Nobita on Easter Island being attacked by the people who are under the influence of Nostradamus, and Nostradamus himself attacking the group with a rain of thunder.\nThe series has not been licensed in English, so there's no way to acquire it legally. However, for the original Japanese language, it can be purchased on Amazon.\n\nReferences:\n\nAniOta Wiki - \u6642\u7a7a\u306e\u9b54\u795e(\u30b6\u2606\u30c9\u30e9\u3048\u3082\u3093\u30ba) (Japanese)\n\n", "topic": "anime", "url": "https://anime.stackexchange.com/questions/54011"}, {"image": "VQAonline_00000893.png", "question": "What's the Piechart about in the Character Panel of Cyber City Oedo 808?", "context": "In the intro of Cyber City Oedo 808 we get a character panel for each of the three main characters, depicting their profile, ie main stats etc. What does the pie chart indicate? What's it about? I can't make out the words exactly. Seems to me like \"NEGRIDE\", \"MONGOLIDE\". My guess is it is related to the origin (as in \"Caucasian\"), but I don't understand it, not sure. Check out the image to see what I'm talking about.\nIf anybody knows kindly share.\n\n", "answer": "It's their origin/race.\nThe labels on the pie chart are:\n\n\nMongolide (Mongoloid)\nEuropide (Caucasian)\nNegride (Negroid)\n\n\nwhich are actually in German.\nThe respective kanji is:\n\n\u4eba\u7a2e (race):\n\n\u9ec4\u8272\u7cfb (Mongoloid)\n\u767d\u8272\u7cfb (Caucasian)\n\u9ed2\u8272\u7cfb (Negroid)\n\u305d\u306e\u4ed6 (others)\n\n\n", "topic": "anime", "url": "https://anime.stackexchange.com/questions/58221"}, {"image": "VQAonline_00001140.png", "question": "How does gravity affect pressure under water?", "context": "How different would being under water in Europa differ from being under water on Earth? Would it be possible to dive deeper on Europa with a submarine than on Earth and by how much?\n\n", "answer": "For a liquid, hydrostatic pressure is $\\rho g h$ where $\\rho$ is density (this is always the same for all water) g is gravitational acceleration and h is depth.\nThe gravitational acceleration on Europa is 1.3 $\\text{m/s}{}^2$ (compared with 9.8 $\\text{m/s}{}^2$ on Earth). But on Europa there is 20km of ice floating on the water. \nAs a rough estimate, the gravity in Europa is 1/10 that on Earth, so the pressure at the bottom of the ice/top of the water would be comparable to the pressure at 2km in the Earth's Oceans. \nSince we can build submarines that can operate at 10km, we could reasonably suppose that we could operate submarines down to about 100km on Europa, which takes us to the rocky surface. However we have no way to pass through 20km of ice crust to reach the water layer. Building the submarine may be the easy part!\n", "topic": "astronomy", "url": "https://astronomy.stackexchange.com/questions/28560"}, {"image": "VQAonline_00000944.png", "question": "Why are most planetary orbits nearly circular", "context": "In our solar system, with the exception of Pluto all planets follow a relatively circular orbit around the Sun, at the same inclination. They also all rotate in the same direction, none are 'retrograde'. \n\nAn image of the orbits of our solar system.\nHow and why have they all formed into these orbits? Is this reflected in most orbital systems? What factors affect the orbits of newly formed systems?\n", "answer": "The combination of the conservation of angular momentum and gravity give you the inclination and direction. Gravity will condense material in the axis direction of the angular momentum, and collisions and gravitational interactions will damp oscillations in that direction, forming a disk of material. The angular momentum resists gravity in the plane perpendicular to the axis, maintaining the extent of the disk. Over time, the direction of rotation of the angular momentum is the direction of the vast majority of the material, due to collisions and interactions with that smaller population going the wrong way on a one-way street.\nThe circularity of the orbits is the result of a more dynamical process. Hopefully someone else here can explain it better than the following. My simplistic understanding is that a bunch of orbits crossing each other is not stable. Planets and planetesimals have their orbits changed continuously until such time as they settle into orbits that have fewer interactions with other bodies. Sort of a natural selection. In the long haul, this results in a relatively stable configuration with circular zones that tend to not interfere with the other circular zones. In fact, this is now part of the definition of the word \"planet\", in that to be called such, a planet needs to clear the neighborhood of its orbit. Though its not clear how it could occur, a bunch of elliptical zones that happen to be co-aligned would also not be stable since orbits precess due to various influences.\nWhere the material was less dense than where the planets are currently must certainly have many objects in eccentric, retrograde, and/or highly inclined orbits. Pluto is the first hint at that, with a relatively high inclination and eccentricity compared to the planets. Sedna is far more eccentric.\nAs for other systems, I'm sure there must be some oddballs out there. However the Kepler mission has observed mostly circular orbits for planets around other stars.\n", "topic": "astronomy", "url": "https://astronomy.stackexchange.com/questions/1979"}, {"image": "VQAonline_00001150.png", "question": "Would drinking melted ice water from Mars taste like sparkling water?", "context": "Does water on Mars form from the atmosphere as ice free from salt?\nDoes water desalinize through the evaporation and transpiration?\nWhen the fresh top ice is placed in a container then pressurized to melt, would the water be carbonated?\nParticulates of salt and dust is less in the atmosphere during a Martian winter so would the ice that forms be also drinkably cleaner?\nhttps://space.stackexchange.com/questions/33884/how-to-drink-water-on-mars\nhttps://earthscience.stackexchange.com/questions/16076/how-salty-can-ice-be\n\nhttps://kottke.org/18/12/a-massive-ice-filled-crater-on-mars\n", "answer": "Any ice that forms from water will be free of salt -- the process of crystallization does an excellent job of removing impurities.\nBut that's only when freezing out of liquid water, which is not a process that happens today at the Martian surface. When ice forms out of the air, it forms as tiny crystals -- \"hoar frost\" we call it on Earth -- and forms on substrates, such as grains of sand. Since the soil of Mars is laden with salts of various sorts (including things a lot nastier than NaCl, such as perchlorate salts), it's hard to believe that the Martian hoar frost wouldn't be contaminated with salty sand and dust.\nAnd, of course, the Martian atmosphere has dust suspended in it which is continually precipitating out and dust which is blown up by the not-infrequent dust storms and dust devils and all of this falls on any newly-formed frost.\nIt's very hard to see how there could be any macroscopic ice deposits on Mars that don't have dust and salt as impurities.\n", "topic": "astronomy", "url": "https://astronomy.stackexchange.com/questions/29361"}, {"image": "VQAonline_00002531.png", "question": "For civil or passenger airplane, is any moment that left and right elevator activated differently?", "context": "\nSource: Wikipedia\nI want to know if any moment that the left and right elevator are activated (used/functioned) differently of a civil or passenger airplane. Here I am talking about civil airplane, not a fighter jet. I.e., one (side) elevator is pulled up/down while another elevator of another side remain \"neutral\" or neither pulled up nor pulled down. If any, when or in what condition?\n", "answer": "No, there is no situation where left and right elevators are intentionally deflected differently in any civil aircraft. (I assume that the \"ruddervators\" of V-tailed airplanes don't count!)\n", "topic": "aviation", "url": "https://aviation.stackexchange.com/questions/95387"}, {"image": "VQAonline_00001871.png", "question": "How did the F28 have such a good L/D ratio?", "context": "The second link given on a Google search for \"Breguet Equation\" is to a PDF document from MIT. On page 7 of this document is a chart showing L/D ratios for various commercial aircraft (turboprop, regional jet, and \"large aircraft\") plotted against the year the aircraft entered service. Below is an image of this chart:\n\nWhat appears to stand out (at the top of the chart) is the Fokker F28 aircraft.\nFrom an engineering and operational perspective, what enabled that aircraft to have such a high L/D ratio?\nAlso, I thought that the F50 / F70 / F100 aircraft had the F28's wing (or an evolutionary version). Why didn't these aircraft have an equally good L/D ratio?\n", "answer": "Taken as I am with the little Fokkers because I used to walk around the flight line and see them being built, I reckon that's a mistake. The F-28 having a better L/D ratio than the F-27 with its long slender wings doesn't sound right to me. The F100 with its supercritical wing definitely should have better L/D than F28.\nThe two entries for F-28 in figure 1.4 seem very similar to the entries in Figure 1.9 of the MIT document that you mention.\n", "topic": "aviation", "url": "https://aviation.stackexchange.com/questions/38594"}, {"image": "VQAonline_00002297.png", "question": "Why does Thrust Reverser need to be operative during the cargo door operation?", "context": "It is stated that when the Electric Pump push button is depressed, the electric pump is automatically running as long as the cargo door selector valve is set to either OPEN or CLOSE position.\nDuring this period, the operation of other systems powered by the Yellow Hydraulic system is inhibited except Alternate Brake and Thrust Reverser of ENG #2.\n\nThe Alternate Braking is understandably essential as it stops aircraft from moving, and cargo door operation is considered that aircraft is in parking, not landing phase when the thrust reversers are only being used.\n\nMy question here is why thrust reverser ENG#2 needs to be operative during cargo door operation.\nThanks for your answers. \n\n(Source: Training Manual A320 Family)\n", "answer": "\u201cIt is stated that when the Electric Pump push button is depressed, the electric pump is automatically running as long as the cargo door selector valve is set to either OPEN or CLOSE position\u201d\nThis is not correct. Cargo door operation will automatically energize the Elec Pump while it is in the OFF position. (not depressed)\n\u201cMy question here is why thrust reverser ENG#2 needs to be operative during cargo door operation\u201d\nIt doesn\u2019t need to be operative. It needs to be Not Inhibited, in case there is a malfunction. \nThe cargo door uses the Yellow system. It is designed to be used on the ground even with the Electric Pump in the OFF position. The pump will come on automatically in this case to supply hydraulic power to the cargo door. It will inhibit all the other items on the yellow system to insure adequate hydraulic power to the cargo door.\nBUT......If there was a malfunction in the Cargo Door system while in flight, it could possibly cause the Yellow Elec Pump to engergize while it is in the OFF position. If that were to happen, it is designed to inhibit all items on the Yellow system. That would be bad, so they designed the switch to inhibit everything except Alt Brakes and #2 Reverse.\nThis situation could only happen if the Yellow Elec Pump was automatically commanded ON while in the OFF position. \n", "topic": "aviation", "url": "https://aviation.stackexchange.com/questions/77051"}, {"image": "VQAonline_00002658.png", "question": "Quality loss when exporting motion graphics videos in Premiere Pro", "context": "I am fairly new to Premiere Pro and I have been having an issue related to exporting my videos. All of the videos I am working on right now are motion graphics/animation videos. For these projects I create high quality images in Flash, export them as PNGs, and then use those images in Premiere Pro to create animations/motion graphics. The issue I'm having is that no matter what export settings I use, some of the images have obvious pixelation on them in the final result, and the colors seem to get slightly darker, even though they look perfect in the editor.\nAs an example, here is a side by side comparison on the image in the editor, versus a screenshot from the final exported video:\n\nYou will probably have to zoom in slightly to see the pixelation, but it's especially noticeable in thered are on the calendar image, on the orange circle attached to the email, and on the awning and sign over the shop entrance.\nThe pixelation issue seems to be most obvious with the red/orange/yellow colors, but I have also seen it with other high saturation colors and some dark blues.\nDoes anyone have any advice about why this might be happening and how I could fix it?\nI've recently been reading about issues that can be caused by having \"broadcast illegal colors\". Could this be related?\nI know that this issue could be occurring due to my export settings and sequence settings, but no matter what ones I use this still seems to occur. The primary export settings I've used are H.264 with Vimeo 1080p as the preset, since that is where I'm uploading, but I also tried a few others and didn't see any differences. For sequence settings I have also tried a few options including DNX 1080p, AVC-Intra 100 1080p, and Custom settings, but haven found any success.\nI also read that I should try exporting with DNX HQ 1080p 29.97, but I don't know what to do with an MXF file, and some of the research Ive done shows Vimeo sometimes has trouble with them, so any advice there would also be appreciated.\nAnyway, I think I've rambled enough for now. Any help you can provide would be greatly appreciated, and I'd be glad to answer any questions about the project settings.\nThanks!\n", "answer": "There are 2 things causing this issue:\n\nChroma sub-sampling. This is explained pretty well in this answer.\nCompression artifacts - For example, the mark above the \"1\"on the email notification is due to the way H.264 compresses data. This is known as \"ringing\" and is quite common with compression.\n\nSome options for dealing with it include:\n\nAvoid using saturated reds and blues especially in areas of high color contrast\nUse a codec or codec settings that don't exhibit ringing (easier said than done when you're uploading to a video service that may recompress it)\nUse settings to reduce how noticeable it is, such as turning on motion blur when exporting, so when things move, they exhibit softer edges\n\nUnfortunately, the reality is that TV is terrible for pure motion graphics. The subsampling that occurs makes it impossible to have nice crisp lines in saturated colors. And unfortunately, most of the codecs that work on all platforms are made to deliver TV and movies to users.\n", "topic": "avp", "url": "https://avp.stackexchange.com/questions/21499"}, {"image": "VQAonline_00002667.png", "question": "Correct workflow in Adobe Premiere for work with interlaced content", "context": "I have in input a DNxHD 120 in mov container 25fps interlaced upper field, I have to do various kind of edits, cuts, replace some pieces with after effects compositions and at the end of all the job export everything as a MXF 25fps again interlaced upper field.\nUntil now I always worked only with progressive contents so I'm unsure if what I'm doing is correct:\n\nI imported the footage into the project\nCreated a new sequence based on what i think is the correct format: 1080 25fps interlaced upper field\nStarted cutting, pasting and replaced some after effects comps\nExport with media encoder using the DNxHD SD 1080i 50 (i wonder why i can't find a 1080i 25 preset), however even if is writtend 50 it seems i can pick 25 fps.\n\nIs all this correct ? I mean what's the best practice ? I'm keeping everything interlaced upper field. I think Premiere is a professional software able to handle all the footage in interlaced without problems.\nor\nShould I instead convert and make everything progressive first and then export again back in interlaced? (i fear this solution is not very good)\nI also have this error if i try to rotobrush the footage:\n\nThis is starting to confuse me even more, why i should use a 50fps ?\n", "answer": "As for how to work... it all depends on what your final destination is. Are you working on a piece to be broadcasted, or for the Web? In the latter case, the final product should be progressive. If the former... the in turn it depends on your source material, which seems to be interlaced.\nSo you should:\n\nCreate an interlaced project, \"1080i50\". The \"50\", in this case,\ndoesn't mean \"frames per second\", but \"fields per second\" (2 fields\n= 1 frame).\nWork as you would normally in Premiere.\nExport it in an interlaced format too. \"1080i50\" sounds good, but again, that\ndepends on the final destination of your piece.\n\n", "topic": "avp", "url": "https://avp.stackexchange.com/questions/22862"}, {"image": "VQAonline_00002696.png", "question": "How to create a video so it can be displayed on 3 screens", "context": "My question is simple.\nIs it possible to create a video (by standard tools and processes) that needs to be displayed on 3 horizontal screens stacked side by side (like a screen wall)?\nThe idea is to put the screens on this setup:\n\nMy designers and video producers are stuck with this task, so I am trying to help with it researching.\n", "answer": "I do this all the time at work, I'm currently rendering a 6 screen video wall for an installation. The easiest way to work is to create the video at the size of all the displays put together, e.g. for 3 \u00d7 4K displays it would be 11520 \u00d7 2160 pixels. This is so that you can work on the whole thing at once, and also so you can justify to your boss the insane amount of money they spent on your workstation (did I say that out loud?). \nYou can use standard video tools like NLEs, e.g. Premiere or Resolve, compositors and animation packages like Fusion and After Effects or 3D applications like Maya, Lightwave or Blender. All modern versions of this kind of software will have the capability to work at custom resolutions, but as mentioned, your hardware will need to be pretty good, you'll have to budget for render time, and your proxy game will need to be strong (proxies are low-res substitutes that you can use to work on, then swap for the high-res originals at export time).\nThen you export, and encode into the playback format suitable for your playback devices. Some playback devices are able to handle large videos and split the playback into multiple screens. Small computer-based devices like Intel NUCs and Mac minis can be hooked up to multiple monitors and standard video players like VLC can be used to play the content full-screen across them all. Alternatively you can get multiple playback devices, one for each screen and synch them over a network. The output format will depend on the hardware you are using to play the material, so talk to whoever is setting that up.\nTo split the composition you re-compose the full composition into the screens that will be displayed on each monitor. In the installation I am working on at the moment there are 6 screens, but the material is being played as three parts, with each playback device playing to two screens. So I've split my 10800 \u00d7 1920 into three 3600 \u00d7 1920 compositions.\nThe tools above all have the ability to take one composition and nest it into another at a different resolution, but as I've found in my current projects you may have to re-edit the main composition to help with renders (I was hitting a GPU memory wall with some GPU-based effects, so I had to resize some of my layers and shuffle things to get it to render without choking).\n", "topic": "avp", "url": "https://avp.stackexchange.com/questions/29409"}, {"image": "VQAonline_00002761.png", "question": "Recommendation for a Sasquatch Day barbecue?", "context": "Group of hunting buddies here in BC, are planning have a Sasquatch Day barbecue on October 20!\nOctober 20 is the anniversary of the alleged Patterson\u2013Gimlin film of a real Bigfoot. Hoping to start a new tradition in this neck of the woods!\n\nThe footage was shot in 1967 in Northern California, and has since been subjected to many attempts to authenticate or debunk it.\nThe footage was filmed alongside Bluff Creek, a tributary of the Klamath River, about 25 logging-road miles (40 km) northwest of Orleans, California, in Del Norte County on the Six Rivers National Forest. The film site is roughly 38 miles (60 km) south of Oregon and 18 miles (30 km) east of the Pacific Ocean. For decades, the exact location of the site was lost, primarily because of re-growth of foliage in the streambed after the flood of 1964. It was rediscovered in 2011. It is just south of a north-running segment of the creek informally known as \"the bowling alley\".\n\n\nWith the onset of the Covid-19 pandemic, Bigfoot became a part of many North American social distancing promotion campaigns, with the creature being referred to as the \"Social Distancing Champion\" and as the subject of various internet memes related to the pandemic.\nWe are cooking up our own version of monster burgers.\nWould anyone be able to make a recommendation of what would go well with a Sasquatch Day themed drinks for our barbecue (beer and other drinks ideas are very welcome).\n", "answer": "A beer that should be relatively easy to find across the United States (and hopefully Canada!) would be Sierra Nevada's Bigfoot Ale. Certainly on target but likely much harder to find outside of Portland, Oregon would by anything by Sasquatch Brewing.\nFor cocktail fans, while I don't see anything specifically Sasquatch themed, some of the recipes for cocktails involving Monster Energy Drink might fit the bill.\n", "topic": "beer", "url": "https://beer.stackexchange.com/questions/9379"}, {"image": "VQAonline_00002759.png", "question": "Can I re-use a spirits measure cup for different spirits?", "context": "I have a stainless steel jigger measuring cup for UK spirits measures, like this image:\n\nThe alcohol does not seem to 'stick' to the jig cup, so is it okay to use the cup to, say, measure a gin drink then re-use it for, say, a vodka drink, without rinsing the cup?\nOr should I always wash or rinse the jig before re-use for either a different spirit or a different brand of the same spirit?\nShould I have separate measuring cups for different spirits?\n", "answer": "A purist would say always rinse.\nA normal person would say don't worry about it.\nA pragmatist would say measure the vodka before the gin and no one will be able to tell the difference.\n", "topic": "beer", "url": "https://beer.stackexchange.com/questions/8252"}, {"image": "VQAonline_00002737.png", "question": "Causes for regional differences in brewery count in the United States", "context": "I recently came across the following map of number of breweries per 1 million population in the United States:\n\nVisually it's apparent that there are the most breweries per capita in the northwest and northeast, and there are the fewest breweries per capita in the south.\nWhat is the main cause for these regional differences? The impact of climate? Laws and regulations in different parts of the U.S.? Regional demand differences? Something else?\n", "answer": "I believe it is mostly due to the laws of each state. Brewers Assoc. stats.\nAfter prohibition ended, it was up to the states and counties to determine their own stance toward alcohol. One part of the repeal of the 18th Amendment was the legalization of homebrewing wine and mead. The law did not allow the homebrewing of beer until 1978. Event then, it was up to each state to determine their stance toward the homebrewing of beer.\nThe states with the most breweries per capita follow the federal statute, which allows for 100 gallons per person of drinking age up to a maximum of 200 gallons per year. The states on the map with the least breweries have only recently relaxed their laws on homebrewing and alcohol sales in general. \nMany homebrewers dream of starting their own brewery or pub, but they need to be able to practice their craft. I am unable to locate the minutes to the Alabama House of Rep. discussion online, but the Basic Brewing Radio Podcast did air a snipped of the floor arguments on the legalization of homebrewing. Some of the arguments against were uniformed and antiquated.\nWhy open up a beer business in a city/county/state where regulations and attitudes are opposed to your product?\n", "topic": "beer", "url": "https://beer.stackexchange.com/questions/4628"}, {"image": "VQAonline_00003034.png", "question": "Adjusting brake rub on a mini-v brake (Giant Propel)", "context": "I have a Giant Propel and the front left brake is constantly rubbing and sticking against the wheel. I've tried adjusting the bolt highlighted below, which I believe is the spring tension, but loosening or tightening this bolt seems to do nothing. \n\nI've also tried adjusting the barrel adjuster but when I loosen it, the right side (which isn't rubbing) will move further out from the wheel, where as the left side (the one that's rubbing) won't move at all no matter how much I twist the barrel. \nDoes anyone know how to centre these brakes to prevent them rubbing?\nThanks.\n", "answer": "The small screw to the northwest of your red circle in the picture is the spring tension adjuster. Tighten it down to increase the pull on the left side. \nYou may also want to loosen the screw on the right side if there is a large imbalance in tension between the two arms.\nThe bolt you tried adjusting simply holds the brake arm on. It doesn't have anything to do with the adjustment of your brake.\n", "topic": "bicycles", "url": "https://bicycles.stackexchange.com/questions/31244"}, {"image": "VQAonline_00002878.png", "question": "How can I tell if the size of a bike is right for me?", "context": "Today I disassembled my current fixed-gear, and reassembled all the parts in another frame, which I am considering to purchase.\nThe current owner of the frame warned me that it might be too big for me, and I agree with him. Even though, I think it has good cost-benefit, so I decided to do a test-ride.\nI noticed the final geometry and rideability was pretty good, and since the frame has a long headtube (which I desire) I guess the inseam problem might not be so important considering the main goal is randonneuring, where \"riding\" characteristics are much more important than \"mounting and dismounting\" characteristics. Most probably the bike will receive a drop handlebar, even though I like flat bars a lot.\nBelow is a photo of the current set. There are 6cm of seatpost visible, and I might want to lower an aditional 1cm.\nThe main reason I want to somewhat stubbornly keep the frame is that I liked riding it, didn't see any major problem except for a slight touch of the upper seatstays between the thighs, but that tends to disappear since I felt the need to put the saddle a bit more forewards.\nWhat I am asking is:\n\nEven considering the frameset is ridiculously larger than I would\n typically choose, if I like riding the bike, is there any problem that\n I'm not considering, a reason NOT to keep the frameset?\n\nI'm 1,88cm (6'3\") tall, and have used 56cm bikes, this one is 64cm (seat tube, bbc to top).\nThanks for any insight!\n\n", "answer": "Frame fit\nPrimarily, a bike fits if you can position pedals, saddle and handlebars relative to each other so that they match your body's proportions. Secondarily, when these contact have been set, other frame parameters should fall into place so that the riding characteristics are as desired. That includes e.g. toe clip overlap, stand over height, bottom bracket height, heel/panniers clearance, steering geometry, or that an extremely short or long stem gives a weird steering balance. Depending on what you use the bike for, the importance of these parameters varies.\nDoes \"frame size\" equal \"seat tube length\"?\nIf you consider frame fit as putting the contact points in the right spots, the length and angle of the seat tube is one of the less important dimensions. By choosing the right seat post length and setback, it is possible to put the saddle in the right position relative to the pedals (or, simplified, the bottom bracket). Generally, on most frames, most riders can put the saddle in the right position relative to the bottom bracket.\nDepending on the use of the bike stand over height (or clearance) is more (MTB) or less (road) important. If you want a certain stand over clearance, of course this limits the frame size to a certain amount. This is easy to determine: With the correct tires and wheels, the height of the top tube should be lower than the length of your inseam.\nHow to determine \"frame size\" instead\nThe critical point is often if you can put your handlebars of choice in the right spot, too. One thing to consider here is that drop handlebars have a significantly longer reach than straight ones. This is because the normal hand position is on or behind the brake lever hoods with a drop bar, and not on the straight section next to the stem eye. If one does not want to resort to extremely short or long stems, a frame usually only allows for a good fit with either drop or straight bars.\nWorking backwards from the hand position through handlebars and a normal sized stem (ca. 8-12cm on a classic frame like this), one ends up at a certain position where the top of the head tube should be.\nStack and reach\nThe corresponding frame dimensions are the vertical and horizontal distances of the head tube upper end from the bottom bracket, called stack and reach. Unfortunately especially frame reach is quite difficult to measure on a frame without complicated measurement setups (or a good picture exactly from the side without perspective distortions). Therefore many tend to use top tube length as an alternative to reach, and (\"virtual\") seat tube length instead of stack. One should be careful though: The position of the top of the head tube not only depends on seat tube and top tube lengths, but also on the seat tube angle (which can be even harder to measure than reach).\nThe easiest way to find out your needs regarding stack and reach is to measure a bike which fits you well and has the relevant handlebars setup. One way to do this is to take a photo exactly from the side and from quite far away, and measure in the picture.\nWhat does this mean for you?\nAs you say, stand over clearance does not bother you with this frame, so the seat tube length is very likely ok. On the picture there is also plenty of room to move the saddle forwards, in the worst case with a zero setback seat post. So the saddle probably position fits you. Good.\nI am more concerned about whether the frame reach is too long, when changing to a drop bar. But as far as I know, the classic Raleigh road geometries do not have overly long reach, and the stem shown on your picture is way longer than what I would consider aesthetically pleasing on a frame like this (especially with quite little seat post to be seen).\nWithout knowing more about the frame and your fit preferences it is very hard to advise if it will fit you in this regard. The fact that the frame was built with a classic quill stem in mind, but has a threadless fork today, does not make it easier: It may be that you can't rise the bars enough with an ahead stem (and the fork already being cut).\nThe easiest thing would be to just try putting a drop bar on the frame (ideally with an adjustable stem temporarily). With a fixed gear that should be easy to do. Ride and adjust, and you will feel if it fits.\nWhat else?\nApart from all that I think that the average 56cm frame is quite a bit too small for the average 1.88m male (horizontal top tube, or \"virtual seat tube length\" assumed). I am 1.83m myself and usually start looking around what is labelled as 58cm.\nI'm pretty sure that the fork is not original. Ask the seller if the original still exists. It probably is a threaded fork, which allows for higher building quill stems. And it probably looks nicer, too.\nFinally: I have never heard about problems with the thighs rubbing on the seat stays before...\n", "topic": "bicycles", "url": "https://bicycles.stackexchange.com/questions/13419"}, {"image": "VQAonline_00002830.png", "question": "What maintenance is required for a thru axle?", "context": "A thru axle is a hollow axle that gets clamped by each fork leg and through the hub. Like what a motorcycle has. Much stiffer and stronger design then the typical QR dropout.\nMy sons newer downhill mountain bike has a thru axle on his front wheel/fork. I've never had a bicycle with an axle like this.\nIs there any different sort of maintenance I should be doing?\nEdit ---------\nThey are on some Marzocchi 55s, probably around 2008/2009 model year:\n\n", "answer": "Keep it lightly greased where it comes into contact with the hub bearing races and dropouts. This helps get it in and stops it seizing against the dropouts (the axle and dropouts are alu).\n", "topic": "bicycles", "url": "https://bicycles.stackexchange.com/questions/7535"}, {"image": "VQAonline_00003810.png", "question": "How can I export a full alignment from IGV as an image?", "context": "The IGV browser lets you export an alignment as an image (File => Save Image). However, this image only contains those reads that fit in the viewing window:\n\nAs you can see in the image above by looking at the scroll bar on the right, this is only a small subset of aligned reads. Is there any way to export an image of the aligned reads that includes all reads and not only those currently shown in the window?\n", "answer": "Running an IGV batch script helps increase the number of reads per image, compared with File | Save image, but it does not solve the issue by itself. \nTo run a script, first make a script file like so with 1 command per line:\nsnapshotDirectory ./\ngoto chr2:28582094-28582790\nmaxPanelHeight 10000\nsnapshot chr2_28582094_28582790.png\n\nRun the script using Tools | Run batch script menu. The importat trick is the maxPanelHeight 10000 which is what makes this include more reads than the default snapshot. \nYou can also convert this into a little shell script:\n#!/bin/bash\nmkdir -p /tmp/igv\nsleep 2\necho \"snapshotDirectory /tmp/igv\"\nsleep 2\ngoto chr2:28582094-28582790\nsleep 2\necho \"maxPanelHeight 10000\"\necho \"snapshot chr2_28582094_28582790.png\"\n\nThen, just open IGV, load the relevant BAM file and run the script like this to connect to IGV via telnet (credit to Pierre Lindenbaum):\nsh foo.sh | telnet 127.0.0.1 60151\n\n", "topic": "bioinformatics", "url": "https://bioinformatics.stackexchange.com/questions/10705"}, {"image": "VQAonline_00003757.png", "question": "Obtaining identifier from plot of sequence GC%", "context": "I was looking at this script from https://biopython.org/DIST/docs/tutorial/Tutorial.html#htoc297 and wanted to use it to determine outliers for tentative Horizontal gene transfer.\nThe data set is provided in the example, under the section 20.2.2 Plot of sequence GC%.\nI want to yield a identifier for the x axis instead of just an arbitrary number of genes or cds' so I can actually determine which sequences have the outlying values, as I can not with the current graph.\n\nI am not great at python or understanding list comprehensions, so if anyone could augment the code provided in the example to do so, it would be appreciated.\nI provided the code below also.\n#from : https://biopython.org/DIST/docs/tutorial/Tutorial.html#htoc297, data set under \n#section 20.2.2 Plot of sequence GC%, ls_orchid.fasta\n\nfrom Bio import SeqIO\nfrom Bio.SeqUtils import GC\n\ngc_values = sorted(GC(rec.seq) for rec in SeqIO.parse(\"ls_orchid.fasta\", \"fasta\"))\nimport pylab\npylab.plot(gc_values)\npylab.title(\"%i orchid sequences\\nGC%% %0.1f to %0.1f\" \\\n % (len(gc_values),min(gc_values),max(gc_values)))\npylab.xlabel(\"Genes\")\npylab.ylabel(\"GC%\")\npylab.show()\n\n", "answer": "The tricky bit here is that you need a list of sequence records sorted in the same order as the CG values. You can accomplish this by making tuples of the seq records and GC values, then sorting the tuples specifying the GC values as the sorting key:\ngc_w_records = sorted(((GC(rec.seq), rec) for rec in SeqIO.parse(\"ls_orchid.fasta\", \"fasta\")), key=lambda tup: tup[0])\ngc_values = [x[0] for x in gc_w_records]\nrecords = [x[1] for x in gc_w_records] \n\nI don't know much about matplotlib and it's going to be challenging to display 100 IDs, but you'll probably be interested in the extremes of the GC distribution, which you can obtain directly from the sorted list of records:\nrecords[0:2]\n# [SeqRecord(seq=Seq('CGTCACGAGGTCTCCGGATGTGACCCTGCGGAAGGATCATTGTTGAGATCACAT...CAT', SingleLetterAlphabet()), id='gi|2765587|emb|Z78462.1|PSZ78462', name='gi|2765587|emb|Z78462.1|PSZ78462', description='gi|2765587|emb|Z78462.1|PSZ78462 P.sukhakulii 5.8S rRNA gene and ITS1 and ITS2 DNA', dbxrefs=[]),\n# SeqRecord(seq=Seq('CGTAACAAGGTTTCCGTAGGTGAACCTGCGGAAGGATCATTGTTGAGATCACAT...AGG', SingleLetterAlphabet()), id='gi|2765568|emb|Z78443.1|PLZ78443', name='gi|2765568|emb|Z78443.1|PLZ78443', description='gi|2765568|emb|Z78443.1|PLZ78443 P.lawrenceanum 5.8S rRNA gene and ITS1 and ITS2 DNA', dbxrefs=[])]\n\nor:\nrecords[-3:-1]\n# [SeqRecord(seq=Seq('CGTAACAAGGTTTCCGTAGGTGAACCTGCGGAAGGATCATTGTTGAGACAGCAG...TAA', SingleLetterAlphabet()), id='gi|2765656|emb|Z78531.1|CFZ78531', name='gi|2765656|emb|Z78531.1|CFZ78531', description='gi|2765656|emb|Z78531.1|CFZ78531 C.fasciculatum 5.8S rRNA gene and ITS1 and ITS2 DNA', dbxrefs=[]),\n# SeqRecord(seq=Seq('CGTAACAAGGTTTCCGTAGGTGAACCTGCGGAAGGCTCATTGTTGAGACCGCAA...AAG', SingleLetterAlphabet()), id='gi|2765625|emb|Z78500.1|PWZ78500', name='gi|2765625|emb|Z78500.1|PWZ78500', description='gi|2765625|emb|Z78500.1|PWZ78500 P.warszewiczianum 5.8S rRNA gene and ITS1 and ITS2 DNA', dbxrefs=[])]\n\n", "topic": "bioinformatics", "url": "https://bioinformatics.stackexchange.com/questions/4104"}, {"image": "VQAonline_00003862.png", "question": "How can I improve or otherwise investigate an unreliable genome tree?", "context": "Summary My genome tree doesn't agree with my gene trees and I get the feeling that my genome tree might be wrong, possibly due to long branch attraction, but I don't know how to check/fix it.\nBackground\nI have a set of genes of interest recovered from genomes and metagenomes. I also have some bins built from the metagenomes of interest.\nPrevious work I've built multiple gene trees, which tend to agree with each other, as well as with other phylogenetic methods (16S rRNA, initial genome trees where available, conserved orientation of the genes of interest). So at this point I though I had a pretty good idea of how my genes / species of interest are linked to the species tree.\nConcern\nI've assembled the metagenomes and recovered bins. I've refined bins of interest with anvio, based on sequence composition / differential coverage, and kept only the bins which appeared quite high quality (>70% completion*, <10 - often <5% - redundancy). Thus contamination in the metagenome assemblies (MAGs) is unlikely. The contamination would have to happen multiple times in the same way to different samples from different environments, at the same coverage, to get the same effect.\n\nwith the exception of a bin of particular interest which I accepted with 62%\n\nI have de-replicated these bins, added some known genomes, and tried to build a species tree with Orthofinder (with the proteomes). There is a problem occurs with a clade of 3 bins I will call 'X'.\n\nTrees Based on the gene-of-interest phylogenies, X looks like probably a sister-group to the rest of known genomes from order 'A'. Which is very interesting.\nWith the 16S rRNA, it's not quite as clear (there are no 16S rRNA sequences in the bins); however, some 16S rRNA amplified from one of the original samples fit the bill as 'sister-clade to basically everything else in group A'). The first (100 or so) BLAST hits for this 16S sequence are all from group 'A', still.\nTree conflict However, in the proteome tree, X as well as Y (Y being a sequenced genome that I suspected was related to X based on 16S data) end up in order 'B' instead of 'A'. GTDBtk also labels X as belonging to clade 'B', not 'A'.\nNow,I know that genes and genomes don't always have the same evolutionary trajectories. But multiple sets of data lead me to a different conclusion than the proteome tree, so I have to at least question whether the proteome tree is correct; what's likely to be the problem (I suspect long-branch attraction), and, if it is long-branch attraction (or anything else) how can I uncover the correct signal?\nThank you for your time!\n", "answer": "The difficulty in answering the question is:\n\nThe absence of \"Y\" in the \"Gene of interest\" tree\nThe sampling bias in the number of taxa mostly between the \"genome tree\" and the \"gene of interest\" and \"16S\" tree\nThe \"genome\" tree is unrooted so its difficult to compare it with the \"16S\" tree, thus its possible that if \"16S\" was unrooted it falls within monophyly \"B\"\nThe trees don't have any branch or statistical support\nWe don't really know the tree building method (I haven't looked it up)\nThe bacteria might have significant recombination (we don't know the bacteria).\n\nThe answer is there is a lot of room for different tree artefacts here, however it could be a biologically real result. I agree \"the tree\" is simply a tool which uses a series of assumptions and obtaining confidence in a tree requires a lot of analysis. Vast amounts of computational time are dedicated to obtaining a robust tree.\nThe principle step is to compare like for like:\n\nsame number of taxa and\nsame rooting position (or unrooted).\nsecond step is to obtain statistical robustness for the observation.\n\nThere is a formal process for identifying conflicting trees and it needs code - I have written stuff that does this and maybe I should release it. Essentially, you would data chunk gene by gene to produce a genome of gene trees and use e.g. ETE3 or similar to plot the tree conflicts. It ain't rocket science. Thus the amount of user time is trivial. The idea is that once the \"problem trees\" are identified, e.g. rapid evolution.\nThe one observation I would say is that the \"Y\" branch looks a little bit weird and this could indicate part of the data is has resulted from different evolutionary rate and that is the basis for long-branch attraction.\n\nScenario 1 It could be that the 16S and \"genome\" tree are part of the same tree (monophyly B), they are just rooted differently. Thus the conflict is just a small number of genes and this could lead to ...\nScenario 2 Keep in mind bacteria do undergo 'horizontal transmission' and if 'gene of interest' is under strong selective pressure recombination is entirely possible.\n", "topic": "bioinformatics", "url": "https://bioinformatics.stackexchange.com/questions/19117"}, {"image": "VQAonline_00004296.png", "question": "What moth is this and how do we get rid of them?", "context": " \nThis bugs are everywhere in my house and we've tried so many home remedies, but they won't leave. Can anyone help us!?\n", "answer": "That is an Indian meal moth Plodia interpunctella. Family Pyralidae. They are very common household pests. https://www.doyourownpestcontrol.com/indian_meal_moth.htm\nThey breed in grains or cereals. You need to look through your pantry, and find out what they have been breeding in. Seal up your grains and cereals in plastic, or put them all in the fridge. \n", "topic": "biology", "url": "https://biology.stackexchange.com/questions/76532"}, {"image": "VQAonline_00004274.png", "question": "Difference between AjuA and ajuA in biosynthesis", "context": "I cannot for the life of me figure this out. In the following figure, you can find ajuA and AjuA for example.\nI do understand that the lower-case ones are the genes. The arrows indicate the direction of transcription. However, what is the difference to the upper-case AjuA? The text speaks of subunits. How are the two related? When do I speak of ajuA and when would I be talking about AjuA?\nI would also be happy if you could point me in the direction of some literature about the basics of this kind of notation.\nPS: I do understand the mechanisms of the biosynthetic route and am familiar with the different domains in the modules as well as PKS and NRPS modules.\n\n", "answer": "It's just the nomenclature:\n\nBacteria: Gene symbols are typically composed of three lower-case, italicized letters that serve as an abbreviation of the process or pathway in which the gene product is involved (e.g., rpo genes encode RNA polymerase). To distinguish among different [but related genes], the abbreviation is followed by an upper-case letter (e.g., the rpoB gene encodes the \u03b2 subunit of RNA polymerase). Protein symbols are not italicized, and the first letter is upper-case (e.g., RpoB). \n\n[source]\nYou can check out the Wikipedia article on Gene Nomenclature for more information.\n", "topic": "biology", "url": "https://biology.stackexchange.com/questions/73591"}, {"image": "VQAonline_00004243.png", "question": "Can anyone identify this beetle?", "context": "\nFruit Chafer found in Zambezi valley at Victoria Falls on the Zambia/Zimbabwe border \n", "answer": "That is a flower beetle in the family Scarabaeidae, subfamily Cetoniinae.\nDerby's flower beetle (Dicronorrhina derbyana)\nhttps://en.wikipedia.org/wiki/Dicronorrhina_derbyana\n", "topic": "biology", "url": "https://biology.stackexchange.com/questions/69859"}, {"image": "VQAonline_00004533.png", "question": "Unnatural Axe Santa vs. Elf and the Mummy card", "context": "In an epic Munchkin game with the Unnatural Axe extension, we had an Elf player encounter Santa. Santa is normally a level 9 monster, but that changes by -5 because Santa trusts Elves. Silly Santa.\nThat player was doing way to well, so someone added Mommy to the fight. Mommy can only be played on a monster of Level 5 or below, or one that is a Baby. We reckoned that since Santa was now a Level 4 monster against Elves, this card could be applied! Mommy adds another monster of the same kind but 10 levels higher.\nThe poor victim Elf player complained loudly in proper Munchkin fashion, but conceded that our interpretation of the cards was probably allowed.\nWas our interpretation correct, and was Mommy playable here? We were going by Santa dropping down 5 levels for Elves here, but perhaps it should not have been allowed because normally Santa is Level 8 instead.\nIn case you feel sorry for that player, they survived (having slain a monster without help to get of Santa's naughty list) to end up with this hand about two rounds before winning, utterly unstoppable:\n\n", "answer": "Unfortunately, the -5 added to Santa's level does not change the actual level. It is simply a Bonus/Penalty. It is much the same way the Elven Bow provides a +4 Bonus. The Bow does not change the character's Level. Rather, it changes the character's Attack, which is a derivative of the Player's Current Level plus all Bonus and Penalties.\nThough there are some cards that specifically state \"Adds X to Monsters Level\", the Race/Class bonus/penalty is not one of those.\n", "topic": "boardgames", "url": "https://boardgames.stackexchange.com/questions/21226"}, {"image": "VQAonline_00004595.png", "question": "Adding dice to checks in Pathfinder: Rise of the Runelord", "context": "In the FAQ, a point says\n\nChoices Matter. Your choices have consequences. (...) If you roll too low on a Combat check, it\u2019s too late to play a Strength spell to improve your result. \n\nDoes that mean I have to play all the 'discard this card to add a die to a check' cards before attempting the check? E.g, I'm doing some combat check, I need 8 to beat it. I roll my hand-to-hand combat strength check, and get a 6. Can I now discard a BotG to add another die to the result, and try to beat the test?\n\nOr would I have to specify before the check that I would discard the BotG to get an extra die?\n", "answer": "You must play all cards that will modify the check before making the check.\nAccording to the rules (starting on page 11), each check is broken into a number of steps. In order (emphasis mine):\n\n\nDetermine Which Die You're Using.\nDetermine the Difficulty.\nPlay Cards and Use Powers That Affect the Check (Optional).\nAssemble Your Dice.\nAttempt the Roll.\nTake Damage, If Necessary.\n\n\nStep three is where you would discard your Blessing of the Gods in order to add the die. If you make it to step five, where you make the roll, it is now too late to play any cards or powers to modify have the check is made.\n", "topic": "boardgames", "url": "https://boardgames.stackexchange.com/questions/33558"}, {"image": "VQAonline_00004575.png", "question": "When to leave the 20 point?", "context": "I'm relatively new to backgammon, and something I often struggle with is deciding when to leave the 20 point once I've made it. Furthermore, I also sometimes struggle with whether or not to leave the 13 point before or after the 20 point. Are there any rules of thumb which can help guide my decision when it isn't immediately obvious?\n\n", "answer": "The question is more generally about when to leave pieces in your opponents home board. If those pieces are safe, you can theoretically leave them there until all of your other pieces are in your home board. The question is, should you? The advantage of leaving these pieces is threefold.\n\nIt forces your opponent to play safe when bringing pieces towards or into their home board, as you have a credible counterattack sitting there. Without that, they can take a much looser approach to their home board and you can only punish them if they hit one of your pieces.\nIt prevents them from completely blocking their home board, meaning you can always (theoretically) bear on.\nIt delays the beginning of the pure race.\n\nThe disadvantages are:\n\nThose pieces can be much harder to get to your home board. This is much more of a consideration if the pieces are sitting on the 24 point rather than the 20 point, as it's very easy for pieces on the 24 point to get stuck.\nYou can be forced to make a disadvantageous move if those are the only pieces not in your home board.\nIt delays the beginning of the pure race (this is a downside if you are ahead as it gives your opponent more chances to turn the game around).\n\nIn the case where you have already moved both pieces to the 20 point, disadvantage #1 is less of a concern. Consequently, whether you want to move these pieces as opposed to others mostly boils down to whether you are ahead or behind: move them if you are ahead so that you can start the pure race earlier (which solidifies your lead) and leave them if you are behind to try to disrupt/hit your opponent and gain an advantage.\nNecessary caveat: all of this depends on both the rolls you make and the situation on the board. If you have a roll that lets you safely advance pieces from one of these positions but not the other, this can force your hand. Additionally, if you can hit a piece in your opponents home or outer board, that should take extremely high strategic priority.\n", "topic": "boardgames", "url": "https://boardgames.stackexchange.com/questions/30277"}, {"image": "VQAonline_00004896.png", "question": "Help identifying window frame with hinge pin holes", "context": "Can anyone identify this piece (picture below)?\nIt looks like the top half of a window frame with some 2 pin holes in it.\nThe size is 1 x 6 x 4, it has no model number on it but does have LEGO branding on the studs.\n\n", "answer": "That would be 821 Garage Door Frame (Old style).\nAs the name implies, it was for the old garage door in these sets from the 50s and 60s.\n", "topic": "bricks", "url": "https://bricks.stackexchange.com/questions/12965"}, {"image": "VQAonline_00004820.png", "question": "Where can I get replacement cables of this kind?", "context": "What I'm looking for:\n\nThis is a 4,5V cable which is part of an old set I once had. I don't have any parts number, but I know this comes in different lengths.\nGoogle told me about bricklink.com and I've searched there but didn't find it.\nHelp please?\n", "answer": "This is an Electric, Wire 12V / 4.5V.\nThey can be found here:\nCatalog: Parts - Electric, Wire & Connector\nJust find the right length.\n", "topic": "bricks", "url": "https://bricks.stackexchange.com/questions/6879"}, {"image": "VQAonline_00004860.png", "question": "Why are these hinged plates slightly different colors?", "context": "Something I've noticed is that certain pieces appear to be slightly different colors for other pieces of the same color. In particular, hinged red pieces seem to be the most noticeably different, especially some versions of the red hinge plate.\n\nAs you can see, half of the hinge plate is quite a bit darker than the other half which matches the regular plates. The 1x2 fork clip piece is also a little darker, though not as much as the hinge plate and it's hard to see in the picture. I remember that hinge plate being that way when I got and 3 others in the 10167 Brickmaster Kit (with Digital Designer CD) back in 2004, so I don't believe it was faded or a defect.\nSo I've long wondered, what's the deal here? Are the hinged plates made of different plastic, like the transparent pieces are?\n", "answer": "Quorneng's answer is spot on, even bricks that were from the same set might have been made from differing batches of a particular color and slight variances will produce differing colors. Purple is the most notorious Lego color for having a great deal of variance between bricks, and red is not surprisingly, also inconsistent at times.\nTo add to the mix, Lego is changing its formulas over time to keep costs down, and to make the bricks more durable and consistent. Earlier ABS bricks where stiffer and more brittle than modern bricks. They have more of a tomato red color, and are easy to spot next to modern bricks.\nUV damage can alter the color of bricks over time, although this can be fixed with oxyclean + h202.\nFinally, I believe that Lego will internally recycle 'bad' batches of bricks, grinding up and melting down ABS from lots that don't pass QA. This probably also affects brick color. I had a friend who made the pilgrimage to Billund to see the main plant. He told me about how they have a display that shows the casting of bricks to visitors, and it re-uses plastic for the demonstration. I recall that he commented on how the plastic colors were slightly odd looking, presumably because it had been recast so often. This makes sense to me, but it is a second hand story, so don't cite me for your thesis...\n", "topic": "bricks", "url": "https://bricks.stackexchange.com/questions/9576"}, {"image": "VQAonline_00004961.png", "question": "Why is Budai often associated with the future Buddha Maitreya?", "context": "Many times I've heard of Budai (pictured below) being associated with Maitreya, the future Buddha. Why is this so and how widespread is this belief? Wikipedia lists a short verse uttered before Budai's death:\n\nMaitreya, the true Maitreya\n has billions of incarnations.\n Often he is shown to people at the time;\n other times they do not recognize him.\n\nIs there more to this association or was the simple utterance of that verse so long ago the sole reason for believing that Budai was the bodhisatta?\n\n", "answer": "Here is a reference to the Japanese Miroku Bodhisatva.\nAnother reference says that Hotei was Kaishi Zen priest and a Boddhisattva, the future Maitreya:\n\nHotei stands out from the other six Lucky Gods because he is the only one known to have been a real person, rather than a mythical being. According to Chiba, his Chinese name was Kaishi. His birth date is unknown, but his date of death was in March of the year 916. He was a Zen priest who could quote Buddhist text verses almost nonstop. Some sources affirm that he was an incarnation of the Bodhisattva Miroku (Maitreya in Sanskrit), the Buddhist Savior of the Future (Giraud 404). However, some of his actions would not be considered particularly saintly. For example, he would beg for meat and fish even though they were foods forbidden to priests.\n\nBits of details about claims to Maitreya and where references of it could be found in the pali canon.\n", "topic": "buddhism", "url": "https://buddhism.stackexchange.com/questions/8775"}, {"image": "VQAonline_00004969.png", "question": "What is this picture?", "context": "Even after referring to several books I simply cannot understand what this picture is depicting. I am not a Buddhist and am hence finding it difficult to decipher.\n\nI found it on this page, whose title is, \"The Buddha of Purification with consort\". The description on that page says it depicts a manifestation of \n\n\"unity of the fully developed masculine and feminine energy: the complete purity and highest state of enlightenment\".\n\nIf that's an accurate description, could you provide the explanation of that phrase?\nThe Buddha himself didn't want to get entangled in samsara, let alone unite with the other sex; so isn't this contradictory?\n", "answer": "That would be yab-yum. I think the wikipedia entry will more than answer your question. You can find that here. The one thing the entry only hints at, however, is that this is an actual practice in Tibetan Tantra. In fact, it's generally what comes to mind, for a popular audience anyway, when the word \"tantra\" is used. I think it's an exercise in futility to try and justify this practice according to Theravada or Mahayana ethics. Within a tantric context, it just is what it is. \n", "topic": "buddhism", "url": "https://buddhism.stackexchange.com/questions/19036"}, {"image": "VQAonline_00004976.png", "question": "Anapanasati - are the 16 considered progressive sequence of steps?", "context": "Answers to this question suggest that the 16 are steps or stages of anapanasati, which implies that they are a progressive sequence of steps or stages. They are also related to the four foundations of mindfulness or the satipatthana.\nOn the other hand, the booklet \"How To Meditate\" (quoted below) by Yuttadhammo Bhikkhu, seems to suggest that one should not use it as progressive steps. Instead, it recommends that if feelings arise, stop what you were trying to do, and go with that. Or if thoughts arise, stop what you were trying to do, and go with that.\nThat does not sound like a progressive sequence of steps. That seems to be more \"go with the flow\" or feel free to move from any of the 16 to any other, depending on what arises.\nSo, which is right approach to anapanasati? Are the 16 progressive sequence of steps approach right? Or is the approach of the \"How To Meditate\" booklet right?\nOr are they two different techniques?\nFrom \"Chapter Two: Sitting Meditation\" of the booklet \"How To Meditate\":\n\nRegarding the body, watching the rising and the falling is sufficient\n for a beginner meditator. At times, one might wish to also acknowledge\n the position of the body as \u201csitting, sitting\u201d, or \u201clying, lying\u201d if\n it is more found to be more conducive for clear observation.\nIn regards to feelings, when a sensation arises in the body, one\n should fix one\u2019s attention on it, discarding the abdomen and focusing\n on the sensation. If a feeling of pain should arise, for example, one\n should take the pain itself as a meditation object.\nAny one of the four foundations may serve as a meditation object, as\n all four are aspects of reality. It isn\u2019t necessary to stay with the\n rising and falling of the abdomen at all times. Instead, when pain\n arises, one should observe the new object, the pain, in order to\n clearly understand it for what it is, rather than judging or\n identifying with it. As explained earlier, the meditator should simply\n focus on the pain and create the clear thought, \u201cpain, pain, pain,\n pain\u2026\u201d until it goes away. Instead of getting upset about the pain,\n one will see it for what it is and let it go.\nWhen happiness arises, one should create the clear thought, \u201chappy.\u201d\n When one feels peaceful or calm, one should create the clear thought,\n \u201cpeaceful,\u201d or \u201ccalm\u201d until that feeling goes away. Here, the object\n is to avoid clinging to the feeling, which would create a dependency\n on it. When one clings to positive feelings, one will be inevitably\n dissatisfied when they are gone.\nOnce the sensation disappears, one should return to the rising and\n falling of the abdomen and continue observing it as \u201crising\u201d and\n \u201cfalling\u201d.\nIn regards to the mind, if thoughts arise during meditation, one\n should acknowledge them as \u201cthinking\u201d. It doesn\u2019t matter whether one\n is thinking about the past or future or whether one\u2019s thoughts are\n good or bad; instead of letting the mind wander and lose track of\n reality, bring the mind back to the reality of the thought with,\n \u201cthinking\u201d. Then return to the rising and falling and continue\n practice as normal.\nIn regards to dhammas, when the mind gives rise to liking, pleased by\n a certain experience, create the clear thought, \u201cliking, liking\u201d. When\n disliking arises \u2013 anger, boredom, frustration, etc. \u2013 create the\n clear thought, \u201cdisliking, disliking\u201d, \u201cangry, angry\u201d, \u201cbored, bored\u201d,\n or \u201cfrustrated, frustrated\u201d. When laziness or drowsiness comes up,\n create the clear thought, \u201clazy, lazy\u201d, or \u201cdrowsy, drowsy\u201d. When\n distraction or worry arise, \u201cdistracted, distracted\u201d or \u201cworried,\n worried\u201d. When doubt or confusion arise, \u201cdoubting, doubting\u201d or\n \u201cconfused, confused\u201d and so on.\nOnce the above hindrances subside, bring the mind back again to a\n clear awareness of the present moment by focusing on the rise and fall\n of the abdomen.\n\nThe 16 steps or stages of anapanasati:\n\n", "answer": "Try not to overthink it. There is a clear path of progressive steps or progress, yes, but for each meditator it varies somewhat as they each have varied conditions. We don't control what arises, so we can't expect to watch it unfold like a textbook.\nThe stages or steps occur naturally, without us having to force or think about it, as long as you are doing the one basic thing correctly: observe with wise attention whatever arises, without judgement. \nYou'll notice each grouping of steps follows a pattern of prescriptive action - this pattern of treatment is the important thing. Apply this pattern to whatever arises, and the Path will unfold before you, eventually without \"effort\". At first we start with the coarse, and naturally go sublter and subtler as our minds get more still and concentrated. The next thing to contemplate will be waiting for you, and arise on its own. Don't try to seek out an expected experience. Just observe whatever it is as it arises, watch it as it disappears, relaxed, without clinging. Its shockingly simple when it finally clicks. No need to wonder if you're at stage 13 or should I now contemplate the next thing on the to-do list or anything like that, that will just stir your mind up.\nThe breath is simply our home base, something we can always return to if we wander. Sticking with the breath trains our focus, trains our concentration, trains our resistance, trains us to not cling or push away. It is important to remember that the training of these skills are the key, not what object you use. \nMaster the method, and the Royal Road to Nibbana will appear. Good luck. May we all become truly free.\n", "topic": "buddhism", "url": "https://buddhism.stackexchange.com/questions/34971"}, {"image": "VQAonline_00005135.png", "question": "What are the fumes that form above liquid nitrogen?", "context": "What is the composition of liquid nitrogen fumes? Is direct contact with these fumes harmful?\n\n", "answer": "The visible fog that forms when liquid nitrogen is poured into an open container is almost entirely water fog: the boiling nitrogen chills the air above it, causing the humidity in the air to condense into fog.\nThere may be some microdroplets of liquid nitrogen in the fog too, but the air, even after it's been chilled, is still much warmer than the boiling point of nitrogen, so there won't be much. (You can condense air by running a continuous stream of liquid nitrogen through a heat exchanger and blowing air over it, but mostly you get liquid oxygen when you do that.)\nLiquid nitrogen has a relatively small heat of vaporization, as these things go: 199.2 kJ/kg according to Air Liquide. (Compare 572.2 kJ/kg for dry ice.) That plus the Leidenfrost effect means it's actually rather difficult to freeze yourself with the stuff. There's a standard demo where you pour liquid nitrogen into an audience member's cupped hands. The Leidenfrost effect prevents it ever coming in contact with their skin, and it only leaves a mild chill behind. Try this with dry ice and your \"volunteer\" will need emergency treatment for frostbite.\nGaseous nitrogen is second only to the noble gases in its inertness, and it makes up 78% (by volume) of the air you're breathing as you read this, so obviously the \"fumes\" aren't harmful in themselves. However, as was mentioned in an earlier answer (which seems to have been deleted), if you let liquid nitrogen evaporate in an enclosed space, it can \"displace\" the oxygen that makes up another 21% of the air, at which point anyone in the area may asphyxiate. The sensation of suffocation is triggered by having too much CO2 in your bloodstream, not by inadequate oxygen supply, so you might not even notice that anything was wrong until you collapsed. That's the second most dangerous thing about evaporating liquid nitrogen.\nThe most dangerous thing is, if you let liquid nitrogen evaporate in a sealed container, it's liable to explode on you.\n", "topic": "chemistry", "url": "https://chemistry.stackexchange.com/questions/15017"}, {"image": "VQAonline_00006124.png", "question": "Why do we subtract the mass of one proton in mass spectrometry (negative mode)?", "context": "This is probably very basic, but I have been given the understanding that we subtract the mass of only one proton when looking for our compound in a mass spectrometry (negative mode). I dont really understand why we are only subtracting the mass of one proton when our compound could potentially loose more than one proton at higher pH? \nIn example, the amino acid glutamate, would it not loose or gain more than one proton is a negative (higher pH) or positive (lower pH) mode mass spectrometry (MS)? Perhaps I have misunderstood the concept of negative and positive mode for MS?\n\n", "answer": "Interesting question, which originates from a major misconception related to pH in LCMS. Nowadays, there are dozens of ionization techniques so in the question always mention the ionization method.\nAnyway, LCMS is usually done with electrospray ionization (ESI) technique and so are amino acids. First of all nobody completely knows the exact process in electrospray ionization as yet. This is not unusual in mass spectrometry where even a common ionization technique like ESI and electron impact, the picture is not 100% clear.\nSecond important point is the pH in ESI does not determine whether a negative ion or a positive analyte ion will be formed. I also used to have this misconception a decade ago about pH. The final ionization process in ESI takes place in a gas phase. However it is well known that certain ion can suppress ionization very badly. Don't try to connect mobile phase pH with the state of ionization of the molecule in the gas phase.\nIn negative ion, you are simply observing a net negatively charged species. Nothing \nFor details, I'd encourage you to read \"pH Effects on Electrospray Ionization Efficiency\" in\u2002the Journal of the American Society for Mass Spectrometry\u00b7 Go to the journal website and locate the paper with this title. I am not providing a link so that students develop the habit of searching themselves. \n", "topic": "chemistry", "url": "https://chemistry.stackexchange.com/questions/135013"}, {"image": "VQAonline_00005950.png", "question": "Identifying orbital given electron probability density plot", "context": "\nBeing a high-school student, I only have a qualitative understanding of the wave mechanical model of atom.\nBelow are my inferences regarding the plot shown in the above picture:\nThe concerned orbital has $3$ radial nodes (I'm counting the places where the value of the y-coordinate is zero excluding the ones at the extreme left and right).\nHence, the orbital should satisfy $n-l-1=3$. The orbitals satisfying the condition are $4s, 5p, 6d, 7f,\\ldots$.\nAre my inferences correct?\n", "answer": "I think your inference is correct, though the plot looks very awkward. There should be an exponential decay at large values of $r$, but in the plot the curve plunges sharply onto the $x$-axis. Also the shapes of each peak should not be so symmetric, but instead skewed towards the left.\n", "topic": "chemistry", "url": "https://chemistry.stackexchange.com/questions/109149"}, {"image": "VQAonline_00006399.png", "question": "What will be the best move in this position?", "context": "I am a lower rated (1200-1300) player, and I am quite hesitant while making sacrifices. In the current game I see that black's king is unprotected and a bishop sacrifice at f5 can open up an attack against the black king. Is this the best move in the current position?\n\n", "answer": "Almost certainly Bxf5 is a winning move, but it may not be what you want to play. I asked if exf6 was legal. If it is, play it. After Black recaptures play Bg6+ and your attack is just as strong without sacrificing.\nIn the diagram position, Blacks position is dreadful, probably lost. If you can win without taking risks, why not? \nYou are not sure if the sacrifice works. That is, you do not trust your analysis. Under those circumstances, should you sacrifice and risk losing? Generally speaking not, if you have a very good position and can see other ways to make progress. Generally yes if you think that your position is bad, and the sacrifice is a last shot.\nBut perhaps you should sacrifice anyway, just to find out how good your analysis is. Much depends then on the circumstances of the game, Friendly skittles? Last round of tournament? But if you find that it does not work, try to learn something. What did you not see? Why did you not see it? Is there something you should now remember to look for? Did you win, but only because your opponent defended badly? You should look at these things post mortem if you opponent is willing.\n", "topic": "chess", "url": "https://chess.stackexchange.com/questions/17919"}, {"image": "VQAonline_00006506.png", "question": "Is this king in checkmate?", "context": "Consider this chess board set up:\n\nIs the black king allowed to move to B4 or D4? The knight on C6 would put their white king in check if it moves.\nI haven't really read any rules about this so thought I'd ask.\n", "answer": "Yes, a piece can check even if it can't move because of a pin. This is explicitly written in the FIDE Laws of Chess (emphasis added):\n\n3.9.1 The king is said to be 'in check' if it is attacked by one or more of the opponent's pieces, even if such pieces are constrained from moving to the square occupied by the king because they would then leave or place their own king in check.\n\nTherefore, if the black king tried to escape to b4 or d4 it would be moving into check, which is not allowed.\n", "topic": "chess", "url": "https://chess.stackexchange.com/questions/36888"}, {"image": "VQAonline_00006362.png", "question": "What is the solution to this mate in 2 puzzle?", "context": "\nWhat could be the answer to it?\n", "answer": "It is a nice little puzzle:\n3k4/3P4/3Q4/8/8/8/8/4K3 w - - 3 13\n\n1. Qd5 Kc7/Ke7 2. d8=Q# 1-0\n\n", "topic": "chess", "url": "https://chess.stackexchange.com/questions/12686"}, {"image": "VQAonline_00006776.png", "question": "Translation of \u7e54\u8349\u70ba\u8335 in context", "context": "I am translating Dai Hongci\u2018s diary. He is describing an Austrian school for young nobles (\u8cb4\u5191\u5b78\u5802\uff09. The above has a slightly unexpected word order. I am wondering, whether I understand it correctly (as grass interwoven into a thick mattress) , or whether \u7e54\u8349 is a plant name (cotton or something).\nBelow are the context (sentence by sentence), my German translation, as well as my rush attempt at an English one. A photography of a print, which I appended below, can be found here:\n\n\u683c\u81f4\u5668\u68b0\u5ba4\u3001\u85cf\u4e66\u5ba4\uff0c\u85cf\u7c4d\u9887\u5bcc\u3002/\nDas Zimmer mit den naturwissenschaftlichen Apparati sowie die\nBibliothek bergen zahlreiche B\u00fccher. /\nThe room with the natural sciences equipment and the library room contain plenty of books.\n\u89c2\u9a6c\u623f\uff0c\u65c1\u4e3a\u7ec3\u9a6c\u573a\u3002/\nBesichtigung der Stallungen.\nDaneben befindet sich das \u00dcbungsgel\u00e4nde der R\u00f6sser. /\nInspection of the stables. Next to them there is the training ground for the horses.\n\n\u5802\u540e\u56e0\u5c71\u4e3a\u56ed\uff0c\u6797\u6728\u4f18\u80dc\uff0c\u7ec7\u8349\u4e3a\u8335\u3002/\nHinter der Schule, an die Berge grenzend,\nist ein Garten zu finden.\nDes Waldes B\u00e4ume sind zahlreich und florieren,\ndie Gr\u00e4ser wachsen dicht wie eine Matte./\nBehind the school, adjacent to the hills, there is a garden (or park). Its trees are numerous and flourishing. Its grass is thickly interwoven as a mattress.\n\nTLDR: I understood \u56e0 without \u8349\u5b57\u982d as \u9023\u63a5, adjacent because of the Song poem: \u6c34\u8fb9\u5c0f\u4e18\u56e0\u53e4\u57ce, \u4e0a\u6709\u5de8\u7af9\u6570\u767e\u4e2a\n\n", "answer": "\u7ec7\u8349\u4e3a\u8335 (Weaving grass into a carpet/ grass is woven into a carpet) is the literal meaning\nIt figuratively describes the scene as 'densely grown grass look like a woven carpet' (you can imagine it was very short grass)\n", "topic": "chinese", "url": "https://chinese.stackexchange.com/questions/48180"}, {"image": "VQAonline_00006555.png", "question": "What does \u5530 (shua1) mean?", "context": "I met the word \u5530 shua1 in a Xi Yang Yang book as the sound \u5530\u5530 one character makes while eating. Two sources agree it is \"onomat. quick slick up.\" I don't know the English \"quick slick up.\" Google translate makes a plausible but unclear suggestion of shabu shabu for \u5530\u5530. \nWhat is this word?\nHere is the context. Lazy lamb is enjoying a snack. The panel caption tells us this is outside of the sheep village school, with a light breeze blowing (\u7f8a\u6751\u201c\u6559\u5ba4\u5916,\u98ce\u513f\u8f7b\u8f7b\u5439\u8fc7).\n\n", "answer": "baike.com describes it as \"swiftly\" or \"swish\"... I believe it is indeed onomatopoeia.\n\u5530 <\u5f62> - Adverb \n\u5f62\u5bb9\u8fc5\u901f\u7684\u6837\u5b50 \u3010swiftly\u3011\u3002\u5982:\u5530\u5730\u4e00\u4e0b\u8df3\u4e0a\u5899;\u5530\u6e9c\u6e9c(\u5f62\u5bb9\u6781\u5176\u8fc5\u901f);\u5530\u5566(\u5f62\u5bb9\u8fc5\u75be) \nDescribes \"swiftly\" or \"quickly\" doing something. Like to \"swiftly\" jump on the wall.\n\u5530 <\u8c61> - Adjective for sound\n\u5f62\u5bb9\u8fc5\u901f\u64e6\u8fc7\u53bb\u7684\u58f0\u97f3 \u3010swish\u3011\u3002\u5982:\u5530\u5530;\u5530\u5587\u5587\nOr describes a sound, to \"swish\" by something.\n", "topic": "chinese", "url": "https://chinese.stackexchange.com/questions/3595"}, {"image": "VQAonline_00006626.png", "question": "How to enter pinyin itself under Windows?", "context": "Is it possible to enter text as if normal Chinese text is entering\n\ni.e. with guessing of correct characters, but later entering Pinyin of entered text, not characters themselves?\nzh\u00e8zh\u012bg\u01d2u\n\n", "answer": "Usual IME's won't have that feature... so I think you have two approaches here.\n\nUse a special IME or IME scheme, for example \u5730\u7403\u62fc\u97f3 from \u4e2d\u5dde\u97f5\u8f93\u5165\u6cd5\u5f15\u64ce(RIME). Reference: this Chinese post\nType Chinese first, then search for a Chinese-Pinyin conversion tool/online app, for example http://hanyu.iciba.com/pinyin.\n\n", "topic": "chinese", "url": "https://chinese.stackexchange.com/questions/22558"}, {"image": "VQAonline_00006822.png", "question": "How to reconcile Kierkegaard's model of worship with Existential authenticity?", "context": "\nI've been told Kierkegaard suggested that worship be a form of theater where the congregants are actors and pastors are directors. (This is the only on-line reference I could find, does anyone have a link to a more authoritative source for this?)\nKierkegaard is regarded as an Existentialist. Existentialists stress the importance of Authenticity.\nCan someone please explain how it would be possible to worship authentically if a worshiper is merely acting?\n", "answer": "Kirk, I just came across this question from way back in December. Actually, one of the primary points that Kierkegaard was making is that what happens in worship is done for the sake of God (who is the audience). It is a corrective to the idea that worship is entertainment for the congregation or that the pastor/priest is the primary performer. The primary action of worship is the praise and prayer of the congregation offered to God; the role of the pastor/priest is to support the work of the congregation. The word \"liturgy\" is actually in its derivation \"the work of the people.\"\n", "topic": "christianity", "url": "https://christianity.stackexchange.com/questions/4804"}, {"image": "VQAonline_00006843.png", "question": "Why are the Psalms numbered this way in some Catholic Bibles?", "context": "I am looking at a Catholic Bible and I see the following:\n\nPSALM 1\n[...]\nPSALM 9-10\n[...]\nPSALM 51 V50 Miserere\nPSLAM 52 V 51\n[...]\nPSALM 150\n\nPlease see image below.\nWhat accounts for this numbering? \nIt is not a straight 1 thru 150 numbering. For example there is Psalm 9-10 and Psalm 51 has V50 alongside it.\nI am looking for a Catholic explanation sought but I assume a Bible scholar ought to be able to explain it.\n\nImage produced from The Jerusalem Bible Popular Edition.\n\n", "answer": "There is a difference in numbering between the Hebrew (Masoretic) text and the Greek and Latin (Septuagint and Vulgate). Psalms 9 and 10 in the Hebrew get combined into one as do Ps 114 & 115. Psalms 116 and 147 both get split in two.\nMost Protestant Bibles follow the Hebrew numbering while most Catholic and Orthodox ones follow the Greek. \nI suspect the reason for the difference is that the numbers were added to the Hebrew and Greek manuscripts at a later date and there were different opinions as to where one finished and the next began. \n\n\nCf. Septuagint (LXX) numbering of the Psalms | The Orthodox Church in America.\nCf. Note on the different numbering of the 150 Psalms | psalmen.wursten.be.\n\n", "topic": "christianity", "url": "https://christianity.stackexchange.com/questions/36613"}, {"image": "VQAonline_00006866.png", "question": "How was the translation of the Book of Mormon shown through Joseph Smith's seer stones and hat?", "context": "The Wikipedia article on the golden plates says that Joseph Smith translated the golden plates into the Book of Mormon with the help of \"seer stones\". Initially, he is said to have used two stones set in frames as glasses to see the new translation. Later, he switched to a different method with one seer stone inside a hat, but I'm confused as to how it was supposed to help in translation.\n\nAfter the loss of the first 116 manuscript pages, Smith translated with a single seer stone, which some sources say he had previously used in treasure-seeking. Smith placed the stone in a hat, buried his face in it to eliminate all outside light, and peered into the stone to see the words of the translation. [\u2026] Smith's translation did not require the use of the plates themselves. Though Smith himself said very little about the translation process, his friends and family said that as he looked into the stone, the written translation of the ancient script appeared to him in English.\n\n\nImage from Wikipedia, licensed under Creative Commons Attribution-Share Alike 4.0\nI'm confused about this process. Obviously, there is some sort of divine power assumed to be going on here, but I\u2019m trying to understand how the miracle was said to have manifested, specifically where these letters appeared.\nWas the stone projecting the words into the darkness? Were the words appearing upon the face of the stone, like the popular depiction of a crystal ball? Was the stone moving around and forming letters? How was the translation of the Book of Mormon shown through Joseph Smith's seer stones and hat?\nIn other words, how does hat + stone = words?\n", "answer": "In 1887, David Whitmer wrote about the translation process, and said the following (emphasis mine):\n\nJoseph Smith put the seer stone into a hat, and put his face in the hat, drawing it closely around his face to exclude the light; and in the darkness the spiritual light would shine. A piece of something resembling parchment would appear, and on that appeared the writing. One character at a time would appear, and under it was the interpretation in English. Brother Joseph would read off the English to Oliver Cowdery, who was his principal scribe, and when it was written down and repeated to Brother Joseph to see if it was correct, then it would disappear, and another character with the interpretation would appear. Thus the Book of Mormon was translated by the gift and power of God, and not by any power of man.\n\u2014\u2009David Whitmer, \"An Address to All Believers in Christ\" (1887), 12; Quinn (1998, p. 172).\n\nSo according to Whitmer, Joseph Smith put his face in his hat to exclude natural light, and the letters appeared through a \"spiritual light\" upon a piece of parchment that appeared.\nThis account does mention the seer stone being in the hat, but doesn't mention any connection between that and the letters appearing. Presumably, it was intended to be some sort of divine focus that was necessary for making the parchment appear.\n", "topic": "christianity", "url": "https://christianity.stackexchange.com/questions/66273"}, {"image": "VQAonline_00006891.png", "question": "What are these spots on the inside of my moka pot? Do I need to remove them?", "context": "I've got these spots on the inside the bottom section of my Bialetti Moka Express:\n\nThey are placed pretty evenly around the ridgy part toward the edge of the bottom of the circle, with a few toward the middle. They are only on the bottom of the pot as far as I can tell, not the sides. They look sort of like black mold, but I gave the pot a gentle scrub with warm water and a mild soap and they didn't budge.\nI don't normally use soap on my pot. Generally I make a cup of coffee, rinse all components, and allow them to air dry. Sometimes when I'm pressed for time, the just-used pot sits in the sink for a few hours without getting disassembled or rinsed. I have a hunch that that moisture may be contributing here. I use filtered water, so I don't think they are mineral deposits (unlike in this case). The pot is pretty new. It's been used 1-3 times per day for about three weeks.\nSo what are these spots? If they aren't affecting the flavor of the result, do I need to worry about them? If so, how can they be removed?\n", "answer": "Short story: They are probably aluminum oxide, but almost certainly some kind of metal oxide. You don't need to take any action unless they're affecting the taste of your coffee, and it shouldn't be. You probably can't (or shouldn't) really permanently remove them even if you wanted to.\nLonger story...\nThis is a classic Bialetti model, and a classic Bialetti symptom. The vessel is probably made out of alumin[i]um alloy (though there are similar models that are made of stainless steel, your picture seems to be looking down into the bottom heating vessel, and it looks like the classic aluminum species). Unfortunately the symptoms you're seeing, from my perspective, is a classic failure of the Bialetti design. Aluminum tarnishes (oxidizes) somewhat readily, especially when it is hot and in contact with water. Correspondingly, the only way to use this thing is, indeed, to heat it with water.\nThe vessel, especially, needs to be rinsed and dried thoroughly in The Right Way. The \"care\" page for Bialetti (follow that link, then click on the \"Care Use\" tab; I can't figure a direct link) suggests to dry thoroughly and most tellingly \"Do not reassemble the product until all parts are completely dry to avoid oxidation.\" Even if you're meticulously drying the thing, it's still going to be a little hot and a little damp; this is going to invite oxidation. \nStandard ways of removing tarnish are even more damaging in the long term than just leaving them alone. Aluminum oxide generally protects the metal from further oxidation (unlike rust -- iron oxide -- which flakes off and facilitates further oxidation).\nI have a stainless steel moka pot, and it's terribly tarnished on the outside from heating on a gas burner hundreds of times, but the inside is clean (and cleanable) as the metaphorical \"whistle\".\nFor completeness: It's also possible that they are mineral deposits; this answer to the other question that you linked has some additional info about that possibility. It would come from a buildup (\"scale\") from using \"hard\" water, and more so when nearly boiling dry (also not a good idea); filtered or distilled water is much less likely to do this. However, I think it's unlikely that you'd see scale buildup over the span of a couple dozen uses.\n", "topic": "coffee", "url": "https://coffee.stackexchange.com/questions/2069"}, {"image": "VQAonline_00006922.png", "question": "Why is my dalgona coffee foamy instead of creamy?", "context": "A dalgona coffee with cold milk is my go-to summer drink. I discovered this golden coffee beauty during the last summer lockdown and since then did not stop drinking it.\nSince I started last year I am following the same recipe:\n\n2 tbsp instant coffee or espresso powder\n1 tbsp sugar\n2 tbsp very hot water\nca. 300ml/13fl oz milk\n\nMixing the ingredients above except the milk with a hand mixer.\n(slightly changed recipe from https://www.bbc.co.uk/food/recipes/dalonga_coffee_20606)\nUsually, my dalgona cream always had a very creamy texture, almost like whipped cream, see below:\n\nBut the last two times, my coffee cream got frothy, like a milk foam instead. I know that reduced sugar can lead to more frothy and less creamy foam, but as said, I did follow the exact same recipe.\nAny idea why the foam is not creamy anymore? And I get a froth instead? Could this be due to mixing it on a higher/quicker level?\nI would like to have my coffee cream back. Thank you!\n", "answer": "\nAny idea why the foam is not creamy anymore? And I get a froth instead? Could this be due to mixing it on a higher/quicker level?\n\nYes, absolutely. Rudimentary speaking the process of whipping something is adhering air bubbles to the fats in your mix (similar to soap bubbles, where the fats act as the soap). The finer these air bubbles are, the finer and more creamy the mixture appears. If you whack it at high speed, it will create large air pockets, which at the next time the whisk comes around at speed either collapse, or create more large air bubbles. Mixing at lower speeds will create smaller bubbles, which at next rotation can subdivide into smaller and smaller bubbles, giving you the smooth consistency you're looking for.\nDon't go too low though, as you'll just be stirring the mixture instead and not incorporating any air. The best way to create the right consistency is doing so on a \"medium\" setting taking roughly 5-6 minutes to whip up your coffee (or longer depending on the amount of mix, more mix takes more time).\n", "topic": "coffee", "url": "https://coffee.stackexchange.com/questions/5720"}, {"image": "VQAonline_00006927.png", "question": "What is this tag/label on Jacobs coffee lid?", "context": "I noticed that the Jacobs coffee containers I buy have such tag/label on their lid:\n\nWhat is the purpose of this tag? According to Google Translate, the first word means \"Coffee\", but can't decipher the meaning of the numbers. 250 isn't the weight or amount of coffee, as it's a 200 gram container.\nThe tag wasn't added by the supermarket from which I buy the coffee.\nI didn't notice any other coffee brand with such tag/label on its lid.\n", "answer": "It seems to be a tax stamp used by the Republic of Belarus. The '\u0420\u0411' on the right side of the sticker stands for Republic of Belarus.\nThe package is rated for content up to 250 grams. In 2016, a Belarusian business news website reported on a duty increase on coffee, it mentions 25 to 250 grams as a category with a fixed duty.\nThis website shows similar stickers for other products (beer). The bottom two rows of numbers for those sample stickers contain zeros and capital As. It seems to me that those numbers and letters are serial numbers used by the Belarusian tax authorities, perhaps to allow trackability (from a tax perspective, not related to the origin of the coffee).\n", "topic": "coffee", "url": "https://coffee.stackexchange.com/questions/5999"}, {"image": "VQAonline_00006969.png", "question": "Is consciousness a sub product of the brain or is there a duality?", "context": "I am trying to understand what consciousness is, based on my basic knowledge of our senses, artificial intelligence (computer vision, specifically) and some philosophy. Here's my reasoning:\nAs far as I understand it from my Neural Network class, each eye captures the light and transmits it through electric signals to the brain. Each action potential can be seen as a binary 1, no activity as a 0.\nThis stimulates some of the neurons in the visual cortex, which allows our brain to interpret and understand what the eyes have captured.\nIn computer vision, this is where \"sight\" ends: the computer detects what's in the image, based on its knowledge base - its training set - and that's it.\nSo why does the human brain take the extra steps of combining both signals into an image - i.e translating the different waves of the electromagnetic spectrum into shapes, colors, etc -, turning it upright and displaying it to ourselves?\nThe same could be said, as far as I know it, about the other senses: hearing would be detecting vibrations in the air, but the brain also translate these into sounds.\nDoes this hint that Descartes' dualistic definition could be true, that our consciousness is separate from the brain? Or could consciousness be a sub product of the brain, a consolidation of what each one of its areas are processing, in order to better understand the world (as in Stanislas Dehaene's workspace neurons cognitive model)?\n\nedit: I've been thinking a bit more about the subject. If there are workspace neurons responsible for consciousness, then there should be a region of the brain that is constantly active, at all times.\nAfter a bit of - online - research, I've stumbled upon the Default mode network (DMN), which is anti-correlated to the Task-positive network (TPN). Interestingly, the DMN is active when the individual is not focused on an activity - at rest. Furthermore, there is a possible increase in connectivity between DMN's nodes during REM sleep - when we dream. On the other hand, TPM is most active during attention-demanding tasks. It has therefore been theorized that both are elements of a single default network with anti-correlated components (in an scientific paper behind a pay-wall, so I'm quoting Wikipedia).\nPlus, both networks involve the fronto-parietal attention network, where it is believed the workspace neurons are located.\nI couldn't help correlating the DMN and TPN to Carl Jung's theories of the Conscious and the Unconscious - which I am particularly fond about. If these wild conjectures I am forming are correct, then we could have the following (oversimplified) neural correlate of consciousness:\n\nIn this idea, exchanges between TPM and DMN occur through neurotransmitters, while exchanges between the conscious and unconscious happen using psychic energy (as in the energy from the psyche, not supernatural powers). Furthermore, the collective unconscious would then be inherited from human to human - embedded in our DNA. If true, then there would be discernible patterns in a newborn's brain...\nSo, going back to A.I, a conscious agent would not, theoretically, be a single agent, but instead a multi-agent collaborative system...\nNow, I have very basic knowledge and lack scientific research on these topics, but I am interested in learning a LOT more. Is this theory absurd and blatantly wrong, or is it worth pursuing as a hypothesis?\n", "answer": "Your question is about the hard problem of consciousness, which is basically the question of how qualia can be explained in a mechanistic way. As alluded to by the name of the problem, it's hard to give a satisfactory answer. The answer right now is: we don't know.\nThere are some theories about how qualia and consciousness could have a neural basis (see this question and this wikipedia article). But none of these theories are widely accepted by cognitive scientists and there is still considerable debate over the issue.\nYour question also illustrates another philosophical issue in the philosophy of mind. You asked:\n\nSo why does the human brain take the extra steps of combining both signals into an image - i.e translating the different waves of the electromagnetic spectrum into shapes, colors, etc -, turning it upright and displaying it to ourselves\n(emphasis mine)\n\nThe idea that conscioussness and qualia reconstruct the world and then we act and perceive on that world creates a problem: how do we act an perceive on that world? Do we reconstruct it? If so, then we are back to the original problem. This is called the homunculus argument, and it illustrates the fact that we haven't explained anything if we posit that the purpose of qualia is to reconstruct what's already out there.\n", "topic": "cogsci", "url": "https://cogsci.stackexchange.com/questions/10072"}, {"image": "VQAonline_00006980.png", "question": "What is the purpose of dead end neurons in this diagram?", "context": "I was reading over some sources about ANNs, and here I found this diagram of a Recurrent Neural Network (RNN, the source calls it a feedback neural network).\n\nI noticed that a few neurons in this diagram lead to other neurons that are dead ends (have no connections to other neurons). For example, neuron U5 leads to no other neurons, and U9 has a connection to U4, but U4 doesn't have a connection to any other neurons.\nIs there a purpose for U5 and U4 to even exist?\nOr am I missing something bigger here?\n", "answer": "As said in the comments, $U_4$ and $U_5$ are not used to compute the values of the output neurons. \nHowever, in some cases one may allow the connections between neurons to change over time, in which case $U_4$ and $U_5$ may be later used to compute the values of the output neurons. Seanny123 gives biology as an example. For another example, you may want to look at the literature on neuro-evolution, i.e. optimization of neural architectures with evolutionary algorithms, which may allow changing the connections between neurons over generations (e.g. that's what I did for my master thesis: Franck Dernoncourt. \"The medial Reticular Formation: a neural substrate for action selection? An evaluation via evolutionary computation.\". Master's Thesis. \u00c9cole Normale\nSup\u00e9rieure Ulm. 2011., though I may have coded the mutation and crossover operators in such a way that dead-end neurons don't appear, I don't recall).\n", "topic": "cogsci", "url": "https://cogsci.stackexchange.com/questions/13683"}, {"image": "VQAonline_00007007.png", "question": "What is a science concept that we can teach preschoolers that's purely visual?", "context": "We are trying to develop a scientific study to test preschoolers (3-5 years old) ability to learn a new science concept. The problem is that children that young can only remember concepts based on how they look but cannot remember any abstract information. So we are trying to find a science concept that can be taught to young children purely visually (feature based), that they can remember and then be tested on. What is a good example of this?\nExample of a purely visual concept. (Differentiating species based on features):\n\n", "answer": "At this age I find it limiting not to use verbal communication in combination with visuals and ACTION. \nIf you can't remove this limitation, please disregard this answer. Otherwise, keep reading it!\n\nIn my experience with kids at this age, they respond positively with:\n\nfun physical activities: ludic games, moving around, challenges using their body (e.g. psychomotricity)\ninstigating stories: using puppets or props, emotions (lightly exaggerated facial expressions, peculiar ton of voice, events happening to characters)\nconcrete projects: arts and crafts, building blocks, toys, music\nmedia: videos, iPad games, etc\nnature: bringing them outdoors (green park, beach, etc), playing with animals or watching them, \nlove: serenity, care and patience from the facilitator, positive stimulation, challenge proportional to the cognitive level, respect for his or her individuality \n\nConsidering that at this age most kids cannot develop abstract thinking, I never expected to test them on the concepts I'm helping them to build (if she doesn't know the answer, it's ok!). I always thought more like I'm facilitating them to form \"building blocks\" internally, like \"planting seeds\", that will grow with them and help them in the future, when their cognitive system is better developed. \nI also follow some educators & philosophers that regard this age as a time to teach moral values, social and emotional skills, rather than science and informational skills [I'll provide some reference later ].\n\nHere are some ideas:\n\nPaleontology concepts, Earth's history, care for species:\n\ntell the story of the extinction of the dinosaurs using puppets and emotions; \nplay a relate video appropriated for the age (maybe scenes from Iced Age?);\ntest (and reinforcement): ask them to draw the characters of the story and talk about what they have learned and how they wanted the story to end\n\nWater cycle / care for environment: \n\n\nplay the video of the water cycle; \npresent a skit with characters showing the consequences for the water system on throwing trash on the floor or using plastic bottles, etc; \ntest (and reinforcement): have them build props using recycled materials and ask them how to protect the environment and what are the consequences of not doing it\n\nHuman body / hygiene: \n\n\nplay a video showing the \"nasties\" (or monsters) inside our mouth, that are flushed away when we brush our teeth; \nplay another video with a catchy song about brushing our teeth (or play the song with them using musical instruments); \nbring a gigantic brush and simulate brushing gigantic teeth made out of toilette paper rolls (or something similar to a tooth); have them playing with the gigantic brush, pretending to brush their teeth; give them challenges\ntest (and reinforcement): have them make the \"mouth monsters\" with playing dough and ask them how to prevent that from happening, using their life examples\n\n\n\nSome educators that inspired this answer: Pestalozzi, Montessori, Comenius, Rousseau, Rivail, Jesus of Nazareth, Socrates\n", "topic": "cogsci", "url": "https://cogsci.stackexchange.com/questions/23466"}, {"image": "VQAonline_00007549.png", "question": "why does my sourdough bread have lumps of flour", "context": "I've made sourdough bread numerous times, however, I sometimes find hard parts in the center of the loaf.\nWhenever I cut into a baked bread (after rest) - there appears to be hard parts towards the centre of the load (seems like lumps of flour).\nI'm wondering if anyone might know what this might be (and should I bake or proof the loaf for longer to eliminate this?)\n\n", "answer": "This looks like an excessive amount of flour worked into the dough when shaping - the spots do absorb humidity from the surrounding dough, but remain dense and hard. \nTo prevent this, be mindful of how much flour you add to your bench and how you shape your loaf. Especially with rather wet doughs, the impulse to flour generously to prevent it from sticking is obviously there. And while you can and probably want to flour what\u2019s going to be the outside of the loaf, you should be careful about not working the floury parts to the inside. There are a few different methods how to solve that, if you search for shaping especially wet dough, you will find various methods that can also be used for medium hydration doughs. For example, Foodgeek has a nice video where you can see the shaping of a boule completely without bench flour and a batard with minimal flouring.\n", "topic": "cooking", "url": "https://cooking.stackexchange.com/questions/107788"}, {"image": "VQAonline_00007122.png", "question": "How to prevent a cookie from becoming spongy like a cake?", "context": "I was following the recipe here: http://allrecipes.com/recipe/best-chocolate-chip-cookies/ \nand the cookies came out a little like cake rather than cookies. It's NOT crispy, not crunchy. It's a little spongy but tastes good actually.\nIs the temperature and timing off? 350F seems a little low. I'm not sure if it's visible from the below picture but still.\n\n", "answer": "They're definitely under cooked. The recipe calls for a 10 minute bake at 350 degrees -- that's seems short. You want to pull them when the edges are darker brown, which probably would have taken another 4-8 minutes.\nIt's hard to tell with the lighting of your photo, but I think your tops weren't browning fast enough either. Before I got a new oven, I'd have that same problem when I baked cookies. I fixed that by starting the cookies on a lower rack (one or two slots from the bottom) and moving them halfway through to a higher rack (one or two slots from the top).\nLetting the cookies cool on the pan also helps set them hold together better.\nEDIT: As for the texture -- you aren't going to get a crunchy cookie from this recipe. This recipe is for a slightly chewy cookie with crisp edges. If your cookie is still cakier than chewy, you may want to make sure you are measuring flour correctly.\n", "topic": "cooking", "url": "https://cooking.stackexchange.com/questions/25248"}, {"image": "VQAonline_00007024.png", "question": "How do I cook toast on an Aga?", "context": "I've an \"Aga toaster\", which is like a couple of disks of 2cm wire mesh joined together with a handle:\n\nBut no matter what I try, most of the time the toast sticks. Any ideas?\n", "answer": "You can try to tossing some Flour on your Aga toaster first, but it will give you extra grill marks on your toast.\n", "topic": "cooking", "url": "https://cooking.stackexchange.com/questions/4635"}, {"image": "VQAonline_00007806.png", "question": "Is there a way to \"revive\" or protect hand sewing needles from corrosion?", "context": "I often find myself hand sewing small seams or buttons, repairing a garment or embroidering something using standard hand sewing needles. After about 8 hours of active use, the needles start to become harder and harder to push through the fabric and they take on a dull, black color. I assume this is corrosion due to the natural sweat and acid on my hands.\nHere's a picture of some of my needles. The ones on the bottom are (mostly) unused, the ones on the top are used for a few hours.\n\nThe problem is not that the tips of the needles are blunt, but the surface seems to become less smooth and has a harder time gliding through the fabric. Sometimes I hear the needle stopping in the fabric for a fraction of a second before going in further. Sewing with these needles gets increasingly frustrating until I realize what's going on and switch to a new one. New needles don't cost a fortune, but I find it frustrating how quick they become unusable.\nIs there a way to restore a used needle to it's former smooth surface? Or is there a way I can treat a brand new needle to prolong the lifetime of it's smooth finish?\n", "answer": "Tl;dr: Your best option may be to buy gold plated or stainless steel needles.\nEveryday needles are nickel plated (says Wikipedia and other sources). Nickel corrodes in contact with sweat, worse for some people than others because some people have saltier sweat. Working in a cool room with freshly (frequently) rinsed hands should help, but this will only slow the corrosion down.\nGold won't corrode, and at a quick look, gold plated needles are only slightly more expensive (mainly because of the existence of cheap brands that don't make gold-plated ones).\nSome stainless steel needles are even marketed as hypoallergenic (nickel-free), and similarly seem to cost only a little more than the normal nickel-plated carbon steel, though it's hard to get a direct comparison.\nBarrier gloves (e.g. latex, nitrile) are likely to be a pain when you need dexterity, but would otherwise be a solution. Only the tips of the finger(s) and thumb(s) that regularly contact the needle need to be covered; finger cots are available or just use the finger cut off disposable gloves.\nIt should be possible to remove the corrosion. I have removed a different type of corrosion from (repurposed) sewing needles using Brasso metal polish, rinsed afterwards with denatured alcohol and dried with paper towel. Steel wool will take it off but will also remove the nickel plating over time. Both of these approaches are perhaps a little tedious anyway, with each needle needing wiping.\nThe most likely dipping process for cleaning would be a jewellery cleaner solution that includes ammonia. You can buy this or make it if you already have household ammonia. It may have adverse effects on the needles but would be worth a try. I might have some but don't hand sew enough to corrode needles so can't test for you.\n", "topic": "crafts", "url": "https://crafts.stackexchange.com/questions/10833"}, {"image": "VQAonline_00007739.png", "question": "What is the name of this painting genre?", "context": "I know the genre is related to photo realism/hyper realism, It's photo realism/hyper realism, But I want to know what's the name of it with this tools!?\nThey use graphite, They use brush and dark colors in it, They use different kind of black powders as color\nHere is an example:\n\n", "answer": "To me, this seems like it is charcoal pencil art. Google that term and check out the Images it brings up. They are very similar.\nThe way this is created is the artist uses a charcoal pencil to draw on the paper, then smudges the charcoal into the picture they want to make. The powder is actually the charcoal. In this case, the light color comes from the paper the eye/face is drawn on.\n", "topic": "crafts", "url": "https://crafts.stackexchange.com/questions/5530"}, {"image": "VQAonline_00007702.png", "question": "Using leather to bind books with substrate", "context": "I have acquired some leather that seems to have been made to be in vehicle interior. I want to use it to bind books with some tooling done to it. This leather seems to be thin compared to the hide that is normally used to bind books. This makes me think that a substrate is needed to allow the leather to be tooled. What sort of material works best? I was thinking of using compressed paper Machee. \nHere is a picture of the leather for reference.\n\n", "answer": "If you are very careful with the tools you can do some shallow tooling. Tools with rounded tips would be best. You might be able to just kind of smooth out the texture of the leather in certain areas to create contrast. Even if you back the leather, deeper lines and sharper tools will cut through. You might be able to get creative with some colored leather backings so that when the tool breaks through the color underneath will show.\n", "topic": "crafts", "url": "https://crafts.stackexchange.com/questions/2865"}, {"image": "VQAonline_00008125.png", "question": "Associativity vs blocks per set in fixed size caches", "context": "I am trying to understand the following claim:\n\nFor a fixed size cache, each increase by a factor of two\n in associativity doubles the number of blocks per set\n (i.e., the number or ways) and halves the number of\n sets \u2013 decreases the size of the index by 1 bit and\n increases the size of the tag by 1 bit.\n\n\nI am just wondering if someone can clear this up for me. For example, if there is a cache with 16 cachelines and it's 2-associative the number of blocks per set is 8, but if we increase by a factor of two associativity then the number of blocks per set decreases to 4.\n", "answer": "Actually he was referring about the blocks that can be placed in the cache from the Main memory.\nfrom your example \n16 cache lines - 2 way set associative | assuming 32 blocks in Main Memory\nso there are 8 blocks per set so each set will have 32/8 = 4 blocks\nnow, made into a 4 way set associative | same 32 blocks of main memory\nso there are 4 blocks per set so each set will have 32/4 = 8 blocks they are doubled so you need 1 more bit to represent it by having same tag but appending it with 1 or 0 at the front\nSimilarly the sets are getting halved, i think you got that part\n", "topic": "cs", "url": "https://cs.stackexchange.com/questions/66906"}, {"image": "VQAonline_00008440.png", "question": "How to get enclosed spaces from a series of connected nodes", "context": "I have a bunch of connected walls in a list and the data for them is like so:\nWall\n{\n Node A;\n Node B;\n}\n\nNode \n{ \n float x; \n float y; \n}\n\nI want to find the rooms from the connected walls as an array of connected points to represent each room's perimeter.\nThis is an example visually of what i am trying to find:\n\nThe red dots are the nodes, and the lines are the walls, the numbers are the identified rooms that the walls created.\nThe walls can be at any angle, not sure if that matters though.\nI am wondering what algorithms exist that can help me solve this problem, what is the best way to approach this?\n", "answer": "One approach is to use a data structure for representing a planar graph. Each node corresponds to a vertex in the graph, and each walls corresponds to an edge in the graph. Then, you are looking for the set of faces in this graph. Standard data structures for representing planar graphs should make it easy to retrieve the set of faces.\nFor instance, one standard data structure is the DCEL data structure. It explicitly contains one record for each face, so once you have converted this to a DCEL data structure, then it is straightforward to iterate over all faces. There are standard algorithms for constructing a DCEL data structure from the set of vertices and edges. Or, instead of a DCEL, it looks like you could alternatively use a quad-edge data structure or a winged-edge data structure. The keyword is to look for data structures for polygon meshes. This has been studied in great detail in the computer graphics and computational geometry fields.\nAlternatively, you could solve your problem directly. For each node, find all of the walls associated with it, sort them by their angle, and store that sorted list associated the node. After doing that for all nodes, then you can iterate through all rooms. Pick an wall, then you can find the room to the \"right\" of that wall by simulating the left-hand rule: stand to the right of that wall, put your left hand on the wall, and walk forward, going in a circle around the perimeter of the room. To simulate that rule, as you walk forward, you'll walk to the endpoint of the current wall; at that node $d$, to find the next wall you proceed to, look in the sorted list of walls incident on $d$, and find the next one in sorted order, then follow that wall. It might be a bit trickier to work out the details of this, than to use an existing implementation of a DCEL data structure.\n", "topic": "cs", "url": "https://cs.stackexchange.com/questions/118306"}, {"image": "VQAonline_00008418.png", "question": "If you have a DFA M, how do you construct a new NFA that accepts L(M) and {e}?", "context": "Suppose you're given a DFA $M$ with a 5 tuple of $(Q, \\Sigma, \\delta, q_0, F)$.\nHow do you construct an NFA that accepts $L(M)$ and $\\{\\varepsilon\\}$, the set containing the empty string? I know that a DFA is an NFA.\nMy thought is that you would simply add a new transition from the start state to the accepting state, which takes epsilon/the empty string.\nFor example (eps. is epsilon/empty string):\n\nSince I've added an empty string transition, the machine is no longer a DFA, but it is an an NFA.\n", "answer": "You've got the right idea, but there's a slight bug.\nIf you add an $\\varepsilon$-transition from the start state to all accepting states, then the new machine recognizes a subtly different language.\nLet $L_k = \\{w_1 \\cdots w_k : \\exists u_1, \\dots, u_{k-1} \\in L(M) \\text{ such that } w_1 u_1 w_2 \\cdots u_{k-1} w_k \\in L(M)\\}$.\nNote that $L_1 = L(M)$.\nThis new machine will recognize the union of the $L_k$, i.e., it recognizes $\\bigcup_{k=1}^{\\infty} L_k$. \nInstead, you can create a new state $q_{\\text{empty}}$, add an $\\varepsilon$-transition from $q_0$ to $q_{\\text{empty}}$, and take the new set of accepting states to be $F \\cup \\{q_{\\text{empty}}\\}$.\n", "topic": "cs", "url": "https://cs.stackexchange.com/questions/115055"}, {"image": "VQAonline_00008653.png", "question": "What is this notation style called?", "context": "I was reading a paper about implementing the statistical programming language R on the JVM, and encountered this notation I haven't seen before (I'm not formally CS-trained). What is the name of notational style? I've seen it before in other papers--usually as it related to functional programming and the REPL.\n\n", "answer": "It is notation for Formal Proofs and specification of language grammar in Backus-Naur Form.\n", "topic": "cstheory", "url": "https://cstheory.stackexchange.com/questions/20124"}, {"image": "VQAonline_00008651.png", "question": "Should the Schedule of ``High-level Operations'' Respect the Linearizability of ``Low-level Operations'' in Proof of Simulation Algorithm?", "context": "Backgroud\nI am reading Chapter 10 ``Fault-Tolerant Simulations of Read/Write Objects'' of the Book Distributed Computing (by Hagit Attiya & Jennifer Welch). Specifically, in section 10.2.3, it simulates a multi-writer multi-reader read/write register from single-writer multi-reader registers.\nSimulation Algorithm\n\nThe idea of the algorithm is to have each writer announce each value it desires to write to all the readers, by writing it in its own (single-writer multi-reader) registers; each reader reads all the values written by the writers and picks the most recent one among them.\n\nThe algorithm uses vector clock to compare the values written by the writers and find the most recent one among them. The pseudocode appears in Figure , where\n\n\n$TS[i]:$ The vector timestamp of writer $p_i$, $0 \\le i \\le m - 1$. It is written by $p_i$ and read by all writers.\n$Val[i]:$ The latest value written by writer $p_i$, $0 \\le i \\le m - 1$, together with the vector timestamp associated with that value. It is written by $p_i$ and read by all readers.\n\n\nCorrectness Proof\nThere are two types of operations in the algorithm:\n\nhigh-level operation: operation on simulated object $R$ (i.e., $read_r(R)$ and $write_w(R,v)$ in the pseudocode).\nlow-level operation: operation on $TS[i]$ and $Val[i]$.\n\n\nTo prove that the simulation is correct we have to show that every admissible execution is linearizable. Consider any admissible execution $\\alpha$. To show that $\\alpha$ is linearizable, we have to show that there is a permutation $\\pi$ of the high-level operations in $\\alpha$ that preserves the order of non-overlapping operations, and in which every read operation returns the value of the latest preceding write.\n\nMy Problem\nAs the last paragraph indicated, we consider the schedule of high-level operations. So my problem is:\n\nShould the schedule of high-level operations also respect the linearizability of low-level operations?\n\nTo illustrate, consider the following admissible (according to the simulation algorithm) execution:\n\nAssume $TS[i][i] = 0, \\forall 0 \\le i \\le m - 1$ initially. $m$ writers write concurrently. They all first execute Line 10 - Line 11 in procedure $NewCTS_w()$ and thus obtain $TS[i][i] = 0, \\forall 0 \\le i \\le m - 1$. Then, they execute Line 12 in some order (for example, in the order of pid). In this scenario, when all writers finish, the system state can be stated as\n $$S = \\left\\{ \\begin{array}{ll}\n\\langle 1, 0 , 0, \\ldots, 0 \\rangle \\\\\n\\langle 0, 1 , 0, \\ldots, 0 \\rangle \\\\\n\\langle 0, 0 , 1, \\ldots, 0 \\rangle \\\\\n\\langle 0, 0 , 0, \\ldots, 1 \\rangle\n\\end{array} \\right.$$ \n\nIn the linearizability proof, the $m$ high-level writes will be put in an order, for example, according to their pids. However, if we consider the low-level operations in such order, the second writer cannot obtain $\\langle 0, 1, 0, \\ldots, 0 \\rangle$. Instead, it should obtain $\\langle 1, 1, 0, \\ldots, 0 \\rangle$, because it is put after the first writer's high-level write operation.\n\nProblem: I am confused about the situation. What is wrong of my argument? Should the schedule respect the linearizability of low-level operations in simulation algorithm? Or, what does it mean to respect it?\n\n", "answer": "I understand the proof as follows.\nStart with an arbitrary execution of the above algorithm. In this execution each operation on $TS[i]$ and $Val[i]$ has already been linearlized by assuming that these are single-writer multi-reader atomic registers. Therefore, by assumption, the schedule linearizes these low level operations.\nThe remainder of the proof shows that there exists some unique assignment of distinct times for each high-level operation such that the following are true.\n\nFor each such operation that starts at time $t_s$ and ends at time\n$t_e$, the assigned time $t_a$ is in the interval $[t_s,t_e]$. \nThe sequence of high-level reads and writes, as determined by this assignment of times, satisfies the linearlizability.\n\n", "topic": "cstheory", "url": "https://cstheory.stackexchange.com/questions/19223"}, {"image": "VQAonline_00008624.png", "question": "Reader, Writer monads", "context": "Let $C$ be a CCC. Let $(\\times)$ be a product bifunctor on $C$. As Cat is CCC, we can curry $(\\times)$:\n$curry (\\times) : C \\rightarrow(C \\Rightarrow C)$\n$curry (\\times) A = \\lambda B. A \\times B$\nFunctor category $C \\Rightarrow C$ has usual monoidal structure. A monoid in $C \\Rightarrow C$ is a monad in $C$. We consider finite products as monoidal structure on $C$.\n$curry (\\times) 1 \\cong id$\n$\\forall A\\ B. curry (\\times) (A\\times B) \\cong (curry (\\times) A) \\circ (curry (\\times) B)$\nTherefore $(curry (\\times))$ preserves monoidal structure, so it transports a monoid to a monad and a comonoid to a comonad. Namely, it transports an arbitrary monoid $w$ to $(Writer\\ w)$ monad (look at the definition \u2014 $w$ must be a monoid). Similarly it transports the diagonal comonoid to the Coreader comonad.\nNow, for concreteness, I unfold the construction of Writer.\nBegin. Actually $Writer = Coreader = curry(\\times)$, they simply have distinct names in Haskell. We have a Haskell monoid $\\langle w, mappend, mempty\\rangle$:\n$mappend : w\\times w \\to w$\n$mempty : 1 \\to w$\nWriter is a functor, so it must map also morphisms, such as $mappend$ and $mempty$. I write this as below, though it is invalid in Haskell:\n$Writer\\ mappend : Writer(w\\times w) \\to Writer\\ w$\n$Writer\\ mappend$ is a natural transformation, a morphism in $C\\Rightarrow C$. By properties of $curry(\\times)$ it is a function, which takes $a \\in Ob(C)$ and gives a morphism in $C$:\n$Writer\\ mappend\\ a = mappend\\times(id(a)) : Writer (w\\times w) a \\to Writer\\ w\\ a$\nInformally, $Writer\\ mappend\\ a$ sums components of type $w$ and pumps $a$ intact. This is exactly the definition of Writer in Haskell. One obstacle is that for the monad $\\langle Writer\\ w,\\mu,\\eta\\rangle$ we need\n$\\mu : Writer\\ w \\circ Writer\\ w \\to Writer\\ w$\ni.e. incompatibility of types. But these functors are isomorphic: $Writer (w\\times w) = \\lambda a. (w\\times w)\\times a$ by the usual associator for finite products which is a natural isomorphism $\\cong \\lambda a. w\\times(w\\times a) = Writer\\ w \\circ Writer\\ w$. Then we define $\\mu$ via $Writer\\ mappend$. I omit a construction of $\\eta$ via $mempty$.\nWriter, being a functor, preserves commutative diagrams, i.e. preserves monoid equalities, so we have for granted proved equalities for $\\langle Writer\\ w,\\mu,\\eta\\rangle$ = a monoid in $(C\\Rightarrow C)$ = a monad in $C$. End.\nWhat about Reader and Cowriter? Reader is adjoint to Coreader, as explained in the definition of Coreader, see link above. Similarly, Cowriter is adjoint to Writer. I did not find a definition of Cowriter, so I made it up by analogy shown in the table:\n\n{- base, Hackage.category-extras -}\nimport Control.Comonad\nimport Data.Monoid\ndata Cowriter w a = Cowriter (w -> a)\ninstance Functor (Cowriter w) where\n fmap f (Cowriter g) = Cowriter (f . g)\ninstance Monoid w => Copointed (Cowriter w) where\n extract (Cowriter g) = g mempty\ninstance Monoid w => Comonad (Cowriter w) where\n duplicate (Cowriter g) = Cowriter\n (\\w' -> Cowriter (\\w -> g (w `mappend` w')))\n\nBelow are the simplified definitions of those (co)monads. fr_ob F denotes a mapping of a functor F on objects, fr_mor F denotes a mapping of a functor F on morphisms. There is a monoid object $\\langle w,\\hat{+},\\hat{0}\\rangle$ in $C$.\n\nWriter\n\n$fr\\_ob (Writer\\ w) a = a\\times w$\n$fr\\_mor (Writer\\ w) f = \\lambda \\langle a_0,w_2\\rangle. \\langle a_0,f\\ w_2\\rangle$\n$\\eta a = \\lambda a_0. \\langle a_0, \\hat{0} \\rangle$\n$\\mu a = \\lambda \\langle \\langle a_0,w_1\\rangle,w_0\\rangle. \\langle a_0, w_0\\hat{+}w_1\\rangle$\n\n\nReader\n\n$fr\\_ob (Reader\\ r) a = r\\to a$\n$fr\\_mor (Reader\\ r) f = \\lambda g\\ r_0. f (g\\ r_0)$\n$\\eta a = \\lambda a_0\\ r_0. a_0$\n$\\mu a = \\lambda f\\ r_0. f\\ r_0\\ r_0$\n\n\nCoreader\n\n$fr\\_ob (Coreader\\ r) a = r\\times a$\n$fr\\_mor (Coreader\\ r) f = \\lambda \\langle r_0,a_0\\rangle. \\langle f\\ r_0,a_0\\rangle$\n$\\eta a = \\lambda \\langle r_0,a_0\\rangle. a_0$\n$\\mu a = \\lambda \\langle r_0,a_0\\rangle. \\langle r_0,\\langle r_0,a_0\\rangle\\rangle$\n\n\nCowriter\n\n$fr\\_ob (Cowriter\\ w) a = w\\to a$\n$fr\\_mor (Cowriter\\ w) f = \\lambda g\\ r_0. f (g\\ r_0)$\n$\\eta a = \\lambda f. f\\ \\hat{0}$\n$\\mu a = \\lambda f\\ w_1 w_0. f (w_0\\hat{+}w_1)$\n\n\n\nThe question is that the adjunction in $C$ relates functors, not monads. I do not see how the adjunction implies \"Coreader is a comonad\" $\\to$ \"Reader is a monad\" and \"Writer is a monad\" $\\to$ \"Cowriter is a comonad\".\nRemark. I am struggling to provide more context. It requires some work. Especially, if you require categorical purity and those (co)monads were introduced for programmers. Keep nagging! ;)\n", "answer": "Yes, if a monad $M : C \\to C$ has a right adjoint $N$, then $N$ automatically inherits a comonad structure.\nThe general category-theoretic setting to understand this is as follows. Let $C$ and $D$ be two categories. Write $\\mathrm{Fun}(C, D)$ for the categeory of functors from $C$ to $D$; Its objects are functors and its morphisms natural transformations. Write $\\mathrm{Fun}^L(C, D)$ for the full subcategory of $\\mathrm{Fun}(C, D)$ on the functors which have right adjoints (in other words, we consider functors $C \\to D$ with right adjoints and arbitrary natural transformations between them). Write $F^R : D \\to C$ for the right adjoint of a functor $F : C \\to D$. Then $-^R : \\mathrm{Fun}^L(C, D) \\to \\mathrm{Fun}(D, C)$ is a contravariant functor: if $\\alpha : F \\to G$ is a natural transformation then there is an induced natural transformation $\\alpha^R : G^R \\to F^R$.\nIf $C = D$, then $\\mathrm{Fun}(C, C)$ has a monoidal structure given by composition and so does $\\mathrm{Fun}^L(C, D)$, because the composition of left adjoints is a left adjoint. Specifically, $(FG)^R = G^R F^R$, so $-^R$ is an antimonoidal contravariant functor. If you apply $-^R$ to the structural natural transformations which equip a functor $M$ with the structure of a monad, what you get out is a comonad.\n", "topic": "cstheory", "url": "https://cstheory.stackexchange.com/questions/2101"}, {"image": "VQAonline_00011322.png", "question": "Treatment for wood under a Trex Deck", "context": "Just moved into a 5 y/o house with a \"maintainence free\" Trex deck. Underneath, it's supported by lumber as shown below\n\nWhen it rains the water drips through the deck slats and leaves this green moss on the lumber. I'm wondering a few things:\n\nDoes this lumber require any maintence? Ie do I have to weather seal this every so often?\nDoes the green moss cause the wood to deteriorate faster?\nCan this type of wood be pressure washed? If so, does it need treatment after?\n\nThanks\n", "answer": "That framing will last decades with no maintenance. I just replaced the decking on my mother's 20-year-old pool deck which had not been well-maintained through the years. The framing was all in good shape though the decking was rotted and warped.\nWith that said, if you want to wash it for cosmetic reasons, it won't hurt to do that.\n", "topic": "diy", "url": "https://diy.stackexchange.com/questions/204825"}, {"image": "VQAonline_00009053.png", "question": "What is the pointed end of a spud or construction adjustable wrench used for?", "context": "I saw one of these at Lowe's the other day and was just wondering what the pointed end is used for. I have seen it called a \"construction\" wrench and a \"spud\" wrench.\n\n", "answer": "Same thing you use a Marlin Spike for. (Tapered steel rod with a mushroom head on one end to strike with a mallet or dead blow) It's an alignment tool for drilled holes in metal building frames.\nDuring assembly, the bolt holes don't necessarily line up, so you stick the tapered tail through the mating holes in the construction item, lever them around till the adjacent holes line up, slap a bolt through, affix a nut and washer and then use the wrench end to tighten.\nNote: Landlubber term is Drift Pin or Bull Pin. Alignment Punch would be the technical name any machinist or mechanic would recognize. When an ocean going tug captain decides that it's more lucrative to run his boat up a slough, pull the engine and use it to power a sawmill operation, he's already been using a tapered steel rod to splice hawser cable. And when he needs to line up pieces of equipment on shore, he repurposes the Marlin Spike for more or less what he's been doing with it all along, using it as a multipurpose prybar, alignment tool and general persuader of anything that isn't quite in the right place. And the name he knew it as followed him into the big mill where he became head millwright.\n", "topic": "diy", "url": "https://diy.stackexchange.com/questions/18795"}, {"image": "VQAonline_00009361.png", "question": "What tool is a solid metal cube with dimples or half spheres on every side?", "context": "My neighbor found this metal tool in a box from his father\u2019s workshop. \n\nIt's a cube with all sorts and sizes of circles and half spheres on every edge.\nWhat is it?\n", "answer": "Die for creating sheetmetal hemispheres. There should be a set of punches with round ball ends somewhere in that mix, unless someone misused them for something else or lost them.\nDoming Block and Punch Set is one term for the tool set. The die is also called a dapping block. You can find them at jewelry tool suppliers.\n", "topic": "diy", "url": "https://diy.stackexchange.com/questions/46399"}, {"image": "VQAonline_00012223.png", "question": "How much does the weight of urban structures (buildings) affect the compaction (permeability, porosity, density) of alluvial sediments below a city?", "context": "I live in Van, Turkey. Van City is situated on an alluvial plain beside Lake Van. General geological structure of the area can be seen on pages 43-44 in this conference book. One of the images is below.\n\nAccording to the article in the link, sediment layers has a slope of 15-30 degrees toward the lake.\nWith the construction of every building, the pressure over the sediments increases. Obviously, this increase is greater in downtown area which has a lot of apartment blocks (mainly 5 to 7 story high reinforced concrete buildings). Downtown area is roughly in the middle of the plain.\nI want to know that how much does the weight of urban structures (buildings) affect the compaction (permeability/porosity/density?) of alluvial sediments below a city and how does this affect the flow of underground water?\n", "answer": "This question might be more appropriate for the SE Engineering site.\nThe thing about soils is that they vary from place to place. Some are sandy, some contain more clay than others and the thicknesses of layers is also variable. All this influences how different soils react to surface loading stresses, such as from building foundations/footings.\nImmediately below the footings the soils will experience the greatest stress increases. As discussed in this document, particularly from page 10 onwards, the magnitude of stress the soil experiences from footings decreases with depth - see the effect of a point load $Q$, applied to the surface, on the upper right of page 12. At a depth $z$, the vertical stress $\\sigma_v$ has a certain higher value. At a depth of $2z$, the value of $\\sigma_v$ is lower.\nOne reason why the affect of surface loads decreases with depth is due to the nature of soil particles and how they lie in relation to each other. Soil particles are not uniform in size or shape, so they rarely lie directly on top of each other. There are gaps between soil particles, called pores. The pores can hold ground water.\nCompaction and consolidation of soils will reduce the volume (size) of pore spaces. Foundation loadings will only affect the soil beneath them.\nBecause soil particles do not lie directly on top of each other, one particle may lie above two or more particles, the stress exprienced by the upper soil particle will be transferred to the other particles. Providing the soil is deep enough, the affect of this is the to, over depth, transfer vertical stresses to horizontal stresses.\nUltimately, if a soil profile is deep enough, the stress the lower reaches of the soil experience due to urban development will be small and will not affect ground water movement.\n", "topic": "earthscience", "url": "https://earthscience.stackexchange.com/questions/18625"}, {"image": "VQAonline_00012153.png", "question": "MODIS Surface Reflectance Data State QA Aerosol Quantity field meaning is unclear", "context": "I am trying to compare NDVI values computed from MODIS Surface Reflectance(MOD09) with the NDVI product (MOD13).\nMODIS Surface reflectance Collection 5 products include a Data State QA field that I am using to estimate which pixels are best suited for computing NDVI.\nA part of this field is labeled Aerosol quantity. The values in this field seem to significantly affect the result of NDVI computation.\nThe explanation of the possible values of this field in the MOD09 Users' Guide seems not entirely clear to me.\n | 00 - climatology\naerosol | 01 - low\nquantity | 10 - average\n | 11 - high\n\nThe last three values are clear, while the meaning of the climatology value is somewhat non-intuitive.\nI have selected a crop field and calculated NDVI for all pixels clear of clouds or cloud shadows during one vegetation period and observed the following:\n\nIt seems that results with low aerosol quantity (green points) match well with the results obtained with the complex rating algorithm used in the standard NDVI MOD13Q1 product. While the data with climatology state (orange points) seems far off.\nWhat does the climatology attribute mean and how to treat it?\n", "answer": "Most likely, climatology means there was no retrieval at all.\nBayesian retrievals combine information from an a priori with information from measurements. When the retrieval fails for whatever reason, or the measurement contains insufficient information, instead of reporting no measurement at all, they copy over information from the a priori and use the flag to indicate that the reported \"measurement\" is in fact simply the climatology.\nFor details, see optimal estimation, in particular the book by Rodgers:\nClive D. Rodgers (2000). Inverse Methods for Atmospheric Sounding: Theory and Practice. World Scientific.\n", "topic": "earthscience", "url": "https://earthscience.stackexchange.com/questions/10046"}, {"image": "VQAonline_00012139.png", "question": "Digitalizing precipitation data from the image of weather map", "context": "I have a set of very old precipitation anomaly maps, from 1995-2006 for Brazil. The data of these maps, is to the best of my knowlegde, lost.\nI would like to have them digitalized, ie, to NetCDF or another similar gridded format. Is there any tools recommendend for this job?\nThe figure below is a recent plot, similar to my files.\n\n", "answer": "You can convert the image to NetCDF data using gdal_translate. The command line looks something like:\ngdal_translate -ot Int16 -of netCDF jpeg_filename nc_filename\n\nYou can use ncl example: click here\n", "topic": "earthscience", "url": "https://earthscience.stackexchange.com/questions/8818"}, {"image": "VQAonline_00012463.png", "question": "Why the definition of productive economy in Leontief Open Model is such?", "context": "Note: as an inter-disciplinary question, it has its twin on Mathematics Exchange: https://math.stackexchange.com/questions/4193896/why-the-definition-of-productive-economy-in-leontief-open-model-is-such . There you can find some potentially useful insights.\nThe Leontief Open (Production) Model is a simplified economic model for an economy in which consumption equals production, or input equals output.\nIn this model, the entries in the consumption matrix represent non-negative monetary values (in some arbitrary monetary unit) of various types of outputs (such as commodities or services). Every $(i,j)$th entry in the consumption matrix is the monetary value of the output of $i$th industry needed by the $j$th industry to produce one monetary unit of its own output. It means that arbitrary consumption matrix, commonly denoted $C$, can be represented in the form of an input-output table:\n\nFor example, in order to obtain a unit of output from industry 3 it takes $c_{13}$ units of output from industry 1, as well as $c_{23}$ of output from industry 2, and $c_{33}$ of output from industry 3 (i.e. itself).\nThe open sector, in this context, refers to the external consumer whose demands are required to be satisfied and its demands are represented in the model by special kind of column vector where each $i$th entry corresponds to the non-negative monetary value of the output required by the external consumer from the $i$th industry. Such column vectors are called demand vectors and such individual vectors are often denoted $D$ (capital \"d\") or $\\textbf{d}$ (boldface lower case \"d\").\nTotal production associated with a particular consumption matrix $C$ and some demand vector $D$ in this model is represented by a column vector where every $i$th column is a non-negative monetary value of the output of $i$th industry required to be produced to satisfy the needs of the external consumer. Such column vectors are called production vectors and are commonly denoted $X$ or $\\textbf{x}$.\nThese triplets of associated matrices (production vector $X$, consumption matrix $C$, and demand vector $D$), based on the assumption that production equals to the sum of internal and external demand/consumption, are related by the matrix equation\n\\begin{equation}\n X = CX + D\n\\end{equation}\nwhere $CX$ is the internal consumption.\nIn this model, an economy is productive by definition if for its consumption matrix $C$ exists Leontief inverse $(I-C)^{-1}$ and $(I-C)^{-1} \\geq 0$.\nHere comes the weird part.\n\nA consumption matrix C is said to be productive if $(I \u2212 C)^{\u22121}$\nexists and $(I \u2212 C)^{\u22121}$ \u2265 0\n\nThe matrix $(I-C)^{-1}$ even has its own name, Leontief inverse, yet it is still unclear to me why it is even needed to exist.\nAccording to Derrick Chung,\n\nAn economy is productive if it can meet any external demand.\nIn other words, for any\nD, there is an X such that X \u2212 CX = D with X \u2265 0 (i.e. X contains only non-negative entries)\n\nand right below there is an analogous definition with Leontief inverse. Why can't an economy be considered productive if $X(I \u2212 C) = D$ (without requiring matrix $(I-C)$ to be invertible)? Or, equivalently, why should the relation between $X$ and $D$ be bijective? Or does it have something to do with the eigen-values?\n", "answer": "This is a specific terminology employed in the literature on Leontief model. Productive here means that all sectors must be profitable (do not confuse it with notions of productivity used elsewhere in the economics, where productivity refers generally to outputs over inputs).\nProfitability requires that:\n$$X-CX>0$$\nActually, in order to arrive at sensible solution the Leontief matrix must be invertible with non-negative inverse (see Zalai 1983), since you are trying to solve for $X$ which should be positive (negative production does not make sense, how do you produce negative number of PCs?):\n$$X(I\u2212C)=D \\implies X = (I-C)^{-1}D$$\nAs far as I understand unless all sectors are profitable its not possible to get non-negative inverse of $I-C$.\n", "topic": "economics", "url": "https://economics.stackexchange.com/questions/45802"}, {"image": "VQAonline_00012397.png", "question": "Providing subsidies affects Marginal Private cost but not Marginal Social Cost?", "context": "According to the theory that I'm seeing on the internet and in textbooks, when a government wants to correct underproduction due to a positive externality of production, they can decide to subsidize the good.\n\nThe subsidy would reduce the costs of production for the firm, and hence shift their Marginal Private Cost to the right towards the Marginal Social Cost curve. In an ideal situation, the curve could be made to actually lie on the MSC curve, and the underproduction would be completely solved.\nThe problem I have with this is that nobody talks about why it's possible for the MPC to ever get closer to the MSC curve. The formula for Social Cost is Private cost + external cost (which is negative in the case of positive externalites of production). So if I were to reduce the private cost of the good, then it's social cost would also reduce. In other words, if I move the MPC curve forward, the MSC curve should also move forward by the same amount.\nDoesn't that mean the only way to get the MPC curve to actually get closer to the MSC curve is to reduce the positive externality itself (which obviously isn't done)? Or is it that people already know that the MSC curve will move, so when they're talking about the MPC curve touching the MSC curve, they're talking about the old MSC curve and not the new one?\n", "answer": "The social cost of a product is the value of the resources that are consumed for its production, irrespective of property rights (and a \"negative externality\" is nothing else than resources consumed, e.g. a contaminated river due to pollution from production reduces the \"natural resources\" of society etc).\nBy subzidizing the product you do not affect the amount of resources that are consumed in order to produce it. What you do is to perform a re-allocation of resources from the public purse to the private producer to increase output. So social cost remains unchanged. \n", "topic": "economics", "url": "https://economics.stackexchange.com/questions/29111"}, {"image": "VQAonline_00012355.png", "question": "Which of Arrow's four desirable properties' is violated in this scenario?", "context": "So the scenario is as follows: there are 3 agents and 4 alternatives, $a,b,c,$ and $d$. Society's ranking of the 4 alternatives is such that \n\nthe highest-ranked alternative is agent 1's highest-ranked\n alternative,\nThe alternative that agent 2 ranks highest among the three remaining alternatives is ranked second highest by society \nThe alternative that agent 3 ranks highest among the two remaining alternatives is ranked third highest by society. \nSociety ranks the 4th alternative lowest.\n\nSo my interpretation of the ranking would look something like this:\n\nWhich of Arrow's four desirable properties is violated?\nI'm guessing that the \"Unanimity\" preference would be violated, because agent 2 prefers $b$ over $c$, but this is not reflected in society's ranking?\nThe four desired properties I am referring to are:\n1/ Complete and transitive preferences\n2/ Respect unanimity\n3/ Non-dictatorial outcomes\n4/ Independence of irrelevant alternatives\n", "answer": "Note that an alternative being highest-ranked by an agent involves comparisons with all other alternatives, which raises doubts that your method satisfies independence of irrelevant alternatives. Indeed, it does not. Independence off irrelevant alternatives is actually the only property that is violated.\nHere are two profiles, only the preferences of the first two agents matter in them, so I will not specify the preferences of the other agents. I only assume the other agents preferences are the same in both profiles.\nAgent 1, Profile $1$: $a\\succ b\\succ c\\succ d$\nAgent 2, Profile $1$: $b\\succ a\\succ c\\succ d$\nIn Profile 1, the social ranking of $a$ and $b$ is $a\\succ b$.\nAgent 1, Profile $2$: $c\\succ a\\succ b\\succ d$\nAgent 2, Profile $2$: $b\\succ a\\succ c\\succ d$\nIn Profile 2, the social ranking of $a$ and $b$ is $b\\succ a$. \nSince the relative ranking of $a$ and $b$ is the same in both profiles, independence of irrelevant alternatives is violated. \n", "topic": "economics", "url": "https://economics.stackexchange.com/questions/21227"}, {"image": "VQAonline_00012503.png", "question": "Villages and hamlets in US native parlance", "context": "How common or uncommon are the words village and hamlet in US native parlance? In the small discussion following this comment, one user said that the word village is not a word we use here, but another one said There are villages in the US! I live in one, and it is inside a town.. There is a Wikipedia article Village (United States) that discusses some of the formal issues.\nI've always taken village as a translation of the Dutch word dorp, a concentration of human settlement ranging from a few dozen people to a few thousand, in a rural area, not connected to any town or city. Here is a nice example with the village of Rougon in the French Alps (photo from Wikimedia Commons):\n\nWhat would such a settlement be called in the US? Do such settlements even exist in North America? Can a place with between 10 and 1000 inhabitants be called a (small) town rather than a village? From this Wikipedia article comes the quote Eighty-nine percent of the cities in Nebraska have fewer than 3,000 people.. Are those really cities?\nP.S. I'm pretty sure the word village in the UK and Ireland matches the meaning that I'm thinking of pretty much.\n", "answer": "The term 'village' is common in some parts of the US, and rare or marked in others. In some States, 'village' is a formal designation for a settlement smaller than a city, for a neighborhood within a city, or for a historical area. In other States village is not formally used, and is simply an informal or quaint term for a small town. So if you ask native speakers from the US about 'village', their answers might vary depending on where they are from, and they might not be aware of different usages in different parts of the country. It's a big place; although the shared language can make it seem smaller there are still distinct regional vocabularies. \nThe term 'hamlet' is rare throughout the US, except in reference to the Prince of Denmark. There are probably a few places that use that name, but it would carry a markedly antique connotation. \n", "topic": "ell", "url": "https://ell.stackexchange.com/questions/502"}, {"image": "VQAonline_00013021.png", "question": "What do you call these large signs hung from the ceilings?", "context": "\nWhat do you call these large signs hung from the ceilings? We often find these at the airport, sometimes, instead of one large sign, we have 3 large ones next to each other with the directions to various places. Is there a specific word for these, I would be very much surprised if they were just called signs.\n", "answer": "We normally call them signs, though you can also say airport terminal signs or direction/directional signs at the terminal.\n", "topic": "ell", "url": "https://ell.stackexchange.com/questions/218809"}, {"image": "VQAonline_00012769.png", "question": "Who is/are they?", "context": "\nA:I assume you know about the latest goings-on with Hessington oil.\nB:I wouldn't be much of a partner if I didn't.\nA:Now they've decided to take on Ava Hessington personally.\nB:Who is they?\nA:The U.S. government.\n\nThese are lines cited from Suits.\n\nWhy did this B say who is they instead of who are they?\n", "answer": "Use-mention distinction\nThis is an example of the use-mention distinction. In the sentence \"Who is they [sic].\", \"they\" isn't being used as a pronoun, but instead is being used a word. That is, it's referring back to the use of the word \"they\" in the previous sentence (\"Now they've decided to take on Ava Hessington personally.\")\nNotice in that sentence by A the \"they\" takes the plural form of the verb (\"they are\"/\"they're\"). This is always the case with \"they\" when it's used as a pronoun; it's always gramatically plural, even when it's used to refer to a single person. (Which, despite moaning by some pedants, is a perfectly valid English construct.)\nIn the sentence by B (\"Who is they?\"), the \"they\" isn't being used as a pronoun, but is being used to refer to the word \"they\" that was used by A in the previous sentence. The convention in English is that when you have this sort of \"mention\" case - where you use a word as a word as opposed to using it for its meaning - you should place that word in quotes or italics. So the closed captioner should really have rendered it as\n\nWho is \"they\"?\n\nor \n\nWho is they?\n\nThe fact that they didn't is either due to limitations of the closed captioning system (for example an inability of the system they were using to represent quotes or italics), or simply because the captioner wasn't aware it was needed.\nNote that the quotes/italics here aren't being used as \"scare quotes\" or as some sort of marking of incorrect usage, but as a literal quote of the word itself. As such, the speaker wouldn't actually make an air quote gesture, but probably would subtly emphasize the \"they\", either by saying it in a slightly different tone or pausing slightly before saying it.\n", "topic": "ell", "url": "https://ell.stackexchange.com/questions/115453"}, {"image": "VQAonline_00013440.png", "question": "Attention Americans: Do you use the SILENT H in the beginning?", "context": "English is my mother tongue and we often follow the British pronunciations.\nHowever, something caught my attention recently. In my Oxford dictionary, I noticed that so many words that begin with 'wh' have a leading 'h' as part of the phonetics. Here are some of them:\n\nwhat\nwhen\nwheeze\nwheel\nwheat\nwhatever\nwhip\n\nand the list goes on...\nI have heard many US accents but never encountered this. Perhaps I never noticed it.\nThus my question: Do Americans really use these words with a leading h or at least are they supposed to?\nThanks!\nEDIT: I am not using the net as my source here. It's a printed book. So for those who are unsure, here it is:\n\nBTW, the exact title of the dictionary is:\nOxford WordPower\nEnglish-Arabic dictionary\nISBN: 0-19-431485-5\nNEWEST EDIT: Please do not consider this a duplicate of Hwat, hwere, and hwy?\nThat discusses the origins of words and various 'accents'. Yes, the examples are related, but my question is different and simple. I'm more keen on knowing if Americans (today) use the leading h pronunciation (at large)?\nEver since I started my study of languages (my mother tongue included), I've tried to stick to a standard pronunciation from the dictionary. Thus in a way I look at the dictionary's pronunciation as the right way to pronounce words, regardless of what people really do. So now after seeing the leading h pronunciation related to Americans, I'm curious to know if they really do so.\nAnd after the kind of responses and links I've seen on this subject today, I don't even think that an average non-academic American really knows this exists. Perhaps I'm wrong.\n", "answer": "Some do! Some don't. I said these words to myself just now to see what I do, and wouldn't you know it, I do put a teensy tiny \"h\" in front of these. Kind of like \"hWat?\" It's barely discernable, even to me. \nAmazing to learn after all these years that I pronounce some words in a way I would never have imagined. Wow! Or is that hWow?\nI'm originally from Southern California, and probably had an original \"O.C\" accent, but since age 15 I've been all over: Toronto, Canada; Cheltenham, England; Germany for 3 years and Yes I speak it fairly fluently; Washington state, USA, for the past 30 years. My accent is probably so muddled up by now there's no way to tell what I sound like.\n", "topic": "english", "url": "https://english.stackexchange.com/questions/127143"}, {"image": "VQAonline_00013638.png", "question": "Is she speaking proper Cockney (or whatever it is she's imitating)?", "context": "\nAt one point in Witness for the Prosecution, Marlene Dietrich's character is at some pains impersonating a guttersnipe. I've been told that the actress spent quite some time working on her accent for this scene. The acting part is well-nigh impeccable (as usual with Dietrich). It's the accent I'm not so sure about. It does sound like a plausible imitation, but is it a little off? And if so, how? Please enlighten me.\nThis clip contains the scene. Dietrich she starts talking at the 2:27 mark:\nhttps://youtu.be/sjkWibjhpIo&t=2m27s\n", "answer": "She is attempting (very badly) to imitate a Cockney accent. She has it in very short snatches but generally it is obvious to any Londoner (or any British person) that she is a long way away from the real thing. She is clearly coming at it from her Germanic natural tongue. Even so, she proably does a better job than Dick Van Dyke in Mary Poppins.\nFor a dramatic representation of Cockney - try Marni Nixon as Audrey Hepburn's singing voice in My Fair Lady\nWhilst 1000 times more authentic than the OP's example, even she doesn't have it exactly.\n", "topic": "english", "url": "https://english.stackexchange.com/questions/290484"}, {"image": "VQAonline_00013924.png", "question": "What do you call a robe that's parted in the middle and is longer in the back?", "context": "I have a reference picture of what I'm actually talking about, it's the.. coat/robe that the elf is wearing below. It's got like this tear-drop skirt, and I'm not sure whether 'coat' or 'robe' is more appropriate or if there's something better. \n\n", "answer": "As a hobbyist fashion designer and seamstress, there is no specific term for this article to account for the tapered hemline. \nAny way you\u2019d call this item, whether coat, sheath, etc, if you wanted to indicate it has tapered hemline, you\u2019d have to expressly call it out. \n", "topic": "english", "url": "https://english.stackexchange.com/questions/501075"}, {"image": "VQAonline_00014031.png", "question": "Need help in translation for a letter written to my grandfather in esperanto", "context": "I have tried google translate, but i will be greatful if someone can help improve the translation. Thank you :)\n\n\nOni Rorespondas en Esperanto Kairo, 16 Marto 1939.\nSinjoro Sajed Ali Multanano, Kalkuto,\nMia kara amiko, antau unu monato mi ricevis vian satatan leteron kiun\n sekvis=post unu semajno=la teo pakajo. koncerne la kvaliton de la teo\n gi ne estas bona tamesn trinkebla sed ne gravas, mi trinkados gian\n sukos gis kiam gi guetos al mi, kaj poste, mi faros cigaredojn el gi\n por regali miajn konatojn. tio estas la plej ne komprenebla ke la\n komercisto kalkuta kalkulis du anaojn pli por la afranko de la pakajo\n olli efektive afrankis. La kaira posto pagigis per mi 12 piastrojn . I\n Rupio 12 anaoj, nome 9 piastrojn por la dogano kaj 3 piastrojn al la\n posto pro la ne sufica afranko.\nNull mia petas vin sendl al mi alion; hindoj generale diras Ke la\n kolza oleo, sarsoun-ka-tel. Hinde estas tre bona konservilo de haroj\n kontrau la blankigo, mi ankau uzis tiun oleon dum mia restado en\n Hindujo. Mi petas bonvulo tui sendi al mi duon-litron da gi. Gi povas\n est parfumi ta nur estu tre atenta car ci tiun oleon oni ofte falsigas\n per alia malplikara oleo. Ankau bonvolu atente enpaki gin ke dum la\n transporto gi ne estu difektebla. Mi pensas ke stanita ferlada vazo\n estas preferinda ol vitra.\nMi intencas sendi al vi belegan libron tre volumenan I6 sur23\n centrimetroj 500 paga, titolita Jarmiloj Pasas, Tiu-ci li ro estas\n mirinda verkajo en bonega stilo verkita originale en Esperanto. Gi\n estas iom kara sedtre interesa kaj scienca traktanta la universalan\n historion, la evoluon de la unu cela besteto gis la nuna civilizita\n homo, pri religioj, politiko ktp. Mi rekomendas al vi ci tiun libron\n car ci konas vin kaj scias ke vi estas sciencema.\nKoncerne la priskribon de la irano bonvolu iom atendi.\nkun sincera saluto Demetrio Fedorcak Mansurah Poste Restanta Egiputjo\n\nTranslation as per Google Translate\n\nOne is answered in Esperanto Kairo, March 16, 1939\nMr. Sajed Ali Multanano, Kalkuta,\nMy dear friend, one month ago I received your satellite letter that\n followed one week after a week = the tea pakajo. In terms of the\n quality of the tea, it is not good enough to drink but it does not\n matter, I will drink its sugar when it boils to me, and then I will\n make cigarettes of it to regain my acquaintances. This is the most\n understandable that the retailer has calculated two ducks more for the\n garlic olli actually cut off. The Kaira post has paid 12 pies with me.\n I Rupio 12 ducks, 9 piasters for the customs and 3 piasters to the\n post due to the not enough thread.\nNo one asks me to send me an attachment; Hindus generally says that\n the collar oil, sarsoun-ka-tel. Hinde is a very good hair keeper\n against the whitening, I also used this oil while staying in India. I\n beg you to send me a half-liter of goodbye. It can be perfumed but be\n very attentive when this oil is often falsified by other lesser oil.\n Also, be careful to pack it at all times that during transport it may\n not be defective. I think that a stanita forage vessel is preferable\n than vitra.\nI intend to send you a beautiful book very large I6 on23 centimeters\n 500 pounds, titled Millennium Passes, this is a wonderful writing in\n an excellent style originally written in Esperanto. It is a bit dear\n to the world's interesting and scientific interest in the history of\n the world, the evolution of the one target animal by the current\n civilized people, religions, politics, etc. I recommend this book to\n you since you know and know that you are scientifically.\nRegarding the Iranian description, please wait a little.\nwith sincere greeting Demetrio Fedorcak Mansurah Later remaining Egypt\n\n", "answer": "Here is my attempt at a translation:\n\nOne makes correspondence in Esperanto.\nCairo, 16th March 1939.\nMr. Sajed Ali Multanano, Calcutta.\nMy dear friend, a month ago I received your much appreciated letter which was followed \u2013 a week later \u2013 by the tea package. About the quality of the tea, it is not good, however it is drinkable, but it doesn\u2019t matter, I will keep drinking its juice until it is tasty to me, and afterwards, I will make cigarettes out of it to treat my acquaintances. That is the most incomprehensible thing that the [shopkeeper/businessman] in Calcutta calculated two annas more for the postage of the package than he actually stamped. The postal service in Cairo made me pay 12 piastres there [a letter seems to be missing so I\u2019m not sure of the last word]. 1 rupee and 12 annas, in other words 9 piastres for the customs officer and 3 piastres to the post office because of the insufficient postage.\nNow I ask you to send me something else; [Indians/Hindus] generally say that the colza oil, \u201csarsoun-ka -tel\u201d [?] in Hindi, is a very good tool for protecting hair against whitening, I also used that oil during my stay in India. Please kindly send me a half litre of it straight away. It can be perfumed, just be very careful because people often fake this oil with another cheaper oil. Also please pack it carefully so that it won\u2019t be possible to damage it in transport. I think a tin can would be preferable to glass.\nI am intending to send you a very large, beautiful book, 16x23cm, 500 pages, called Millenniums Pass, this book is an amazing work in an excellent style, written originally in Esperanto. It\u2019s a bit expensive but very interesting and scientific, covering universal history, the evolution of the single-celled creature until the current civilised man, religion, politics etc. I recommend this book to you because I know you and know that you are scientifically minded.\nAbout the description of Iran, please wait a while.\nWith sincere greetings,\nDemetrio Fedor\u0109ak\nMansurah [?]\nPost-ly remaining,\nEgypt\n\nThank you for sharing this very interesting letter with us.\n", "topic": "esperanto", "url": "https://esperanto.stackexchange.com/questions/5221"}, {"image": "VQAonline_00014023.png", "question": "What is the origin of the Esperanto green star (\"Verda Stelo\")?", "context": "What is the origin of the Esperanto green star symbol (Verda Stelo)?\n\n", "answer": "The Verda Stelo was created by Zamenhof in 1892 as a symbol for Esperantists.\nWhen asked about this, he said,\n\n\"It seems to me, that my attention was drawn to the color green by Mr. [R. H.] Geoghegan and from that time I began to publish all of my works with green covers . . . Looking at one of my pamphlets that I had entirely by chance printed with a green cover, he pointed out that this was the color of his homeland, Ireland; at that time it came to me, that we could certainly look at that color as a symbol of HOPE. About the five-pointed star, it seems to me, that at first Mr. de Beaufront had it imprinted on his grammar [of Esperanto]. I liked that and I adopted it as a symbol. Afterward by association of ideas, the star appeared with a green color.\"\n\nFrom Wikipedia\nNote that this quote wasn't sourced, but I've seen this explanation often enough that it seems to be quite likely.\n", "topic": "esperanto", "url": "https://esperanto.stackexchange.com/questions/317"}, {"image": "VQAonline_00014028.png", "question": "What's the current status of Esperanto movement in North Korea?", "context": "Is there an active Esperanto movement in North Korea today?\nAccording to Vikipedio, a North Korean Esperanto Association was founded in 1959 and published a dictionary. But there are no more information about it after.\nFrom the same page in Vikipedio, the KFA used to have an Esperanto page as late as 2005, but not anymore it seems.\nIn a book about North Korea, I read that there was an Esperanto movement there:\n\nAll this makes me very interested in knowing:\n\nIs the movement there still active today? How many members do they have?\nDo they attend the UEA? Or have any external relationships?\nHow did their association develop?\n\n", "answer": "The Vikipedia article you linked to appears to be a reposting of an article in Libera Folio. Both versions of the article contain the following line:\n\nLa sola konata ligo inter Esperanto kaj Norda Koreio dum la lastaj jaroj estis Asocio por Amikeco kun Koreujo, KFA, fondita en la jaro 2000. \n\nThe web site in question appears to have been taken down. (The link from the LF article does not work.) Using the Wayback Machine I verified that the page was still working in 2007 and was written in decent, but not perfect, Esperanto. It's difficult to get a sense of who wrote it or what their contact with the outside world was. \nIn 2005 Monato published an Article by Ulrich Lins who had traveled to North Korea. The last line in the article seems to be an answer to your questions:\n\nSed de preska\u016d 40 jaroj mankas \u0109ia informo pri Esperanto-movado en Norda Koreio.\n\nIf someone like Ulrich Lins says there's no information about Esperantists in North Korea, I'm inclined to believe him. Certainly no North Korean Esperantist has been able to travel to a foreign Esperanto meeting in decades.\n", "topic": "esperanto", "url": "https://esperanto.stackexchange.com/questions/4855"}, {"image": "VQAonline_00014043.png", "question": "EEA Family permit application... please help", "context": "\nMy wife is filling our her EEa FP and on the EEA National page there is this question \"what is your permanent residential address and contact details\" my question is who's address do we put there the EEA'S national address or my wife address. Also what should be put for the following question. Thank you \n", "answer": "The page refers elsewhere to \"the EEA national\" in the third person. One is therefore tempted to conclude that the second person (\"you\") refers to the applicant.\nHowever, I used this form a little over a year ago, and I remember that it's truly awful. If I recall correctly, there are places where \"you\" is clearly used (improperly, in my opinion) to refer to the EEA national. It is also difficult to give too much credence to grammar-based arguments regarding an application that asks ungrammatical questions like\n\n\nWhat is your permanent residential address and contact details?\nAre your correspondence address different ... ?\nDoes EEA national currently living in UK?\n\n\nWhen I filled out this form, I resolved several ambiguities by making reference to the paper form: https://www.gov.uk/government/uploads/system/uploads/attachment_data/file/270530/vaf5.pdf.\nThat form asks for contact details for the EEA national, including permanent home address and correspondence address. This supports the conclusion that the page you are asking about seeks that information as well.\nIf there's no other place in the online application that seeks the EEA national's address, then I would conclude that the screen you show implements section 8.3 of the paper application, and you should enter the EEA national's address.\nIn the unlikely event that you want to take time to be a test case, you can fill it out with the applicant's address. Take a screen shot of the page with the applicant's information. Submit that with your application to show that you had reason to believe that you were being asked for the applicant's address.\n", "topic": "expatriates", "url": "https://expatriates.stackexchange.com/questions/8340"}, {"image": "VQAonline_00014054.png", "question": "Do I need to be employed at the moment of AOS interview for my employment-based EB1?", "context": "I just received a notice from USCIS for my employment-based EB1b green card interview, set for mid-April. I already left my previous job, and am currently unemployed, searching for another job. \nWhile all supplements for Bona Fide job offer (I485-J) has been submitted already at the time of filing, do I need to have a job/be employed at the very moment of the interview?\nBased on definitions, my I-140 is already approved, meaning that I am already qualified. Plus I also had an offer at the time of filing.\nMy I-485 received date is October 18, 2017, and the interview will be on the day #183 (right now I'm on day #159). Here is what they asked to bring.\n\n", "answer": "You must either be employed at a similar job and your I-485 has been pending for 180 days, or the company that petitioned you must still be planning to employ you, and you plan to accept, after you immigrate.\nSo to answer your literal question, no, you don't need to be employed, and don't even need to be in the US, but in that case, the employer that petitioned you (the one you left) must still be planning to employ you after you immigrate. I am guessing that the company that petitioned you is no longer offering you a job, so you will need to have a similar job at the time of the interview and your I-485 needs to have been pending for 180 days, and I believe they will probably as you to file a Supplement J confirming this.\n", "topic": "expatriates", "url": "https://expatriates.stackexchange.com/questions/13404"}, {"image": "VQAonline_00014059.png", "question": "what is the place of issue of a passport?", "context": "I am an Italian citizen who lives in the UK.\nwhen applying for an Indian Visa (or any other Visa) they ask for the place of issue of my passport.\nThat is the Italian consulate in London UK.\nwhat is the right answer to this question?\nIs it London Uk or Italy?\n\n", "answer": "Your passport's data page (the page with your picture) displays an entry for \"Autorit\u00e0/Authority/Autorit\u00e9.\" As noted in the comments below, the entry there is likely to display \"Ministro Degli Affari Esteri,\" the Italian Ministry of Foreign Affairs, as the issuing governmental agency. The Passport itself does not say where (physically) the passport was issued.\nIf in this Indian visa application you may only choose \"London (UK)\" or \"Italy\" as the \"place of issuance,\" I'd say the correct response is \"London.\"\n", "topic": "expatriates", "url": "https://expatriates.stackexchange.com/questions/19653"}, {"image": "VQAonline_00014083.png", "question": "Taking protein right before the workout", "context": "The label of one protein supplement suggests to have a shake before the workout and one after. I understand having a couple of protein shakes during the day after the workout, but is there any actual benefit from taking it right before you start? Is this a marketing trick to finish your protein quicker?\nThese are is the ingredients on my current protein:\n\n", "answer": "The theory that many body builders believe is that the body will absorb more of the protein if consumed within one hour of training. Having the shake before will also add to the queue of protein the body can absorb.\nIt is true that the body absorbs protein at different rates depending on the type of protein and whether you eat a regular meal close to when you take the protein. When you train, it does set many things in motion regarding your hormone balances. For instance your muscles are more sensitive to insulin, which having carbs after training helps recovery and minimizes the risk of turning those carbs into fat.\nHowever, the studies in relation to protein timing are fairly inconclusive. As long as you have the protein you need throughout the day, you will be fine.\nPerhaps a better alternative to a protein shake would be a dose of Branched Chain Amino Acids (BCAA) before and/or during training. These don't tax your body as much to digest because they are pretty much broken down already. However, they do help your recovery and energy levels while you lift.\n", "topic": "fitness", "url": "https://fitness.stackexchange.com/questions/4788"}, {"image": "VQAonline_00014150.png", "question": "Can only squat extremely narrow", "context": "I'm having a lot of trouble working on my (high-bar) squat. The only way I'm able to squat to proper depth right now is with a hip wide stance, with toes pointed out very far (45 degree or even more). When I try to squat with a wider stance, I feel pain inside my hip. I'm not able to squat to parallel with a shoulder width stance.\nI have already performed stretches for my hip flexors and hamstrings, but this doesn't feel like it's helping with the problem.\nI tried to do this kneeling frog squat stretch and this also hurts, when spreading the legs more than hip wide. \n \nIs this stretch a good way to improve my hip mobility?\nAre there any other good stretches that I can try?\n", "answer": "Everyone's hip mobility is a little bit different. The only thing that should change between your low and high bar squats is the bar position and torso lean. Unless you are specifically squatting with a different stance as in its programmed in as a squat variation. Mobility work will help you change your stance width but generally a more natural approach of wherever your feet feel most comfortable is usually best in my opinion.\nIf you are hell bent on getting your stance in though I always felt like breathing pause squats helped me with hitting depth when I had a closer stance.\n", "topic": "fitness", "url": "https://fitness.stackexchange.com/questions/33464"}, {"image": "VQAonline_00014090.png", "question": "Squat calf machine v/s free weights Squats: Efficiency and knee safety", "context": "I recently tried to squat using the squat calf machine and I found it very practical and efficient for when you are training alone without a spotter.\n\nI have few questions:\n\nWhat are the main differences between the \"squat calf machine\" and \"free weights squats\"?\nWhich creates less \"probability\" for knee injury while squatting?\n\nOn a side note, does the Inclination of the feet stand in the \"squat calf machine\" serve for sthg? or is it just a matter of shape/design?\n", "answer": "Probability of injury\nThere is a very low probability of knee injury while squatting if you keep your knees tracking parallel to your feet, and descend to a depth that places the top of your thighs slightly lower than parallel to the ground. The same is true for leg presses (and I will assume also for the machine you're asking about).\nTwo references that demonstrate the lack of ACL force during a squat or leg press are listed in this answer.\nDifferences between your machine and a barbell squat\nWith a machine, the path of the weight is defined by the mechanics of the machine. You are not responsible for controlling the path the weight takes, only for pushing the weight along that path. This makes the machine exercise an easier exercise than the barbell lift. It is also less functional. You will need to use your back and abs more when you do the lift with a barbell.\nThe path that the machine defines may not be optimal for your particular limb dimensions. When you lift a barbell, you can move it in a way that is optimal for you. (For a squat, this means keeping the bar directly over your midfoot, you'll be sitting back, with your knees not extending too far past your toes.) When you let the machine define the path, that path may vary from the optimal path for your body.\n", "topic": "fitness", "url": "https://fitness.stackexchange.com/questions/6200"}, {"image": "VQAonline_00014315.png", "question": "What are the cognate patterns for adjectives in French?", "context": "I'm reading \"French for Reading\" by Sandberg and there is a section for cognate patterns for adjectives that I do not understand on page 4:\n\nThere are four items here that I'd like to clarify\n\nSandberg states that -en and -euse in French often correspond to -ous in English. Should this be -eux and -euse in French correspond to -ous in English?\nHe also states that -eux and -elle in French often correspond to -al in English. Should this be -el and -elle in French correspond to -al in English?\nNext, he states that -el and -ive in French usually correspond to -ive in English. Should this be -if and -ive in French usually correspond to -ive in English?\nFinally, he states that -if and -enne in French often correspond\nto -an in English. Should this be -en and -enne in French\noften correspond to -an in English?\n\n", "answer": "All of your corrections are right.\n\n1/ eux, euse (ous) \u2014 vicieux/vicieuse (vicious), pieux/pieuse (pious), pr\u00e9cieux/pr\u00e9cieuse (precious), s\u00e9rieux/s\u00e9rieuse (serious), bulbeux/bulbeuse (bulbous)\n2/ el, elle (al) \u2014 officiel/officielle (official), constitutionnel/constitutionnelle (constitutional), r\u00e9el/r\u00e9elle (real) \n3/ if, ive (ive) \u2014 cons\u00e9cutif/cons\u00e9cutive (consecutive), abrasif/abrasive (abrasive), abusif/abusive (abusive), constructif/constructive (constructive), conclusif/conclusive (conclusive) \n4/ en, enne (an) \u2014 autrichien/autrichienne (Austrian), canadien/canadienne (Canadian), martien/martienne (Martian), di\u00e9t\u00e9ticien/di\u00e9t\u00e9ticienne (dietician), m\u00e9diterran\u00e9en/m\u00e9diterran\u00e9enne (mediterranean), math\u00e9maticien/math\u00e9maticienne (mathematician), parisien/parisienne (Parisian), londonien/londonienne (Londonian) \n\n", "topic": "french", "url": "https://french.stackexchange.com/questions/40748"}, {"image": "VQAonline_00014259.png", "question": "Help with French Rebus", "context": "What is the berry and what does the cow say in this puzzle \n\n\n\"Nous sommes abandonn\u00e9s\"\n\n", "answer": "That holly berry is the houx. The cow mooing is either meugler/beugler/mugir but the typical sound is meuh ! Spoilers ahead.\n\n N' - houx - seau - meuh - A - banc - dos - nez ! (Nous sommes abandonn\u00e9s !)\n\n", "topic": "french", "url": "https://french.stackexchange.com/questions/22467"}, {"image": "VQAonline_00014310.png", "question": "What's this phrase on the wall of a toilet of a French ch\u00e2teau ?", "context": "The image below is a screenshot from Johnny English (2003):\n\nOn the wall I can figure out the first three words that reads NE PAS CRACHER, but the last word is too distorted in the view to be identified.\nWhat is the full phrase on the wall? And what does it mean in this scenario?\n", "answer": "It's written S.V.P., a standard abbreviation for S'il vous pla\u00eet. You will most of the time find it written as SVP.\n", "topic": "french", "url": "https://french.stackexchange.com/questions/37963"}, {"image": "VQAonline_00015486.png", "question": "How to make concentric circle roads?", "context": "I can make a circle road. I use the shift key to make it an equal roundness.\nMy problem is trying to make another road outside or within that circle. It is rarely anywhere near going inside or outside of it. Usually it is trying to make a half overlap circle offset to a side (like the olympic symbol).\nI have tried placing my cursor in various places. I can't seem to find the formula.\nWhat is a procedure for making easily making a donut road?\n\n", "answer": "First, create a circle road using the circle shape.\nAfter that, SimCity will display guidelines in circles above and below the road. Switch to the curved road tool, and then draw the inner circle by following the inner guidelines.\n", "topic": "gaming", "url": "https://gaming.stackexchange.com/questions/107698"}, {"image": "VQAonline_00014734.png", "question": "What do the skill percentages in the description of some skills mean?", "context": "In the tooltip for some skills, just above the detailed statistics, there are one or more icons that look like one of the attributes, and next to them some percentage, as is pointed to by the first question mark in this screenshot:\n\nWhat does that mean? I used to think that it means how much from the current weapon damage is transferred to the skill, but that appears to be answered by the other circled part.\nSome skills even list more than one of those.\n", "answer": "The \"magic icon 50%\" means that the damage output of the spell scales with your magic stat at a rate of 50%. I don't think this means that 50% of your magic stat will be applied to the damage, rather I believe it is there to compare against other skills that may have different percentages here. The base damage of Ember Bolt at level 1 is 10-15, so I believe that the 11-16 in your case indicates an additional point of damage due to your magic stat level.\nYou are correct that the damage also is impacted by the power of your weapon, in this case 10% of the damage your weapon would normally do is applied to the damage you'd do with Ember Bolt.\nNote also that this damage is per bolt, and you fire multiple bolts, which also can go up as you level the skill, I believe.\nOnce upon a time there was a skill calculator that I believe helped you play with the various statistics and determine the output damage you'd end up with, but the site's gone offline. It seems like Runic plays with the various formulae when they patch, so getting accurate, detailed information that is up to date is challenging. There's some info in the last post here, but I have no idea if it's accurate with the current revision of the game. \n", "topic": "gaming", "url": "https://gaming.stackexchange.com/questions/41881"}, {"image": "VQAonline_00016897.png", "question": "Is there a way to decipher Prof. Goopton's language using in-game cues", "context": "Are there hints in the game to decrypt his characters to our alphabet? \nI.e. by our own observation and without (Spoiler):\n\n Without using the Sponge\n\n\n", "answer": "No, there are no in-game cues to help translate what he says. (Except the item you mentioned)\nIf you want to translate it yourself:\n\n He speaks Alienese, a language from futurama\n\n", "topic": "gaming", "url": "https://gaming.stackexchange.com/questions/272487"}, {"image": "VQAonline_00018671.png", "question": "What is this epiphytic cactus with small orange flowers along the stems?", "context": "Does anyone know the species of this epiphytic cactus? Even the genus would be a step forward. Could it be a Disocactus sp.?\nThe open flower is less than 2 cm (just under an inch) in diameter. The colours on the photo look pretty realistic.\n\n", "answer": "I believe this to be Pfeiffera monacantha.\nSynonym(s): Rhipsalis monacantha, Acanthorhipsalis monacantha, Pfeiffera monacantha, Acanthorhipsalis incahuasina, Pfeiffera incahuasina, Acanthorhipsalis monacantha, Lepismium monacanthum.\nIn regards to its taxonomy, the International Plant Names Index (http://www.ipni.org) shows that the generic name Lepismium was changed to Pfeiffera in 1994 (and nothing about it changing back to Lepismium).\nTo fortify this, The New Cactus Lexicon (http://www.newcactuslexicon.org/) reaffirms the genus Pfeiffera with 9 species.\nDetails about its range and and taxonomy can be found here: http://media.eol.org/pages/5181624/hierarchy_entries/57218552/overview\nGeneral culture for epiphytic cacti: http://cactus-epiphytes.eu/z_page_culture.htm and http://www.abbeybrookcacti.com/epi_culture.htm\n", "topic": "gardening", "url": "https://gardening.stackexchange.com/questions/24322"}, {"image": "VQAonline_00018238.png", "question": "Orange spherical fungus in garden", "context": "We have a weird orange spherical fungus growing in our garden, any idea what it is and if we need to do anything special to get rid of it?\n\n", "answer": "This appears to be a slime mold (not a fungus) in the genus Lycogala; most likely either L. epidendron or L.terrestre. It is harmless to your garden, feeding on microorganisms and other detritus.\n", "topic": "gardening", "url": "https://gardening.stackexchange.com/questions/5624"}, {"image": "VQAonline_00019156.png", "question": "What plant is this? Large leaves and pink flowers.", "context": "\nCan anyone identify the plant in the attached image. Thank you in advance. \n", "answer": "This looks like a Rhododendron, an example image here, but flowers are not full open yet. You can recognize a Rhododendron by the shape of the (large and dark green) leaves, which typically form a round pattern under the flower or bud. \n", "topic": "gardening", "url": "https://gardening.stackexchange.com/questions/41417"}, {"image": "VQAonline_00019693.png", "question": "Origin of young German lady's attire with fore-sleeves and head cover in 1878 photo from Hannover?", "context": "I have attached a scan of the cover and photograph of a picture taken in Hannover in 1878. We believe the young lady to be a sister to my Great-Great Grandmother (Catherine Sophia Dorothea Wilkering) who immigrated to the US from Germany (Schaumburg-Lippe?) in about 1868.\nWhile we do not know the young lady's name, my question has to do with her attire. What is the significance of her outfit with the fore-sleeves and the head covering?\n\n", "answer": "Folk costumes - Volkstrachten - are traditional clothing that can be highly specific to a region or even a village. The woman in your photo is wearing parts of the Oesterten-Tracht or Lindhorster_Tracht, which was characteristic of eastern Schaumburg-Lippe.\nIt consists of a characteristic red skirt with colored ribbon trim (\"Want\"), apron, sleeveless vest with applied lace, ruff (H\u00e4llschen), embroidered shawls, and knitted forearm guards (\"Handschen\") as well as the headgear and headscarves. \nOnce commonly worn, there are efforts underway to help preserve this and other regional costumes (Tracht des Jahres 2010).\n", "topic": "genealogy", "url": "https://genealogy.stackexchange.com/questions/9415"}, {"image": "VQAonline_00019828.png", "question": "What is the location on this 16th century baptism record?", "context": "Randle Case was baptised in the parish of St Oswald, Chester, in May 1589. His father was Thomas Case and the record seems to read \"Randle Case, sonne of Thomas Case of ----\", but I can't figure out this last bit.\nAny suggestions? I think it could be Flookersbrook, a part of modern day Hoole Village, but I'm not sure and would like confirmation.\n\n", "answer": "Fluker's Brooke, which is now Flookersbrook in Hoole.\nhttp://chester.shoutwiki.com/wiki/Flookersbrook\n", "topic": "genealogy", "url": "https://genealogy.stackexchange.com/questions/16282"}, {"image": "VQAonline_00019777.png", "question": "Where is (was) the Mormon Post Office in Utah Territory in 1860", "context": "In trying to unravel some family relations in Utah Territory, one of the source materials is the 1860 Federal Census:\n\nPaging through that census there are several Post Offices in Salt Lake County: Union, Great Salt Lake City. \nDoes anyone know where the \"Mormon Post Office\" was located (in relation to maps of today)\n", "answer": "The easy way to find census locations on a map is to consult William Thorndale and William Dollarhide's Map Guide to the U.S. Federal Census, 1790-1920 (Baltimore: Genealogical Publishing Company, 1992). \nBut if you want to try to plot it on the map yourself, you could look for descriptions of the districts.\nThe online guide at the US National Archives, 1790-1890 Federal Population Censuses - Part 1 has information about what maps and ED descriptions can be found at NARA. The section Enumeration District Descriptions and Maps explains:\n\nAn ED refers to the area assigned to a single census-taker. ED descriptions pertinent to the schedules covered by this catalog are in Descriptions of Census Enumeration Districts, 1830-1890 and 1910-1950 (T1224). Table 2, ED Descriptions, 1830-90, in T1224, explains coverage of the 17 rolls pertinent to this catalog. \n\nafter Table 2, they note:\n\nThe title of T1224 contains a misnomer because EDs, strictly defined, were not used until the 1880 census. The early censuses used the term subdivision to refer to part of a supervisor's or marshal's division or district. Subdivisions in the early censuses comprised towns, townships, or other units comparable to MCDs.\n\nMCDs = minor civil division (see Claire Kluskens' article from Prologue, Plans of Division, which describes the 1930 Census)\nUnfortunately, if you look in the microfilm catalog for T1224, the description says it hasn't been digitized yet, and it doesn't seem to be on the list Microfilm Publications and Original Records Digitized by Our Digitization Partners, so you would have to consult the microfilm at the National Archives or at another repository that has a copy of the microfilm. A DP, T1224.PDF is available. \nThe USPS' Publication 119, Sources of Historical Information on Post Offices, Postal Employees, Mail Routes, and Mail Contractors says:\n\nLists, tables, and directories of Post Offices \n are available for nearly half of the years from 1803 to 1870\n\nIf you can find those titles online, you can see if they include a map. For instance, Hathi Trust has a listing for List of post offices in the United States, with the names of postmasters, on the 13th of July 1857. Also the regulations and laws of the Post Office Department. Comp. from the records of the Post Office Department, by D.D.T. Leech. \nPublication 119 says:\n\nSelected editions of the List of Post Offices in the United States,\n Table of Post Offices in the United States, Directory of Post Offices,\n and National Five-Digit ZIP Code and Post Office Directory (titles\n vary slightly) may be available from your local library through\n inter-library loan.\n\nCheck Worldcat.org for libraries near you.\nNARA's guide to P.O. Reports of Site Locations, 1837 - 1950 has a section on How to Locate the Microfilm Roll Number Containing a Site Report for a Particular Post Office. For Utah the rolls are:\n\n\nRoll 594: Utah, Beaver - Garfield Counties\nRoll 595: Utah, Grand - Salt Lake Counties\nRoll 596: Utah, San Juan - Sevier Counties\nRoll 597: Utah, Summit - Weber Counties\n\n\nThese rolls are part of M1126, Post Office Department Reports of Site \nLocations, 1837\u20131950 (683 rolls); the catalog includes them as part of M1126C (200 Rolls) and the catalog says they have not yet been digitized, so like T1224, you'd have to find somewhere to view the microfilm or get it via interlibrary loan. Earlier rolls are available through the National Archives' catalog, so check back periodically to see if more rolls are added. A DP, M1126.pdf, is available for download. \nOther resources:\n\nPrinted Maps of Utah to 1900: An Annotated Cartobibliography by Riley Moore Moffatt (available via BYU ScholarsArchive)\nMaps at the Library of Congress, such as this Rand McNally Indexed map of Utah with a new and original compilation and index. published in `876 which \"Shows relief by hachures, drainage, post offices, stations, counties, cities and towns, roads, and railroads with names.\"\nOld Maps of Utah at Old Maps Online\n\n\nUpdate: I searched for all post offices in Salt Lake County on the USGS's Geographic Names Information System (GNIS) but I do not see a Post Office by that name in the search results (GNIS does include some historical entries). \nYou could try looking for historical topographic maps for the area at the USGS's Site The National Map or by using the USGS Historical Topographic Map Explorer to see if you can find nearby post offices in the county.\nFurther reading and more resources:\n\nMapping the census by Judy G. Russell (posted Nov 25, 2015)\nCensus tools at Stephen P. Morse's One-Step Web Pages\n\n", "topic": "genealogy", "url": "https://genealogy.stackexchange.com/questions/14536"}, {"image": "VQAonline_00019974.png", "question": "CD dust cover auf Deutsch", "context": "Wie nennt man im Deutschen eine Pappabdeckung einer CD?\nAuf Englisch nennt man sie a dust cover. Ich habe gegoogelt, aber ich denke, dass der Staubschutz einer CD was anderes ist.\nWie sagt man: the dust cover has minor scratches?\nKann ich das englisches Wort benutzen? z.B:\n\nDer Dust cover weist ein paar leichte Kratzer auf.\n\nWenn ja, wie soll ich das Wort schreiben?\nDustcover, Dust cover, Dust-cover, Dust-Cover?\n\n", "answer": "Bei B\u00fcchern hei\u00dft das Schuber. Also:\n\nder [CD]-Schuber\n\n", "topic": "german", "url": "https://german.stackexchange.com/questions/38888"}, {"image": "VQAonline_00019946.png", "question": "Translations for Weather-Descriptions", "context": "I am trying to translate some weather descriptions for a weather-application to German. I managed to translate many of them, but I am still stuck with a few. I hope you can help me with it:\n\nragged shower rain\nshower sleet\nlight rain and snow\nrain and snow\nsand\ndust\n\nMy attempts in German:\n\nRegenschauer (but what is ragged in this context)\nSchneeregen?\nLeichter Schneeregen?\nSchnee und Regen?\nSandig?\nStaubig?\n\nDo you have better translations, because I\u2019m not quite happy with them, yet.\n\nAbout the App:\nIt just shows some basic weather information of the current weather and a 7 days forecast.\nFor the current weather I also have a weather description, that my weather provider gives me. For these texts I need translations. See attached screenshot. In this example it would be sunny = (sonnig).\nBut I have some problems with sand or dust for example. What kind of weather-description is sandig or staubig? I have never seen this before \u2026\n\n", "answer": "For standardised weather descriptions, one can always fall back to the METAR format. The German Wikipedia provides an overview including translations for some but not all of your problematic terms.\nRagged showers or thunderstorms seems to indicate on and off according to bits and bobs of information I\u00a0found on the internet. Thus I would propose wiederkehrende Regenschauer for ragged shower rain.\nSleet is Schneeregen and showers are of course Schauer, so shower sleet is Schneeregenschauer.\nDistinguish between Schneeregen\u00a0(sleet) and Schnee und Regen (snow and rain). The former is both in one, the latter is both but not at the same time. Probably, we need something like leichter, in Schnee \u00fcbergehender Regen for light rain and snow.\nSand and dust are visibility issues due to sand or dust. The Wikipedia translations list Sand und verbreitet Staub.\n", "topic": "german", "url": "https://german.stackexchange.com/questions/29548"}, {"image": "VQAonline_00020009.png", "question": "Help deciphering a Kurrent letter", "context": "My abilities in neither German nor Kurrent are very advanced, and i need some help trying to decipher a letter from 1758. I checked on the Wikimedia Commons page for the image of this letter, but sadly found no information about its contents there\u2014let alone a full transcription. I'm interested in this because seeing a transcription should help me to better my Kurrent-reading skills, and also i'm curious for an example of some earlier modern German.I've tried my hand at identifying some of the words, but haven't really got that far!\nThe image is attached and any native help would be much appreciated.\n\n(From https://commons.m.wikimedia.org/wiki/File:Lessing_Kleist-Brief.jpg)\nEdit after answer: I accidentally used the wrong /das/ in one of my notes. On the topic of missing eszetts, i thought one of those big long words said 'Geheimnis', but the double s threw me off, and so i assumed the first letter to be a B, despite its similarity to how i write my Gs, and abandoned trying to work it out. Nice to see this did indeed bring to light some differences to note.\n", "answer": "Transcribed in Latin script it is here:\nhttps://de.wikisource.org/wiki/Gotthold_Ephraim_Lessing_an_Ewald_von_Kleist\n\nLiebster Freund,\nMorgen geht das Bataillon Garde von hier weg; nach Bre\u00dflau, wie man\n sagt das ist die einzige Neuigkeit, die ich Ihnen von hier melden\n kann. Oder wollen Sie noch etwas neues von Gottscheden[1] wi\u00dfen? Er\n wird mit den Gesalbten unsres Gleims[2] immer bekannter; immer\n vertrauter. Er hat wieder franz\u00f6sische Verse gesetzt, nebst einer\n goldnen Tabatiere[3] und einem Ringe. Er macht gar kein Geheimni\u00df\n draus; er ist vielmehr so stolz drauf, da\u00df er die ganze Unterredung,\n die er hier mit dem K\u00f6nige gehabt hat, in sein Neu[e]stes eindrucken\n la\u00dfen. Gott wolle nicht, da\u00df unser Gleim seinen Patriotismum auch so\n weit treibt, da\u00df ihm Gottsched durch diese Bekanntschaft respectabler\n wird! Jetzt ist es vielmehr die rechte Zeit, neue und blutigere\n Satyr[e]n wider ihn zu machen, als man noch je gemacht hat. Und wenn\n wir damit zaudern, so wird er uns selbst zuvorkommen. Denn es ist ganz\n gewi\u00df, da\u00df er wieder eine neue Aesthetik in einer Nu\u00df drucken l\u00e4\u00dft.\n Ihre neuen Gedichte werden ihm gleich noch zur rechten Zeit gekommen\n seyn. Wenn ich doch nur auch unterde\u00dfen etwas geschrieben h\u00e4tte, damit\n ich nicht etwa verge\u00dfen w\u00fcrde!\nLeben Sie wohl theuerster, liebster Freund und kommen Sie ja bald\n wieder. Ich bin Zeit Lebens\nganz der ihrige Le\u00dfing\nLeipzig den 14 M\u00e4rz 1758\n\n", "topic": "german", "url": "https://german.stackexchange.com/questions/52570"}, {"image": "VQAonline_00020896.png", "question": "What is this atom affect called?", "context": "I've seen lots of websites with these types of effects/design in the background. What is it called?\n\nSource: https://www.simpla.io/\n", "answer": "I believe called a Network Graph diagram, and it's associated with a the mathematical graph theory\nhttps://en.wikipedia.org/wiki/Graph_theory\nThat said, I'm not sure if there might be perhaps a more Graphic Design naming for it.\nEdit:\nHere's an example of a javascript library to create one:\nhttp://christophergandrud.github.io/d3Network/\nIt also seems to use the term Network Graph, so that's your best bet.\n", "topic": "graphicdesign", "url": "https://graphicdesign.stackexchange.com/questions/74981"}, {"image": "VQAonline_00020927.png", "question": "Colors in design manual, doesn't fits to real color displayed on monitor", "context": "I have an interesting problem. I opened design manual for one logo in Preview in mac. There was showed the colour of a logo with the numeric representation of that colour.\nBut when I picked the colour with Color picker, I have got a different colour number. See picture. Am I missing something or it is normal?\nThanks for a help.\n\n", "answer": "It is normal and it should not be... but it is.\nEvery Corporate Image Manual should include what color profiles are being used.\nIn RGB mode probably you should stay witn adobe 1998, but try to experiment on the CMYK value depending on where you live, Europe or US.\nOne problem is that in US there are more profiles to choose from. In my opinion the basic one (and I do not mean limited, but as fundamental) is Swop V2. But they probably used some version of Gracol or some version of Fogra.\nThe problem is that yes, there is much discrepancy.\nFor now try to stick with the manual.\n", "topic": "graphicdesign", "url": "https://graphicdesign.stackexchange.com/questions/80741"}, {"image": "VQAonline_00021004.png", "question": "InDesign: Prevent table headers from repeating", "context": "I have a long table that breaks across multiple pages. It has a header which I only want to appear once at the beginning of the table. But InDesign is adding the header at the top of every page when the table breaks onto the next page. How can I prevent the header from appearing on every page?\nI checked out Table Options but, as show below, there is no option to say, \"Never repeat the header\".\n\n", "answer": "Set the Header Rows to 0. \nThen manually adjust the first row of the table to appear like the header you want.\nIn short.. do not use the \"header row\" function for the table. It's entire function is to repeat rows as headers.\n", "topic": "graphicdesign", "url": "https://graphicdesign.stackexchange.com/questions/93417"}, {"image": "VQAonline_00021460.png", "question": "What is this burst transmission sequence across the entire band?", "context": "Fairly regularly on the 40m band I hear a quick blip of a tone, and when I look at the waterfall I see these brief transmissions all across the band that look like little dots. What is causing that?\n\n", "answer": "It's probably an ionosonde \u2014 sending out HF radio signals not to communicate but to measure the reflections and thereby determine the current characteristics of the ionosphere.\nThe signal is probably not actually a series of brief transmissions, but a continuous chirp transmission with continuously increasing frequency. Two reasons:\n\nAs far as I know, this is how ionosondes work, and I've heard many chirps on HF, but never a stepped transmission.\nIf you look closely at the display, it appears that only one pixel in each horizontal line is lit by this transmission. If it were short bursts on single frequencies, we would expect to either see the transmission either taking up multiple pixels in the vertical time axis occasionally, or for some of the transmissions to be missed entirely, rather than a constant-ish spacing along the spectrum.\nIt looks like a series of dots because your waterfall display is displaying a series of momentary snapshots over time, not using the entire period of the signal between one line of the display and the next. (This is possible, but requires more computation. If it did, you'd see a continuous slanted line, but much more faintly than the dots because you'd be looking at the power effectively averaged over a longer time period.)\n\nIf this is a chirp, then if you catch the sound of it in SSB mode, you should hear a quickly increasing or decreasing tone, not a stable tone.\n", "topic": "ham", "url": "https://ham.stackexchange.com/questions/14854"}, {"image": "VQAonline_00021447.png", "question": "RF Antenna - V pole - Should one leg go to ground and the other to signal?", "context": "I'm attempting to follow this white paper in order to build an antenna. This is my first antenna and as such I have a rather simple question.\nDIY 137 MHz APT Weather satellite antenna by Adam-9A4QV\nhttps://www.youtube.com/watch?v=9WNmhfpWxdk\nShould one of these legs go to ground and the other to signal from the coaxial cable? It's unclear to me from the diagram:\n\nWhether or not the terminal block is conductive through the center horizontal plane. IE is the top and bottom legs of the antenna separated and thus one goes to signal and one to ground or are they connected in which case they both are connected to both (which I admit makes no sense)?\n", "answer": "One leg of the antenna goes to the center conductor and the other leg of the antenna goes to the braided shield of the coaxial cable. The terminal block is just a way to make the connections, and it has two electrically separate positions. You can use any means of making the connection you have handy, as long as it is small compared to the wavelength.\n(Vocabulary tip: the junction between the cable (feed line) and the antenna is called the feed point.)\n\nHowever, it's worth noting that this is a poor design. Specifically, it has no balun \u2014 a device for converting between a balanced device (the antenna, which as you can see is symmetric) and an unbalanced one (the coaxial cable, which has an inside and outside conductor that are very different). This means that the shield of the coaxial cable will act as part of the antenna, and the performance will vary depending on how the cable is routed. It may pick up common-mode noise traveling from your receiving equipment along the cable, which can be a consideration when trying to receive weak signals from sources such as satellites.\nYou can read more about what a balun is and how it works in this application in the question Using a balun with a resonant dipole.\n", "topic": "ham", "url": "https://ham.stackexchange.com/questions/12244"}, {"image": "VQAonline_00021420.png", "question": "Detecting signals using statistics", "context": "I have sampled a section of spectrum every few Hz. I have mean, standard deviation, and range data for every frequency sampled (and I can gather more statistics if necessary). I've sampled for several seconds before storing the spectrum statistics.\nI would like to be able to guess where man-made signals (i.e. channels) exist in this spectrum (as opposed to noise floor). This is proving to be a difficult task, especially since the noise-floor power sometimes changes (different antenna inputs for different bands, for example.)\nI doubt anyone has a ready solution for this, but what are some starting points, or tips, to help me begin to identify channels?\nIn the below image 'normal' is really autocorrelation from Pandas\n\n", "answer": "You might try taking a image capture of the spectrum waterfall for some duration, and feeding that image to a machine learning inference engine, perhaps a DNN.\nThe inference engine could be trained on a large image database with lots of waterfalls of lots of known or suspected signal types, similar to these signal ID databases: \nhttps://www.sigidwiki.com/wiki/Signal_Identification_Guide \nhttp://qrznow.com/signal-identification-guide/ \n", "topic": "ham", "url": "https://ham.stackexchange.com/questions/8919"}, {"image": "VQAonline_00021574.png", "question": "Are \"Wrist\" blood pressure monitors less accurate then Upper arm?", "context": "There are thoughts that Wrist blood pressure monitors like the one shown below are less accurate than Upper Arm ones. Also, one of the respectable site mentions same with reasonable arguments. Is that generally acceptable opinion?\n\n", "answer": "Yes, they are less accurate.\nMayo Clinic is generally held to be a credible source and in the link you provided they have this to say about wrist monitors:\n\nSome wrist blood pressure monitors may be accurate if used exactly as\n directed. However, the American Heart Association recommends using a\n home blood pressure monitor that measures blood pressure in your upper\n arm and not using wrist or finger blood pressure monitors.\nWrist blood pressure monitors are extremely sensitive to body\n position. To get an accurate reading when taking your blood pressure\n with a wrist monitor, your arm and wrist must be at heart level. Even\n then, blood pressure measurements taken at the wrist are usually\n higher and less accurate than those taken at your upper arm.\n\nThe American Heart Association has this to say:\n\nThe American Heart Association recommends an automatic, cuff-style,\n bicep (upper-arm) monitor.\nWrist and finger monitors are not recommended because they yield less\n reliable readings.\n\nI think those two sources are more than enough to answer the question, but I'll add this as a third source:\n\nThe use of wrist devices for home self-measurement, therefore, leads\n to frequent detection of falsely elevated blood pressure values likely\n because of a poor memory and rendition of the instructions, leading to\n the wrong position of the wrist.\n\nAnd a fourth:\n\nIn conclusion, radial systolic BP is not representative of brachial\n systolic BP, with most participants having a radial systolic BP >5\n mm\u2009Hg higher than brachial and many with differences >15 mm\u2009Hg.\n\n", "topic": "health", "url": "https://health.stackexchange.com/questions/20286"}, {"image": "VQAonline_00021557.png", "question": "\"Copay with deductible\" vs. \"Copay after deductible\"", "context": "Pertaining to health insurance, what does \"Copay with deductible\" mean, in contrast to \"Copay after deductible\"?\nI get \"Copay after deductible\" -- you must pay for the service fully out of pocket until your deductible is met, after which you must only pay the copay amount and the insurance pays for the rest.\nSo what does \"Copay with deductible\" mean?\n\n", "answer": "I've been looking at their plans too, and wondered the same thing. I called Sendero today at 844-800-4963 and the rep told me that it means the same thing as \"after deductible\". She said once you've reached your deductible, then you only pay the copay.\n", "topic": "health", "url": "https://health.stackexchange.com/questions/14328"}, {"image": "VQAonline_00021533.png", "question": "How to deal with pale fingers in cold weather?", "context": "What is this condition called when one or more fingers go pale every time when being in a cold environment?\n\nIs it something that can be handled without having to relocate to exotic countries? \n", "answer": "The phenomenon you are describing and the one shown in the picture is known as Reynaud's phenomenon. In cold temperatures, the body constricts the peripheral blood vessels to prevent losing heat from the blood to the outside air. In Reynaud's phenomenon, this constriction is exaggerated and may prevent blood flow to the fingers or toes (and sometimes other areas) giving rise to the pale appearance shown in your picture.\nReynaud's phenomenon can be classified as primary or secondary. \nPrimary Reynaud's phenomenon, often simply referred to as Reynaud's disease is where the phenomenon is not associated with any underlying disease, and the cause is ideopathic (unknown). Primary Reynaud's phenomenon is considered to be a benign condition.\nSecondary Reynaud's phenomenon occurs when there is an underlying disease that is causing the condition, the most common of which are connective tissue disorders such as lupus (SLE).\nFor treating the phenomemon, your doctor will first want to ensure that you do not have secondary Reynaud's phenomenon, and if you do, the treatment will vary depending on the underlying disease.\nIf you are found to have primary Reynaud's disease, then there are also a variety of treatment options available that should be discussed with your health care provider. Some of the options include:\n\nVasodilators - Vasodilators relax blood vessels increasing blood flow. Creams such as nitroglycerin cream applied to base of fingers may help heal ulcers.\nCalcium channel blockers - Relax and open small blood vessels in your\nhands and feat, decreasing frequency of attacks.\nAlpha blockers - These drugs counter the actions of noradrenaline, a\nhormone that acts to constrict blood vessels.\n\nIn addition, your doctor will likely advise you to not smoke (as smoking is associated with lower skin temperature due to constriction of blood vessels), exercise more (as exercising can increase circulation) and control stress (which may help avoid attacks).\n", "topic": "health", "url": "https://health.stackexchange.com/questions/648"}, {"image": "VQAonline_00021643.png", "question": "In Judges 19:2, did she get into prostitution?", "context": "In Judges 19:2 (NASB)\n\nBut his concubine found him repugnant, and she left him and went to her father\u2019s house in Bethlehem in Judah, and remained there for a period of four months.\n\nIn BPT09 comments it states that in Hebrew the \"she left him\" means that she got into prostitution (as the following screenshot shows).\n\n\n19,2 Em hebraico: entrou na prostitui\u00e7\u00e3o.\n\nwhich means\n\n19,2 In hebrew: got into prostitution.\n\nEven though, apart from the comment in BPT09, no suggestion of prostitution is found in BPT09 and NASB (they basically mention \"she just left\"), the KJV makes it a little more clear\n\nAnd his concubine played the whore against him, and went away from him unto her father's house...\n\nSo, from KJV she first played the whore and only then she left.\n", "answer": "Judges 19:2a\nNew International Version\n\nBut she was unfaithful [H2181] to him. She left him and went back to her parents' home in Bethlehem, Judah.\n\nThe Hebrew word here has a wide range of meanings.\nBrown-Driver-Briggs\n\n1 be or act as a harlot ...\n2 figurative of improper intercourse with foreign nations ...\n3 of intercourse with other deities, considered as harlotry, sometimes involving actual prostitution ...\n4 moral defection\n\nHere is some background info about adultery and prostitution.\nIt was serious, worthy of death as described in Deuteronomy 22:22\n\nIf a man is found sleeping with another man\u2019s wife, both the man who slept with her and the woman must die. You must purge the evil from Israel.\n\nEven if the husband was just suspicious, it was serious. He would bring her to the priest according to Numbers 5\n\n19 Then the priest shall put the woman under oath and say to her, \u201cIf no other man has had sexual relations with you and you have not gone astray and become impure while married to your husband, may this bitter water that brings a curse not harm you. 20But if you have gone astray while married to your husband and you have made yourself impure by having sexual relations with a man other than your husband\u201d\u2014 21here the priest is to put the woman under this curse\u2014\u201cmay the Lord cause you to become a curse among your people when he makes your womb miscarry and your abdomen swell. 22May this water that brings a curse enter your body so that your abdomen swells or your womb miscarries.\u201d\n\u201c \u2018Then the woman is to say, \u201cAmen. So be it.\u201d\n\nThe husband didn't do any of these damaging things to her. Instead, he did the opposite. He forgave whatever that she did.\nLet's see the context:\n\nJudges 19:1 In those days Israel had no king.\nNow a Levite who lived in a remote area in the hill country of Ephraim took a concubine from Bethlehem in Judah. 2 But she was unfaithful to him. She left him and went back to her parents\u2019 home in Bethlehem, Judah. After she had been there four months, 3her husband went to her to persuade her to return. He had with him his servant and two donkeys. She took him into her parents\u2019 home, and when her father saw him, he gladly welcomed him.\n\nShe initiated the leaving. To where? To her father's home. If she had committed prostitution, it's unlikely that her father would welcome her back.\nWhen her father saw the son-in-law, he was glad. Why? If she had committed adultery, both the father and the concubine were in big trouble. It didn't seem that she had committed anything that serious. The three seem to be getting along well. The father kept wanting the husband to enjoy his hospitality.\nFinally, the husband traveled a long way to get the concubine back. After she died a terrible death. He wanted justice. Would he have done it and raised such an outcry, if she was a prostitute? I don't think so.\nCambridge Bible suggests that it could be as simple as that she was just angry with her husband without being unfaithful:\n\nplayed the harlot against him] The text is open to suspicion. LXX. cod. A reads was angry with him; this suits the context, which implies a quarrel, but not unfaithfulness, on the woman\u2019s part; she left him in anger and returned to her father\u2019s house, whither the Levite followed to pacify her (Jdg 19:3 f.). How are we to account for the reading of the text? Moore ingeniously suggests that by the transposition of two letters she was angry (te\u2019\u0115naph) might have become \u2018she committed adultery\u2019 (tin\u2019aph), which was altered by the Jews to \u2018played the harlot,\u2019\n\nI don't know what she did wrong that caused her to run away from the husband. Whatever it was, I don't think it was prostitution or adultery.\n", "topic": "hermeneutics", "url": "https://hermeneutics.stackexchange.com/questions/56902"}, {"image": "VQAonline_00021626.png", "question": "Rev 1:8: Why is 'legei Kurios' not in the critical apparatus of UBS5, as a variant of 'legei Kurios o Theos'?", "context": "Background:\nToday I bought a UBS5 Greek New Testament. As I looked up Revelation 1:8, I noticed that the critical apparatus doesn't include the variant 'legei Kurios' (in stead of 'legei Kurios o Theos', as the critical text reads).\nHowever, this variant, 'legei Kurios' (without o Theos), was used for the King James Version, and all other major translations which were based on Erasmus' Greek text etc. To me, this seems enough reason to include it in the critical apparatus.\nBecause of this, I've started to question the reliability of the critical apparatus. Therefore I'm considering sending the UBS5 back to the shop (and perhaps buying an NA28 instead).\nBefore I do that, I would like to know the following...\nMain question:\nCan anyone explain to me why 'legei Kurios' (without 'o Theos') is not in the critical apparatus of Rev 1:8 in UBS5? (In other words: is it reasonable that they left it out?) \nSubquestions:\n\nWhat manuscripts (and other sources) support omitting 'o Theos'?\nThe Tischendorf critical text does mention the variant:\n\nHowever, I don't know how to interpret it. What does 'o Kurios cum 30.?' mean?\nDoes the NA28 include the variant? (Then I might consider buying that one instead).\nEdit: I now know the NA28 does not include it.\n\nAny other suggestions are welcome too. P.S. I already read the somewhat similar question about the passage at Who is speaking in Revelation 1:8?\nBut I don't think that answers my question.\nThanks in advance!\n", "answer": "\u03db means the edition by Stephanus of 1550. \u201ccum 30.?\u201d means that Tischendorf is guessing that Stephanus found this reading in the miniscule manuscript \u201c30\u201d, but is not able to check the manuscript himself. The upshot is that the shorter version of the sentence is not in any of the old manuscripts or ancient citations.\n", "topic": "hermeneutics", "url": "https://hermeneutics.stackexchange.com/questions/29781"}, {"image": "VQAonline_00021601.png", "question": "Is 1 Corinthians 9:27 about literally beating oneself up?", "context": "I'm used to reading the Bible in Finnish, so 1 Corinthians 9:27 seems very non-literal in most English translations. The Finnish 1992 translation is usually quite dynamic, but in this case it starts \"Kohdistan iskut omaan ruumiiseeni\" which literally means \"I focus the blows to my own body\". The way for example ESV renders this almost seems like a paraphrase in comparison:\n\n1 Corinthians 9:27 (ESV)\n But I discipline my body and keep it under control, lest after preaching to others I myself should be disqualified. \n\nWhat really interests me is that someone suggested the Greek really literally means \"I focus the blows under my eye\". This would really make the passage way more severe. So what does the passage literally say?\nFor reference, an interlinear translation. I looked at it and it looks to confirm the Finnish (vs. English) translation, but also suggests to me that the \"under my eye\" part is inconceivable.\n\n\n\n", "answer": "The confusion with the translation lies in this word: hyp\u014dpiaz\u014d. If we look at Strong's Concordance for this, we see the following definition:\n\nStrongs G5299\n\nto beat black and blue, to smite so as to cause bruises and livid spots\n \nlike a boxer one buffets his body, handle it roughly, discipline by hardships \n\nmetaph.\n\nto give one intolerable annoyance\n a) beat one out, wear one out\n b) by entreaties\n\nthat part of the face that is under the eyes\n\n\nThe first two definitions seem pretty clear that it involves beating or annoyance. The third makes me seriously question Strong's...\nEtymology\nThe Blue Letter Bible shows that that the source of this word is the conjunction of hypo with a derivitive of optanomai. These two words combined form the basis for that third definition of hypopiazo.\nThe definition of these two words:\nhypo - \"by, under\" \noptanomai - \"to look at, behold\" or \"to be seen, to appear\"\nSo the idea of the conjunction of these two concepts could be seen as being defined as \"the part of the face under the eyes\".\nTranslation/Context\nI believe that Strong's adds that third definition of the word purely based on the etymology. If we keep in mind the fact that this is a verb, we see that \"the part of the face that is under the eyes\" doesn't really do anything. If this third translation was related to some other part of our body that had action (mouth, legs, feet, lips, eyelids, hair, etc.) I can see that the definition could be that. However, with the etymology being an area that has no action and cannot take part in action, I feel that the third definition is probably not useful for this.\nSo we see the passage:\n\u03b1\u03bb\u03bb' \u03c5\u03c0\u03c9\u03c0\u03b9\u03ac\u03b6\u03c9 \u03bc\u03bf\u03c5 \u03c4\u03bf \u03c3\u03ce\u03bc\u03b1\n[But] [?????] [My] [... body]\nWhat is Paul doing to his body? Is it something to do with the part of the face under his eyes? I doubt it.\nI think we can be confident in the translation of \"bruise\". We could even be confident of the translation \"beat\" or \"beat black and blue\" in this sense. The translation \"discipline\" is more of a loose translation than \"bruise\". In this sense \"bruise\" is the literal translation.\nSee Translation Philosophy of the English Standard Version for more information.\nSummary\nThe literal, word-for-word translation from Greek would be:\n\"But [I beat-black-and-blue] my body\"\nAlthough that word has roots that mean \"under the eye\", it is a verb and should be understood based on the word definition, which is \"to bruise\" or \"to beat black and blue\", rather than the source of the word, which would be \"under the eyes\".\n", "topic": "hermeneutics", "url": "https://hermeneutics.stackexchange.com/questions/262"}, {"image": "VQAonline_00021746.png", "question": "Where Epics and Puranas are mentioned as 5th Veda?", "context": "There are 4 vedas - Rigveda, Yajurveda, Samaveda and Atharvaveda.\nBut in several places few scriptures are mentioned as 5th Veda, such as:\nMahabharata\n\n\nThe boon-giving great one then taught Sumanta, Jaimini, Paila, his son Suka, and Vaisampayana, the Vedas having the Mahabharata for their fifth. -Adi Parva: Adivansavatarana Parva of Mahabharata\n\nPur\u0101\u1e47as \n\n\u1e5bg-yaju\u1e25-s\u0101m\u0101tharv\u0101khy\u0101\nved\u0101\u015b catv\u0101ra uddh\u1e5bt\u0101\u1e25\nitih\u0101sa-pur\u0101\u1e47a\u1e41 ca\npa\u00f1camo veda ucyate\nThe four divisions of the original sources of knowledge [the Vedas] were made separately. But the historical facts and authentic stories mentioned in the Pur\u0101\u1e47as are called the fifth Veda. -SB 1.4.20\n\nSimilarly, I heard that Epics (Ramayana and Mahabharata) are called as 5th Veda or sometimes Epics and Puranas collectively called as 5th Veda. \nSo, is there any scripture where Ramayana is mentioned as 5th Veda or Epics collectively are mentioned as 5th Veda?\n", "answer": "\nSimilarly, I heard that Epics (Ramayana and Mahabharata) are called as 5th Veda or sometimes Epics and Puranas collectively called as 5th Veda. So, is there any scripture where Ramayana is mentioned as 5th Veda or Epics collectively are mentioned as 5th Veda?\n\nYes, there is. First of all, the Bhagavatam is one of them, in fact the very Bhagavatam verse you quoted:\n\n\u1e5bg-yaju\u1e25-s\u0101m\u0101tharv\u0101khy ved\u0101\u015b catv\u0101ra uddh\u1e5bt\u0101\u1e25 \n itih\u0101sa-pur\u0101\u1e47a\u1e41 ca pa\u00f1camo veda ucyate\n\nIt's just a translation issue; the verse is actually saying that Itihasas (epics) and Puranas together constitute the fifth Veda.\nIn any case, another chapter of the Sriman Bhagavatam provides more detail on this:\n\nBeginning from the front face of Brahm\u0101, gradually the four Vedas \u2014 \u1e5ak, Yajur, S\u0101ma and Atharva \u2014 became manifest. Thereafter, Vedic hymns which had not been pronounced before, priestly rituals, the subject matters of the recitation, and transcendental activities were all established, one after another. He also created the medical science, military art, musical art and architectural science, all from the Vedas. They all emanated one after another, beginning from the front face. Then he created the fifth Veda \u2014 the Pur\u0101\u1e47as and the histories \u2014 from all his mouths, since he could see all the past, present and future.\n\nThis chapter of the Chandogya Upanishad also says that Itihasas and Puranas are the fifth Veda:\n\nI know the Rig-veda, Sir, the Yagur-veda, the S\u00e2ma-veda, as the fourth the \u00c2tharvana, as the fifth the Itih\u00e2sa-pur\u00e2na; the Veda of the Vedas; the Pitrya; the R\u00e2si; the Daiva; the Nidhi; the V\u00e2kov\u00e2ky; the Ek\u00e2yana; the Deva-vidy\u00e2; the Brahma-vidy\u00e2; the Bh\u00fbta-vidy\u00e2; the Kshatra-vidy\u00e2; the Nakshatra-vidy\u00e2 (astronomy); the Sarpa and Devagana-vidy\u00e2.\n\n", "topic": "hinduism", "url": "https://hinduism.stackexchange.com/questions/23409"}, {"image": "VQAonline_00021779.png", "question": "Why is there a Bheem temple in Gorakhnath Mandir ,Gorakhpur?", "context": "There is a temple of Bheem also known as Bhimsena or Mahabali Bhim in the premises of Gorakhnath math in Gorakhpur. What is the importance of this temple? Give references from scriptures to validate your answer. Bheem is the Pandava who was very strong.\n\n", "answer": "As to the story goes, Pandavas were performing 'Rajsuya Yagya' to strengthen their kingdom. Thus, Yudhisthir sent Bheem to Gorakhnath to invite him. \nBut Gorakhnath was in samadhi at that time so Bheem had to wait. Meanwhile, Bheem laid down on the floor and fell asleep and when he woke up a pit was formed in the ground because of his body weight.\nSo, a temple was made in the memory of this story and it's an attraction now.\n", "topic": "hinduism", "url": "https://hinduism.stackexchange.com/questions/34484"}, {"image": "VQAonline_00021735.png", "question": "Do conjoined twins have same soul?", "context": "\nThe picture you see is of - Abigail Hensel and Brittany Hensel, two conjoined twins. Wikipedia- \nhttps://en.m.wikipedia.org/wiki/Abby_and_Brittany_Hensel\nDo conjoined twins have different souls? Both have different feelings, taste, sensations.\nWhat happens to their Karma?\n", "answer": "No, they have different soul because they have different Consciousness. Here is passage from Aitreya Upanishad showing it:\n\nIII-i-3: This One is Brahman; this is Indra, this is Prajapati; this is all these gods; and this is these five elements, viz. earth, air, space, water, fire; and this is all these (big creatures), together with the small ones, that are the procreators of others and referable in pairs to wit, those that are born of eggs, of wombs, of moisture of the \n earth, viz. horses, cattle, men, elephants, and all the creatures that there are which move or fly and those which do not move. All these have Consciousness as the giver of their reality; all these are impelled by Consciousness; the universe has Consciousness as its eye and\n Consciousness is its end. Consciousness is Brahman. \n\n III-i-4: Through this Self that is Consciousness, he ascended higher up from this world, and getting all desires fulfilled in that heavenly world, he became immortal, he became immortal.\n\nFirst Sutra of Shiva Sutras also state 'Chaitanyam Atman' or 'Consciousness is the self.'\n", "topic": "hinduism", "url": "https://hinduism.stackexchange.com/questions/18323"}, {"image": "VQAonline_00022001.png", "question": "From 1936-45 what was the prescribed punishment in Nazi Germany for failing to join or participate in the Hitler Youth?", "context": "\nHitler reviewing members of the Hitlerjugend\nQuestion\nI know all other youth programs were abolished in the early or mid 30's, and then membership in the Hitler Youth was mandatory for \"aryans\" by the end of 1936 under the Gesetz \u00fcber die Hitlerjugend (Hitler Youth Law), which was later made compulsory (regardless of parents' permission) in 1939 under the Jugenddienstpflicht (Youth Service Duty). \nBut if a youth chose not to enlist or participate, were they imprisoned, or were the parents imprisoned? Or was there a fine? Or both?\nThe \"Membership\" tag of the Wiki article merely indicates \"Parents who refused to allow their children to join were subject to an investigation by the authorities\". But to what end were they \"investigated\"? What did the law prescribe as punishment? Was the punishment subjective based on the investigator, or were jail terms or fines prescribed in the law?\n", "answer": "There was no upper limit to sanctions.\nFrom to 1936 to 1939, membership in the HJ was \"voluntary\". So actual punishment was not put into law until the Second Executive Order on the law about the Hitler Youth (Jugenddienstverordnung) from March 25th, 1939.\nWhat follows is my translation. I am neither a lawyer nor a professional translator.\n\nArticle 12. Sanctions.\n(1) A legal guardian will be punished to a fine of up to 150 Reichsmark or imprisonment if intentionally violating article 9 of this order.\n(2) Anyone who maliciously keeps an adolescent away from service in the Hitler Youth, or attempts to do so, will be punished by fine and imprisonment, or one of these punishments.\n(3) Criminal prosecution shall only commence on request by the Jugendf\u00fchrer of the German Reich. The request can be withdrawn.\n(4) The appropriate municipal police office can urge adolescents to meet the obligations placed upon them by this order and the corresponding implementation rules.\n\nThe referenced article 9 was what made membership obligatory.\nNote how (1) and (2) do not specify any maximum punishment, and (4) being a carte blanche for any kind of harassment without requiring (3) to have taken effect.\n\nThis is the original German text, included for completeness:\n\n\u00a7 12. Strafbestimmungen.\n(1) Ein gesetzlicher Vertreter wird mit Geldstrafe bis zu 150 Reichsmark oder mit Haft bestraft, wenn er den Bestimmungen des \u00a7 9 dieser Verordnung vors\u00e4tzlich zuwiderhandelt.\n(2) Mit Gef\u00e4ngnis und Geldstrafe oder mit einer dieser Strafen wird bestraft, wer b\u00f6swillig einen Jugendlichen vom Dienst in der Hitler-Jugend abh\u00e4lt oder abzuhalten versucht.\n(3) Die Strafverfolgung tritt nur auf Antrag des Jugendf\u00fchrers des Deutschen Reichs ein. Der Antrag kann zur\u00fcckgenommen werden.\n(4) Jugendliche k\u00f6nnen durch die zust\u00e4ndige Ortspolizeibeh\u00f6rde angehalten werden, den Pflichten nachzukommen, die ihnen auf Grund dieser Verordnung und den zu ihr ergangenen Ausf\u00fchrungsbestimmungen auferlegt worden sind.\n\n", "topic": "history", "url": "https://history.stackexchange.com/questions/47836"}, {"image": "VQAonline_00021838.png", "question": "What is the state of the art historical analysis of claims that Carthaginians discovered America?", "context": "The Carthaginians, and the Phoenicians in general, are renowned for their seafaring abilities. There is evidence for their circumnavigation of Africa, and strong evidence for the fact that Hanno the Navigator reached modern Cameroon.\nThe geologist Mark McMenamin, working with computer-enhanced images of gold coins minted in Carthage between 350 and 320 BC, analyzed a series of misterious designs appearing on these coins, the meaning of which has long puzzled scholars. McMenamin interpreted the designs as a map of the ancient world, including the area surrounding the Mediterranean Sea and the land mass representing the Americas.\n\nIf this is true, these coins not only represent the oldest maps found to date, but would also indicate that Carthaginian explorers had sailed to the New World.\nAccording to Diodorus Siculus:\n\n[...] in the deep off Africa is an island of considerable size...fruitful, much of it mountainous.... Through it flow navigable rivers....The Phoenicians had discovered it by accident after having planted many colonies throughout Africa.\n\nWe also have another clue. In 1872, four pieces of a stone tablet inscribed with strange characters were found on a Brazilian plantation near the Paraiba River. A copy of the inscription was sent by the owner of the property to Dr. Ladislau Netto, director of the Museu Nacional in Rio de Janeiro. After studying the document carefully, Dr. Netto announced to a startled world that the inscription recorded the arrival of Phoenician mariners in Brazil centuries before Christ. Unfortunately, an Indian rebellion broke out in the Paraiba region that same year and in the ensuing confusion, the plantation in question was never located and the stone itself was never recovered. A copy of the inscription was sent to the eminent French historian and philologist Ernest Renan who declared it a fake, and Netto was ridiculed by the academic establishment of his day.\nRenan based his conclusion on the fact that the text contained certain grammatical errors and incorrect expressions that forced him to question its authenticity. A century later, an American scholar, Cyrus H. Gordon, revisited the Paraiba inscription and arrived at the opposite conclusion. The inscription, he claims, contains grammatical forms and expressions that have been recently discovered and were unknown to linguistic experts of the 19th century like Renan and Netto. Therefore, he contends, the document could not have been a fake. Gordon's translation reads, in part:\n\nWe are sons of Canaan from Sidon...We sailed from Ezion-geber into the Red Sea and voyaged with ten ships. We were at sea together for two years around Africa but were separated by the hand of Baal and we were no longer with our companions. So we have come here, twelve men and three women...may the exalted gods and goddesses favor us.\n\nWhat is the state of the art take on these claims?\n", "answer": "The tablet is almost certainly a modern fake:\n\nDespite Gordon\u2019s certainty about the genuineness of the inscription,\n he failed to find support from colleagues and, notably, entered into a\n bitter dispute with Frank Moore Cross Jr (born 1921), Hancock\n Professor of Hebrew and Other Oriental Languages Emeritus at Harvard.\n Cross pointed to problems with the script, vocabulary and spelling\n (just about every aspect of the inscription, in fact) demonstrating\n conclusively that the text was a modern forgery.\n\nThis seemed to be something of a pass-time among antiquarians in the 19th century.\nAs for the coin, if it is indeed a map, I see no reason to interpret the landmass as anything other than Britain, and the little bits labeled \"British Isles\" as the Channel Islands or, if you want to get really wild, the southern edge of Scandinavia, which is just a tad closer than North America. (\"Asia\" is likely the Arabian Peninsula)\nWhen you see hoofprints, think horses, not zebras. \n", "topic": "history", "url": "https://history.stackexchange.com/questions/7326"}, {"image": "VQAonline_00021919.png", "question": "What impact does colonial legacy have on current US presence in the South China Sea dispute?", "context": "In recent years, the South China Sea has become somewhat important from a geo-political standpoint. China is building military bases, among many other things, and the US has been speaking about containing a militarily aggressive China. As a student of history, it is important to look at this dispute in context not only of the current political motives, but also in terms of the history.\n\n(picture from Wikipedia)\nHistorically speaking, most spoken about in the media and in Wikipedia are the various territorial claims by different claimant entities. Anyone who glances at a map can see that several countries might have valid claims: China, Vietnam, Malaysia, Brunei, Singapore, and the Philippines. Which of these countries ultimately control this water has nothing to do with my question. It is notable that the United States does not have any territory in this area, and that the United States has no territorial claim.\nSo why does the US Navy have a significant presence in the South China Sea? Why is this an important election issue for Hillary Clinton and Donald Trump? Why is this front page news in the US? The media gives a few \"standard narrative\" answers for this, related to international freedom of navigation, international law, trade, and to protect US business interests. Strangely absent from the standard narrative is the US colonial legacy in the Philippines. The US didn't recognize the Philippines as an independent country until 1946, and had a significant military presence there until base closures in the 1990's. It would not be hard to characterize the Philippines as a client state of the US, or at least under neo-colonial control. Edit: It is suggested in the comments that the word \"neo-colonial control\" is to strong. The characterization is considered incorrect by many; but it is undeniable that the US has a history of economically punishing countries to not follow its policies)\nIt is significant that Vietnam, Malaysia, Singapore, and Brunei were not fully independent from the colonial powers until after the 1960's.\nIt seems that to fully understand the South China Sea territorial dispute, it is critically important to understand the colonial history of the region.\nSo the preliminary questions are prior to 1930, were there any formal colonial claimants to this area? Did any have recognized sovereignty over this area of water?\nThe Big Question:\nWhat impact does this colonial legacy have on the current US presence in this dispute?\nEdit: Perhaps the US colonial legacy has no impact on US interests in this area! Any and all answers are possibly acceptable.\n", "answer": "Prior to 1930 (1946), actually, the U.S. had claims in this area, through its possession of the Philippines. This is because the Philippines are one of 10 so-called ASEAN (Southeast Asian) nations. Even to this day, the U.S. has certain treaty rights in the Philippines. That is to say that the U.S. retains a defensive interest in Philippine affairs, even though she has granted independence to the Philippines.\nOther formal claimants in the general area before 1945 were the French (Indochina), the Dutch (East Indies, modern Indonesia), and the British (Malaysia, Singapore, Brunei). These three powers are gone, the U.S. is not.\nThe ASEAN nations basically surround this waterway shown in the map. It is the \"fourth\" great region of Asia (after China, India, and Japan). If they were a country, they would have 500 million people and over 1 million square miles, giving them great economic and strategic importance. And while ASEAN is less rich than the other countries, the waterway affects ASEAN much more; that is, all of ASEAN versus e.g. only part of China.\n", "topic": "history", "url": "https://history.stackexchange.com/questions/30905"}, {"image": "VQAonline_00022133.png", "question": "HERMS waterproof hole in heat exchanger", "context": "I'm in the initial phase of my HERMS project, and I'm struggling to come up with an elegant solution to waterproofing the hole where my copper pipe will enter the hot liquor tank. Ideally I'd like some kind of thru-hull (below) that has fittings on either side I can attach the pipe to, but for the life of me I can't find one. What is the typical solution for this problem?\n\n", "answer": "The Electric Brewery has a step-by-step guide for mounting a valve and quick disconnect to the kettle wall. \nTo summarize, pass a 1/2\" threaded nipple through a hole in the kettle. On the outside, secure with a lock nut and high temperature silicone o-ring. On the inside, secure with an FPT compression fitting. The stainless tubing is connected on the inside to the compression fitting.\n", "topic": "homebrew", "url": "https://homebrew.stackexchange.com/questions/12226"}, {"image": "VQAonline_00022107.png", "question": "How do I identify the type of poppet I need for a corny keg?", "context": "I just got my first kegs - a couple used, reconditioned corny kegs from beveragefactory.com, still smelling like soda (yay). I've been reading up on the process and it seems like it's a best practice to replace the poppets, along with the O-rings. Wish I knew that before I made the order, or I would have gotten those too.\nThere are several types of poppets. \nHow do I identify the the type of poppet I need for my keg? \nAlso how to I remove and replace them?\nI found a couple pages that list some different types:\n\nPoppets from CHI Co. \nPoppets from beverage factory\n\nAlso any recommendations on other cheap places to get the poppets would be great. eBay seems like it might be a way to go.\nI have ball lock Pepsi kegs, and the lids say Corco on them.\ntops of my kegs:\n\n", "answer": "I imagine the safest bet is to unscrew one of the posts and get the same kind of poppet as the one you've already got.\nIt's hard to tell just looking at the top of the keg, so don't take this as certainty, but those keg posts look like firestone posts. I have pepsi kegs, with firestone posts, and use poppet nr. 1 from CHI. \nYou can also narrow it down by elimination. On the CHI page for #2, it says that #1 is a cheaper alternative, so we rule that out. I'm guessing you don't have a plastic insert, which rules out #3 and #4. You'll see immediately from looking at the poppet in your own kegs if it has long legs. I'm guessing not, ruling out #5 and #6. \n", "topic": "homebrew", "url": "https://homebrew.stackexchange.com/questions/7265"}, {"image": "VQAonline_00022111.png", "question": "How to sparge with a inner mash cylinder?", "context": "I have bought a BeerMaker 30 and now I wonder how to do the sparging so that I get the clearest wort possible.\n\nThe issue for me is, that I have a steel cylinder that goes into my electric mash/brew kettle. The bottom of the cylinder i a metal plate with holes in it. I suspect that while I do the mashing, some particles will get through there and end up in the brew kettle.\nWhen I am done with mashing, and start sparging I will have a hard time getting all the particles out, since a Vorlauf would be the whole liquid volume. \nI am thinking about getting a mash-bag that I could put on the inside of the steel cylinder (or even on the outside). \nAm I thinking too complicated? Is this effort really necessary? How would you use this?\nI have not yet used this, but I hope to avoid errors that are unnecessary.\n", "answer": "Just treat it like any other mash tun. Before you pull that thing out of the kettle start drawing off wort through the valve at the base (at least it looks like a valve in the picture). Collect it in a pitcher that you can easily pour from. Then slowly pour in back in at the top of the cylinder. Do this repeatedly until you think the wort looks clear enough. It may take 2-4L of circulation total.\nKeep in mind that the wort won't look like clear beer though. It will be cloudy. What you are looking for is that there are no big pieces of grain or husk still coming out. You'll be able to see them as you pour from the pitcher back in.\nGood luck\n", "topic": "homebrew", "url": "https://homebrew.stackexchange.com/questions/8009"}, {"image": "VQAonline_00022249.png", "question": "Were Gauss's 1846 remarks on the distinction between right and left related to orientability of surfaces?", "context": "Recently i was striked by a quotation of Gauss from a letter to his student Gerling from the date June 23, 1846. This letter states in very concise words that the distinction between right-handed and left-handed 3D cartesian coordinates system is not of aprioric nature; it cannot be determined by the first principles of spatial logic. According to what i read up to now, Gauss remarked this in order to state his disagreement with Kant's philosophy of space and of space perception (Gauss expressed similar disagreement with Kant in his private letters on non-euclidean geometry). Gauss's relevant remarks are those:\n\nOne cannot reduce to concepts the distinction between two systems of three straight lines each (directed lines, of which the one system points forward, upward to the right, the other forward, upward to the left) but one can only demonstrate by holding to actually concrete spatial things. Two minds cannot reach agreement about it unless their views connect up with one and the same system present in the real world.\n\nand there is also this quote from his letter to Schumacher (Februar 8 ,1846):\n\nThe distinction between right and left cannot be defined, but only shown, so that it is thereby a case similar to sweet and bitter...two such minds, however, cannot make themselves directly understood concerning right and left unless one and the same individual thing forms a bridge between them... i find in it the striking refutation of Kant's imagination that space is merely the form of our external perception.\n\nThe reason why i was striked by reading Gauss's quotes is that it anticipates- not in the mathematical details, but rather in the philosophical level - the notion of orientability of surfaces, and perhaps also - though not in the mathematical details - the possibility of non-orientable surfaces and spaces like Mobius's strip and it's higher dimensional analogs. As an illustration of the impossibility of drawing a distinction between right and left at certain spaces, i added here this gif:\n\nA crab walking on a Mobius strip cannot decide between right and left.\nWhat i'd like to know is:\n\nCan anyone tell something about the state of \"philosophy of right and left\" at Gauss's time? because perhaps i'm misinterpreting Gauss's letter and he didn't really stood against Kant's view.\nCan anyone tell a little bit about the background to this correspondence of Gauss? \n\n", "answer": "The short answer to the title question is likely yes. There are two separate issues discussed in the OP, one is more philosophical on Gauss's disagreements with Kant's apriorism about space, and the other, concerning his insights into the orientation of surfaces and non-orientable surfaces. The first question is addressed to some extent in Was Kant a factor in forming Gauss's abstract view of mathematical objects?, so I will focus on the second.\nGauss corresponded with Gerling for a long time, at least since 1819, when he mentioned the angle sum defect (\"Gauss-Bonnet theorem\") for hyperbolic triangles in a letter to him. At the time, the main subject was non-Euclidean geometry, and this is discussed at length in What did Gauss read in the Appendix? by Abardia, Revent\u00f3s and Rodr\u00edguez (the reference is to Bolyai's Appendix from 1832). Issues related to what Gauss called theorema egregium (invariance of curvature under isometries) are discussed in 1825 letters to Schumacher. Gauss's celebrated work on geometry of curved surfaces Disquisitiones Generales circa Superficies Curvas came out in 1827. His relevant writings on the subject, both published and unpublished, are discussed at length in 150 years after Gauss\u2019 \u00abDisquisitiones generales circa superficies curvas\u00bb by Dombrowski. Dombrowski quotes an unpublished 1825 draft of Disquisitiones Generales, which shows that Gauss possessed the concept of surface orientation in 1825 (and possibly as early as 1794). In his modernized \"free translation\":\n\n\"The sum of the angles of a (small) geodesic triangle \u0394 in a curved surface in $\\mathbb{E}^3$ is equal to the sum of \u03c0 and the oriented surface area of the spherical image of \u0394, where the oriented area is taken to be positive or negative according to whether the bounda\u00adry of the spherical image of \u0394 winds around the image in the same direction or in the opposite direction as the boundary of \u0394 winds around \u0394.\" (57)\n\nGauss then generalizes it to geodesic polygons. Dombrowski discusses the context and speculates that Gauss was led to the general statement for curved surfaces by the hyperbolic case combined with his practical experience with geodesic measurements in 1812-1822:\n\n\"The result (57) was already widely known in the special case of a deve\u00adlopable or a spherical surface. Gauss possessed the result analogous to \n (57) for hyperbolic geometry (and therefore essentially for surfaces of constant \n negative curvature, the author) already in 1794 (see G.W. 8, p. 266), and announced it to Gerling in a letter in 1819 (see G.W. 8, p. 182). It is there\u00adfore easy to imagine that Gauss was led to a geometrically intuitive \"insight\" concerning the validity of (57) for the general case of a surface of non-constant Gaussian curvature from the knowledge of this fundamental case and on the basis of his rich differential-geometric experience in geodesy (with geodesics and trigonometry spheroids, but also with questions of mapping and bending) obtained in the years \n 1812 to 1822. \n[...] Although Gauss had studied intensively the concept of \"oriented surface area\" as used by him in (57) (see G.W. 8, p. 398, line 2 from below), he did \n not think that his corresponding studies had \"matured\" sufficiently... The proof (with its so impressive geometric arguments, which we have just retraced from the fragment (55)) was never published by Gauss! The reason for this lies on the one hand certainly in his self-criticism of his own above mentioned sketch of a proof for (57), and particularly of his concept of the \"oriented surface area of the spherical image on a curved surface\" (involved in that proof) which he has not defined with that analytic rigour usually applied by him.\" \n\nThis result leads to theorema egregium discussed in 1825 letters to Schumacher. The Gauss-Bonnet theorem itself is brought up in the 10 October 1846 letter to Gerling (Dombrowski's translation):\n\n\"The theorem which Mr. Schweikart mentioned to vou,that in any eeometrv the sum of all outer angles of a polygon differs from 360 by a quantity,..., which is proportional to the surface area, is the first theorem lying almost on the threshold of that theory, a theorem whose necessity I already recognized in 1794\".\n\nSo the appearance of orientation related issues in the letters to Gerling and Schumacher in 1846 (connected to the older themes of his agreements and disagreements with Kant) is not surprising. \nAs for the non-orientable surfaces, the issue is much more murky. The \"M\u00f6bius band\" is described in private papers of Listing and M\u00f6bius in 1858, almost simultaneously and in similar terms, Listing's description is earlier by several months. Gauss died in 1855, but both were Gauss's students, and many historians attribute many of Listing's topological ideas to Gauss. Listing himself stated something to this effect, although without specifics. Biggs in his chapter in M\u00f6bius and his band explicitly speculates that Gauss might have been the common source here, but admits that the issue is undecidable:\n\n\"Both of them describe the construction in\n very similar terms. Is this another one of those istances which sometimes\n happen in scientific discovery, where an idea whose time is ripe\n appears iI1dependentiy in different places but at the same time? That\n is certainly a possibility. Or was there a common reason for the fact that both M\u00f6bius and Listing described the M\u00f6bius band around\n the same time? If the latter, then the likelihood is that the common\n reason was connected with the work or Gauss who, as we know, had\n been very interested in this kind or topic. Gauss had died in 1855, so\n the idea cannot have been communicated by him directly, but the\n possibility of some link with his work remains. I doubt that the question will ever be completely resolved.\"\n\n", "topic": "hsm", "url": "https://hsm.stackexchange.com/questions/11469"}, {"image": "VQAonline_00022212.png", "question": "When did the people start using the portraits of the two Euclids interchangeably?", "context": "According to the folks of Wikipedia, this is the portrait of Euclid of Alexandria (the \"Father of Geometry\"):\n\nYet, I read several years ago on MathOverflow that the individual in the picture is actually a namesake of Euclid of Alexandria who was born in Megara like a hundred years before Euclid of Alexandria.\nThus, I would like to know if it is known when it was that the people started using the portraits of the two Euclids interchangeably...\n", "answer": "We have very few information regarding the author of the Elements, called Euclid of Alexandria.\nSee Euclid, The Thirteen Books of the Elements, Vol. 1 : Books 1-2 (ed. T.Heath, Dover reprint), page 1-on :\n\nAs in the case of the other great mathematicians of Greece, so in Euclid's case, we have on eagre particulars of the life and personality of the man.\nMost of what we have is contained in the passage of Proclus' summary relating to him [...]. This passage shows that even Proclus had no direct knowledge of Euclid's birthplace or of the date of his birth or death. He proceeds by inference.\nThis man lived in the time of the first Ptolemy. [...] Thus Euclid must have flourished ca.300 B.C., which date agrees well with the fact that Ptolemy reigned from 306 to 283 B.C.\n\nThus, if we know quite nothing about it, it is vey difficult that someone can know his face...\nSee page 3 :\n\nIn the middle ages most translators and editors spoke of Euclid as Euclid of Megara. This description arose out of a confusion between our Euclid and the philosopher Euclid of Megara who lived about 400 B.C. The first trace of this confusion appears in Valerius\n Maximus (in the time of Tiberius) who says that Plato, on being appealed to for a solution of the problem of doubling the cubical altar, sent the inquirers to \"Euclid the geometer.\" [...] if Valerius Maximus took Euclid the geometer for a contemporary of Plato, it could only be through confusing him with Euclid of Megara. \nThe first specific reference to Euclid as Euclid of Megara belongs to the 14th century, occurring in [...] Theodorus Metochita (d. 1332) who speaks of \"Euclid of Megara, the Socratic philosopher, contemporary of Plato,\" as the author of treatises on plane and solid geometry, data, optics etc.: and a Paris MS. of the 14th century has \"Euclidis philosophi\n Socratici liber elementorum.\" \nThe misunderstanding was general in the period from Campanus' translation (Venice (1482) to those of Tartaglia (Venice 1565) and Candalla (Paris 1566). But one Constantinus Lascaris (d. about 1493) had already made the proper distinction by saying of our Euclid that \"he was different from him of Megara of whom Laertius wrote, and who wrote dialogues \"; and\n to Commandinus belongs the credit of being the first translator to put the matter beyond ,doubt: \"Let us then free a number of people from the error by which they have been induced to believe that our Euclid is the same as the philosopher of Megara \" etc.\n\nThus, it is quite clear that the portrait of Euclid by the Dutch painter Justus van Gent, circa 1474, was intended to be a portrait of the geometer, made in a time when he was still known by many as Euclid of Megara.\n\nAdded\nOn the overall issue regarding the re-discovery of Euclid during the Renaissance, see :\n\nRobert Goulding, Defending Hypatia : Ramus, Savile, and the Renaissance Rediscovery of Mathematical History (2010).\n\nIn particular, see Ch.5 : The Puzzling Lives of Euclid, page 117-on, devoted to \"the Renaissance unmasking of Euclid\" [page 118], from :\n\nIn 1505, the Venetian humanist Bartolomeo Zamberti published his monumental Latin translation of the works of Euclid, containing not only the Elements, but also the Phaenomena, Optics, Catoptrics and Data. In his preface, Zamberti acknowledged that his readers would likely want to know something about the author of these works, styled \u201cEuclid of Megara, the Platonic Philosopher\u201d on the title page [page 118].\n\nto \"the final resolution\" [page 141]:\n\nIn 1572, Federico Commandino published a great edition of the Elements to which he prefaced a few biographical remarks. He noted that many had taken Diogenes Laertius\u2019s life of Euclid of Megara to be that of the geometer, so that the author of the Elements was thought to be one of the Socratic philosophers. But, he observed, that could not be the case, since Proclus stated quite clearly that Euclid lived in the time of Ptolemy I: it would be impossible for the two to be the same person.\n\n", "topic": "hsm", "url": "https://hsm.stackexchange.com/questions/325"}, {"image": "VQAonline_00022220.png", "question": "What are the early applications of differential equations to social sciences?", "context": "I am reading Karatzas and Shreeve (Brownian Motion and Stochastic Calculus) and on page 128, at the beginning of chapter 3 on Stochastic Integrals, one reads:\n\nI can't think of any social problem that was treated by Newton and Leibniz. Speaking of differential equations generally financial applications would count as a social problem, but that's only at the turn of the XX century. Are there any earlier applications? Do you know any? Which?\n", "answer": "One of the earliest works containing a \"mathematical model\" in sociology was Malthus's An Essay on the Principle of Population (1798). It did not however contain differential equations, or even formulas for that matter, Malthus expressed that population multiplies geometrically and food arithmetically in words. Malthus's model did influence early work in political economy and sociology, and even Darwin's work. Only in 1838 Comte, the founder of philosophical positivism, declared that \"social physics\" was ready to join the ranks of empirical sciences, and dubbed it \"sociology\" (Siey\u00e8s used the word in 1780, but not in print). \nBut like Malthus the early sociologists, Durkheim and Weber, did not use differential equations. One does appear in Verhulst's 1838 work on population growth, which was motivated by Malthus's model, the differential equation for the now ubiquitous logistic growth is derived and solved there. And in Mathematical Researches into the Law of Population Growth Increase (1845) Verhulst gave the logistic its name. \nMarginalists applied calculus methods to economics (marginals are economists' name for derivatives). In Cournot's Researches into the Mathematical Principles of Wealth (1838) differentiation is used to find equilibrium quantities by maximizing profit, he also anticipates the idea of non-cooperative games and the Nash equilibruium in his models of oligopoly and duopoly. In Walras's Elements of Pure Economics (1874) Cournot's approach is generalized to multiple market agents and the global equilibrium. Edgeworth modeled market behavior based on utility function in Mathematical Psychics: An Essay on the Application of Mathematics to the Moral Sciences (1881), and later even criticized others for lack of mathematical rigor. But neither Walras, nor Edgeworth, nor Marshall in Principles of Economics (1890) use differential equations proper in their models. That had to wait until later. \nPossibly the earliest entry was Bachelier's PhD thesis Theory of Speculation (1900) that somewhat informally introduced mathematical model that we now call stochastic process of Brownian motion to evaluate stock options even before Einstein and Smoluchowski first studied it in kinetic theory in 1905-06. The model leads to diffusion equations. Poincare, Bachelier's instructor, called the work \"very original, and all the more interesting in that Fourier's reasoning can be extended with a few changes to the theory of errors\". Bachelier's model of option pricing closely anticipated the Black-Scholes model widely used today, see Schachermayer-Teichmann's How Close Are the Option Pricing Formulas of Bachelier and Black-Merton-Scholes?\nEDIT: While epidemiology may be borderline between biology and social sciences I found out that Bernoulli's work, pointed out in skol's answer, led to early applications of differential equations to actuarial science, which clearly belongs to the latter. According to Daw's Smallpox and the Double Decrement Table: a Piece of Actuarial Pre-history Lambert (1772) \"gives the earliest application known to me of what would now be called actuarial formulae for dealing with mortality data\". Although Lambert's formulae were based on finite differences he was well aware of Bernoulli's work and their relation to the continuous case. In his 1806 book one of the first French actuaries Duvillard combines Bernoulli type differential equations and Lambert's methods to discriminate between effects of different causes of death, his tables came to be used to estimate risk exposure in life insurance calculations. Interestingly, when later the data collection became more systematic calculations based on Bernoulli's equations became ineffective, and new ones were developed starting with Wittstein (1862), see Multiple Decrements or Competing Risks by Seal.\n", "topic": "hsm", "url": "https://hsm.stackexchange.com/questions/2799"}, {"image": "VQAonline_00022272.png", "question": "What exactly is the difference of meaning of nazzala and anzala in Qur'an verse 136?", "context": "In Surah Nisa, verse 136, Allah says:\n\nBefore the first stop (jeem), Allah describes the Qur'an with \"nazzala\" (\u0646\u064e\u0632\u064e\u0651\u0644\u064e), while he describes the previous books with \"anzala\" (\u0623\u064e\u0646\u0632\u064e\u0644\u064e).\nBoth of these come from the same root verb, nazala, which means \"to descend, to go down.\" The first is baab two, which is a more severe form, while the second looks like the form used often when you cause something to happen (eg. make something descend).\nBut my Arabic is rusty. What exactly is the difference of meaning of these two verbs, and what's the implication here?\n", "answer": "MashaAllah this is a beautiful observation and profound use of the fi'l. \nThere are scholarly opinions on the different usage and the beautiful gems behind it. \n\u0623\u0646\u0632\u0644 comes from the \u0625\u0641\u0639\u0627\u0644 pattern which denotes something to be sent all at once, whereas \u0646\u0632\u0651\u0644 is from the \u062a\u0641\u0639\u064a\u0644 pattern which denotes something to be sent down piecemeal or gradually. \nIn the ayah mentioned, and also the third ayah of Surah Ale Imran (http://quran.com/3/3) - the previous scriptures have been attached to \u0623\u0646\u0632\u0644 which again denotes they were sent down or revealed all at once, whereas the Quran was sent down piecemeal depending on various occasions and situations (http://quran.com/17/106) and most appropriately the \u0646\u0632\u0651\u0644 form of the fi'l was used - SubhanAllah \nNow, one might say what about Surah Qadr where Allah SWT explained the Quran as \u0623\u0646\u0632\u0644\u0646\u0627\u0647 (we sent it down - http://quran.com/97/1) using the pattern which denotes to send down all at once? The scholars have said, there are two revelations of the Quran. One from the Al Lawh Al Mahfoadh to the heavens and the other from the heavens to the heart of the Messenger S. The first revelation was sent down all at once \u0625\u0646\u0632\u0627\u0644 and the second revelation was \u062a\u0646\u0632\u064a\u0644 or piecemeal. \n\u0648\u0627\u0644\u0644\u0647 \u0627\u0639\u0644\u0645 \n", "topic": "islam", "url": "https://islam.stackexchange.com/questions/1809"}, {"image": "VQAonline_00022271.png", "question": "What is the significance of Prophet Yahya being \"hasuwran\"?", "context": "Allah mentions in verse 39 of Surah Al-e-Imran:\nThe fourth word from the end is \"hasuwran\" (\u062d\u064e\u0635\u064f\u0648\u0631\u064b\u0627 ), which Saheeh International (presumably accurately) translates as \"abstaining (from women).\"\nQuestions are:\n\nWhat exactly is the (complete) translation of this word?\nWhat is the significance of this property of Yahya (alayhi salaam)?\n\nI read through a bunch of biography notes, but could not find any mention (let alone any relevance) to this specific word mentioned.\n", "answer": "My answer is what I know from Tafsir Ibn Katheer:\nIt says that \"Hasuran\" with respect to Yahya (alayhi salam) means he is immune to illegal sexual relations. That is what Allah has made him. It does not mean he refrains from sexual relations with women he marries (i.e. legal), but that he is immune from illegal sexual relations. \nAnd Allah knows the best.\n", "topic": "islam", "url": "https://islam.stackexchange.com/questions/1689"}, {"image": "VQAonline_00022306.png", "question": "In Iran, before the second ruku in prayer, what is said during the standing dua?", "context": "I experienced this repeatedly here in Iran, so I suspect this is a Shia thing. In prayer, during the second rakat just before ruku, we make dua while standing. I'm wondering what people are saying during this dua.\nQuestion: In Iran, before the second ruku in prayer, what is said during the standing dua?\nWikipedia mentions that dua is particularly important in Shia Islam, and even has a photo of women performing (what I would guess is) this particular dua.\n\n", "answer": "For Shias:\nIt's not mandatory. You can just move on and not do it all. But It's recommended. It's the moment where you ask Allah anything and everything.\nThere are many many recommendations:\n\n\u0631\u0628\u0646\u0627 \u0622\u062a\u0646\u0627 \u0641\u064a \u0627\u0644\u062f\u0646\u064a\u0627 \u062d\u0633\u0646\u0629 \u0648\u0641\u064a \u0627\u0644\u0622\u062e\u0631\u0629 \u062d\u0633\u0646\u0629 \u0648\u0642\u0646\u0627 \u0639\u0630\u0627\u0628 \u0627\u0644\u0646\u0627\u0631\n\"O Allah! Give to us in the world that which is good and in the\n Hereafter that which is good, and save us from the torment of the\n Fire.\"\n\nor to simply recite a dua faraj* for Imam Mahdi as his ease would be the greatest ease for mankind!\nDua that Imam Ridha did in his Jum'a Qunuts aka Dua'a-e-Allahummah Asleh:\n\n\u0627\u0644\u0644\u0647\u0645 \u0627\u0635\u0644\u062d \u0639\u0628\u062f\u0643 \u0648\u062e\u0644\u064a\u0641\u062a\u0643 \u0628\u0645\u0627 \u0627\u0635\u0644\u062d\u062a \u0628\u0647 \u0627\u0646\u0628\u064a\u0627\u0626\u0643 \u0648 \u0631\u0633\u0644\u0643, \u0648\u062d\u0641\u064e\u0647 \u0628\u0645\u0644\u0627\u0626\u0643\u062a\u0643,\n \u0648\u0623\u064a\u064e\u062f\u0647 \u0628\u0631\u0648\u062d \u0627\u0644\u0642\u062f\u0633 \u0645\u0646 \u0639\u0646\u062f\u0643, \u0648\u0627\u0633\u0644\u0643\u0647 \u0645\u0646 \u0628\u064a\u0646 \u064a\u062f\u064a\u0647 \u0648\u0645\u0646 \u062e\u0644\u0641\u0647 \u0631\u0635\u062f\u0627 \u064a\u062d\u0641\u0638\u0648\u0646\u0647 \u0645\u0646\n \u0643\u0644 \u0633\u0648\u0621, \u0648\u0627\u0628\u062f\u0644\u0647 \u0645\u0646 \u0628\u0639\u062f \u062e\u0648\u0641\u0647 \u0627\u0645\u0646\u0627\u064b \u064a\u0639\u0628\u062f\u0643 \u0644\u0627 \u064a\u0634\u0631\u0643 \u0628\u0643 \u0634\u064a\u0626\u0627\u064b, \u0648\u0644\u0627 \u062a\u062c\u0639\u0644 \u0645\u0646\n \u062e\u0644\u0642\u0643 \u0639\u0644\u0649 \u0648\u0644\u064a\u0643 \u0633\u0644\u0637\u0627\u0646\u0627, \u0648\u0623\u0630\u0646 \u0644\u0647 \u0641\u064a \u062c\u0647\u0627\u062f \u0639\u062f\u0648\u0643 \u0648\u0639\u062f\u0648\u0647, \u0648\u0627\u0644\u062c\u0639\u0644\u0646\u064a \u0645\u0646 \u0627\u0646\u0635\u0627\u0631\u0643\n \u0627\u0646\u0643 \u0639\u0644\u0649 \u0643\u0644 \u0634\u064a\u064d \u0642\u062f\u064a\u0631\nO Allah! ease conditions (his reappearance) of Your servant and caliph with those means\n by which You have eased (hastened) of Your messengers and prophets (Moses, Noah, Jesus, etc).\n Surround him with the angels and support him with the Holy Spirit.\n Appoint protectors from the front and behind him, (those) who would\n guard him from all troubles and calamities. Convert his fear into\n security, (so that) he may worship only You and doesn't associate\n anybody with You. Do not make any of Your creatures an authority over\n Your vicegerent. Permit him to fight against Your enemies and his\n enemies and include me among his helpers. Surely You possess power\n over everything.*\n\nDua Faraj\n\n\u0627\u0644\u0644\u064e\u0651\u0647\u064f\u0645\u064e\u0651 \u06a9\u064f\u0646\u0652 \u0644\u0650\u0648\u064e\u0644\u0650\u06cc\u0650\u0651\u06a9\u064e \u0627\u0644\u062d\u064f\u062c\u064e\u0629\u0650 \u0628\u0646\u0650 \u0627\u0644\u062d\u064e\u0633\u064e\u0646 \u060c \u0635\u064e\u0644\u064e\u0648\u0627\u062a\u064f\u06a9\u064e \u0639\u0644\u064e\u06cc\u0647\u0650 \u0648 \u0639\u064e\u0644\u06cc \u0622\u0628\u0627\u0626\u0650\u0647\u0650 \u060c \u0641\u0650\u06cc \u0647\u064e\u0630\u0650\u0647\u0650 \u0627\u0644\u0633\u064e\u0651\u0627\u0639\u064e\u0629\u0650 \u0648\u064e \u0641\u0650\u06cc \u06a9\u064f\u0644\u0650\u0651 \u0633\u064e\u0627\u0639\u064e\u0629\u064d \u060c \u0648\u064e\u0644\u0650\u06cc\u0651\u0627\u064b\n \u0648\u064e \u062d\u064e\u0627\u0641\u0650\u0638\u0627\u064b \u0648\u064e \u0642\u064e\u0627\u0626\u0650\u062f\u0627\u064b \u0648\u064e \u0646\u064e\u0627\u0635\u0650\u0631\u0627\u064b \u0648\u064e \u062f\u064e\u0644\u0650\u06cc\u0644\u064b\u0627 \u0648\u064e \u0639\u064e\u06cc\u0652\u0646\u0627\u064b \u060c \u062d\u064e\u062a\u064e\u0651\u0649\n \u062a\u064f\u0633\u0652\u06a9\u0650\u0646\u064e\u0647\u064f \u0623\u064e\u0631\u0652\u0636\u064e\u06a9\u064e \u0637\u064e\u0648\u0652\u0639\u0627\u064b \u0648\u064e \u062a\u064f\u0645\u064e\u062a\u0639\u064e\u0647\u064f \u0641\u0650\u06cc\u0647\u064e\u0627 \u0637\u064e\u0648\u0650\u06cc\u0644\u0627\" \u060c\nO Allah, be, for Your representative, the Hujjat (proof), son of\n Al\u00adHassan, Your blessings be on him and his forefathers, in this hour\n and in every hour, a guardian, a protector, a leader, a helper, a\n proof, and an eye. until You make him live on the earth, in obedience\n (to You), and cause him to live in it for a long time.\n\nor other things like:\npray for believers and parents\n\n\u0627\u064e\u0644\u0644\u0647\u064f\u0645\u0651\u064e \u0627\u063a\u0652\u0641\u0650\u0631\u0652\u0644\u0650\u0649\u0652 \u0630\u064f\u0646\u064f\u0648\u0652\u0628\u0650\u0649\u0652 \u0648\u064e\u0644\u0650\u0648\u064e\u0627\u0644\u0650\u062f\u064e\u0649\u0651\u064e \u0648\u064e\u0627\u0631\u0652\u062d\u064e\u0645\u0652\u0647\u064f\u0645\u064e\u0627 \u0643\u064e\u0645\u064e\u0627 \u0631\u064e\u0628\u0651\u064e\u064a\u064e\u0627\u0646\u0650\u0649\u0652 \u0635\u064e\u063a\u0650\u064a\u0652\u0631\u064b\u0627. \u0648\u064e\u0644\u0650\u062c\u064e\u0645\u0650\u064a\u0652\u0639\u0650 \u0627\u0644\u0652\u0645\u064f\u0633\u0652\u0644\u0650\u0645\u0650\u064a\u0652\u0646\u064e \u0648\u064e\u0627\u0644\u0652\u0645\u064f\u0633\u0652\u0644\u0650\u0645\u064e\u0627\u062a\u0650\n \u0648\u064e\u0627\u0644\u0652\u0645\u064f\u0624\u0652\u0645\u0650\u0646\u0650\u064a\u0652\u0646\u064e \u0648\u064e\u0627\u0644\u0652\u0645\u064f\u0624\u0652\u0645\u0650\u0646\u064e\u0627\u062a\u0650\u060c \u064e\u0627\u0652\u0644\u0627\u064e\u062d\u0652\u064a\u064e\u0622\u0621\u0650 \u0645\u0650\u0646\u0652\u0647\u064f\u0645\u0652\n \u0648\u064e\u0627\u0652\u0644\u0627\u064e\u0645\u0652\u0648\u064e\u0627\u062a\u0650\u060c \u0648\u064e\u062a\u064e\u0627\u0628\u0650\u0639\u0652 \u0628\u064e\u064a\u0652\u0646\u064e\u0646\u064e\u0627 \u0648\u064e\u0628\u064e\u064a\u0652\u0646\u064e\u0647\u064f\u0645\u0652 \u0628\u0650\u0627\u0644\u0652\u062e\u064e\u064a\u0652\u0631\u064e\u0627\u062a\u0650\u060c \u0631\u064e\u0628\u0650\u0651\n \u0627\u063a\u0652\u0641\u0650\u0631\u0652 \u0648\u064e\u0627\u0631\u0652\u062d\u064e\u0645\u0652 \u0648\u064e\u0627\u064e\u0646\u0652\u062a\u064e \u062e\u064e\u064a\u0652\u0631\u064f\u0627\u0644\u0631\u0651\u064e\u0627\u062d\u0650\u0645\u0650\u064a\u0652\u0646\u064e\u060c \u0648\u064e\u0644\u0627\u064e\u062d\u064e\u0648\u0652\u0644\u064e\n \u0648\u064e\u0644\u0627\u064e\u0642\u064f\u0648\u0651\u064e\u0629\u064e \u0627\u0650\u0644\u0627\u0651\u064e\u0628\u0650\u0627\u0644\u0644\u0647\u0650 \u0627\u0644\u0652\u0639\u064e\u0644\u0650\u064a\u0651\u0650 \u0627\u0644\u0652\u0639\u064e\u0638\u0650\u064a\u0652\u0645\u0650\nO Allah, grant forgiveness to me for my sins and the sins of my\n parents, and mercy on both of them as he both took care of me when I\n was little, so are all Muslims and muslimat, all believers, male and\n women who are still alive or already dead, and ikutkanlah between us\n and them with kindness. O Allah, grant forgiveness and mercy of God be\n merciful because You are more compassionate and no power and effort\n but with your help.\n\nBut other than recommendations you can ask Allah to make you rich, heal the sick, bless the hearts of the Muslims, feed the hunger, etc.\n\nFor Sunnis: \nFrom WikiPedia: According to the Hanafi opinion, one is supposed to give Takbir (Say Allahu Akbar and raise his palms till ear lobes and hold them back below or above navel with right hand over left) before going into Ruku in 3rd Rakah and recite the Following Qunut Prayer also called as Dua-e-Qunoot (Prayer of Qunoot). After reciting the Dua, Muslims then bend in ruku and perform the rest of the Salath.\n\nshafi'i says: Qunut during morning Salat is mustahab at all time. (\u0628\u062f\u0627\u06cc\u0647 \u0627\u0644\u0645\u062c\u062a\u0647\u062f:\u06f1/\u06f1\u06f3\u06f1\u061b \u0627\u0644\u0645\u063a\u0646\u06cc: \u06f1/\u06f8\u06f2\u06f3)\nAbu Hanifa says: Qunut other than Witr prayer is makruh. (\u0647\u062f\u0627\u06cc\u0647 \u0645\u0631\u063a\u06cc\u0646\u0627\u0646\u06cc:\u06f1/\u06f6\u06f6\u061b \u0628\u062f\u0627\u06cc\u0647 \u0627\u0644\u0645\u062c\u062a\u0647\u062f:\u06f1/\u06f1\u06f3\u06f2)\n\n\n\nFaraj \u0641\u0631\u062c means ease, easement, hast so that our awaiting for Imam Mahdi would be reduced.\n\n", "topic": "islam", "url": "https://islam.stackexchange.com/questions/39925"}, {"image": "VQAonline_00022336.png", "question": "Perch\u00e9 si usava mettere un trattino sotto e sopra alcune vocali?", "context": "\nQual \u00e8 il significato del trattino visibile sotto e sopra ad alcune vocali di quella frase?\nPerch\u00e9 la prima \"E\" di Venezia ha un trattino sopra ma la seconda \"E\" non lo ha? \n", "answer": "I trattini non hanno alcun significato linguistico e sono solo un accorgimento grafico.\nPer dirimere la questione, ho posto la domanda direttamente all'Ufficio per le relazioni con il pubblico della Regione Autonoma Friuli Venezia Giulia, tramite il loro modulo in rete, che mi ha risposto in modo cortese e sollecito:\n\nSig. Gewurz buongiorno,\ni \"trattini\" presenti nel logotipo della Regione Autonoma Friuli Venezia Giulia non hanno nessuna implicazione di carattere linguistico e non esprimono alcun suono dialettale. Sono una scelta puramente grafica che prende spunto direttamente dalle iscrizioni lapidee romane di Aquileia.\nCordiali saluti.\nGrazie per il suo interessamento ed il suo contributo\nFilo diretto con i cittadini\n Ufficio relazioni con il pubblico\n Valter Cabas - URP\n regurp.ud@regione.fvg.it \nhttp://filodiretto.regione.fvg.it\n\n", "topic": "italian", "url": "https://italian.stackexchange.com/questions/6380"}, {"image": "VQAonline_00022349.png", "question": "Sulla pronuncia del nome di Natalia Ginzburg", "context": "Ho sempre avuto il dubbio su come pronunciare il nome dell'autrice del Lessico famigliare: dovrei dire Natal\u00eca oppure Nat\u00e0lia?\nPer influenza delle mie lingue, che hanno una pronuncia diversa tra di loro di questo nome, ma coincidono nella posizione dell'accento tonico, anche in italiano avevo sempre messo l'accento sulla seconda \"a\", pronunciando Nat\u00e0lia, ma non sono sicura se con o senza dittongo in \"ia\".\nHo chiaramente presente, per\u00f2, che, in un corso della Scuola ufficiale di lingue di Barcellona, un fiorentino esperto di letteratura ci disse a proposito di Natalia Ginzburg che la pronuncia standard in italiano fosse Nat\u00e0lia (sulla presenza o no del dittongo non mi ricordo bene).\nIl problema \u00e8 che, in una recente discussione sul Lessico famigliare, un'italiana, anche lei esperta di letteratura, ha affermato che gli italiani pronunciano questo nome nella forma Nat\u00e0lia (mi pare che con dittongo su \"ia\", ma non ne sono del tutto sicura).\nHo cercato il\u00a0nome \"Natalia\" sul Dizionario d\u2019ortografia e di pronunzia della RAI, ed ecco quello che sono riuscita a trovare:\n\nA dire il vero, la spiegazione del DOP non \u00e8 del tutto chiara: per qualche ragione a volte si preferisce la pronuncia \"alla russa\"? E poi la voce \"Natalio\" con cui si dovrebbe confrontare non si trova.\nCome pronunciate o avete sentito pronunciare voi questo nome? Potreste aiutarmi a risolvere il mio dubbio?\n", "answer": "Io ho sempre sentito e detto \u201cNatal\u00eca Ginzburg\u201d, ma questo non vorrebbe dire molto.\nQuello che direi faccia testo \u00e8 che lo pronunciano con l'accento sulla \u201ci\u201d:\n\nil critico letterario Giulio Ferroni;\nil regista Luciano Salce in presenza della Ginzburg stessa;\nla scrittrice Sandra Petrignani, autrice di un libro sulla Ginzburg;\nil conduttore Luigi Silori, ancora una volta alla presenza della scrittrice;\nma soprattutto Carlo Ginzburg, grande storico e figlio di Natalia Ginzburg.\n\n", "topic": "italian", "url": "https://italian.stackexchange.com/questions/13328"}, {"image": "VQAonline_00022327.png", "question": "Is \"scancellare\" really not correct?", "context": "When I was at the elementary schools, I was taught that scancellare is not correct, and that I should always say cancellare. When I grew up, I have heard that scancellare is correct, and that the initial S is, someway, an intensifier.\nIs scancellare really correct/acceptable? If it is not in standard Italian, is it acceptable in some contexts? For example, could be acceptable as regionalism?\nLo Zingarelli 2013 reports scancellare, but it says it is lett. pop. (I guess it means it is literary and popular.) The application I use on my Mac links scancellare to cancellare, which means that, looking for scancellare, I get the description for cancellare with just a small reference to scancellare. \n\n\n\nWhat I am asking is if it is a word I could use in a formal context, during school tests, or similar contexts.\n", "answer": "I would avoid it in anything but informal conversation; the correct Italian form is \"cancellare\", and in my experience using \"scancellare\" in formal contexts (or even just in written form) is definitely inappropriate.\nMaybe I'm biased because I heard it often in elementary school, but I'd say that, more than a \"popular\" word, it feels like a childhood word, like other dubious intensives (I remember that at some point we all called the pencil sharpener \"attemperamatite\" instead of \"temperamatite\" :) ).\n", "topic": "italian", "url": "https://italian.stackexchange.com/questions/1173"}, {"image": "VQAonline_00022706.png", "question": "Is \u300c\u3063\u3057\u3087\u300dpossibly a colloquial sentence ending particle?", "context": "In this chapter of Chainsawman I saw this \u300c\u3044\u3044\u3063\u3057\u3087\u300d and am confused by what it means.\n\nAre there any other particles like this? I don't see this on any lists online.\n", "answer": "I'm not familiar with the manga, but this is likely a colloquial way of saying \u300c\u3044\u3044\u3067\u3057\u3087\u3046\uff01\u300dor \u300c\u3044\u3044\u3088\u306d\uff01\u300d.\nPlease visit here for a little more background (assuming characters are speaking one of the northern dialects).\nKeep in mind, this is not a particle and not necessarily always sentence-ending either.\n", "topic": "japanese", "url": "https://japanese.stackexchange.com/questions/70003"}, {"image": "VQAonline_00022512.png", "question": "What does \u3068 in \"\u5bfe\u8c61\u3068\u3059\u308b\" mean?", "context": "In Japanese version of Magic: the Gathering cards, there is a common idiom\n\"[target description] \u3092\u5bfe\u8c61\u3068\u3059\u308b\u3002[subject] \u305d\u308c\u306b [action].\"\nFor example, in this card:\n\nthe Japanese text says \"\u30af\u30ea\u30fc\u30c1\u30e3\u30fc\uff11\u4f53\u304b\u30d7\u30ec\u30a4\u30e4\u30fc\uff11\u4eba\u3092\u5bfe\u8c61\u3068\u3059\u308b\u3002\u7a32\u59bb\u306f\u305d\u308c\u306b\uff13\u70b9\u306e\u30c0\u30e1\u30fc\u30b8\u3092\u4e0e\u3048\u308b\u3002\"\nThe english text says \"Lightning Bolt deals 3 damage to target creature or player.\"\n\nWhat does the \u3068 in the sentence mean? Wouldn't \u5bfe\u8c61\u3059\u308b work alone?\nIs there a simpler, less wordy, way to say the same thing, similar to the way that in English, we can use \"target\" as an adjective? \n\n", "answer": "\n\nWhat does the \u3068 in the sentence mean? Wouldn't \u5bfe\u8c61\u3059\u308b work alone?\n\n\nNo, \u5bfe\u8c61\u3059\u308b doesn't work alone (since \u5bfe\u8c61 is not a suru-verb).\n\uff21\u3092\uff22\u3068\u3059\u308b (or \uff21\u3092\uff22\u306b\u3059\u308b) means \"make A B\", so \u30af\u30ea\u30fc\u30c1\u30e3\u30fc\u3092\u5bfe\u8c61\u3068\u3059\u308b (or \u30af\u30ea\u30fc\u30c1\u30e3\u30fc\u3092\u5bfe\u8c61\u306b\u3059\u308b) literally means \"make a creature the target\", \"set/have a creature as the target,\" i.e., \"Your target will be a creature.\" \n\n\nIs there a simpler, less wordy, way to say the same thing, similar to the way that in English, we can use \"target\" as an adjective?\n\n\n\"Target\" as an adjective would be \u300c\u5bfe\u8c61\u3068\u306a\u308b\u300d or \u300c\u5bfe\u8c61\u306e\u300d in Japanese. \nTo literally translate \"Lightning Bolt deals 3 damage to target creature or player\" it'd be like: \n\n\u7a32\u59bb\u306f\u3001\u5bfe\u8c61\u3068\u306a\u308b / \u5bfe\u8c61\u306e\u30af\u30ea\u30fc\u30c1\u30e3\u30fc\u304b\u30d7\u30ec\u30a4\u30e4\u30fc\u306b\uff13\u30c0\u30e1\u30fc\u30b8\u3092\u4e0e\u3048\u308b\u3002\n\n... although this does not exactly mean the same thing as the original text; saying \u5bfe\u8c61\u3068\u306a\u308b/\u5bfe\u8c61\u306e\u30af\u30ea\u30fc\u30c1\u30e3\u30fc\u304b\u30d7\u30ec\u30a4\u30e4\u30fc (a creature or player that is your target) implies that you may also encounter \u5bfe\u8c61\u3068\u306a\u3089\u306a\u3044\u30af\u30ea\u30fc\u30c1\u30e3\u30fc\u304b\u30d7\u30ec\u30a4\u30e4\u30fc (a creature or player that is not your target).\n", "topic": "japanese", "url": "https://japanese.stackexchange.com/questions/35982"}, {"image": "VQAonline_00022376.png", "question": "What to call the phenomenon where a rectangle is shown because a font is missing a glyph", "context": "Is there a name to describe the situation where a particular character is shown on a computer screen in a particular font, but this font does not have a glyph for this particular character?\nUsually, the result looks like this: \u25a1\nA kind of empty rectangle. Sometimes it shows the code for that character within the rectangle.\n\nHow do yo call this phenomenon in Japanese?\nIf there is no name, is there a common term referring to this \"missing glyph square\" in Japanese?\n(It is not \u6587\u5b57\u5316\u3051. \u6587\u5b57\u5316\u3051 happens when character encoding meta-information is lost. It is not the problem here. Using a better font solves this phenomenon, but does not solve \u6587\u5b57\u5316\u3051. Selecting a character encoding solves \u6587\u5b57\u5316\u3051, but does not solve this phenomenon.)\n", "answer": "As far as Internet slang goes, the word \u8c46\u8150{\u3068\u3046\u3075} is often used as a term for replacement glyphs because of their rectangular shapes, and \u8c46\u8150[\u5316]{\u3070}\u3051 or \u8c46\u8150[\u5316]{\u304b} describe the phenomenon in one word.\n\n\n\u753b\u9762\u4e0a\u306e\u65e5\u672c\u8a9e\u304c\u3059\u3079\u3066\u8c46\u8150\u306b\u306a\u3063\u3066\u3057\u307e\u3063\u3066\u3044\u3066\u8aad\u3081\u306a\u3044\n\u30d5\u30a9\u30f3\u30c8\u8a2d\u5b9a\u3092\u5909\u66f4\u3057\u3066\u3082\u8c46\u8150\u5316\u3051\u304c\u76f4\u3089\u306a\u3044\n\n\n", "topic": "japanese", "url": "https://japanese.stackexchange.com/questions/5129"}, {"image": "VQAonline_00022975.png", "question": "Translation of Hebrew letters", "context": "I hope that this is not off-topic for this forum. If so, please let me know and I will delete it.\nI'm trying to understand the meaning of these letters which I believe are Hebrew. Can someone who is familiar with the language please provide a translation? I realize that it's a relatively poor image but it's the best I can come up with at this time.\nThank you in advance.\n\n", "answer": "Looks like it says \u05d4\u05e9\u05dd \u05dc\u05e6\u05d9\u05d3\u05d9 (Hashem is on my side/next to me), but with the letters in reverse order for some reason.\n", "topic": "judaism", "url": "https://judaism.stackexchange.com/questions/130594"}, {"image": "VQAonline_00022868.png", "question": "How does Hagbah work on a Sephardic Torah case?", "context": "How does one do Hagbah on a Torah in a Sephardic Torah case, such as one of these silver cases (picture courtesy of Hadad Brothers) :\n \n\nSee also the Ashkenazi version of this question.\n", "answer": "\nWhile it is on the bimah, turn the Torah around so that the opening is away from you and then open up the scroll. You should be looking at the back of the Torah, standing immediately next to the bimah.\nPut your right hand so that it is on the right vertical edge of the right side of the case (reverse instructions for lefties), about half way up.\nSlowly shift the Torah backwards until its edge is starting to come off of the bimah (you can support it with your chest). Once you have room to do so, stick your left hand underneath the bottom of the left side of the case (so you are supporting it underneath with your left hand, and on the side with your right).\nLet the Torah come the rest of the way off the bimah and support it completely with your hands and chest. Try to lift it up higher if you can.\nTurn around in a circle (turning to your right). And then carefully put it back down.\n\nCaution: don't try this unless you have some measure of physical strength. This is not like an Ashkenazi torah where balance is the key. \nAlso, this has nothing to do with the silver case - it is related to the Sefardi-style Torah's (many of which will not have a silver case like the one shown above).\n", "topic": "judaism", "url": "https://judaism.stackexchange.com/questions/10195"}, {"image": "VQAonline_00022944.png", "question": "Does anybody recognize this kashrus symbol? Rectangle K US", "context": "\nAs the title says, does anybody know this kosher symbol? It looks like a large K with the letters U and S on top and bottom of it, inside of a rectangle. The product brand is Super Cakes, subsidiary of Super Bread. A lot of their other products have this symbol, but I can't seem to find it online.\n", "answer": "It looks like this is the trademark of \"United Kosher Supervision\", which is apparently headed by a R' Yaakov Spivak of Monsey, NY.\n", "topic": "judaism", "url": "https://judaism.stackexchange.com/questions/103803"}, {"image": "VQAonline_00022979.png", "question": "Why is \"\ub824\uad8c\" North Korean for \"passport?\"", "context": "I just noticed the North Korean passport says \"\ub824\uad8c\". Indeed, Naverdic says \"\ub824\uad8c\" is North Korean for \"\uc5ec\uad8c\". Apparently both words use the same hanja, just pronouncing it differently.\nCan anyone explain the history/reason behind this difference?\n\n", "answer": "As I explained in the linked answer Why is the Korean name \uc774 often Romanised as 'Lee'?, there is\n'\ub450\uc74c\ubc95\uce59 (First Initial Sound Rule / Law)' in Korean:\n\n<\uc5b8\uc5b4> \uc77c\ubd80 \uc18c\ub9ac\uac00 \ub2e8\uc5b4\uc758 \uccab\uba38\ub9ac\uc5d0 \ubc1c\uc74c\ub418\ub294 \uac83\uc744 \uaebc\ub824 \ub2e4\ub978 \uc18c\ub9ac\ub85c \ubc1c\uc74c\ub418\ub294 \uc77c. \u2018\u3163, \u3151, \u3155, \u315b, \u3160\u2019 \uc55e\uc5d0\uc11c\uc758\n\u2018\u3139\u2019\uacfc \u2018\u3134\u2019\uc774 \u2018\u3147\u2019\uc774 \ub418\uace0, \u2018\u314f, \u3153, \u3157, \u315c, \u3161, \u3150, \u3154, \u315a\u2019 \uc55e\uc758 \u2018\u3139\u2019\uc740 \u2018\u3134\u2019\uc73c\ub85c \ubcc0\ud558\ub294 \uac83 \ub530\uc704\uc774\ub2e4.\n[\ube44\uc2b7\ud55c \ub9d0] \uba38\ub9ac\uc18c\ub9ac \ubc95\uce59.\n\ubd80\uac00\uc815\ubcf4 : \ud55c\uae00 \ub9de\ucda4\ubc95 \uc81c5 \uc808 \uc81c10 \ud56d\uc5d0\uc11c \uc81c12 \ud56d\uc5d0 \ub530\ub974\uba74 \u201c\ud55c\uc790\uc74c \u2018\ub140, \ub1e8, \ub274, \ub2c8\u2019\uac00 \ub2e8\uc5b4 \uccab\uba38\ub9ac\uc5d0 \uc62c \uc801\uc5d0\ub294\n\ub450\uc74c \ubc95\uce59\uc5d0 \ub530\ub77c \u2018\uc5ec, \uc694, \uc720, \uc774\u2019\ub85c \uc801\uace0, \u2018\ub7b4, \ub824, \ub840, \ub8cc, \ub958, \ub9ac\u2019\uac00 \ub2e8\uc5b4\uc758 \uccab\uba38\ub9ac\uc5d0 \uc62c \uc801\uc5d0\ub294 \u2018\uc57c, \uc5ec,\n\uc608, \uc694, \uc720, \uc774\u2019\ub85c \uc801\uc73c\uba70, \u2018\ub77c, \ub798, \ub85c, \ub8b0, \ub8e8, \ub974\u2019\uac00 \ub2e8\uc5b4\uc758 \uccab\uba38\ub9ac\uc5d0 \uc62c \uc801\uc5d0\ub294 \u2018\ub098, \ub0b4, \ub178, \ub1cc, \ub204,\n\ub290\u2019\ub85c \uc801\ub294\ub2e4. \uc608\ub97c \ub4e4\uc5b4 \u2018\uc5ec\uc790(\uf981\u5b50), \uc5f0\uc138(\uf98e\u6b72), \uc694\uc18c(\uf9bd\u7d20), \uc720\ub300(\uf9cf\u5e36), \uc774\ud1a0(\uf9e3\u571f), \uc775\uba85(\uf9eb\u540d)\u2019\uc740 \u2018\ub140\uc790,\n\ub144\uc138, \ub1e8\uc18c, \ub274\ub300, \ub2c8\ud1a0, \ub2c9\uba85\u2019\uc774 \uc544\ub2cc \u2018\uc5ec\uc790, \uc5f0\uc138, \uc694\uc18c, \uc720\ub300, \uc774\ud1a0, \uc775\uba85\u2019\uc73c\ub85c \uc801\ub294 \ub530\uc704\uc774\ub2e4.\n\nShort summary:\n\nWhen consonants such as '\u3139' and '\u3134' are placed at the first block of a\nword as an initial consonant, it is pronounced in a different way. For\nexample:\n\nIn front of '\u3163, \u3151, \u3155, \u315b, \u3160' , '\u3134' and '\u3139' are pronounced as '\u3147': \ub140\uc790 -> \uc5ec\uc790, \ub824\uad8c -> \uc5ec\uad8c\n\nIn front of '\u314f, \u3153, \u3157, \u315c, \u3161, \u3150, \u3154, \u315a' , '\u3139' is pronounced as '\u3134': \ub77d\uc6d0 -> \ub099\uc6d0, \ub85c\ub3d9 -> \ub178\ub3d9\n\n\n*This rule applies to only Chinese characters and doesn't apply to original Korean words such as '\ub9ac\uc744 (\u3139)' and '\ub140\uc11d', etc. Sometimes, it applies to a compound noun such as '\uc2e0\uc5ec\uc131', '\ub0a8\uc874\uc5ec\ube44', '\ub0a8\ub140\ub178\uc18c' and '\uc911\ub178\ub3d9', etc.\n\nHowever, North Korea has ignored this rule and written '\ub85c\ub3d9 (Labor)' instead of '\ub178\ub3d9' and '\ub140\uc790 (woman)' instead of '\uc5ec\uc790'.\n'\ub824\uad8c' is another case where North Korea doesn't follow this rule. It is '\uc5ec\uad8c' in South Korea. There is no such rule in North Korea.\n", "topic": "korean", "url": "https://korean.stackexchange.com/questions/538"}, {"image": "VQAonline_00022988.png", "question": "How to understand the feel and use of written sentences without main verbs?", "context": "I am currently reading existential masterpiece '\ub9ac\ub77d\ucfe0\ub9c8\uc758 \ud558\ub8e8'.\n\nHere's the first page:\n\ub9ac\ub77d\ucfe0\ub9c8\uc758 \uce5c\uad6c\ub4e4\n\n\ubbf8\uc2a4 \uce74\uc624\ub8e8 \uc2dc\ub0b4\uc5d0\uc11c \ud68c\uc0ac\uc6d0\uc73c\ub85c \uc77c\ud558\ub294 \uc5ec\uc131. \ub9e4\uc77c \uc794\uc5c5\uc73c\ub85c \ud53c\uace4\ud55c \ub098\ub0a0\ub4e4\uc744 \ubcf4\ub0b8\ub2e4. \ud3b8\uc758\uc810\uc5d0\uc11c '\ubb50, \uc0c8\ub85c\uc6b4 \uac70 \uc5c6\ub098' \uc0b4\ud53c\ub294 \uac74, \ub2e4\uc774\uc5b4\ud2b8 \ud560\uae4c \ub9d8\uba39\uc744 \ub54c\ub9c8\ub2e4 \ubc18\ubcf5\ub418\ub294 \uc77c\uc0c1. \uc5b4\ub290 \ub0a0 \ud63c\uc790\n \uc0ac\ub294 \uc544\ud30c\ud2b8\uc5d0 \ub3cc\uc544\uc624\ub2c8 \ucfe0\ub9c8\uac00 \uc788\uc5c8\ub2e4. \uc544\ub07c\ub294 \uad6c\uc2ac \ucfe0\uc158\uc744 \ucfe0\ub9c8\uc5d0\uac8c \ube7c\uc557\uae30\uae30\ub3c4 \ud558\uace0 \ucfe0\ub9c8\uc758 \uc7a5\ub09c\uae30 \ub54c\ubb38\uc5d0 \n \ud798\ub4e4\uc5b4 \ud558\uba74\uc11c\ub3c4 \ubbf8\uc6cc\ud558\uc9c0 \uc54a\uace0 \ucfe0\ub9c8\ub97c \ub3cc\ubd10\uc900\ub2e4.\n\n...\n\n\ub9ac\ub77d\ucfe0\ub9c8 \uac11\uc790\uae30 \ubbf8\uc2a4 \uce74\uc624\ub8e8\uc758 \uc544\ud30c\ud2b8\uc5d0 \ub098\ud0c0\ub09c \ucfe0\ub9c8. \ub4f1\uc5d0 \uc9c0\ud37c\uac00 \ub2ec\ub9b0 \uac78\ub85c \ubd10\uc11c \uac89\uac00\uc8fd\uc744 \uc885\uc885 \uac08\uc544\uc785\ub294 \ub4ef. \uc0ac\uc0c1 \ucd5c\uac15\uc758 \uadc0\ucc28\ub2c8\uc2a4\ud2b8\ub2f5\uac8c \ub9e4\uc77c\uac19\uc774 \uc124\ub801\uc124\ub801 \ub4b9\uad74\ub4b9\uad74\ud558\ub294 \uc911. \uc990\uaca8 \uba39\ub294 \uac74 \ud56b\ucf00\uc774\ud06c, \uc644\uc790,\n \uc624\ubbc0\ub77c\uc774\uc2a4, \ud478\ub529. \ub208\ub3c5 \ub4e4\uc774\uace0 \uc788\ub294\uac74 \ubbf8\uc2a4 \uce74\uc624\ub8e8\uc758 \uae08\uc0c9 \uad6c\uc2ac \ucfe0\uc158. \uc88b\uc544\ud558\ub294 \uac74 \uc74c\uc545\ub4e3\uae30, TV\ubcf4\uae30,\n \uc628\ucc9c\uc695\ud558\uae30.\n\n...\n\n\ub178\ub780\ubcd1\uc544\ub9ac \ubbf8\uc2a4 \uce74\uc624\ub8e8\ub294 \ub530\ub85c \uc774\ub984\uc744 \ubd99\uc774\uc9c0 \uc54a\ub294 \ub4ef...? '\ub178\ub791\ubcd1\uc544\ub9ac'\ub85c \ubd88\ub9ac\ub294 \ub178\ub780\uc0c9\uc758 \ubcd1\uc544\ub9ac. \ub300\uccb4\ub85c \uc0c8\uc7a5\uc5d0 \ub4e4\uc5b4\uac00 \uc788\uc9c0\ub9cc, \uc885\uc885 \uc81c\uba4b\ub300\ub85c \ub098\uc640\uc11c \ub9ac\ub77d\ucfe0\ub9c8\ub97c \uae68\uc6b4\ub2e4. \ucd5c\uadfc\uc758 \ud070 \uc1fc\ud06c\ub294 \ub9ac\ub77d\ucfe0\ub9c8 \ub4f1\uc5d0\n \ub2ec\ub9b0 \uc9c0\ud37c\uc758 \uc5f4\ub9b0 \ud2c8\uc0c8\ub85c \ubb3c\ubc29\uc6b8\ubb34\ub2ac \uac19\uc740 \ud53c\ubd80\ub97c \ubaa9\uaca9\ud55c \uc77c\uc774\ub2e4.\n\nI was intrigued how many sentences there (italicized above) don't seem to have main verbs in. \nSome of these might be written in a similarly verb-free way in English - e.g. the ones that are just like a noun that's an alternative identification, like '\uc2dc\ub0b4\uc5d0\uc11c \ud68c\uc0ac\uc6d0\uc73c\ub85c \uc77c\ud558\ub294 \uc5ec\uc131'.\nOther sentences seem very long to not be completed by a verb, e.g. \ud3b8\uc758\uc810\uc5d0\uc11c '\ubb50, \uc0c8\ub85c\uc6b4 \uac70 \uc5c6\ub098' \uc0b4\ud53c\ub294 \uac74, \ub2e4\uc774\uc5b4\ud2b8 \ud560\uae4c \ub9d8\uba39\uc744 \ub54c\ub9c8\ub2e4 \ubc18\ubcf5\ub418\ub294 \uc77c\uc0c1.\nIs this a common way to write in Korean, or is it something to do with it being a translation from Japanese?\nIf it's something that is encountered in Korean, how should I understand the 'feel'? Does it have a shorthand / note-taking / diary style?\n", "answer": "To me, the text looks as though it needed pictures to go with it.\nComic book bubbles often contain sentence fragments because, I believe, the picture is then supposed to complete the thought. For instance, 'After the fall of the Empire. . .' with a picture of some smoldering ruins, to mean, 'Things looked like this.' So when the text stops at 'Miss Kaoru,' 'a woman with a desk job in the city,' or 'each day fatigued with unfinished work,' I feel as though I ought to stop and picture something to myself.\nA diary would be a good comparison too. There we often find fragments, or a succession of memorable sights. I think the idea is that the author can say more, but since he is writing only to himself he need not.\nOr compare how one may react to this (from Camus's The Stranger):\n\nAnd I can remember the look of the church, the villagers in the street, the red geraniums on the graves, P\u00e9rez\u2019s fainting fit\u2014he crumpled up like a rag doll\u2014the tawny-red earth pattering on Mother\u2019s coffin, the bits of white roots mixed up with it; then more people, voices, the wait outside a caf\u00e9 for the bus, the rumble of the engine, and my little thrill of pleasure when we entered the first brightly lit streets of Algiers, and I pictured myself going straight to bed and sleeping twelve hours at a stretch.\n\nTo some readers, the style (of the Korean text) may just look distracted, or sloppy, or--worse yet--trying to achieve some result without working for it.\nOn the suggestion in the other answer, I don't think you should see these fragments as combining to form whole sentences. That may be a useful grammatical exercise, but also may defeat the whole purpose.\nOn the comments now, yes, there is a poetic feel to this type of text probably because we now associate poetry with greater freedom. (But isn't that just modern poetry?)\nThere would be implied \uc774\ub2e4 or \uc788\ub2e4 or other things in these fragments in the sense that they are capable of acquiring it and thus turning into well-formed sentences, but probably not in the sense that the speaker meant it. (In 'Are you hungry?' - 'I am,' we may say that the second person meant 'hungry.')\nI don't believe the style is common outside comic books.\n", "topic": "korean", "url": "https://korean.stackexchange.com/questions/2360"}, {"image": "VQAonline_00023004.png", "question": "Correct usage of '~\ub2e4' and '~\uc774\ub2e4'", "context": "Of course, '~\uc774\ub2e4' is followed by a noun ends with a letter with a final consonant whereas '~\ub2e4' is followed by a noun ends with a letter without final consonants. Let this rule be (*) for reference. For example, '\uc774\uac74 \ud2b8\ub7ed\uc774\ub2e4' which is correct whereas '\uc774\uac74 \ud2b8\ub7ed\ub2e4' which is wrong.\nAnother example is '\uc774\uac74 \ubc84\uc2a4\ub2e4' is correct whereas '\uc774\uac74 \ubc84\uc2a4\uc774\ub2e4' which looks somewhat strange. The following is the first question.\n\nQuestion 1: Is the sentence '\uc774\uac74 \ubc84\uc2a4\uc774\ub2e4' wrong?\n\nEspecially, in academic writing, '~\ub2e4' seems to be avoided. For example, in almost all mathematics textbooks in Korean, we see a sentence like '\uc774\uac83\uc740 \ud568\uc218\uc774\ub2e4' but not '\uc774\uac83\uc740 \ud568\uc218\ub2e4'. The figure following is the final question of math type \uac00 of \ub300\ud559\uc218\ud559\ub2a5\ub825\uc2dc\ud5d8(something like Korean SAT). We can see the phrase '\uadf9\uc18c\uc774\uace0' which should be '\uadf9\uc18c\uace0' according to the rule (*).\n\nSo the next question is\n\nQuestion 2 Is the rule () correct? If it is correct, should we write '\uadf9\uc18c\uace0' in place with '\uadf9\uc18c\uc774\uace0'? If the rule () is not correct, is there any difference between '\uadf9\uc18c\uace0' and '\uadf9\uc18c\uc774\uace0'? Can we say which one is recommanded?\n\n", "answer": "The rule set you described \u2015 \u201cAlpha\u201d\n\nYou have to use \u201c-\uc774\ub2e4\u201d after a consonant.\nYou have to use \u201c-\ub2e4\u201d after a vowel.\n\nis wrong.\n\nThe right rule set \u2015 \u201cBeta\u201d\n\nYou have to use \u201c-\uc774\ub2e4\u201d after consonant.\nYou may use either \u201c-\uc774\ub2e4\u201d or \u201c-\ub2e4\u201d after a vowel.\n\n\n\u201c-\uc774\ub2e4\u201d is the basic form. Sounds formal, because you do not elide the \u201c\uc774\u201d when you can. Used in academic or formal writings, often in the indicative mood.\n\u201c-\ub2e4\u201d is the contraction. Used when you don't use \u201c-\uc774\ub2e4.\u201d Slightly casual compared to \u201c-\uc774\ub2e4,\u201d but not too casual. It's just usual than \u201c-\uc774\ub2e4.\u201d\n\n\n\n\nQuestion 1 | Is the sentence \u201c\uc774\uac74 \ubc84\uc2a4\uc774\ub2e4\u201d wrong?\n\nNo, it's possible construction. Doesn't sound off, but sounds like you're saying a proposition.\n\nQuestion 2-1 | Is the rule correct?\n\nNo.\n\nQuestion 2-2 | If the rule is not correct, is there any difference between \u201c\uadf9\uc18c\uace0\u201d and \u201c\uadf9\uc18c\uc774\uace0?\u201d Can we say which one is recommended?\n\n\n\u201c\uadf9\uc18c\uace0\u201d for everyday speech and writing, in your workplace, home, school, wherever.\n\u201c\uadf9\uc18c\uc774\uace0\u201d for academic writing, when you're describing your math questions.\n\nNot sure which to use? Stick to \u201c\uadf9\uc18c\uace0\u201d and everything's good.\n", "topic": "korean", "url": "https://korean.stackexchange.com/questions/5068"}, {"image": "VQAonline_00023084.png", "question": "About Latin Translation of Quran", "context": "Robert of Ketton's Latin Translation of Quran (4:157):\n\nseque Christum Mariae filium Dei nuncium peremisse perhibent. Sed nequaquam eum, imo sui similem suspendentes interfecerunt. profites etiam se suae caedis authores, cordibus suis non minimam ambiguitatem inde gerunt. Sed eum nullatenus interfecerunt\n\nMy translation:\n\nAnd they say that they killed Christ son of Mary, God\u2019s messenger. But by no means they killed him. On the contrary they hanged similar of him. You declare that they are the performers of his murder. Therefore in their hearts they do not have any uncertainty. But by no means they killed him.\n\nI'm not certain about my translation, specially of profites etiam se suae caedis authores.\n", "answer": "Like fdb said, it should be 'profitentes' instead of 'profites'.\nNicolaus Cusanus cited Ketton's translation like this: \"Profitentes etiam se suae caedis authores cordibus suis non minimam ambiguitatem inde gerunt, [sed eum nullatenus interfecerunt]\" (https://books.google.nl/books?id=mQ-KDwAAQBAJ&pg=PA46).\nTranslation by Hagemann and Glei: \"Selbst wenn sie sich als Anstifter seiner Ermordung erkennen, haben sie in ihren Herzen nicht geringe Zweifel; [get\u00f6tet jedoch haben sie ihn keineswegs.]\" \nNote also that 'non minimam ambiguitatem' is not translated as 'not the least/smallest bit of doubt', but rather as 'not (very) little/small doubt', i.e. litotically: 'considerable doubt'. A Google search for 'non minimam' seems to confirm that it tends to mean the same as 'non parvam'.\nSo it would be: \"Even when declaring themselves to be the instigators of his murder, they have no little doubt in their hearts about it; [yet by no means did they kill him.]\"\nThis is an adequate paraphrase or summary of the Arabic:\n\n\u0648\u064e\u0625\u0650\u0646\u0651\u064e \u0671\u0644\u0651\u064e\u0630\u0650\u064a\u0646\u064e \u0671\u062e\u06e1\u062a\u064e\u0644\u064e\u0641\u064f\u0648\u0627\u0652 \u0641\u0650\u064a\u0647\u0650 \u0644\u064e\u0641\u0650\u0649 \u0634\u064e\u0643\u0651\u064d\u06ec \u0645\u0651\u0650\u0646\u06e1\u0647\u064f\u200c\u06da \u0645\u064e\u0627 \u0644\u064e\u0647\u064f\u0645 \u0628\u0650\u0647\u0650\u06e6 \u0645\u0650\u0646\u06e1 \u0639\u0650\u0644\u06e1\u0645\u064d \u0625\u0650\u0644\u0651\u064e\u0627 \u0671\u062a\u0651\u0650\u0628\u064e\u0627\u0639\u064e \u0671\u0644\u0638\u0651\u064e\u0646\u0651\u0650\u200c\u06da [\u0648\u064e\u0645\u064e\u0627 \u0642\u064e\u062a\u064e\u0644\u064f\u0648\u0647\u064f \u064a\u064e\u0642\u0650\u064a\u0646\u064e\u06e2\u0627]\n\nWhich in Marracci's more literal Latin is translated: \"& profect\u00f2 qui discordes fuerunt circa eum; esset ne ipse, an alius similis ipsi; cert\u00e8 fuerunt in dubitatione de hoc (de vera morte ejus) non fuit illis de hoc ulla scientia, sed tant\u00f9m sectatio opinionis. [Et non occiderunt eum ver\u00e8]\" (https://books.google.nl/books?id=IQdRAAAAcAAJ&pg=RA1-PA171, under verse 156).\nSo the Quran's \"\u0648\u064e\u0625\u0650\u0646\u0651\u064e \u0671\u0644\u0651\u064e\u0630\u0650\u064a\u0646\u064e \u0671\u062e\u06e1\u062a\u064e\u0644\u064e\u0641\u064f\u0648\u0627\u0652 \u0641\u0650\u064a\u0647\u0650\" (\"and indeed those who differ in this matter/concerning him\") and Marracci's \"& profect\u00f2 qui discordes fuerunt circa eum\" correspond to Ketton's \"Profitentes etiam se suae caedis authores\". \n", "topic": "latin", "url": "https://latin.stackexchange.com/questions/13808"}, {"image": "VQAonline_00023041.png", "question": "What does the Latin abbreviation \"Form.\" stand for?", "context": "I came across a medal commemorating the coronation of Charles XV in 1860. The inscriptions on the medal are all in Latin. I am aware of a few latin abbreviations centered around the creators of medals like these:\nFEC. = fecit\nSC. = sculpsit\nDIR. = direxit\nOne of the inscriptions on the medal is sculpsit:\n\nJ. E. ERICSSON SC.\n\nBut there is another abbreviation on this medal that I am not familiar with:\n\nC. G. OVARNSTR\u04e6M FORM.\n\nWhat Latin verb is this supposed to be?\nMy guess would be \"format\" but how is that different than \"sculpsit\"? I have included a picture that I took of the medal for reference:\n\n", "answer": "To me it seems that formavit here means that someone moulded (minted, coined) the medal based on what someone else sculpted (sculpsit) first.\nThis dictionary supports the idea of formare meaning to mould.\nNote that the dictionary was edited in the XVIII Century, while L&S doesn't even list the verb formare (I think formam dare is more classical.)\nFrom what I can understand from this article (and what I remember,) coins and medals are designed by sculptors, leaving the serial production to mints, so I would say that Ericsson was the designer and Ovarnstr\u00f6m was in charge of the minting process.\n", "topic": "latin", "url": "https://latin.stackexchange.com/questions/4375"}, {"image": "VQAonline_00023083.png", "question": "Can I conclude something about pronunciation from I/J and U/V variation?", "context": "I reread the inscription in this question about the abbreviation D. O. M. dated to 1749:\n\nThe variation between I and J caught my eye.\nIt seems to me that:\n\nJ is used in all consonantal positions.\nJ is used in hujus.\nJ is used after I: varijs, munijs, junij.\nIn all other positions I is used, whether it is (classically) short or long.\n\nU and V behave in a similar way:\nU is used in all vocalic positions, V in all consonantal ones.\nThe only (perhaps) exception is that after Q we always have U.\nThe spelling choices seem very consistent, so I can't attribute this to chance.\nThis seems to suggest that the U after Q is pronounced as a vowel, and the J in hujus, varijs, munijs, junij is pronounced as a consonant.\nIs this conclusion justified?\nAll other make sense to me, but varijs, munijs, junij sound weird to me.\nIs it indeed a matter of spelling convention where no two consecutive Is can appear.\n", "answer": "One of your conclusions is, I feel, justified: that the j in hujus is due to it being pronounced as a consonant/semivowel, since that was always the case. More than that: from Classical times (and actually earlier than that) almost all instances of consonantal i between vowels were geminated, since intervocalic short /j/ was lost at an earlier stage (e.g. PIE *tr\u00e9yes > Proto-Italic *tr\u0113s; the exceptions are all compound words where the second part starts with /j/), so huius was /\u02c8huj.jus/. This is still the case in Church Latin today (/\u02c8uj.jus/), so the only question to my mind isn't about j, but if the h is pronounced or not (presumably not).\nii being written as ij I wouldn't consider evidence of anything other than a desire to be easier to read. At least since the Middle Ages people have occasionally (and at points frequently) written the final i in Roman numerals ending in runs of them as j, and in, for example, Middle Dutch /i\u02d0/ came to be written rather than for aesthetic reasons as well. It's just a common ligature.\nqu has always been an odd duck, but I see no reason to believe the u there suggests that that part of the sound came to be pronounced as a vowel so much as that it didn't become fricative like consonantal u in other positions; i.e. the normal state of affairs.\n", "topic": "latin", "url": "https://latin.stackexchange.com/questions/13721"}, {"image": "VQAonline_00023137.png", "question": "Costa charging different prices for same item", "context": "Last week I stopped at services and ordered in Costa two coffees. One Caramel Cappuccino (\u00a33.89) and one Speciality Vanilla (\u00a33.99). When I got to pay and bill was \u00a38.18 I figured something must have gone wrong since it should not exceed \u00a38. It turns out they have charged me Cappuccino (\u00a33.59) + Caramel (\u00a30.50) and another Cappuccino (\u00a33.59) + Vanilla (\u00a30.50) that totalled up to \u00a38.18. \n\nI then asked is there a difference in preparation of drinks (Caramel Cappuccino vs Cappuccino + Caramel and Speciality Vanilla vs Cappuccino + Vanilla), and there was no difference, they are exactly same drink that are being charged differently depending on how cashier feels like. \nAre they allowed by law to charge different prices for same things?\n", "answer": "In common law contract law, the price tag or posted price is only an invitation to treat. The offer and acceptance of the offer happen at the cash register. You are free to back out of the deal when you become aware of the higher price at the register.\nBarring a consumer protection law that forces vendors to honour listed prices, common law contract law applies.\nIt is hard to prove a negative and I have limited exposure to UK law, but these two articles lead me to believe that the UK doesn't have a law requiring vendors to honour posted prices:\n\nhttp://www.telegraph.co.uk/finance/personalfinance/money-saving-tips/10602641/Price-glitches-Do-retailers-have-to-honour-pricing-mistakes.html\n\n\nIf an item is priced incorrectly on the shelf, or scans at the wrong price at the till, retailers are under no obligation to honour it, under the Sale of Goods Act.\n\n\nhttp://www.theguardian.com/money/2012/sep/10/does-shop-honour-price-shelf\n\n\nprice tag is not a contract. It is an \"invitation to treat\" ie it is inviting the customer to make an offer to purchase and the retailer doesn't have to accept that offer. A contract is only formed when the shop accepts a payment\n\n", "topic": "law", "url": "https://law.stackexchange.com/questions/4340"}, {"image": "VQAonline_00023217.png", "question": "How can you legally openly carry rifles to the Michigan governor's office?", "context": "I'm British, and was befuddled to see this picture on /r/Cringetopia of lockdown protestors carrying guns right up to the Michigan's Governor office door. How's this legal?\nThere are just so many dangers to this open carry. Did the security at Gov Gretchen Whitmer's office tail these protestors? What if Gov. Whitmer doesn't have enough bodyguards, or if they don't have enough firepower fight off these protesters, if they start shooting with their rifles? \n\nAre these Reddit comments correct?\n\nthemosey. \n 42 points 23 hours ago \nHonest question: are they allowed to carry guns in the statehouse, legally? Most states have metal detectors and laws against it (not from Michigan just following the news).\nSame for \u201cdemanding\u201d to see the governor and the house floor.\nAnd if it is illegal (and they are straight up terrorists) why aren\u2019t they arrested? They can call it civil disobedience but that still makes them terrorists.\nuniballout. 35 points 22 hours ago\nYes, they are. I think a few years ago they had a bill to stop it. Instead they voted to allow guns, but prohibit protest/political signs. My guess is the black marker and poster board lobbies couldn\u2019t outspend the gun lobbies. So now we have AR-15s waving at our legislators as they vote.\n\nKingslyman. 946 points 1 day ago \nProtestors are somehow allowed to carry guns right up to the Michigan's Governor office door. : pics\n\nThey are \u201callowed\u201d to carry guns in the governor\u2019s office because their own laws allow for it\n\u201cThe Michigan State Police describe Michigan's open carry law as follows: \"In Michigan, it is legal for a person to carry a firearm in public as long as the person is carrying the firearm with lawful intent and the firearm is not concealed. You will not find a law that states it is legal to openly carry a firearm.\u201d\n\n", "answer": "The basic reason that they are allowed to carry weapons is the First and Second Amendments, meaning that you can express yourself, and you can carry weapons. There are various minimal limitations on the right to free speech, so that you can't threaten a person's life; there are more limits on the right to bear arms. In both cases, laws limiting the exercise of those fundamental rights must be restrained, and a law against openly carrying guns to a political protest would be subject to \"strict scrutiny\", meaning that there would have to be a compelling reason to infringe on these rights. The legal rule, here and in the UK, is that it's legal unless it is specifically illegal. That is not to say that Michigan could not pass a law outlawing the carrying of weapons in government buildings, or outlaw political protests right at the door of the governor's office,\nbut until they do, it's legal. As to why certain states have enacted laws outlawing open carry and potentially-disruptive protests, that's mostly a political question rather than a legal one.\n", "topic": "law", "url": "https://law.stackexchange.com/questions/51217"}, {"image": "VQAonline_00023155.png", "question": "Can I used an ASCII generated version of a copyrighted image in my game?", "context": "I'm going to guess the likely answer is no, but I'm interested to know more. So let's say I have an image of James Bond from Goldeneye. \n\nThen let's say I run it through ASCII art generator such as this http://picascii.com/ and it produces the following ASCII code made for the web as such (can't post here but here is a JSFiddle of the generated code).\nhttps://jsfiddle.net/caimen/ope1j9w0/ \nWhy or why would I not be allowed to use this version of a copyrighted image in my game?\n", "answer": "The reason is 17 USC 106: \n\nthe owner of copyright under this title has the exclusive rights to do\n and to authorize any of the following...\n(2) to prepare derivative works based upon the copyrighted work\n\nThe original picture is the underlying protected work. The ASCII reproduction is a derivative work. If you get permission to make the derivative work, it is okay. Otherwise, it is copyright infringement. There is an escape clause, \"fair use\", which amounts to taking a chance that you won't be sued and then arguing that you didn't do them any prohibited harm. If you make any money off of the game, you have a major strike against you. I suggest reading the fair use FAQ; basically, it is really hard to know how a fair use defense will fare, but based on prior cases, I'd say it's infringement, not fair use.\n", "topic": "law", "url": "https://law.stackexchange.com/questions/16376"}, {"image": "VQAonline_00023274.png", "question": "How can I remove the bag from my bin more easily?", "context": "When removing a plastic bin bag from the bin, the vacuum created and subsequent air pressure makes it difficult to remove. \nWith drilling holes into the bottom to let air in as a last resort as this would cause problems cleaning the bin, what else can I do? \nThe bin is a metal pedal bin, similar to this one. Inside there is a hard plastic receptacle, into which I put my bin liner.\n\n", "answer": "If you don't want to let air in from the bottom with holes, you need to somehow let air in from the top. \nAn easy way to do this is by placing a hollow tube/pipe in the bin. Before you put the liner in, place the tube against the side of the bin (it should be tall enough to reach from bottom to top without preventing it from closing). \nThis could be something rigid like PVC, or flexible like a hose. You'll lose a very slight bit of capacity, but it will prevent (or at least lessen, depending on the diameter of the tube) the vacuum effect in the bottom of the bin.\nTry it out a couple times. If it works well for you, you should be able to hack up a way to mount it in the bin permanently.\n", "topic": "lifehacks", "url": "https://lifehacks.stackexchange.com/questions/1671"}, {"image": "VQAonline_00023324.png", "question": "How to avoid background reflection Computer display", "context": "At work I am sitting in front of a conference room which has lots of glass wall. It is always very bright and it makes my computer display look like a mirror. I tried playing with my display setting but it does not help.\nI can not change anything in room.\nAny thing which can be done to make it better for me..\n\n", "answer": "Tell your employer they are likely breaking health and safety rules regarding screen glare and need to do something about it, ideally a blind on the window would be the best (as then you can also adjust it as needed).\n", "topic": "lifehacks", "url": "https://lifehacks.stackexchange.com/questions/9768"}, {"image": "VQAonline_00023302.png", "question": "How to keep dust-free when digitizing photos", "context": "I recently purchased a photo digitizer which looks like this:\n\nUnder the cover there was a clear plastic surface upon which the photos are placed so that they can be digitized. (Unit takes a picture of the photo with an internal high res digital camera and writes it to an SD card). The plastic surface was very hard to deal with because it was very static prone and it was hard to place and remove the photos. So I have replaced the original surface with a piece of glass cut to the proper shape and sanded smooth on the edges and rounded the corners. \nThe glass works great except for one problem (that was the same in the plastic surface). Any dust that may be on photos that one wants to scan get onto the glass and is very, very hard to wipe away. I am looking for the following:\n\nEasy method or technique to remove accumulated dust from the glass surface where it will stay off. Presently it just seems to smear around and is very hard to get totally clean.\nA surface conditioner that could be applied to the glass to make it much less able to act like a dust magnet.\nA way to pre-wipe old photos that may be dusty before placing them on the scanner.\n\n", "answer": "Since you used real glass, you can really clean it. I used acetone to clean the scanner glass when it had fogged from outgassing of plastics inside the scanner. \nSo when you get something onnthe glass that doesn't wipe off easily, remove the glass and clean it in the dishwasher or with acetone or whatever. It is not limited to a dainty little damp rag!\nFor material to be scanned, use compressed air to blow off dust. Use a microfiber cloth to wipe away finger oils: typical microfiber cleaning cloths have both oil-loving a d watar-loving fibers, which is why they can just wipe smudges off your touch screen without any liquids. It will work the same for the photo prints.\nUse cotton gloves. They are sold for specifically for photography/concervation/archiving.\n", "topic": "lifehacks", "url": "https://lifehacks.stackexchange.com/questions/6006"}, {"image": "VQAonline_00023459.png", "question": "Transcripts for Languages with Few Resources (Such as Native American Languages)", "context": "Wondering if there exist any direct Native American or other Indigenous people's transcript of a story or a complete something in their language. By indigenous I mean those living in the Amazon, New Guinea, or Arctic regions, for example. Even Mayan, Aztec, or Incan would work too (in the native language).\nMost linguistic examples from these languages are snippets or perhaps as much as a single phrase or sentence, but I haven't seen complete stories multi-sentence structures.\nI can find Native American stories here, and click into one of the texts (e.g. Yana Texts), but it seems like an English retelling of stories heard from indigenous peoples. Same with stories from the Inuit. Basically looking for a written transcription (either a Romanization or in their original language like Canadian Aboriginal Syllabics) of some story or content from an indigenous language, not written in English but instead written in the native language (or Romanization of it).\nI would love to see something like what follows (but for any Native/Indigenous language), but even longer and more complex.\n\nI understand that most (all perhaps) indigenous languages didn't have writing until it was invented in the 1800's or 1900's or so, so Romanizations work well too.\nUpdate\nHere are some I have since encountered a while later:\n\nhttps://cloudfront.escholarship.org/dist/prd/content/qt1gx6543n/qt1gx6543n.pdf?t=noidx4\nhttps://www.quechantribe.com/documents/documents/Stories-Quechan-Oral-Literature.pdf\nhttps://www.quechantribe.com/documents/documents/Xiipuktan_(First_of_All).pdf\nhttps://www.uwgb.edu/oneida/Texts.html\nhttps://cloudfront.escholarship.org/dist/prd/content/qt1gx6543n/qt1gx6543n.pdf?t=noidx4\nhttp://soda2.sou.edu/awdata/030124c1.pdf\nhttp://www.mingolanguage.org/texts/tom/\nhttps://cloudfront.escholarship.org/dist/prd/content/qt1gx6543n/qt1gx6543n.pdf?t=noidx4\nhttps://pub-jschol-prd.escholarship.org/uc/item/1gx6543n\nhttp://linguistics.berkeley.edu/~jspence/wailaki/wailaki-lexicon.php?mode=browse\nhttps://archive.org/details/rosettaproject_crk_book-1/\nhttps://openaccess.leidenuniv.nl/bitstream/handle/1887/28287/9789087280109-Text.pdf?sequence=4\n\n", "answer": "It's not clear to me exactly what your criteria are: the most stringent version that I can reasonably imagine would be substantial (2-page?) narratives in written form, composed by a native speaker in the language and written in the language by a native speaker. That would exclude translations of texts from other languages \u2013 however, if you do include translations, then there are myriad Bible translations performed by native speakers (the contemporary standard for Bible translation is that the linguists assist in developing a writing system but the native speakers do the translating). This is an example of the Bible in Zapotec\nThere are stories in Lushootseed which may qualify. For example, the story s\u010d\u0259tx\u02b7\u0259d \u0294i tsi\u0294i\u026c \u019b\u0315a\u019b\u0315ac\u0313ap\u0259d \"by\" s\u0294adacut is available. This is a traditional story, so it would be more accurate to say that he told the story, rather than composing it. However, it is not clear who did the transcribing/writing. In light of the fact that he died in 1973, I suspect that the written version is based on the recorded telling of the story (which is available in that link). It is highly likely that some of the stories from tsi si\u0294ab taq\u02b7\u0161\u0259blu are written by her, since she did write in Lushootseed. This story is attributed to her, although the recording is not of her voice. I don't have a copy of the book, but she published a collection of stories, Haboo: Native American Stories from Puget Sound: but it is possible that these are just translations and they do not have the original texts. It is probably also unclear which stories were written down by taq\u02b7\u0161\u0259blu, and which were written by Tom Hess, the main linguist involved in the language.\n", "topic": "linguistics", "url": "https://linguistics.stackexchange.com/questions/29057"}, {"image": "VQAonline_00023430.png", "question": "Japanese ji versus dji and zu versus dzu (pronunciation and keyboard input issue)", "context": "I have found the following partial hiragana chart on t he internet. I was wondering, in IPA, how does ji (\u3058) differ from dji and zu (\u305a) differ from dzu? Technically, how are these pronounced? Thanks.\nAlso, with a Japanese input keyboard for Android, how do I enter the latter characters (those prefixed with a 'd' in the picture)?\n\n", "answer": "The difference between these four is the topic of the article on yotsugana (\u56db\u3064\u4eee\u540d). In south Shikoku, they appear to be all different. However, in standard Tokyo Japanese, \u3058 = \u3062 and \u305a = \u3065. \nThe actual diachronic change of [dz] to [z] is variable in standard Japanese. According to \"The Sounds of Japanese\", the affricate [dz] occurs at the beginning of a word or immediately following a syllable-final consonant, and the fricative [z] occurs in the middle immediately following a vowel. This isn't affected by which kana is used to transcribe it. The kana \u3058 and \u305a are more common anyway since the gendai kanazukai (\u73fe\u4ee3\u4eee\u540d\u9063\u3044) post-WW2.\nAs for R\u014dmaji input, I've always used t + u / s + i with dakuten. \n", "topic": "linguistics", "url": "https://linguistics.stackexchange.com/questions/17718"}, {"image": "VQAonline_00023463.png", "question": "The difference between [w] and [u] (especially between the states of the lips)", "context": "While searching labialization on the Wikipedia, it is easy to find these statements: 'Labialized sounds involve the lips <...> When vowels involve the lips, they are called rounded.'\nIn Russian, as in many other languages, there is a rounded vowel [u]. It is very typical of Russian people, who learn English, to pronounce English labialised (~rounded) [w] with the same state of the lips, look at the\u00a0picture:\n\nBut when these Russian speakers come to a teacher who tells them not to use their lips at all and pronounce [w] in a 'smiling' manner, a question arises: if both sounds are labialised, why cannot one use his/her lips in the same way as in pronouncing [u]?\nWell, what is the difference between the state of the lips while producing [u] and [w]? Why both sounds are 'labialised/rounded', but do not really resemble one another in terms of the lips' state? Is there a more useful recommendation on how to hear/see the difference between [w] and [u]?\n", "answer": "The sounds [u] and [w] really do resemble each other, just as [u] and [o] resemble each other: resemblance is weaker than identity. I don't know what the actual problem is that Russian language teachers are addressing, but given that Russian does not have [w], I assume it's sometimes challenging for speakers to produce appropriate English outputs; perhaps [wi] is produced more like [u\u0306i]. Both [w] and [u] involve protruded lips (rounding, a form of labialization), and [v] is labial, but is not labialized and not rounded.\nThe lip difference between [u] and [w] is primarily one of timing and secondarily one of degree. That is, [w] involves a more ballistic (rapid) gesture that gives a greater construction, and [u] is slower and less constricted (depending on dialect it is also fronted and/or diphthongized). \n", "topic": "linguistics", "url": "https://linguistics.stackexchange.com/questions/30356"}, {"image": "VQAonline_00023510.png", "question": "Did Camus ever say \u201cAlways go too far, because that's where you'll find the truth\u201d?", "context": "This statement has been attributed to Albert Camus in various places on the internet. For example, this one:\n\nHowever, his Wikiquote page doesn't contain this quote. So did he actually say it? If he didn't, then who would likely say it?\n", "answer": "Almost certainly not.\nWhile the quotation is attributed to Camus on many sites, two factors make the attribution dubious:\n\nNo source is ever provided.\nThe quote is at odds with Camus's well-documented philosophical positions.\n\nFirst, the absence of a specified source is in itself suspicious. Any quotation attributed to an author should be easy to track down to a source in that author's writings. If the quotation is free-floating, with no specific citation ever provided, the chances of that quotation's actually originating from that author are vanishingly small. Such synthetic quotes are long attested. Here are some well-known examples:\n\nThe coldest winter I ever spent was a summer in San Francisco. (Mark Twain never said it.)\nTime flies like an arrow; fruit flies like a banana. (Not by Groucho Marx.)\nIt is never too late to be what you might have been. (George Eliot? Nope.)\n\nThese examples predate the internet, but dubious attributions certainly spread thick and fast over that medium. The \"Camus\" quote, for example, turns up everywhere from Goodreads to Pinterest. Yet the absence of an actual citation to any of his works is a giveaway that the attribution is dubious.\nSecond, Camus resisted philosophical labels, but is generally regarded as an existentialist and an absurdist. The Stanford Encyclopedia of Philosophy says:\n\nAlthough he forcefully separated himself from existentialism, Camus posed one of the twentieth century\u2019s best-known existentialist questions, which launches The Myth of Sisyphus: \u201cThere is only one really serious philosophical question, and that is suicide\u201d. And his philosophy of the absurd has left us with a striking image of the human fate: Sisyphus endlessly pushing his rock up the mountain only to see it roll back down each time he gains the top.\n\nAbsurdism stresses the gap between human longing for certainty and the contingency of our experience that makes such certainty impossible. In his best-known philosophical work, The Myth of Sisyphus, Camus writes:\n\nDe qui et de quoi en effet puis-je dire: \u00abJe connais cela!\u00bb Ce c\u0153ur en moi, je puis l\u2019\u00e9prouver et je juge qu\u2019il existe. Ce monde, je puis le toucher et je juge encore qu\u2019il existe. L\u00e0 s\u2019arr\u00eate toute ma science, le reste est construction. Car si j\u2019essaie de saisir ce moi dont je m\u2019assure, si j\u2019essaie de le d\u00e9finir et de le r\u00e9sumer, il n\u2019est plus qu\u2019une eau qui coule entre mes doigts. ... Entre la certitude que j\u2019ai de mon existence et le contenu que j\u2019essaie de donner \u00e0 cette assurance, le foss\u00e9 ne sera jamais combl\u00e9. Pour toujours, je serai \u00e9tranger \u00e0 moi-m\u00eame. En psychologie comme en logique, il y a des v\u00e9rit\u00e9s mais point de v\u00e9rit\u00e9.\n\nTranslation:\n\nOf whom and of what indeed can I say: \"I know that!\" This heart within me I can feel, and I judge that is exists. This world I can touch, and I likewise judge that it exists. There ends all my knowledge, and the rest is construction. For if I try to seize this self of which I feel sure, if I try to define and summarize it, it is nothing but water slipping through my fingers. ... Between the certainty I have of my existence and the content I try to give that assurance, the gap will never be filled. Forever I shall be a stranger to myself. In psychology as in logic, there are truths but no truth. (p. 7, emphasis added)\n\nThis passage summarizes Camus's philosophy pretty well. The only certainty is existence, and knowledge ends there. The idea that \"existence precedes essence\" is of course definitive of existentialism, but Camus goes further in noting the \"gap\" between the certainty of existence and the impossibility of any certainty about it. Camus rejects outright the possibility that we can escape our limited existence and arrive at any kind of objective knowledge; hence, \"there are truths but no truth.\"\nCamus goes on to say, with regard to philosophical statements, that \"Ils ne sont l\u00e9gitimes que dans la mesure exacte o\u00f9 ils sont approximatifs\"; i.e., \"They are legitimate only in precisely so far as they are approximate\". This suspicion of any and all truth-claims was fundamental to Camus's outlook. Given this epistemological stance, it is not feasible that he would endorse, let alone utter, a statement as banal as \"Always go too far, because that's where you'll find the truth\". The idea of the truth would be ridiculous to Camus.\nEdit. OP asks in a comment:\n\nJust for the sake of the argument, let's assume the quote is actually Camus'[s]. I wonder if we can interpret \"the truth\" in here as \"there are truths but no truth\", and \"go too far\" as \"far enough that you see the absurdity of the world\"? Would that interpretation be... too far?\n\nCamus would not make that argument. Absurdity is the human condition, not the human truth. That is to say, we find ourselves trapped in absurdity by virtue of our efforts to make meaning of our existence, by virtue of trying to find the \"truth\" about it. To say that the absurdity is the truth would reify what is existential to what is essential. Camus had his quarrel with the existential label, but in this case he would likely agree that absurdity is existential rather than essential, and so can't be described as an objective truth. That said, this particular extension of your question is better addressed on Philosophy Stack Exchange.\n", "topic": "literature", "url": "https://literature.stackexchange.com/questions/16662"}, {"image": "VQAonline_00023515.png", "question": "Swedish novel from 2013 on a researcher in the isolated P\u00e5rte observatory in the Sarek Mountains, Sweden", "context": "I'm looking for a Swedish literary novel, published 2013, set in the P\u00e5rtetj\u00e5kko observatory in the Sarek mountains in Sweden. In the book, the (fictional) main character is alone in the remote mountain summit observatory, and contemplating his (real) predecessor Axel Hamberg.\nIn the same year, the documentary book Axel Hamberg och Sarek was published. This is NOT the book I am looking for; I'm looking for a work of fiction.\nDue to various transliterations between Sami and Swedish, the mountain may be called p\u00e5rtetj\u00e5kka, p\u00e5rtetj\u00e5kko, p\u00e5rte, b\u00e5rddetj\u00e5hkk\u00e5, boarek, or similar variations. The observatory is called p\u00e5rteobservatoriet in Swedish.\n\nSetting of (much of) the novel. Source: SibFreak, Wikimedia Commons, CC-BY-SA\nI don't remember the author or the title of the book, and I didn't manage to find it with a web search. Would anyone know what book this may be or have suggestions how I can search for it in a more targeted way (such as limiting to all works of fiction published in Sweden in 2013)? I know I read it in 2013 and I know it was new at the time.\n", "answer": "This sounds like Observatoriet by Mats S\u00f6derlund, published in 2013 by Albert Bonniers F\u00f6rlag. The Nordin literary agency blurbed the books as follows:\n\nThe Observatory is a merciless novel that gets under the skin and lingers on the mind for a long time.\nErik is alone in the mountains on a scientific mission to observe and record the wind, the air pressure and temperature levels. While a snowstorm turns into a hurricane Erik is shivering inside the cabin\u2019s shaking walls, haunted by his memories. He no longer wants to be alone and he longs for Kristine, a fellow student that he fell in love with but is afraid to open up to. Afraid that she will see trough his intellectual surface and discover the coward who never fought back and defended himself from his boundless and abusive mother. Erik is vulnerable and fragile and feels trapped inside the isolated cabin which feeds his anxiety and childhood memories. The reader is given a harrowing portrayal of a small northern village, where the bullied Erik lives among insensitive adults and their constant breaches of privacy, at home and in the village school.\nMats S\u00f6derlund tells the story from the boy\u2019s perspective and it\u2019s cleverly done with psychological insight, speaking directly to the reader. And although this is a love story or a tale of tentative love and self-destructive behaviour, it is above all a poignant coming-of-age story.\nThe Observatory is Mats S\u00f6derlund\u2019s first novel, a story with the same linguistic responsiveness and power that characterizes his poetry.\n\nSee Google Books for the first few pages:\n\nDet \u00e4r ingen som saknar mig, ingen som undrar vart jag tagit v\u00e4gen. Jag kan lika g\u00e4rna ruttna h\u00e4r uppe. Ettusen\u00e5ttahundratrettiofyra meter \u00f6ver havet. Det \u00e4r natt. Kylan kommer i korta pustar. Jag kan inte ligga l\u00e4ngre. \u00d6ppnar blixtl\u00e5set. Kall luft rusar in.\nNo one misses me, no one wonders where I have gone. I might as well rot up here. One thousand eight hundred and thirty-four meters above sea level. It\u2019s night. The cold comes in short breaths. I can\u2019t lie down anymore. Open the zipper. Cold air rushes in.\n\nSearch confirms that \u201cp\u00e5rteobservatoriet\u201d and \u201cAxel Hamberg\u201d appear in the text.\n", "topic": "literature", "url": "https://literature.stackexchange.com/questions/18478"}, {"image": "VQAonline_00023512.png", "question": "Does \"a country junction\" mean a crossing of roads in the countryside in this passage?", "context": "I would like to know what \"a country junction\" means in the following sentences:\n\nThe first car that stopped took us east. The driver, a middle-aged\nman, eyed us from time to time but asked no questions. We drove\nsilently along country avenues lined with tall chestnut trees, past\nfields bordered by poppies. I had no idea where we were. We had no map\nand there were few road signs, but even if there had been more the\nnames of the places would have meant nothing to me. While I took it\nin, this nameless expanse, you slept with your face against the\nwindow.\nAt some point in the afternoon the driver let us out at a country\njunction. You tore a coupon from your hitch-hiker\u2019s booklet and\npassed it to him.\n\u2018Hope you send it in and win a hairdryer or something!\u2019 you cried, and\nswung the doors shut. He nodded and sped off into the horizon.\n\n\nTomasz Jedrowski, Swimming in the Dark, Chapter 3\n\nIn this novel which is set in the 1980's in Poland under the socialist regime, where homosexuality was socially unacceptable, the protagonist Ludwik (a university graduate) left Poland in 1981 to live in the United States of America. And he remembers what it was like back then in Poland, where he went to the agricultural camp (which was mandatory for college graduation). At the camp, he met Janusz, who would soon become his lover. After the camp, Ludwik decided to go to a lake district with Janusz. So they hitchhiked a car, whose driver let them out at a \"country junction.\"\nIn this part, I wonder whether this \"country junction\" means a crossing of roads in the countryside, as in the following picture, or has some other meaning that I couldn't guess.\n\nI am an English learner from South Korea, so thank you for your patience in advance as I may not know obvious things. I would very much appreciate your help. :)\n", "answer": "Yes, your interpretation is right.\nA junction (see meaning 2b) in this context means:\n\n2 a a place or point of meeting\nb an intersection of roads especially where one terminates\n\nWhile \"country\" here is used as an adjective related to meaning 4 of the noun, in the same way as in the phrase \"country avenues\" in the preceding paragraph:\n\n(n.) 4 rural as distinguished from urban areas\n\n\n(adj.) 1 of, relating to, or characteristic of the country\n\nThis makes sense in context, since we already know they're driving through the countryside, and the only kind of \"junction\" to be found there would be a junction of roads (not e.g. an electronic junction).\n", "topic": "literature", "url": "https://literature.stackexchange.com/questions/16750"}, {"image": "VQAonline_00023580.png", "question": "Why don't Muay Thai fighters bounce like Karate fighters?", "context": "Why don't Muay Thai fighters bounce like Karate fighters, with their leg forward? In Karate they teach that gives energy with the bounce. Just curious, not sure what is the best way. It seems Muay thai fighers are more static when they bob back and forth between one leg.\nhttps://youtu.be/w9kxtJpRmw4?t=43\n\nNote: If Muay Thai fighters were to bounce, probably they should keep front leg facing forward to prevent round house leg trips.\n", "answer": "This clip is of a points karate match, not a full contact contest. Points karate prioritises speed over impact.\nPronounced bouncing is typically viewed as undesirable in full contact fights, because the rhythm created by the bouncing makes your strikes more predictable, and also allows your opponent to time strikes to land either when you're unweighted (and therefore easier to unbalance), or fully-weighted on the downward, landing, phase (and therefore unable to counter or check when defending a leg kick, for example).\nWith the exception of certain strikes such as flying knees and kicks which utilise the power of leaps, or the gravitational force of a descending body or limb, most basic techniques are rendered more powerful when launched off a solid base (ie: bodyweight anchored firmly to the ground via one or two legs).\nMuch of the power generated by punches rises from the feet, to be amplified by the legs, then again by the hips, the shoulders and arms. A kick such as a roundhouse or a front kick also transfers far more energy into an opponent if the opposite leg remains in contact with the ground. Without this contact, some of the energy gets wasted; it is redirected back into the attacker, sometimes manifesting as undesirable loss of posture, balance, offensive and defensive capacity.\nSee this clip of a Kyokushin tournament. Yes, they still bounce slightly, but just enough to maintain motion; to avoid a static stance and promote explosivity. The reduced bounce means it is easier for them to ground themselves quickly prior to executing a technique when they need to.\n", "topic": "martialarts", "url": "https://martialarts.stackexchange.com/questions/12846"}, {"image": "VQAonline_00023561.png", "question": "Fix body protector strap holders", "context": "The lower strap holders in Taekwondo body protectors seem to be their weakest spots. I have seen protectors in decent condition, other than the broken strap holders, that make them impossible or difficult to use.\n\nIs there a way to mend them well enough to be usable for practice again?\n", "answer": "Well you can just take it to any clothes alterations place, and they can stitch it up fairly easily. Something like this would take about 5 to 10 minutes of time for them to do. Make sure to tell them it has to be a very strong stitch.\nThey might even recommend replacing the loops instead of stitching the existing ones. And they can suggest a material that will hold up better, maybe a thick nylon material.\nSpeaking of nylon, if they're made of nylon fibers, you can try melting them together with a lighter or some kind of flame.\nI don't recommend gluing with any kind of glue. The bond probably won't hold up over time. And if it doesn't work, you now have to deal with the glue somehow.\nOne alternative to stitching would be to use metal rivets. You can get them on Amazon. You might not even need any tools, just a hammer. But keep in mind adding rivets means you lose about a centimeter or two along the perimeter of the loop, because you have to overlap the two ends to make the rivet. That might make the loop too small.\nHere's a web site describing how to do it:\nhttps://sew4home.com/tips-resources/sewing-tips-tricks/how-install-metal-rivets-sewing-projects\nBut my recommendation is to just go with a clothes alterations place instead. It's fairly cheap, and they know what they're doing.\nHope that helps.\n", "topic": "martialarts", "url": "https://martialarts.stackexchange.com/questions/8311"}, {"image": "VQAonline_00023544.png", "question": "How do you escape from a front head lock?", "context": "A real experience: \nWell about a year ago I got in fight with a bullying thug in street, and we tackled each other. He was way bigger than I am, though we pushed each other locked-arm swinging around for some time, and soon he got my head in a lock with his arm and tried to pull me around. Despite my resistance, he succeeded and I lost my balance, being pushed to the ground with my feet bent to the ground due to extreme pressure he was putting on me. I tried to catch his legs but he was deliberately standing away, so I could not reach them. He kept punching on the side of my head with hammer fist, I was left helpless, if people were not coming around, I do not know what would happen. \nAlthough the best solution might be not to get in fight in the first place, especially when opponent is stronger, in case it happens how should one release oneself or counteract when put in such a situation locked especially on the head in a lower position?\nUpdate:\nWell I guess I finally got the technical name for this situation: a front headlock as shown in this YouTube video and picture below. I hope it makes clear the situation I described above. I had a similar situation:\n\n", "answer": "Basic headlock release:\n1) twist your body to get the front of your neck at the gap (if your throat is being closed by the arm this is top priority).\n2) \nWith the arm that is closest to the attacker - do a front crawl motion (bring the arm between you and the attacker low to get the arm behind you both). \n3)\nBring the arm up and between yours and the attackers heads.\n4) Push your hand upwards (in a knife hand ideally) into the attackers nose.\nNOTE this is a basic technique we teach - mainly to allow children to get out of grabs in a playground setting without hurting the other child to avoid further repercussions. \nI would adapt this slightly for a real life situation as described. Below are little additions to help protect you and increase the likelihood of escape.\nGuard - id place the other hand infront of my face for a bit of protection.\nMovement in a headlock your centre of gravity will be generally lower than theirs. Keep pushing in different directions to keep them adjusting and not able to focus fully on hurting you.\nAttack your feet and knees should be free to attack the legs of your attacker. As with the movement above will be a distraction to the attacker helping with the defence. There are many pressure points in the legs which may be used to your advantage (e.g. back of the knees has many)\nWARNING The basic technique will work in class. It may be required to add the extras in with a less willing \"partner\".\n", "topic": "martialarts", "url": "https://martialarts.stackexchange.com/questions/6545"}, {"image": "VQAonline_00023627.png", "question": "Where is this extra plane coming from?", "context": "\nWhy are there solid lines drawn connecting the solid circles after twinning has taken place? Ideally, that explains a real plane, isn't it? But why are we going to get a plane out of nothing if we mechanically twin or a twinning is followed after heat treatment(annealing twins)?\n", "answer": "solid circles represent the positions of atoms in the twinned region; these black circles represents the only atoms that are present there, whereas dashed circles are their original positions that after twinning are no more occupied on account of displacement.\nThese black atoms are displaced from their original positions on account of twinning and hence form novel planes that are not found in the untwinned regions.\n", "topic": "materials", "url": "https://materials.stackexchange.com/questions/4105"}, {"image": "VQAonline_00023626.png", "question": "Band structure of Weyl semimetal?", "context": "Weyl semimetals are topological quantum materials whose low energy excitations emerges as massles Weyl Fermions. They have a band touching point near the Fermi level called Weyl node.\n\nWhat is actually the energy dispersion relation of Weyl Fermions? More specifically, do we have to consider the band as a linear dispersion relation or as a conduction and valence band which takes the shape of two cones?\nI came up with this doubt because if we consider the band dispersion relation as linear, then\n$$E_k=C\\times (k-k_0)$$(For convenience consider 1D. but I know that Weyl semimetals are three dimensional)\nthen the effective mass is given by $$m*=\\frac{\\hbar^2}{\\frac{\\partial^2E_k}{\\partial k^2}}$$\nthat gives\n$$m*=\\frac{\\hbar^2}{0}=\\infty$$\nWhich contradicts the fact that Weyl Fermions are massless.\nBut If we consider it as the latter one then the derivative is not defined at the Weyl node?\nWhere am I wrong in my calculations and concepts?\n", "answer": "A Weyl point does indeed have linear dispersion. The effective mass tensor you are using $\\frac{1}{m^*_{ij}}\\propto\\frac{\\partial^2\\varepsilon(k)}{\\partial k_i \\partial k_j}$ is not really well-defined here. After all, we only define the effective mass that way to match the common non-relativistic equation $\\varepsilon = \\frac{\\hbar^2 k^2}{2m^*}$. A Weyl point is in no sense described by such an equation since its low-energy effective Hamiltonian is relativistic. Specifically, the effective Hamiltonian is a generalization of the Weyl Hamiltonian from high energy physics. This generalized form is\n$$ \\tag{1}H(k) = (a_\\mu\\cdot k)\\sigma^\\mu $$\n(more on this in the next section). The ordinary Weyl equation from high energy physics is given by the special case $H_{weyl}=k\\cdot \\sigma$. This describes massless chiral fermions as described in many other places.\nGeneral Weyl Point Dispersion\nNow I will sketch the derivation of the general Weyl point dispersion (both Type-I and Type-II). In general, a Weyl point is an isolated linear crossing of two bands. In the vicinity of that crossing, we can focus on just those two bands which are described by a two-band Hamiltonian which is generally of the form\n$$\\tag{2}H(k) = h_\\mu(k)\\sigma^\\mu = h_0(k) \\sigma^0 + h_x(k) \\sigma^x + h_y(k)\\sigma^y + h_z(k)\\sigma^z$$\nTaking the degenerate point to be at $k=0$ and at $\\varepsilon=0$, we can expand the functions $h_\\mu$ as $h_\\mu(k) = \\nabla_k h_\\mu\\big|_0 \\cdot k + \\dots$\nso that the effective Hamiltonian at the Weyl point is\n$$ H(k) = (a_\\mu\\cdot k)\\sigma^\\mu $$\nwhere $a_\\mu = \\nabla_k h_\\mu\\big|_0$ are model-dependent vector parameters. This can be diagonalized to arrive at the general Weyl point dispersion relation:\n$$ \\tag{3}\\varepsilon_\\pm(k) = a_0\\cdot k \\pm \\sqrt{(a_x\\cdot k)^2 + (a_y\\cdot k)^2 + (a_z\\cdot k)^2}$$\nDepending on $a_0$, this can be either a Type-I or Type-II Weyl point.\nMulti-Weyl points\nAs a final note, I just want to mention that there are also so-called multi-Weyl points which have non-linear dispersion and have higher chiralities. More about them here\n", "topic": "materials", "url": "https://materials.stackexchange.com/questions/4033"}, {"image": "VQAonline_00023640.png", "question": "When import volumetric data (spin density of CHGCAR) to VESTA which option should we pick here?", "context": "I am trying to plot SPIN DENSITY isosurfaces using VESTA. I have used vaspkit option 312 to create SPIN.vasp file which contains spin density from the CHGCAR. Now, when I import data to VESTA it shows the window below to convert or not convert data.\nMy question is : What is the unit of data in CHGCAR ? and which option should I pick to plot isosurfaces in e/Ang\u00b3 unit.\n\n", "answer": "This is a bit of a boring answer, but chances are you normally want to replace the current data and do nothing. The operation can be used to add or subtract densities to visualize charge differences, but if you just want to plot a spin density you can likely just use replace.\nYou also probably do not need to convert the units since most VASP-like tools tend to use angstroms and I am assuming vaspkit does not decide to do a unit conversion away from angstrom.\n", "topic": "materials", "url": "https://materials.stackexchange.com/questions/4777"}, {"image": "VQAonline_00042174.png", "question": "Total average of averages not same as the average of total values", "context": "I am having a strange problem while computing the overall percentage. Let me demonstrate my problem using an example.\nAssume that I am receiving multiple batches of apple from the vendor and the number of good apple received in each batch are computed to display as percentage as below:\n\nIf I find the average of the last column (Good Apple Percentage),\nI get $\\frac{(98.5+98+99)}{3} = 98.5 $%\nHowever, if I find the total no of apples and total number of bad apples first (column 2 and 3),\nTotal No of Apples $= 200+100+300 = 600$\nTotal No of Bad Apples $= 3+2+3= 8$\nThen compute the total percentage, $\\frac{600-8}{600}= 98.67$%\nWhy is the total average of the average not the same as the average of total values? There is no rounding up or rounding down necessary for individual percentages (Column 4), so shouldn't the two percentage be the same? What am I missing?\n", "answer": "The reason you are getting different numbers is because if you look at just the averages, it completely obliterates any sense of the size of the original data. Let me give you a more extreme example to illustrate:\nBatch A: 2 apples total, 1 bad apple, 50% good apples\nBatch B: 100 total apples, 1 bad apple, 99% good apples\nTotaling the two we have 102 apples, 2 of which are bad, 100 good. But if we average just the percentages, we are at $74.5%$.\nThe moral of the story is, you can't just average the averages because it loses all sense of how big the individual samples are.\n", "topic": "math", "url": "https://math.stackexchange.com/questions/4205313"}, {"image": "VQAonline_00036995.png", "question": "Defining the Cosine Function from First Principles, intuitively", "context": "Throughout most of my mathematics education, the cosine function has been defined formally either using its series expansion, as $\\mathfrak{Re}(\\exp i\\theta)$, or as the unique solution to $y+y''=0$ with $y(0)=1$ and $y'(0) = 0$. \nAlthough these definitions make the mathematics more convenient, I've always been interested to see what it would be like to try to formalise it directly with the geometric intuition from the unit circle, assuming only the notion of distance (i.e. that we have a complete metric space with metric $d\\colon\\mathbb R^2 \\times \\mathbb R^2 \\to \\mathbb R$).\nSince, by geometric intuition, we have that the cosine function is the $x$-coordinate obtained by walking along the unit circle, my idea was to take a line segment from the point $(1,0)$ to another point on the circle such that the distance is $\\theta$, and split it into two line segments whose sum of lengths is also $\\theta$. We then split into 3 line segments, and so on, in a limiting manner, \nin such a way that the sum of lengths is always $\\theta$, as illustrated below with $\\theta = 2\\pi/5$. \n\n[Desmos link]\nWhen this procedure converges, the $x$- and $y$-coordinates of the limiting point should be the cosine and sine of $\\theta$ respectively. The problem is I can't find an easy way to express the $n$th coordinate of this procedure (assuming we are splitting the line into $n-1$ segments) so that I can take the limit.\nI was going to try and do things the other way round, that is, define $\\cos^{-1}\\colon [-1,1] \\to [0,\\pi]$ by approaching the circle from below and finding the length. I did manage to do this, and got that \n\\begin{align*}\n \\cos^{-1}(x) &= \\sum_{k=1}^\\infty d\\left[\\left(\\frac{(k-1)x}{n}, \\sqrt{1-\\left(\\frac{(k-1)x}{n}\\right)^2}\\right), \\left(\\frac{kx}{n}, \\sqrt{1-\\left(\\frac{kx}{n}\\right)^2}\\right)\\right]\\\\[4pt]\n &= \\sum_{k=1}^\\infty \\sqrt{2\n \\left(1-\\sqrt{1-\\frac{(k-1)^2\n x^2}{n^2}}\n \\sqrt{1-\\frac{k^2\n x^2}{n^2}}\\right)-\\frac{2\n k(k-1) x^2}{n^2}},\n\\end{align*}\nand I suppose one can extrapolate from here to obtain the $\\cos$ and $\\sin$ functions and extend them appropriately. \nBut it feels like the other way, that is, finding that limiting point, should be possible. I appreciate any assistance with this.\n", "answer": "Let $(x_1, y_1)$ be the first point in your construction. Using the similar right triangles $(0, 0)$\u2013$\\left(\\frac{x_1 + 1}{2}, \\frac{y_1}{2}\\right)$\u2013$(1, 0)$ and $\\left(\\frac{x_1 + 1}{2}, \\frac{y_1}{2}\\right)$\u2013$\\left(\\frac{x_1 + 1}{2}, 0\\right)$\u2013$(1, 0)$, we compute\n$$(x_1, y_1) = \\left(1 - \\frac{(\\theta/n)^2}{2}, \\frac\\theta n\\sqrt{1 - \\frac{(\\theta/n)^2}{4}}\\right).$$\nHow do we go from $(x_k, y_k)$ to $(x_{k + 1}, y_{k + 1})$? This is just a rotation, which is a job easily done by complex multiplication by the constant $x_1 + iy_1$:\n$$x_{k + 1} + iy_{k + 1} = (x_k + iy_k)(x_1 + iy_1).$$\nIterating this $n$ times gives\n$$x_n + iy_n = (x_1 + iy_1)^n.$$\nAt this point, some calculus would show that this approaches $e^{i\\theta}$, but we don\u2019t want to use calculus, so instead we\u2019ll expand this using the binomial theorem:\n$$x_n + iy_n = \\sum_{k = 0}^n \\binom{n}{k} x_1^{n - k}i^ky_1^k.$$\nThe real part is\n$$\\begin{split}\nx_n &= \\sum_{k = 0}^{\\lfloor n/2\\rfloor} \\binom{n}{2k} (-1)^kx_1^{n - 2k}y_1^{2k} \\\\\n&= \\sum_{k = 0}^{\\lfloor n/2\\rfloor} \\binom{n}{2k} (-1)^k\\left(1 - \\frac{(\\theta/n)^2}{2}\\right)^{n - 2k}\\left(\\frac{\\theta}{n}\\right)^{2k}\\left(1 - \\frac{(\\theta/n)^2}{4}\\right)^k,\n\\end{split}$$\nwhich I suppose is an expression.\nMiraculously, in the limit as $n \\to \\infty$, we have\n$$\\binom{n}{2k} \\frac{1}{n^{2k}} \\to \\frac{1}{(2k)!}, \\quad\n\\left(1 - \\frac{(\\theta/n)^2}{2}\\right)^{n - 2k} \\to 1, \\quad\n\\left(1 - \\frac{(\\theta/n)^2}{4}\\right)^k \\to 1,$$\nso the $k$th term converges to simply\n$$\\frac{(-1)^k}{(2k)!}\\theta^{2k}.$$\nAfter checking some details about exchanging the sum with the limit, we get back the power series we know and love.\n$$\\cos \\theta = \\sum_{k = 1}^\\infty \\frac{(-1)^k}{(2k)!}\\theta^{2k}.$$\nWe can get $\\sin \\theta$ from the imaginary part in a similar way.\n", "topic": "math", "url": "https://math.stackexchange.com/questions/3060125"}, {"image": "VQAonline_00025752.png", "question": "F is a submersion of $X \\times S$, and for a fixed $x$.", "context": "The following text confuses me very much:\n(1) For fixed $x \\in X$, is my understanding correct?\n\n$F_{(x,s)}$ takes $(x,s)$ and gives $f(x)+s$, so $dF_{(x,s)}$ takes $(x,s)$ and gives $df(x)+ds$. But $x$ is fixed, so $df(x) = 0$. Also, $ds$ is just identity. Hence, $\\forall v \\in T_{(x,s)} (X,S), \\exists u \\in (X,S): dF_{(x,s)}(u)=v.$ Namely, $u = (x,s).$ Therefore, $dF$ is surjective hence a submersion.\n\n(2) Why $F$ is a submersion of $X \\times S$?\n(3) And why therefore transversal to any submanifold $Z$ of $\\mathbb{R}^M$?\n\nThank you~\n", "answer": "Your argument for (1) looks good, except that $u$ should be $(0,v)$, where the first component is the zero tangent vector at $x$, not $x$ itself (since the arguments of $dF$ need to be tangent vectors, tangent to the submanifold $\\{x\\}\\times S$).\nFor (2), just observe that the tangent space to $X\\times S$ at $(x,s)$ includes, as a subspace, the tangent space to $\\{x\\}\\times S$. Since the latter maps, via $dF$, onto the whole tangent space to $\\mathbb R^M$, so does the former.\nFor (3), recall that the desired transversality of $F$ to $Z$ means only that the image of $dF$ and the tangent space to $Z$ together generate the whole tangent space to $\\mathbb R^M$. In the case at hand, the image of $dF$ is already this whole tangent space, so we don't even need the tangent space of $Z$ to generate that whole tangent space. (In other words, submersions are transversal to everything.)\n", "topic": "math", "url": "https://math.stackexchange.com/questions/442556"}, {"image": "VQAonline_00043901.png", "question": "How to formulate this type of arcsin problem?", "context": "Reading and commenting on What are some common ways students get confused about finding an inverse of a function? I was kindly set straight that the use of $\\sin^{^{-1}}(x)$ to mean $\\arcsin(x)$ has been rendered obsolete. International Standard ISO 80000-2 offers the following:\n\nThis notation is preferable to me, with one point of confusion. The text we use, Algebra and Trigonometry by Paul Foerster, uses $\\textrm{Arcsin}(x)$ to indicate the restricted domain, and the lower case as above for all real domain. This notation leaves me with my question - \nHow do we ask for the arcfunction with no domain restriction, i.e we actually want the multiple answer as in \"solve for the first 6 values when $y=2$ for the following sinusoidal function\" ? \nDisclosure - This question is in reaction to my discovering that a notation that I never cared for has been eliminated by an international standard for nearly 10 years, but still used at my high school. I am trying to understand where to go with this as people in general aren't so open to change. \n", "answer": "\nSolve the equation $\\sin x=0.5$ in $[0,7\\pi]$.\nThe equation $y=-10\\cos(3t)$ represent the motion of a weight hanging on a spring after it has been pulled $10$ cm below its equilibrium point and released. The output $y$ gives the position of the weight in cm above (positive $y$) or below (negative $y$) the equilibrium point after $t$ seconds. Find the first four times when the weight is $5$ cm above its equilibrium point. \nA ferris wheel with a diameter of $36$ m rotates counterclockwise at $5$ revolutions per minute. At its lowest point, each seat is $0.9$ m from the ground. For a rider starting at the lowest point, his height (in meters) above the ground after $t$ minutes is given by $y=18.9-18\\cos(10\\pi t)$. Find all the times in the first half minute when the rider is $24$ m above the ground. Round to the nearest tenth of a second.\nAn alternating current generator produces the current (in amperes) $i=30\\sin(120\\pi t)$ where $t$ is the time in seconds. Find the least positive $t$ for which $i=25$ A. \nA polarizing filter for a camera contains two parallel plates of polarizing glass, one fixed and the other able to rotate. If $\\theta$ is the angle of rotation from the position of maximum light transmission then the intensity of light leaving the filter is $\\cos^2 \\theta$ times the intensity entering the filter. Find the least positive $\\theta$ so that the intensity of light leaving is $70$ % of that entering.\nThe planet Mercury travels around the sun in an elliptical orbit given by $\\displaystyle r=\\frac{3.44\\times 10^7}{1-0.206\\cos\\theta}$. Find the least positive $\\theta$ for which mercury is $3.78\\times 10^7$ miles from the sun.\nGiven the equation $xy=-2$ replace $x$ and $y$ with $x=u\\cos\\alpha-v\\sin\\alpha$ and $y=u\\sin\\alpha+v\\cos\\alpha$. Find the least positive $\\alpha$ so that coefficient of the $uv$ term will be $0$.\nWhat are the $x$-intercepts of the graph of the function $f$ defined by $f(x)=4\\sin^2x-3$ on the interval $[0,2\\pi]$?\nBlood pressure is a way of measuring the amount of force exerted on the walls of blood vessels. It is measured using two numbers: systolic (as the heart beats) blood pressure and diastolic (as the heart rests) blood pressure. Blood pressures vary substantially from person to person, but a typical blood pressure is 120 80, which means the systolic blood pressure is 120 mmHg and the diastolic blood pressure is 80 mmHg. Assuming that a person\u2019s heart beats 70 times per minute, the blood pressure P of an individual after t seconds can be modeled by the function $P(t)=100+20\\sin\\left(\\frac{7\\pi t}{3}\\right)$. In the interval $[0,1]$, determine the times at which the blood pressure is between 100 and 105 mmHg.\n\n", "topic": "matheducators", "url": "https://matheducators.stackexchange.com/questions/16577"}, {"image": "VQAonline_00043907.png", "question": "Definition of Trapezoid", "context": "From one textbook we use in our High School -\n\nTranscription:\n\nA trapezoid is a quadrilateral with exactly one pair of parallel sides. The parallel sides are called bases of the trapezoid.\n\nAnd from Wikipedia -\n\nIn Euclidean geometry, a convex quadrilateral with at least one pair\nof parallel sides is referred to as a trapezium (/tr\u0259\u02c8pi\u02d0zi\u0259m/) in\nEnglish outside North America, but as a trapezoid[1][2] (/\u02c8tr\u00e6p\u0259z\u0254\u026ad/)\nin American and Canadian English.\n\nOne other textbook in my school follows the Wikipedia definition.\nThe former definition excludes parallelograms and rectangles. The latter, defines both to be a subset of trapezoids.\nHow do we address this with students? I'm starting to get objections to the textbook image I posted, with the student either recalling having learned it differently in a prior class, or searching and declaring another source as the contradiction to our textbook. I can offer a response of \"This is one of those math naming properties that doesn't have 100% agreement.\" Although this doesn't seem satisfying.\n", "answer": "I would use this to help students understand three \"meta\" ideas:\n(1) Math is not about memorizing lots of random trivia. In the real world, if you go up to a mathematician and ask them which definition of a trapezoid is right, they will just smile indulgently. They don't know or care.\n(2) There is not always a consensus about definitions. Get over it, boys and girls! In STEM, it's very common that when you read something, you need to check which definitions they're using.\n(3) In general, in math, we prefer to make our definitions in such a way that theorems come out tidy and with a minimum of special-casing. For this purpose, it's usually good to have the things that fit definition A be a subset of the things that fit definition B. By this rule of thumb, it's preferable to define a parallelogram as being a trapezoid. If not, then any time you want to prove a theorem whose conclusion is \"X is a trapezoid,\" you will probably have to uglify it by making the conclusion \"X is either a trapezoid or a parallelogram.\"\nOften, a reason why books will sometimes choose exclusive definitions (so that a square is not a rectangle, and a parallelogram is not a trapezoid) is that they have a low estimate of their students' intelligence. Students operating at lower intellectual levels (as well as very young kids) have trouble understanding how these definitions can be inclusive.\nIn this particular example, there is a possible advantage of choosing the exclusive definition, which is that then we have two sides that we can pick out as the \"bases.\" It's a trade-off.\n", "topic": "matheducators", "url": "https://matheducators.stackexchange.com/questions/19410"}, {"image": "VQAonline_00043879.png", "question": "Worksheet: Homology in Intro Lin Al", "context": "I am about to start grad school and I am trying to think seriously about teaching [you know, before I get swamped with my own coursework]. I wrote a hypothetical worksheet for an introductory linear algebra class which tries to do something a little more interesting than \"Here is a random LT I made up, compute the nullspace\". Here is an excerpt from the beginning:\n\nTriangle (T). We will use the following graphic to create two vector spaces and a linear transformation between them:\n$\\qquad\\qquad\\qquad\\qquad$ \nThe vector space $C_0$ is given by formal linear combinations of the labelled points. In other words, $(x,a ,b )$ form a basis for $C_0$, so a typical vector in $C_0$ looks like\n$$ 5x-3a -2b $$\nSimilarly, the vector space $C_1$ is given by formal linear combinations of the line segments. By analogy to the above construction, an explicit basis might be $(xa , ab, bx)$, so that a typical vector in $C_1$ looks like\n$$ -6xa+2ab-8bx. $$\nFinally, consider the linear transformation $D:C_1\\to C_0 $ which takes an line segment $vw$ to $v-w$, if $v$ is the vertex the arrow is pointing toward, and $w$ is the vertex the arrow is pointing away from. For instance, $D(xa )=x-a$.\nT1. Compute $D(-6xa+2bx-8ab)$.\nT2. Write the matrix for $D$ using the two bases provided.\nT3. By Gaussian elimination or otherwise, find all $\\alpha\\in C_1$ so that $D(\\alpha)=5x-3a-2b$.\nT4. Compute the nullspace of $D$ and its dimension (nullity).\n\nTwo more examples follow, all with \"compute the nullity of $D$\" ($4\\times 4$ and $4\\times 5$; a little large but not terrible) with the kicker at the end:\n\nFurther Thinking. Can you describe a relationship between the nullity of $D$ and the geometry of the graphic? Write a conjecture, using evidence (from your answers to the previous questions, or elsewhere) to justify it.\n\nIdeally, this would be accompanied by a discussion of formal linear combinations in class, since the treatment above is clearly inadequate as a first introduction.\nI came from an undergrad program with a high emphasis on STEM, so linear algebra education was unusual, as was the math motivation of the student body. I guess what I really want to know is, from people with actual experience in a classroom, would this fly? and if not, can it be salvaged?\nBut to make the question more amenable to the SE model:\n\nFor what style/level of linear algebra course, if any, would this worksheet be appropriate?\nAre there obvious local improvements to what you see here? (i.e., could one of the questions be altered slightly to probe an additional concept)\nCan you point me toward education research that talks about the effectiveness of incorporating high-level ideas into lower-division courses?\n\n", "answer": "I have a dim memory of a simple exercise I came up with as a TA. It was something like the vector space over the reals of polynomials in two variables, including x^2, y^2, and (x+y)^2. I then asked them to compute something like the dimension of the space S spanned by x^2 + y^2, (x+y)^2, and xy. I also asked them if xy was in S, and then if y was in S, and to explain why or why not.\nThe professor for the course thought it was a great problem, and that none of the students would understand it. He put it on a final exam with my name for attribution, and had me grade the result. He was right. A few students came close, but none were able to demonstrate full understanding of what was going on with the problem. \nI think your exercise is good for an introductory graduate student or an ambitious undergraduate. Unless you have a top rate class though, the exercise will be under-appreciated. Or, it will take a long time to appreciate. This is not to discourage you so much as to alert you. If\nyou turned this exercise into a lengthy blog post with some more detail worked out, you might reach a suitable audience.\nGerhard \"Society Needs More Good Exercises\" Paseman, 2015.08.08\n", "topic": "matheducators", "url": "https://matheducators.stackexchange.com/questions/8489"}, {"image": "VQAonline_00043942.png", "question": "Is a rhombus rigid on a sphere or torus? And generalizations", "context": "If a rectangle is formed from rigid bars for edges and joints\nat vertices, then it is flexible in the plane: it can flex\nto a parallelogram.\nOn any smooth surface with a metric, one can define a linkage\n(e.g., a rectangle) whose edges are geodesics of fixed length,\nand whose vertices are joints, and again ask if it is rigid\nor flexible on the surface.\nThis leads to my first, specific question:\n\nQ1.\nIs a rhombus, or a rectangle,\nalways flexible on a sphere?\n\n\nIt seems the answer should be Yes but I am a bit uncertain\nif there must be a restriction on the edge lengths.\n(In the above figure, the four arcs are each $49^\\circ$ in length, comfortably short.)\n\nQ2.\nThe same question for other surfaces: Arbitrary convex surfaces? A torus?\n\nI am especially interested to learn if there are situations where a linkage that is flexible\nin the plane is rendered rigid when embedded on some surface.\nIt seems this should be possible...?\n\nQ3.\nMore generally,\nLaman's theorem\nprovides a combinatorial characterization of the rigid linkages in the plane.\nThe $n{=}4$ rectangle is not rigid because it has fewer than $2n-3 = 5$ bars:\nit needs a 5th diagonal bar to rigidify.\nHas Laman's theorem been extended to arbitary (closed, smooth) surfaces\nembedded in $\\mathbb{R}^3$?\nPerhaps at least to spheres, or to all convex surfaces?\n\nThanks for any ideas or pointers to relevant literature!\nAddendum.\nI found one paper related to my question:\n\"Rigidity of Frameworks Supported on Surfaces\"\nby A. Nixon, J.C. Owen, S.C. Power.\narXiv:1009.3772v1 math.CO\nIn it they prove an analog of Laman's theorem for\nthe circular cylinder in $\\mathbb{R}^3$.\nIf one phrases Laman's theorem as requiring for\nrigidity that the number of edges $E \\ge 2 V - 3$\nin both the graph and in all its subgraphs,\nthen their result (Thm. 5.3) is that, on the cylinder, rigidity requires\n$E \\ge 2 V -2$ in the graph and in all its subgraphs.\nThis is not the precise statement of their theorem.\nThey must also insist that the graph be regular\nin a sense that depends on the rigidity matrix achieving maximal rank\n(Def. 3.3).\nThey give as examples of irregular linkages on a sphere\none that\ncontains an edge with antipodal endpoints, or one that includes\na triangle all three of whose vertices lie on a great circle.\nBut modulo excluding irregular graphs and other minor technical\ndetails, they essentially replace the constant 3 in Laman's\ntheorem for the plane with 2 for the cylinder.\nTheirs is a very recent paper but contains few citations to related\nwork on surfaces, suggesting that perhaps the area\nof linkages embedded on surfaces is\nnot yet well explored.\nIn light of this apparent paucity of information, it seems appropriate that I 'accept'\none of the excellent answers received. Thanks!\nAddendum [31Jan11].\nI just learned of a 2010 paper by Justin Malestein and Louis Theran,\n\"Generic combinatorial rigidity of periodic frameworks\"\narXiv:1008.1837v2 (math.CO),\nwhich pretty much completely solves the problem of linkages on a flat 2-torus,\ngeneralizing to flat orbifolds. They obtain a combinatorial characterization for generic minimal\nrigidity for \"planar periodic frameworks,\" which encompass these surfaces.\n", "answer": "Q3: Laman's theorem is the same on the sphere.\nIndeed, a configuration with $n$ vertices and $m$ edges is defined by a system of $m$ equations in $2n-3$ variables (there are $2n$ coordinates of points, but we may assume that the first point is fixed and the direction of one of the edges from the first point is fixed too). The the left-hand sides are analytic functions of our variables and the right-hand sides are the squares of the lengths of the bars (on the sphere, cosines rather than squares).\nConsider this system as a map $f:\\mathbb R^{2n-3}\\to\\mathbb R^m$. The rigidity means that a generic point $x\\in\\mathbb R^{2n-3}$ cannot be moved within the pre-image of $f(x)$. This implies that $rank(df)=2n-3$ on an open dense set. Choose a configuration from this set and project it to the sphere of radius $R\\to\\infty$. The equations on the sphere converge to those in the plane, hence the rank of the linearization on the sphere will be maximal ($=2n-3$) for all large $R$. So we get an open set of configuration on the sphere where the linearization has the maximal rank (and this implies rigidity). Since all functions involved are analytic and the rank is maximal on an open set, it is maximal generically. So our linkage is generically rigid on the sphere.\nConversely, consider a flexible linkage on the plane. If $m<2n-3$, it will be flexible on the sphere by a dimension counting argument. Otherwise, by Laman's theorem, there is a subgraph with $r$ vertices and more than $2r-3$ edges. Consider such a subgraph for which $r$ is minimal. Then, by Laman's theorem, we can remove some edges so that this subgraph remains rigid. And, by the above argument, it is rigid on the sphere too. So the edges that we removed were redundant both in the plane and in the sphere. Let's forget about them and repeat the procedure. Eventually we will get a linkage with fewer than $2n-3$ edges.\n", "topic": "mathoverflow.net", "url": "https://mathoverflow.net.stackexchange.com/questions/40463"}, {"image": "VQAonline_00044606.png", "question": "Constructing a 1-planar graph that has no rectilinear drawing", "context": "A 1-planar graph is a graph that can be drawn in the Euclidean plane in such a way that each edge has at most one crossing point, where it crosses a single additional edge.\n1 planar graph\nI read the following paper, which stated the following fact about the rectilinear drawing of a 1-plane graph.\nThomassen, C. (1988), Rectilinear drawings of graphs. J. Graph Theory, 12: 335-341.https://doi.org/10.1002/jgt.3190120306\nTheorem 3.1. Let $G$ be a graph represented as a normal drawing of index $0$ or $1$. Then there exists a rectilinear drawing equivalent to $G$ if and only if $G$ contains no $B-$ or $W-$configuration.\n\nFrom this theorem we know that a 1-planar graph satisfies any 1-plane drawing of it without $B-$ and $W-$configuration, then it has no rectilinear drawing.\nBut I haven't constructed a 1-planar graph without rectilinear drawing so far. It is even doubtful that it exists. This is one of my confusing points. Any advice would be helpful and thanks in advance!\nSome concepts are added below:\n\nEvery finite graph can be drawn in the Euclidean plane such that any two edges\nhave at most one point in common (in particular, two edges with a common end\nhave no other point in common), and such that every point that is not a vertex\nand that is on two edges is on precisely two edges and is a cross-point for these\ntwo edges. We call such a drawing normal.\n\nThe maximum number of crosspoints\non an edge is called the cross-index of the drawing.\n\n\n", "answer": "The main result of the following paper states that a graph which has a 1-planar straight-line drawing has at most $4n-9$ edges, where $n$ is (as usual) the number of vertices:\nDidimo, Walter, Density of straight-line 1-planar graph drawings, Inf. Process. Lett. 113, No. 7, 236-240 (2013). ZBL1259.05107.\nOn the other hand, it is not hard to come up with a 1-planar graph with $4n-8$ edges: take any drawing of a planar graph in which every face is a quadrilateral and add a pair of crossing edges to each face. For instance, applying this to the cube gives $K_8$ with a matching removed.\n", "topic": "mathoverflow.net", "url": "https://mathoverflow.net.stackexchange.com/questions/390547"}, {"image": "VQAonline_00044466.png", "question": "Immersion of non-orientable surface in $\\mathbb R^3$ with conditions on the height function", "context": "EDIT: The answer is trivially positive; the question arose from my misunderstanding of the figure below.\nCan a non-orientable closed surface of odd genus be immersed in $\\mathbb R^3$ so that the associated height function be of Morse-Bott type and have no centers?\nThat is, the height function would have only Bott-type extrema and saddle singularities. A Bott-type singularity is a non-degenerate singular circle: a circle where the derivative is zero with the function being quadratic on transverse curves. A center is a Morse-type local extremum: an isolated singularity around which the function is $\\pm(x_1^2+x_2^2)$ in some local coordinates.\nMy intuition is that no. (I've asked this question on math.SE but did not get any answer.)\nConsider the projective plane $\\mathbb RP^2$ as the Boy surface (left) [EDIT: wrong, both figures are not immersions, and the left figure is not the Boy surface] and the Klein bottle $K^2$ (right):\n\n(image from the book). The surfaces shown are images of immersions (i.e., locally smooth embeddings) in the complements of the singular points (white dots). The vertical line in the right-side figure is a homologically non-trivial cycle.\nFor even genera $g$ (except $g=2$, which is a different story), it is easy to do: e.g., connect the top and bottom of $K^2$ (right) by a tube (as if you drill a wormhole along the vertical axis), which will form a surface of genus $g=4$ immersed [EDIT: wrong, this is not an immersion] with two Bott-type extrema (circles) and two Morse-type saddles. (You can get any even genus $g\\ge4$ by adding more handles.)\nHowever, adding such a handle to the $\\mathbb RP^2$ (left) seems not possible. Suppose you add such a handle connecting the bottom to the top of the figure (left). There must be a singularity on that handle. Indeed, consider the evolution of the level sets from the bottom to the top along this handle. The level sets at its endpoints are circles $S^1$ immersed in the plane: O-shaped at the bottom and 8-shaped at the top, which are not regularly homotopic by the Whitney\u2013Graustein theorem. Therefore, there must be a singularity in between.\nMy intuition is that the singularity will be similar to the saddle shown on the picture (roughly speaking, not preserving the parity of the total turning number below and above). Though the singular contour can be more complicated (e.g., connecting more handles), it will effectively convert the left-side picture into the right-side one: it would cause an additional cycle passing through the first singularity (like the cycle between the two singularities shown on the right), thus making the genus $g$ even.\nI think this argument would generalize to a surface with more handles, as soon as any cycle exists between the \"bottom\" and \"top\" of the singular level of the type shown in the figure (left): in the absence of centers, for each saddle \"that changes the orientation,\" along the evolution of the level sets (passing transparently the Bott-type extrema along the handle) there will occur another similar saddle adding a second cycle.\nUnfortunately, I lack the skill to convert this into a formal proof, and even if I could do it for this particular type of immersion [EDIT: wrong, this is not an immersion] of $\\mathbb RP^2$ (Boy surface), it would not prove the claim in the general case. Could you provide such a proof, or point to sources where a proof can be found? Detailed explanations would be greatly appreciated, since I am not an expert.\n", "answer": "This is not correct, and here is why.\nTake an immersion of $\\mathbb RP^2$ in $\\mathbb R^3$, for example, https://en.wikipedia.org/wiki/Boy%27s_surface .\nAfter a little perturbation, we can assume that the height function is a Morse function. After a further perturbation we can assume that this function has an even number of local maxima and minima. Indeed, we can always create additional pairs of critical points, one local max and one saddle. Moreover for any large enough $n$ there will be a perturbation of the original immersion with the total $2n$ local maxima and minima. \nAssume that these maxima minima are on different level sets. Let us pair them together in any way and join them by segments in $\\mathbb R^3$ along which the height function has no critical points. It is not hard to see then, that we can attach a cylinder to the surface that will go along the segment. We might need to create up to two Bott circles on this cylinder so that the condition on the absence of centres is satisfied.\nRemark. Note that both surfaces depicted on the figure are not immersed surfaces. The first one has one singularity at the top, the second has two singularities. \n", "topic": "mathoverflow.net", "url": "https://mathoverflow.net.stackexchange.com/questions/343606"}, {"image": "VQAonline_00044795.png", "question": "Is this a \"warped\" rotor?", "context": "I am experiencing some pulsating while braking in my 99 Ford Escort. After removing the wheel and inspecting the discs they both look like this.\n\nAre the lateral lines something that could cause the pulsation I am experiencing? Do these need to be replaced or just resurfaced?\n", "answer": "It very difficult to say if a rotor is warped by visual inspection. The tolerences involved are very small in the .004\" range. That is about the thickness of 3 or 4 human hairs. My recent experience (if you are doing your own repairs) is to replace rather than resurface. The thickness of used and resurfaced rotors may exceed the minimum allowed thickness, however they seem prone to quickly warping after being reinstalled. When you take the cost of having the rotors machined versus the cost of new ones it often isn't cost effective to reuse them. There are of course exceptions on vehicles with very expensive rotors but the average front wheel drive hubless rotor is pretty inexpensive. \n", "topic": "mechanics", "url": "https://mechanics.stackexchange.com/questions/7213"}, {"image": "VQAonline_00045433.png", "question": "Air chuck replacement", "context": "I have an air chuck similar to the one pictured. The gasket in the chuck at the end of the short hose is missing so it won't seal around a tire valve and as a result, cant inflate the tire.\nWhen I try to remove the chuck by turning the hex-nut-shaped point just below the chuck, the whole assembly just spins. I can't tell if the chuck is threaded on or if its some kind of compression fitting.\n\nIf anyone has any experience with these and can guide me on how to replace the chuck, or maybe even where I could find replacement little gaskets, it would be much appreciated.\nThanks to you all in advance.\n", "answer": "The joint at the end you're working with is a compression fitting and you'll not be able to remove it there. You might be able to remove it at the body of the air chuck, but I'd bet it'll break the body. Realistically, the type which is shown is considered a \"throw away\", which means, once it's broke or not useable anymore, you should just look to replace it. I get it, as it is sort of a waste ... but that's what way of things these days.\n", "topic": "mechanics", "url": "https://mechanics.stackexchange.com/questions/88631"}, {"image": "VQAonline_00044774.png", "question": "Central lock button blinking on Peugeot 407 when car is locked", "context": "When I lock my car, the central lock button (the one with padlock) on the panel starts blinking with a 1 sec. rate. I have check if the boot is closed, and it is (if it wasn't, it would also show an error on the display. \nIt's not that long ago since I bought, but I can't recall seeing this blinking earlier. Can anyone check if theirs are blinking as well, or does anyone know what's wrong, if there is anything wrong at all?\nPanel: \n\n", "answer": "Been told that this is normal. That it is blinking when car is locked.\n", "topic": "mechanics", "url": "https://mechanics.stackexchange.com/questions/3775"}, {"image": "VQAonline_00045759.png", "question": "What does Turbotax mean by \"This account did not have income, deductions, or credits associated with it in [year]\"?", "context": "I use Turbotax to prepare my taxes in the United States. When declaring foreign financial account, Turbotax asks:\n\nWhat does Turbotax mean by \"This account did not have income, deductions, or credits associated with it in [year]\"?\n\nWhat does it mean for a financial account to have an income, deductions, or credits?\n\n", "answer": "The question is asking: in your Form 1040, was there no income (interest, dividends, royalties, other income, gains), deductions, or credits that specifically pertain to this foreign financial account?\nFor example, for your foreign investment account that made $500 in dividends last year, you would have reported this interest as income on Line 2b Taxable Interest of Form 1040. You would then leave this box unchecked. For your foreign bank account that made $0 in interest, you would check this box.\nMore details: this screen in TurboTax is helping you fill out Form 8938, Statement of Foreign Financial Assets. Part III of this form asks you to aggregate all the income, deductions, and credits that pertain to all the foreign financial assets you're declaring on this form. Part V of the form asks you to provide further details on each foreign financial account. One of the lines in Part V is the checkbox you highlighted, Box 3d, which is labelled \"No tax item reported in Part III with respect to this asset\".\n", "topic": "money", "url": "https://money.stackexchange.com/questions/139395"}, {"image": "VQAonline_00045597.png", "question": "Forex & CFDs good profits, but why?", "context": "First off, i am a total newcomer to trading. I opened an account with an online broker, am using their App with a 10k demo account.\nI am under no illusions when it comes to trading. I am not interested in doing real trades until i spent at least 2 years in simulation. I don't believe in get-rich quick schemes although i understand it's possible, at high risk only though. I also understand that things would feel entirely different if that were real money at stake.\nI started doing some basic trades to see how things go. I quickly developed my trading style (i don't dare call it a strategy) \u2013\u00a0it does involve waiting out positions to recover sometimes. 1-2 days sometimes. I use TP but never used SL.\nThe basic approach is very amateur-like: i look at a chart of the past hours or days, i imagine where it will go without any knowledge of any economical background whatsoever (sorry), and then i just buy or sell, based on what i feel will happen. I usually never invest more than a 100 EUR in one sale/purchase, and i often wait for longer durations before even opening the App again. I just let it sit. I often have margins that leave me almost no funds to trade with, because i tend to open multiple positions of multiple instruments at once.\nOverall, in less than two weeks i was able to almost double my account value. However, i am surprised to find that i have very few losses.\nWhat i am trying to understand is, why it is that my way of holding on to positions for longer durations (i never got beyond the margin) has turned out to be so beneficial? Is that in any way reliable? Because almost every trade i made turned out positive, even though most trades first went the other direction and i just waited it out (which was possible due to margins).\nOr is it because i am using a demo account, and things are tweaked to look good and inviting, but look totally different with a real account? (i don't think i'm allowed to post the name of the App/platform here, so i refrain from that, but i guess most of you will know it).\nIf someone could enlighten me as to why this approach is good or why it is a totally silly idea which will lead me down into deep loss the moment i start trading with real funds, i would be glad to hear you out.\n(Please note: i have opened another demo account which i use via my smartphone, here i use the same \"strategy\" and was able to make EUR 6.000 from the 10k demo account in two weeks).\nI am posting my results page from the app (made screenshots and merged them together):\n\n", "answer": "This was all luck, that amount of leverage will destroy your account in a single bad trade.\nYou profit is way less than it should be because you are getting killed on fees. \nTake a look at the bitcoin trade, you should have 2,157.30 in profit but you only have 1825.42.\nAnd your currency trades were consistently positions that were worth $400,000 dollars, where you were pulling out ~$50 in profits, even though they should have been ~$80 profits.\nYou are consistently getting 30% less than you should be, and consistently betting waaaaay bigger amounts than you account can really handle.\nBad trades will probably have 30% greater losses than actual, and when the market moves the wrong direction then a single position will wipe out your account.\nYes, you could have just bought bitcoin and gotten great profits. You totally nailed the directions of the markets! It is just a matter of time before you blow up, the trailing profits won't always help you when the market starts going down first.\n", "topic": "money", "url": "https://money.stackexchange.com/questions/85897"}, {"image": "VQAonline_00045678.png", "question": "HSA eligibility when with employer plans", "context": "My employer offers two health plans:\n\nPPO with optional FSA\nHDHP (high deductible health plan) with an HSA option.\n\nThe plans have identical coverage, the difference is in the premiums, deductibles, and oop (out-of-pocket) maxes. The PPO has a deductable of 2k individual, 4k family and an oop max of 4k individual and 8k family. \nI am in the process of running the numbers, but so far it is looking like I am going with the PPO plan, after having been on some form of HDHP for the last decade.\nIf I do go with the PPO plan, am I allowed to open my own HSA (like the one with Fidelity: https://www.fidelity.com/go/hsa/why-hsa\n for example), since the deductable and OOP max is within the IRS rules for HSA eligibility? Here is the IRS publication: https://www.irs.gov/publications/p969\n\n", "answer": "In order to be considered an HSA-compatible High Deductible Health Plan, there are very specific features that a health plan needs to have (or not have). Since you have two options, and only one is labeled as \u201cHDHP with HSA option,\u201d it is most likely that the other plan is not compatible with HSA eligibility. The feature that usually trips up \u201cPPO\u201d plans are discounted copays before you hit your deductible. As you can see from the comparison images you provided, your PPO plan does indeed have copays which is why it isn't HSA compatibile. But if you still aren\u2019t sure, ask your employer (or the health insurance company), as they should be able to tell you if the plan is an HSA-compatible HDHP. \nIf you currently have money in an HSA, you won\u2019t lose that money if you become HSA ineligible. You would still be able to use that money on medical expenses until it is gone. You would just be unable to contribute new money to your HSA. \n", "topic": "money", "url": "https://money.stackexchange.com/questions/115059"}, {"image": "VQAonline_00046061.png", "question": "Star trek episode with Mr. Spock holding an \"exotic animal\"", "context": "I have found on 9-gag this still:\n\nwith Mr. Spock holding an \"exotic animal\" which is actually an annoyed dog in a unicorn-suit (sorta...). \nFrom which episode is it? Unless it is a photoshopped image (can't really trust 9-gag...)\n", "answer": "If you Google \"Spock Unicorn Dog\", it seems that this image is from episode called \"The Enemy Within\" (Season 1 Episode 5).\n\nScott assists in beaming a dog-like animal specimen from the planet, but he and the others are surprised when two identical creatures materialize - one completely docile and the other vicious. \n\nFrom IMDb's synopsis page:\n\nFirst, after repairs are made, they try the experiment with a unicorn-dog, which was split into two when it was beamed up. One half of the unicorn-dog was peaceful, the other was snarling and biting.\n\nAlso, this image appears on a site called thegeektwins.com in a caption contest where it specifically notes the source of the image:\n\nWhile the original Star Trek: The Original Series series had a lot of things going for it (a great cast, engaging characters, thought-provoking science-fiction, resonant themes), I don't think anyone would list \"big budget\" among them. Nowhere was it more evident than in \"The Enemy Within,\" when an alien creature was depicted by a little dog in a fur coat and a horn. \n\n", "topic": "movies", "url": "https://movies.stackexchange.com/questions/36326"}, {"image": "VQAonline_00046638.png", "question": "Scar on Justin's neck", "context": "After the shooting episode of season 4 of 13 Reasons Why, when Jessica hit Justin with a stapler, he got injuries on his forehead. But after that, till the last episode, there was a visible scar on his neck like that of skin infected with fungi. It was not a scar of injury. Did they show how he got it or was it something that the actor had and they chose not to cover it for the character too?\n\n", "answer": "So in an interview with the director, he explained that they were Kaposi's sarcoma (KS)\n\nLesions on the skin and/or internal organs caused by abnormal growth\nof blood vessels. In people living with HIV, Kaposi's sarcoma is an\nAIDS-defining cancer.\n\nHence, they hinted that Justin had HIV and the scars were willingly put on the character's skin.\n", "topic": "movies", "url": "https://movies.stackexchange.com/questions/109541"}, {"image": "VQAonline_00046318.png", "question": "Why didn't Jason kill his last victim?", "context": "In Friday The 13th, Jason pulled his last victim into the lake.\n\nBut why did Jason spare his last victim's life?\n", "answer": "This was mentioned in the comments, but needs to be an answer. \nThe scene described was just a dream sequence, and had no real bearing on the 'facts' of the movie, which were that Jason wasn't actually even alive in this movie, and that the killings ended with the death of his mother.\nUntil Part II that is!\n", "topic": "movies", "url": "https://movies.stackexchange.com/questions/70287"}, {"image": "VQAonline_00046720.png", "question": "The tonal function of a bar in minuet by krieger", "context": "This is Minuet by J.Krieger \n\nIf I look at the 6-7-8 bars, there is the following chord progression:\nE - E(7) - Am\nWith a trill in the 7th bar. I feel like the 6 bar does a kind of \"teasing\" in a tonal sense i.e from the dominant E you want to go straight to the tonic but the trill in the 7th prolongs the tension.\nIs my analysis correct? and is there a name for this \"structure\"?\n", "answer": "The analysis is correct. This piece is in A minor. The chord progression that you've spelled out (E-E7-Am or V-V7-I) is called a cadence. If you wanted to put a name on the structure you've noted it would be a cadential extension. Usually this term is applied to a longer delay in the return to the tonic (Am) chord but any place where the Dominant (V or E in this key) function chord is continued past the expected point of resolution could be considered a cadential extension. \nYou can find some more in depth information about the larger musical theoretical structures in Common Practice music in the book \"A Practical Approach to Form in Music\" by Peter Spencer.\n", "topic": "music", "url": "https://music.stackexchange.com/questions/3332"}, {"image": "VQAonline_00047779.png", "question": "Sheet music time signature question", "context": "I don't understand how the first measure only has one quarter note and how the third one has three quarter notes and a half note, both measures don't seem to add up to 4 quarter notes like the time signature.\n\n", "answer": "The first measure is called a pickup measure. The music stars on the fourth beat, so the music would start with rests. Sometimes people will put the rests in, other times (as here) they will leave them out. This is also known as an 'anacrusis'.\nThe quarter notes in the third bar are 'triplets' (three notes in the space of two) recognizable from the '3' with the bracket above them. Triplet are one type of 'tuplet'.\nA tuplet is:\n\nany rhythm that involves dividing the beat into a different number of equal subdivisions from that usually permitted by the time-signature (e.g., triplets, duplets, etc.)\n [Carl Humphries, The Piano Handbook, 2002]\n\n", "topic": "music", "url": "https://music.stackexchange.com/questions/100777"}, {"image": "VQAonline_00047076.png", "question": "A weird form of Anacrusis", "context": "I saw this in a piece the other day and was wondering if there is a term for it. The piece begins on a trill that ends on the first beat of a a measure. The rhytm then mimics a upbeat yet there is actually none.\nI'm wondering if this is a typical Spanish rhythms or something common to the Romantic era.\nFor reference here is the sheets.\n \n", "answer": "You are apparently confusing the notational peculiarity of an anacrusis right at the beginning of a piece (which is notated using a partial bar, usually without a bar number of its own) with the musical function of the anacrusis, a short leading phrase before its principal reference point, usually the beginning of a bar.\nBy far most anacruses don't interrupt an ongoing rhythm, the only slight exception being if they immediately follow a fermata. Even then it is unusual to not make the bar have its nominal length. Where that would not leave any room for the anacrusis, it's more common to add a full additional bar (starting with rests) rather than put down a double thin bar after the fermata and let the anacrusis start in a shortened bar right behind that.\n", "topic": "music", "url": "https://music.stackexchange.com/questions/41356"}, {"image": "VQAonline_00048244.png", "question": "Phrase opening, three decending octave intervals with timpani roll", "context": "I think of a big orchestral piece that starts or has this at a major phrase.\nTwo descending staccato octave intervals with a pause between on the 1st (8th) and 5th. Then a timp roll and a 3rd descending interval on the 1st.\nScored below.\n\nWhat is this piece? Might be Tchaikovsky.\n", "answer": "It's the scherzo, the second movement, of Beethoven's 9th symphony:\n\n\n\nNote the dotted rhythms; the tympani rhythm is actually the same rather than being a roll.\n", "topic": "musicfans", "url": "https://musicfans.stackexchange.com/questions/12141"}, {"image": "VQAonline_00048231.png", "question": "Song identification help, Big band tune", "context": "Had this stuck in me head for a couple of days now and it's really bothering me.\nCan you help identify it?\nI seem to remeber it starts with a bari or trombone maybe with a snare rim click in the background.\nI've written out how I think it goes.\n\n", "answer": "One of my big bands has just found the charts for this - and it's a great number! Birdland. \n", "topic": "musicfans", "url": "https://musicfans.stackexchange.com/questions/9857"}, {"image": "VQAonline_00048213.png", "question": "Identify the number of 11 Beethoven piano sonatas", "context": "I pursue a hobby called \"geocaching\". Currently, I am trying to solve a puzzle which requires that I identify the sonata number of 11 of Beethoven's piano sonatas, from sheet music (see attached image). As I am a neophyte when it comes to reading sheet music I'd be most appreciative if someone might be able to assist.\nUpon further scrutiny, there are several sonatas which appear twice. That would mean the solution has numbers in it that repeat.\nThanks in advance.\n\n", "answer": "It's unclear if you're looking for the opus numbers or the numbering from 1 to 32, so I'll offer both.\nIn order:\n\n4 (Op. 7)\n\n9 (Op. 14, No. 1)\n\n15 (Op. 28)\n\n9 (Op. 14, No. 1)\n\n8 (Op. 13)\n\n5 (Op. 10, No. 1)\n\n1 (Op. 2, No. 1)\n\n2 (Op. 2, No. 2)\n\n30 (Op. 109)\n\n4 (Op. 7)\n\n22 (Op. 54)\n\n\nThe images in your post are from the two volumes of the Beethoven Piano Sonatas edited by Heinrich Schenker; you can see the table of contents for the two volumes here and here.\n", "topic": "musicfans", "url": "https://musicfans.stackexchange.com/questions/5884"}, {"image": "VQAonline_00048248.png", "question": "What is the story of the Chimera?", "context": "The Neo-Hittite Chimera (850\u2013750 BC) from Karkemish is housed in the Museum of Anatolian Civilizations in Ankara, Turkey: \n\n\"Museum of Anatolian Civilizations080\" by Georges Jansoone (JoJan) - Self-photographed. Licensed under Public Domain via Wikimedia Commons.\nThis chimera is presumably a precursor to the more well known Greek chimera. Do we know of any stories associated with it? Is it mentioned in Hittite mythology?\n", "answer": "After researching on the topic for a considerable amount of time, it seems most of the sources which describe the Neo-Hittite Chimera, do not contain any myths about the creature nor any in-depth details.\nTherefore, it leads me to believe what another source said (emphasis mine):\n\n\"On the orthostats at Neo-Hittite sites, the Chimaera had no narrative context and served only to ward off evil, but in Greek myth the beast was to be conquered by a hero\"\nsource: http://cuartmuseum.colorado.edu/collection.bak/classics/greekvases/orientalizing.html\n\nWhich means, the Neo-Hittite Chimera was created just to \"ward off evil\" and hence does not have any myths associated with it.\n", "topic": "mythology", "url": "https://mythology.stackexchange.com/questions/610"}, {"image": "VQAonline_00048246.png", "question": "How and why was Talos killed?", "context": "In Greek mythology there was this ancient robot like (or god like) creature named Talos who was responsible for protecting Crete.\n\nIn the Image above we can see depicted the death of Talos. But who killed him? How and why was he murdered?\n", "answer": "When the Argonauts reached Crete, Talos - the bronze guardian of Europa - managed to push them back to the sea with ease. The Argonauts were desperate for food and water, and Medea decided to use her magic to kill Talos. She \"sent forth baneful phantoms\" and as Talos was trying to stop them from reaching Dikti (Europa's hideout), he accidentaly \"grazed his ankle on a pointed crag\", releasing the ichor from his single vein.\nHere's the story, as told in Argonautica:\n\nAnd Talos, the man of bronze, as he broke off rocks from the hard cliff, stayed them from fastening hawsers to the shore, when they came to the roadstead of Dicte's haven. He was of the stock of bronze, of the men sprung from ash-trees, the last left among the sons of the gods; and the son of Cronos gave him to Europa to be the warder of Crete and to stride round the island thrice a day with his feet of bronze. Now in all the rest of his body and limbs was he fashioned of bronze and invulnerable; but beneath the sinew by his ankle was a blood-red vein; and this, with its issues of life and death, was covered by a thin skin. So the heroes, though outworn with toil, quickly backed their ship from the land in sore dismay. And now far from Crete would they have been borne in wretched plight, distressed both by thirst and pain, had not Medea addressed them as they turned away:\n\"Hearken to me. For I deem that I alone can subdue for you that man, whoever he be, even though his frame be of bronze throughout, unless his life too is everlasting. But be ready to keep your ship here beyond the cast of his stones, till he yield the victory to me.\"\nThus she spake; and they drew the ship out of range, resting on their oars, waiting to see what plan unlooked for she would bring to pass; and she, holding the fold of her purple robe over her cheeks on each side, mounted on the deck; and Aeson's son took her hand in his and guided her way along the thwarts. And with songs did she propitiate and invoke the Death-spirits, devourers of life, the swift hounds of Hades, who, hovering through all the air, swoop down on the living. Kneeling in supplication, thrice she called on them with songs, and thrice with prayers; and, shaping her soul to mischief, with her hostile glance she bewitched the eyes of Talos, the man of bronze; and her teeth gnashed bitter wrath against him, and she sent forth baneful phantoms in the frenzy of her rage.\nFather Zeus, surely great wonder rises in my mind, seeing that dire destruction meets us not from disease and wounds alone, but lo! even from afar, may be, it tortures us! So Talos, for all his frame of bronze, yielded the victory to the might of Medea the sorceress. And as he was heaving massy rocks to stay them from reaching the haven, he grazed his ankle on a pointed crag; and the ichor gushed forth like melted lead; and not long thereafter did he stand towering on the jutting cliff. But even as some huge pine, high up on the mountains, which woodmen have left half hewn through by their sharp axes when they returned from the forest\u2014at first it shivers in the wind by night, then at last snaps at the stump and crashes down; so Talos for a while stood on his tireless feet, swaying to and fro, when at last, all strengthless, fell with a mighty thud. For that night there in Crete the heroes lay; then, just as dawn was growing bright, they built a shrine to Minoan Athena, and drew water and went aboard, so that first of all they might by rowing pass beyond Salmone's height.\nSource: The Project Gutenberg EBook of The Argonautica, by Apollonius Rhodius\n\n", "topic": "mythology", "url": "https://mythology.stackexchange.com/questions/218"}, {"image": "VQAonline_00048260.png", "question": "What was the parentage of the Korean King Suro?", "context": "According to Korean myth, King Suro of Geumgwan Gaya was one of six eggs that descended from heaven.\nDoes he have any parentage other then that? Any details about who laid the eggs? Or any other accounts of parentage?\nAnd why on this website are his parents listed as Jeonggyeon Moju and Lee Biga? And who are they?\nRegarding the Sansin Jeonggyeon Moju I found this depiction of her which is located in Simwonsa Temple in Gayasan National Park just outside Seongju, Gyeongsangbuk-do. \n\nIt gives no information about her except that she is a \"female Mountain Spirit ... that occupies Mt. Gayasan.\"\nAre there any more information about her and Lee Biga and any account of their parenting King Suro?\n", "answer": "There are two separate, somewhat incomptaible traditions regarding King Suro.\nThe first origin story comes from a now-lost work, but is quoted in the Samguk Yusa \uc0bc\uad6d\uc720\uc0ac (\u4e09\u570b\u907a\u4e8b) by the 13th century monk Iryeon. This states that a purple rope was spotted hanging from heaven; beneath the rope was a golden box wrapped in red cloth; and inside there were six golden \"eggs\" that are as round like the sun. Apparently, the next day these eggs \"turned\" into six boys.\n\n\u552f\u7d2b\u7e69\u81ea\u5929\u5782\u800c\u8457\u5730\u3002\u5c0b\u7e69\u4e4b\u4e0b\u3002\u4e43\u898b\u7d05\u5e45\u88f9\u91d1\u5408\u5b50\u3002\u958b\u800c\u8996\u4e4b\u3002\u6709\u9ec3\u91d1\u5375\u516d\u5713\u5982\u65e5\u8005\u3002\u773e\u4eba\u6089\u7686\u9a5a\u559c\u3002\u4ff1\u4f38\u767e\u62dc\u3002\u5c0b\u9084\u3002\u88f9\u8457\u62b1\u6301\u800c\u6b78\u6211\u5200\u5bb6\u7ab4\u69bb\u4e0a\u3002\u5176\u773e\u5404\u6563\u3002\u904e\u6d79\u8fb0\u3002\u7fcc\u65e5\u5e73\u660e\u773e\u5eb6\u5fa9\u76f8\u805a\u96c6\u958b\u5408\u3002\u800c\u516d\u5375\u5316\u70ba\u7ae5\u5b50\u3002\n\nThere is no indication of human-like parentage in this origin story - he is simply descended from heaven.\n\nThe second parentage comes from the Chos\u014fn era geography book Sinjeung Donggukyeojiseungram \uc2e0\uc99d\ub3d9\uad6d\uc5ec\uc9c0\uc2b9\ub78c (\u6771\u570b\u8f3f\u5730\u52dd\u89bd). This text names Jeonggyeon Moju the mountain goddess of Mt Gaya, located in the region of the Gaya Confederacy. Traditions has it that, while bathing in a pond, she attracted a divine voyeur: Ibiga, \"a heavenly god\".\n\nWhen she opened her eyes, Chonggyon saw the round shadow of the sun falling on the surface of the pond still like a mirror. Something sparkled momentarily. She suddenly felt ashamed. The moment she raised her head and looked up into the sky, her eyes met with the burning eyes of someone along with the sunlinght. At that instant, she cried, \"Ah!\" and felt the burning sttream of someone's vision penetrating her body. The burning light was none other than that of the sun. Although she tried to hdie her body with her hands in a hurry, it was too late. To be sure, she had already looked Ibiga straight in the face.\nPae-Gang, Hwang. Korean Myths and Folk Legends. Jain Publishing Company, 2006.\n\nAnd that apparently was enough to get her pregnant. Jeonggyeon Moju is said to have given birth to two boys: \u60f1\u5ba4\u6731\u65e5 and \u60f1\u7a92\u9751\u88d4. The former is identified as an alternative name for Ijinashi, the founding king of Daegaya, lit. Great Gaya; the latter is said to be King Suro.\nNot much else is known about the goddess. However, given her province, it is inferred that she was a kind of patron deity to the people who later made up the Gaya Confederacy. To this day, the Sanshingak shrine on the mountain performs rites every October, praying to her for national prosperity.\n", "topic": "mythology", "url": "https://mythology.stackexchange.com/questions/5296"}, {"image": "VQAonline_00048267.png", "question": "Free Art License Proper Method Of Attribution For An Image", "context": "I am working on an (as of yet) small project, a wallpaper generator that overlays SVG images like stencils on solid backgrounds, and I am unsure of how to properly go about licensing the art (largely under the Free Art License 1.3). \nThe section that is troubling me is:\n\nYou have the right to distribute copies of this work; whether modified or not, whatever the medium and the place, with or without any charge, provided that you:\nattach this license without any modification to the copies of this work or indicate precisely where the license can be found,\n specify to the recipient the names of the author(s) of the originals, including yours if you have modified the work,\n specify to the recipient where to access the originals (either initial or subsequent).\n\nAnd later summarised:\n\n\u2013 How to use the Free Art License? \nTo benefit from the Free Art License, you only need to mention the following elements on your work:\n [Name of the author, title, date of the work. When applicable, names of authors of the common work and, if possible, where to find the originals].\n Copyleft: This is a free work, you can copy, distribute, and modify it under the terms of the Free Art License http://artlibre.org/licence/lal/en/\n\nGiven that I'm generating the images client side, using a simple element, I cannot include this information in the png Description line, or anything like that\nPutting it in the corner of the wallpapers would be unseemly; so I am wondering if it would be enough to give the information in:\n\nIn the source of the web app and\nIn the application itself, say in a 'Licensing/Attribution' tab (this is how the art section of the GNU site appears to do it)?\n\n\nAny help that can be offered is greatly appreciated.\n", "answer": "\nIt is a copyleft license because any larger work that includes part of the work you received must be released, as a whole, either under the same license or under a similar license that meets stated criteria. Please don't use it for software...\n\nhttps://www.gnu.org/licenses/license-list.html#FreeArt\nYou might consider CC-BY or CC-BY-SA.\nAn example of attribution can be seen on Wikimedia Commons.\nI would also place the information in a README or LICENSE file,\nwhich is in the same folder as the art.\nI would also add the information to the equivalent project level file.\n", "topic": "opensource", "url": "https://opensource.stackexchange.com/questions/4620"}, {"image": "VQAonline_00048283.png", "question": "What does trademark use limitation mean in open-source licenses?", "context": "A lot of open-source licenses have a limitation on trademark use, meaning that they \"do not grant trademark rights, even though licenses without such a statement probably do not grant any implicit trademark rights.\"\n\nBut to whom does this limitation apply to? Does it prevent the trademark, such as the name and logo of the project from being used or modified; or does it stop contributors from registering their own trademarks regarding the name and logo of the project (or in other words, to patent the project).\nIs there a clearer explanation for the trademark use limitation?\n", "answer": "\nDoes it prevent the trademark, such as the name and logo of the project from being used or modified\n\nThe limitation does not prevent this - but this is because it is the default in law for any names, logos etc which meet the threshold to count as a trademark; it it likely that the names and logos of most open source projects do not meet that threshold. What choosealicense is doing here is classifying licenses into two sets:\n\nThose which make this explicit.\nThose which do not make this explicit and instead rely on the default in law.\n\nRealistically, the difference here is probably moot, but if it's something that you have a specific concern on for a specific project, you should take professional advice.\n\nor does it stop contributors from registering their own trademarks regarding the name and logo of the project?\n\nIt is very unlikely that contributors would be able to make a successful trademark claim for the name and/or logo of an open source project. They are, by their very nature, something available \"in the commons\" and not indicative of a particular relationship. So again, this is not prevented by the limitation but instead by the very nature of trademark law.\n\nor in other words, to patent the project\n\nPatents and trademarks are very different things. Do not confuse the two.\n", "topic": "opensource", "url": "https://opensource.stackexchange.com/questions/12134"}, {"image": "VQAonline_00048272.png", "question": "Using GPL licensed library over a service for non-GPL softwares", "context": "I want to use a GPL v2 licensed library(esper) for commercially distributed non-GPL softwares but I need to make sure non-GPL software not violate GPL license.\nCan I use this library indirectly using a wrapper service(can be a windows service, web service, rest api etc.) ? This wrapper service will provide a generic interface to utialize this library and will be licensed with GPL. Any commercial software can communicate with this service using its plugins. \nLets explain through the figure below.\n\nalternative image link : https://ibb.co/nRjxr7\n\nE is GPL licensed library that is I want to use. It is not modified in any way.\nC is my GPL licensed service that using E and running independently of A/B . Any client software can use it.\nA and B are my commercial web apps that will use C over adaptors. A and B can work without C. So there is not strong dependency for C. A, B and C are not running in the same process. They can work even on different machines.\nAdaptor A/B will call C over tcp/http. \nC service will provide generic operation interfaces. Adaptors will use the data contracts that provided by C. So, Adaptors needs to know what data models are supported on C operations\n\nIs this scenario appropriate? Is there any proper way to achive my problem ?\nHow should I distrubute Adaptor A/B ? Are they need to be GPL licensed ( A/B distrubuted with an .exe file to installation without containing adaptors)\nLastly, what is the way of I distributing C and mentioning in my commercial applications to tell users to install it and integrate with other applications. (I can specify with a description that explains how to utilize C service for the A/B users. Also I can make C program source and installation file available on my server for download.)\n", "answer": "IANAL/IANYL. That said, the devil, as usual, is in the details, and you don't give us much information about how these adaptors work.\nIf, for example, the B-C adapter is a piece of software that works by linking against libraries that form part of B, and also against libraries that form part of C, then you've created a single, large, derivative work of both B and C, and you have a big licensing problem on your hands.\nIf, instead, the adapter uses userspace as a mediating tool (eg, B puts files on the file system, then the adapter moves them into C's bit of filespace and calls C through fork-and-exec to deal with them, or C runs as a daemon that periodically checks whether there are new files to deal with), then B and C are separate works for copyright purposes, and you don't have a problem.\nThe one about your obligations with respect to C is pretty well covered elsewhere, but in brief, if you tell your users that they must download and install it, and you make both source and executable available from the same place on your website, that would suffice. If you distribute C directly to them, your obligations with respect to source provision are a bit more extensive, but not much so.\n\nEdit: you have clarified that both A and B adapters will interact with C via HTTP. Provided they do this using RFC-standard HTTP operations (POST, GET, etc.), that seems likely to me to be at arm's-length enough to make them separate for copyright purposes; certainly my web browser doesn't need to know about the GPL status of the web servers with which it interacts. If they're thus separated, then A and B adapters can be proprietary software which you distribute along with A and B.\nI'm still a bit nervous about \"Adaptors will use the data contracts that provided by C. So, Adaptors needs to know what data models are supported on C operations\", because I don't understand what you mean.\nEdit 2: given your subsequent clarification, it depends on how knowledge of the structures is acquired. Generally, if there is a separate written specification, and you implement this de novo in your code, all is likely to be fine (there has been at least one attempt to claim that an API can be copyrighted, but it wasn't hugely successful). If there is no separate written specification, and you read (in your example's) Amazon's library code to work out what the API is, or worse still, use their code as a basis for your own, there is a problem.\nI note that you have yourself written C, wrapping it around the pre-existing GPL library E. That puts you in a position to write that specification, but not to implement it. Clean-room practice is that person A reads the \"contaminated\" code and writes the functional spec, and a person B implements that spec, without any further reference to person A. You cannot be both of those people.\n\nNote that this whole analysis assumes that linking creates derivative works, and as we note elsewhere, this is not yet a certainty; arguments can be made both for and against the issue. It is generally accepted, however, that interaction-through-userspace keeps two programs far enough apart that neither is entangled by the other from a copyright point of view.\n", "topic": "opensource", "url": "https://opensource.stackexchange.com/questions/6394"}, {"image": "VQAonline_00048298.png", "question": "Dynamic programming example", "context": "\nI am going to buy a family car at the beginning of the New Year. I am going to stay in the UK\n for the next 4 years. I am considering the possibility of being a customer of company A which\n sells BMW models. Not only I can buy a car from this company but I can also trade in my car\n at this company and have my car serviced by the company. Company A gives a warranty that\n the annual maintenance costs and trade-in prices will remain the same for the next 4 years.\n They are: \n\n\\begin{array}{cc}\\hline\\text{Age of Car}&\\text{Annual Maintenance}&\\text{Trade-in Price at the}\\\\&\\text{Cost}&\\text{end of the period}\\\\\\hline0&{\\it\\unicode{xA3}}2,000&{\\it\\unicode{xA3}}6,000\\\\1&{\\it\\unicode{xA3}}4,000&{\\it\\unicode{xA3}}5,000\\\\2&{\\it\\unicode{xA3}}6,000&{\\it\\unicode{xA3}}4,000\\\\3&{\\it\\unicode{xA3}}7,000&{\\it\\unicode{xA3}}2,000\\\\\\hline\\end{array}\n\na) Suppose that I can buy a new car at \u00a312000, a price which is fixed for the next 4 years.\n What strategy should I choose to minimize the net cost (purchasing costs + maintenance\n costs - money received in trade-ins) incurred during the next four years? Given the optimal\n strategy found, do I have to solve the problem again if the company changes the price from\n \u00a312000 to \u00a3$x$ before I buy a car?\n\nSolution to a)\n\nQuestion: Could somebody please explain how do they get the values in the first table? Isn't the cost for using a car for 1 year supposed to be $2000+4000$?\n", "answer": "The \"cost\" values in the lower left table are calculated as the sum of the cost of buying the car ($12000$), plus the total maintenance cost of each year of owning the car, minus the trade in cost after the specified number of years has elapsed.\nSo, the total cost of owning the car for one year is $12000$ (price of car) $+$ $2000$ (maintenance cost during year $0$) $-$ $6000$ (the value of the car at the end of period $0$, i.e., after $1$ year) $= 8000$.\nSimilarly, the total cost of owning the car for two years is $12000$ $+$ $2000$ (maintenance cost during year $0$) $+$ $4000$ (maintenance cost during year $1$) $-$ $5000$ (value of the car at the end of period $1$, i.e., after the second year) $=$ $13000$.\n", "topic": "or", "url": "https://or.stackexchange.com/questions/3265"}, {"image": "VQAonline_00048290.png", "question": "Excel Solver linear programming - Is it possible to use average of values as a constraint without #DIV/0! errors or sacrificing linearity?", "context": "I'm trying to create an assignment optimization model where the areas are assigned to either the south or north school districts so that the total distance is minimized. Each school must have at least 1500 students, an average income of at least $85,000 and a minority % of at least 10%. \nThe issue I am having is that when I use solver to find a solution by changing cells G4:G13 (H4:H13 is calculated to be the opposite), there seems to be at least one iteration where the denominator of the average income of a school is 0 (in other words, no districts assigned to one school) and of course this causes a dividing-by-0 error. I tried adding a constraint to ensure each school had at least one district in it which did nothing to solve my problem and I also tried suppressing the error with =IFERROR() which only made the model non-linear. \nI need to use the Simplex LP method in solver for this assignment. Is there a way I can add these \"Average\" constraints without issue? \n\n", "answer": "Instead of\n$$\\frac{\\textrm{Total school income}}{\\textrm{Number of areas}} \\ge \\$ 85000$$\nyou could have a constraint\n$$\\textrm{Total school income} \\ge \\$ 85000 \\times \\textrm{Number of areas}.$$\nIn this case, you won't have any problems when the number of areas is zero. It also makes the model linear instead of non-linear, which is usually a good thing.\nAn alternative would be to add constraints to require that each district has at least one area.\nI would probably use both suggestions at the same time.\n", "topic": "or", "url": "https://or.stackexchange.com/questions/2615"}, {"image": "VQAonline_00048345.png", "question": "Transform nonlinear cost function to get LP or MILP", "context": "I'm trying to schedule power of multiple prosumers in a microgrid.\nThe problem includes a cost function with min and max statements since I want to distinguish between the prices for importing and exporting energy.\nThe following is a minimal example for the cost function without summing over all timesteps at the moment:\nI have two decision variables $P_{charge}$ and $P_{discharge}$ for the battery storage (one of it is assumed to be always zero) and one other controllable load $P_{load}$. Consumption is denoted with positive and generation with negative values. $C_{import}$ and $C_{export}$ are the per unit prices for importing and exporting energy.\n$C = max((P_{charge} + P_{discharge} + P_{load}) \\cdot \\Delta T,0) \\cdot C_{import} + min((P_{charge} + P_{discharge} + P_{load}) \\cdot \\Delta T,0) \\cdot C_{export}$\nThe min and max functions form if-else statements to check if the sum of power flows is negative or positive.\nIs it possible to introduce some binary decision variables to transform it into a MILP? I want to use a Matlab solver later.\nThanks for every hint!\nEdit 1:\nI have tried to introduce the proposed auxiliary variables but they do not match the desired values I computed from my simulation. So maybe I have to clarify the problem. At first, I simplify power flow of battery to one bidirectional power variable to reduce the problem further.\n\nThe image shows a part of my problem.\nMy objective is to maximize $R - C$, where $R \\ge 0$ is revenue and $C \\ge 0$ is cost. Per unit prices are always greater than or equal to zero and $C_{import} > C_{export}$ is valid for now. The nonlinear version would be:\n$$R = min(P_{load} + P_{BSS},0) \\cdot (-1) \\cdot C_{export}$$\n$$C = max(P_{load} + P_{BSS},0) \\cdot C_{import}$$\nI have defined non-negative decision variables $X_{import}$ and $X_{export}$ with the line capacity (energy per timestep) as upper boundary. Additionally I've introduced following equality constraint:\n$$(P_{load} + P_{BSS}) \\cdot \\Delta T == X_{import} - X_{export}$$\nI neglected the injected PV power at this node for now. My goal is to be somehow able to use the auxiliary variables for an upper level node.\nMaybe I didn't fully understand the proposed answers which might led to the undesired result.\n", "answer": "Taking a different tack from Rob, I'm going to assume that the original objective function $C$ is correct as stated and is being minimized, with $C_\\text{export}>0$ being a per-unit compensation value for exported energy (i.e., you pay to import and get paid to export). In that case, splitting the energy expression into $X_\\text{import} - X_\\text{export}$ as Rob does will work if $C_\\text{import} > C_\\text{export}$ but will not work with $C_\\text{import} < C_\\text{export},$ since in the latter case the solver could add an arbitrary amount $\\delta > 0$ to both $X_\\text{import}$ and $X_\\text{export},$ keeping the energy value the same while reaping an undeserved profit of $(C_\\text{export} - C_\\text{import})\\delta.$ In this case, you have to introduce a binary variable $Y$ and constraints $$X_\\text{import}\\le M_\\text{import}\\cdot Y$$ and $$X_\\text{export}\\le M_\\text{export}\\cdot (1-Y),$$ where $M_\\text{import}$ and $M_\\text{export}$ are upper bounds on the amount of energy that could be imported or exported.\n", "topic": "or", "url": "https://or.stackexchange.com/questions/8669"}, {"image": "VQAonline_00048615.png", "question": "Silva Polaris compass, use of the second dial numbered from 0 to 7", "context": "\nI am looking for the use of the second (outer) dial numbered from 0 to 7 on the Silva Polaris compass.\nThis second dial rotates independently of the inner dial.\n", "answer": "It is used to measure distance. You turn it on its side and roll it along a path on the map. This means it has to correspond to a particular map scale (my first guess is 1:50,000, but that looks a bit off). For a different scale you could do a simple conversion.\nCan you tell me what the arc length of one major division is? From the picture, it kinda looks like a little more than one inch (maybe 1.25\" ?? ). I can get the associated scale from that (i.e. what is the arc length between 6 and 7)\n", "topic": "outdoors", "url": "https://outdoors.stackexchange.com/questions/25376"}, {"image": "VQAonline_00048484.png", "question": "What Suction Cups Were Used To Climb Trump Tower?", "context": "There is a video here.\n\nBasically I won't say I'm planning to do the same thing but would like to find out more behind how he did it. Someone told me the suction cups were intended for construction and not climbing but I'm not really sure.\nThe following which looks similar requires an air compressor. If it requires a small air compressor how did Steve do it?\n", "answer": "It looks like they were four NV11 Fairs Air Powered Suction Cups as pointed out by this article.\n", "topic": "outdoors", "url": "https://outdoors.stackexchange.com/questions/13803"}, {"image": "VQAonline_00048478.png", "question": "What is a so called \"Weiderost\" (cattle guards)?", "context": "Last time we visited Austria for holidays we've recognized devices called \"Weiderost\" (literally translates into pasture grate). It's a coarse meshed grate laying in the street. It can be found where a fence is crossing a street. The grate kind of connects the two ends of the fence (the fence can't just pass it of course because driving and stuff). We know that it's called \"Weiderost\" because there were signs telling us so - \"Caution! Weiderost!\" - every time we came across one. \n\nWhat is this used for?\n", "answer": "There are different kind of obstacles so that farm animals like cows on alpine pastures stay in the related territory. If you hike, you often pass little doors or Z-shaped wooden obstacles or have to lift gates.\nFor dirt roads this is similar, the cattle needs to be restricted so e.g. with a \"Weiderost\". In comparison to a gate you can always overcome the bars by car, but cattle and sheep aren't likely to overcome. So they are often build on roads for agricultural use or the kind, where they have to drive by regularly.\n", "topic": "outdoors", "url": "https://outdoors.stackexchange.com/questions/13444"}, {"image": "VQAonline_00048651.png", "question": "What are these? Received from children's centre in free pack", "context": "A friend received this baby pack from our local Sure Start center (they're a UK children's initiative for those who don't know). It contained various, obviously useful items for people with a baby and these (pictured). We don't know what they are but we're assuming they're some kind of baby proofing device? Does anyone recognize what they are? There's no information or instructions in the pack.\n\n", "answer": "Those look like something for wrapping up and securing draw cords on blinds. You would wrap the cord between the diamond shaped piece and the long piece. You can then insert the cord into the holes in the diamond piece to keep it from coming unraveled. This way you can shorten the strings so kids can't choke themselves on it.\n", "topic": "parenting", "url": "https://parenting.stackexchange.com/questions/16232"}, {"image": "VQAonline_00048664.png", "question": "How to child-safe a ball faucet", "context": "We've got a ball faucet1 in our kitchen, and a toddler (2 years, 10 months) who is really into playing in water and strong enough to haul around chairs - so we're regularly having to get her out of the sink and clean up the huge puddle she leaves behind.\nI've been utterly unable to find a way to child-safe this thing. Searching Google leads to a bunch of articles on how to repair them (apparently, they're prone to leaking), or information on bar faucets (which have a similar name).\nDIY'ing it has also been a big let-down because, while I can secure it's vertical movement fairly well, it takes a surprisingly small amount of twist for the water to start flowing.\nDoes anyone know how to secure one of these?\n1: While not the exact model, this is a good example of the type, and very close to what we have.\n\n", "answer": "After a little discussion in the comments, I'll put forward one suggestion.\nI'd recommend looking for something like this (on amazon) for in front of the sink. It's a memory foam, non-slip bath mat and they're sometimes on offer at places like Costco (UK), so I believe cheaper models are available. That one is fuzzy, but I've seen an equivalent that is more rubbery and smoother (more like wipe clean). In my experience, they're like big, heavy, rubbery sponges. They're quite thick and actually quite fun/comfortable to stand on, so it might provide a nice distraction, too.\nIt'll (hopefully) do two jobs:\n\nIt might prevent the chair from being moved in front of the sink. If she's dragging the chair rather than lifting it, the non-slip and slightly higher profile of the bath mat will make it harder to get it in front of the sink (buying you time to catch her moving the chair).\nIf the chair makes it into position and the water flows, it'll catch at least some of the puddle. Because the mat is spongey, do be warned that it might make the chair less stable.\n\nI'd also recommend \"rug gripper tape\" to keep the mat in place if it's too easy to move (in my experience, they have rubber backing and are quite \"solid\" so you might not need this), and I'd install the mat when she's not looking and say it's for mummy/daddy's comfort on their feet when they have to stand at the sink. I'd probably waddle over it a few times for fun at this point, too - I genuinely do like the feel of these things!\nAnd, I'm sure you've already done this, but check the thermostat on your hot water is high enough for comfort but low enough for practicality.\n", "topic": "parenting", "url": "https://parenting.stackexchange.com/questions/42092"}, {"image": "VQAonline_00048645.png", "question": "Silicone bottle nipple, usually has a whitish film after washing and drying? Why is this the case?", "context": "I've notice that after we wash & air dry our silicone nipples, frequently they will have a light whitish kind of film on the inside. This film survives boiling water sterilization too. I would then have to wash it again with a lot more soap to remove this.\nWhy does this happen? Is it from the milk fat?\n\n", "answer": "The milk fat is easily washed away, so I would rule that out. \nI think what you're seeing is the dried calcium of the tap water, especially if it seems powdery and can be rubbed off when dry. I think this is what user77907 means by scale. This is especially the case if your tap water is very hard. \nYou need not worry about these deposits: according to Wikipedia, the World Health Organization says that \"there does not appear to be any convincing evidence that water hardness causes adverse health effects in humans.\" \nI installed a water filter for drinking water (and for sterilization, coffee maker, etc.) in my kitchen, and all appliances as well as washed/sterilized items no longer have these deposits. If you're concerned about the purity of your tap water, I recommend this filter from APEC Water Systems; it's the best I could find, and many alternatives were much more expensive.\n", "topic": "parenting", "url": "https://parenting.stackexchange.com/questions/4400"}, {"image": "VQAonline_00048708.png", "question": "What information should we leave for a cat sitter/cattery?", "context": "I keep a single sheet document on each cat that gets printed out whenever they are left in someone else's care (a cat sitter, a family member, taken to be boarded, etc). What information should be provided on this sheet that may be relevant in case of an emergency?\nI've attached an example of my current form, with the personal information redacted.\n\n", "answer": "Everyone is going to be a little different, but with respect to your current form (which I quite like), I would consider:\n\nRemove the \"Rescued From\" information, unless it's relevant to the care in which case the relevancy should be indicated.\nAdd a \"willing to spend\" line by the vets to indicate how much cost you would be willing to bear in an medical situation. Our vets had this on their boarding form and I think it's wise, even if you intend no limit.\nAdd an \"in the event of death\" line for to do the event that your pet dies. Not a pleasant thing to think about, I know, but consider how the person caring would react if something does happen. \nAdd a feeding schedule along with any instructions (for ours we need to add water to the wet cat food to deal with a megacolon issue).\nAdd a treat schedule or some basic instructions on how much treats can be given.\nAdd a medication schedule and instructions. For example, our boy gets 3 ml of lactulose daily orally before evening feeding. Even if you don't have any medication to give, saying so is a good thing.\nAdd an alternative contact (if necessary). When we boarded ours at the vets, we had my mother as a contact for any decision making. Now, that depends on how much decision making you want to leave up to the sitter. As an aside, if you do have a sitter, notifying the vets that this is the case and whether or not they can make certain decisions isn't a bad idea.\nAdd a general \"notes\" section for basic behavioral information and what not.\n\nThat's all that I've managed to come up with.\n", "topic": "pets", "url": "https://pets.stackexchange.com/questions/7478"}, {"image": "VQAonline_00048813.png", "question": "Is it possible to tell shrimp eggs if they\u2019re cross bred?", "context": "\nI have noticed that my shrimp are recently becomin g pregnant, also I keep my shrimps in one 40 liter tank so I wonder if it\u2019s possible to tell by only the eye if the eggs are cross bred.\nYou may be interested in the species I had and currently have.\n1 week ago:\n10 Red cherries shrimp\n1 Fire Red shrimp\n1 Red Rili shrimp\n3 Blue velvets shrimp\n3 Sunkist shrimp\nAfter my nasty Otto mass murder, I have...\n6 Red cherry shrimp\n1 Fire Red shrimps\n3 Blue velvets shrimps\n2 Sunkist Shrimps\nSo, now you know that, I have 2 pregnant Sunkist shrimp who have eggs just after the killings, and 1 pregnant Red Cherry shrimp who has had eggs 2 weeks before the killing.\nI have seen the pregnant RC shrimp hanging around the BV males often, But I have no idea what the Sunkist shrimps Mates could possibly be.\nI just wonder what their babies would look like, or if their a cross breed.\nExtra Information:\nPh Levels - neutral\nAmmonia levels - N/A\nNitrate levels - N/A\nCatfish included: yes\nThe nasty Otto, a Oil Tatia and a Pleco\nI would like to hear the answers and find out soon! There is a new tag for, \u2018shrimp\u2019 now.\n", "answer": "I'll preface with the fact that I have not kept shrimp for very long, and I've not bred them. But I have done a lot of research on the fact (as much as I could, there's not a lot of easily accessible, reliable resources out there), since keeping varying types of shrimp together runs a number of breeding risks.\nYou really can't tell who bred with who at this point, not until the shrimp eggs hatch, and the shrimps mature, which is only a matter of about 2 months (varies per type).\nHave you sexed your shrimp? You could make an educated guess based on your female to male ratios. (I.e., if your 1 Red Rili was female, you can remove that shrimp as a possible parent of your current eggs.)\nIt's important to know that it's not uncommon for crossbred (breeding between different species) shrimp to return to \"wild\" coloring (brown), instead of taking on one of a parent's trait. Interbreeding (breeding between same species) is more likely to keep the vibrant colors.\nThere's a number of helpful interbreeding and crossbreeding charts you should check out, which can help you narrow down if who might have bred with who, if some parings wouldn't be possible. I felt that this article was particularly useful, and has a chart linked at the end: Interbreeding Chart. For example: it is possible for your Blue Velvet to have bred with your Red Cherry, but is highly likely to produce a brown shrimp.\n(a lot of shrimp have varying names, or are falsely labeled, so you may need to look up the specific genus of each of your shrimp to match them up on that chart.)\nI'd also like to note that, if you have a fish that is killing your shrimp, the baby shrimp may not live long enough for you to tell. I would recommend keeping your breeding and baby shrimp separate from the fish.\n", "topic": "pets", "url": "https://pets.stackexchange.com/questions/29446"}, {"image": "VQAonline_00048742.png", "question": "I think my cat broke it's leg", "context": "My cat got its leg trapped in a door and his leg is swollen. It's the leg on th upper part of the picture. When it first happened he could put a little bit of pressure on it but now its swollen and won't put any pressure on it. I have given him some food and he can walk but he limps. He doesn't seem to be in pain but when I touch his leg he screams. For this reason I have not felt the leg to see if it's broken. He also has a small cut on the side of his leg where he trapped it in the door. I was wondering if anyone could tell me if it's broken and tell me what to do other than take him to the vet tomorrow.\n", "answer": "Take him to the vet tonight!\nIf there is absolutely no potential for an emergency vet. Do your best to keep him inactive, until you can reach the vet first thing tomorrow morning. \n", "topic": "pets", "url": "https://pets.stackexchange.com/questions/16355"}, {"image": "VQAonline_00048857.png", "question": "Is physics semantically deterministic?", "context": "\nThe Philosophy of Mathematics Today, By Matthias Schirn, p518\nIs physics today about a \"single object\"? Or is it \"indeterministic\"?\n", "answer": "The most popular opinion right now is that physics is deterministic, simply because that assumption has lead to great success. We don't know whether it is actually true or not, but we do often assume it.\nHowever, in all real world cases, we do not have perfect information about the system. Thus real world physics' predictive power is less than perfect, even if the underlying theories were perfect. This would suggest that, in real life situations, it is effective to treat physics as a nondeterministic process.\n", "topic": "philosophy", "url": "https://philosophy.stackexchange.com/questions/36356"}, {"image": "VQAonline_00048891.png", "question": "What is the name of this fallacy: 101 is either binary or decimal?", "context": "\nA similar example would be of binary and decimal number where we are not sure about the number system.\nFor example, The person who proposed a number is not remotely available to disclose about number system.\n\ne.g. 101. Is 101 decimal? [T/F] or Is 101 binary? [T/F]?\n\nWhen context is not clear or not given, so, it could be one hundred and one in decimal, or it could be a decimal representation of five in binary.\n\nIs it even a fallacy, if so then which one?\n\nIt is like an argument without proper context or domain.\n", "answer": "EDITED 9/11/2020\n\nWhat is the name of this fallacy: Is 101 binary or decimal?\n\nI prefer to call it the either-or fallacy, but it is also known as false dilemma among others. The idea is that a choice is given that constrains to two or perhaps a few options when a much broader reading of responses is both possible and warranted. In your example, 101 is a number in base-2 and base-10 and every base in between and beyond.\nIn regards to your picture, it highlights the likely cognitive source of the fallacy, and that is ambiguity. For instance, it is ambiguous when one writes 101 and does not explicitly explain which base it is.\nWhat's the difference between ambiguity and a false dichotomy? Well, ambiguity is a lack of context to indicate the intention of the author, whereas a false dichotomy is an argument that exploits the lack of context. To be technically precise, an argument must have two or more premises and result in a conclusion, and many logicians accept that unstated premises can be reasonably understood by context. For instance, one could posit that an either-or fallacy is essentially understood as:\nP1: 101 could be binary.\nP2: 101 could be decimal.\nTherefore, 101 is either binary or decimal.\nSo, technically the question alone independent of context might not be accepted as an argument and therefore fallacy, but ANY act of drawing a conclusion regarding 101 as binary or decimal and presuming those are the only two choices is clearly a fallacy.\nNote, to resolve the ambiguity, mathematicians generally use brackets or subscripts or both to indicate base. For instance [101]_2, [101]_10, and [101]_16.\n", "topic": "philosophy", "url": "https://philosophy.stackexchange.com/questions/76231"}, {"image": "VQAonline_00048860.png", "question": "Why are 2 Constants introduced in proving: \u2203y \u2200x (x = y) \u2234 \u2200y \u2200x (x = y)?", "context": "Source: Sweet Reason: A Field Guide to Modern Logic (2010 2 ed) by Henle, Garfield, Tymoczko.\n\n[Question, p 330, Section 12.1.] [Justify:]\n 7. \n [Premise 1:] \u2203y \u2200x (x = y)\n \u2234 \u2200y \u2200x (x = y) \n[Answer: p 379, also online.]\n\n\nI understand, and so ask not about, steps 1-3 and the justification for each step that is clearly stated. Instead, probably because I do not understand the intuition behind this argument, I do not understand the proof strategy, especially 4, 4.1-4.3.3. E.g.:\n\nWhy must we introduce b and c in 4.1 and 4.2 (as Assumptions for Conditional Proof)? Why not only one Constant? Why any at all? \nIntuitively, how does \u2203y expand and so universalise itself into \u2200y? \n\n", "answer": "The intuition is that if there is something (y) that is equal to everything (x), then everything must be equal to everything by transitivity of equality.\n\na is the variable that is known to exist because of the \u2203 quantifier in the premise.\nb and c are the x and y in the conclusion. a cannot be used here, because it is not 'clean' (it comes from 1.).\n\nBoth b and c are known to be equal to a by application of 3 (4.2 and 4.3.2). By transitivity of equality, then also c = b (4.3.3).\nYou could see 4.1 and 4.3.1 as eliminating the \u2200 quantifiers in the conclusion. On both instantiations, 3 is applied to achieve equality with a. Then transitivity is applied in 4.3.3.\nTo concretely answer your questions:\n\nWe have to eliminate two universal quantifiers, so we need two clean variables. a is not clean, because it is introduced by \u2203-elimination.\nI'm not entirely sure what you mean here. The variable introduced from the \u2203 quantifier is not universalised, because a clean variable is needed for that. a is used in the transitivity step (4.3.3.). This step basically says b = a = c.\n\n", "topic": "philosophy", "url": "https://philosophy.stackexchange.com/questions/37333"}, {"image": "VQAonline_00049336.png", "question": "Who is the photographer of this black and white photo of two female nudes standing over a pile of clothes?", "context": "A print was given to a friend long time ago and he vaguely remembers that he was told the artist was German. I feel like I have seen it somewhere before. I have tried searching by the below image in both Google and Bing and got no good result. Does anyone know some information about the photographer?\n(Warning: NSFW image behind spoiler)\n\n \n\n", "answer": "I don't know anything about the photo but did do a bit of digging on Google.\nGoogle Images has an option to search by image that returned the following results: Here\nThis led me to what looks like an art auction website that is in German: Here\nFinally, the page did not resolve with the pictures, but it did have the alt text visible. Down at the very bottom one of the images has the text \"Albert Watson 1942 - Two nude models\".\nAlbert Watson is a Scottish photographer well known for his fashion, celebrity and art photography; more info at wikipedia here.\nFor these reasons, I believe this image is from the artist: Albert Watson and is possibly titled Two nude models\n", "topic": "photo", "url": "https://photo.stackexchange.com/questions/61552"}, {"image": "VQAonline_00048997.png", "question": "How to photograph a snowy landscape?", "context": "I am about visit Nepal for 8 weeks, planning to trek the Annapurna circuit, Everest Base Camp and Imja Tse. Most of the landscape is extremely bright and snowy.\n\nCredit: Gianni Scopinaro\n\nHow do I avoid getting the snow appear as uniform white surface? \nIs there a difference between close-ups and scenery?\nDo I need to purchase equipment, like polarizers?\n\n", "answer": "In the photo you put in the question, note how the foreground is extremely underexposed. This is because the exposure metering was made relative to the snow. However, if you set the snow to the standard exposure, you will get a \"dark\", gray snow. You need to add about 2 stops with Exposure Compensation (or use manual mode) in order to get a bright white snow.\nA polarizer can certainly help by eliminating some of the specular reflections from the snow, to make it more \"uniform\".\nUpdate: If you wish to shoot people on a snowy background, you can easily run into the problem of overexposed snow or underexposed persons. Except for using grad-ND filter, like mentioned in the other answers, for reducing the brightness of whatever is above your subjects, you need to find other, smart ways to equalize the brightness levels.\nYour options are to try shooting when the person is facing the sun, to use large reflectors or to use active lighting like (off-camera) flash.\nAnother link on the subject: How to cope with high contrast?, and see Matt's answer.\n(*) I am assuming you know how your light meter works and how to use it.\n", "topic": "photo", "url": "https://photo.stackexchange.com/questions/15591"}, {"image": "VQAonline_00049174.png", "question": "Upgrading EOS 350D or changing to full frame for low-light improvement?", "context": "My question is as follows: In 2006 I bought a Canon EOS350D with a Tamron 18-200, which was a great choice for shooting pictures on vacations, not having to carry around much, etc. Now, this seems obviously not the best choice for taking low-light pictures without flash. Here is an example with ISO 1600 and quite some noise.\n\nI now think about upgrading my camera/replace it to improve the low-light performance. Two strategies showed up when browsing the web (both quite expensive):\n\nMany seem to \"Canon EF 24-70mm f/2.8L II USM\" (about 2000\u20ac). (Will it be compatible?)\nAnother strategy seems to be bying a full frame like Canon EOS6D (about 1700\u20ac). Here my old lenses will not be compatible, so I'll need a new one, the above one would be perfect, but unfortunately not in my budget. What would be possible then (but already slightly over my budget) is the Canon EF 24-105mm f/4L IS USM (1200\u20ac)\n\nWhat would be preferable of the above two options? Do you have other suggestions, especially with lower budget (around 1000\u20ac), e.g. would it be worth upgrading to the Canon EOS 700D?\nAdditional information: I also own a Sigma 10-20. The pictures of the series from which the above picture is taken had focal length 18-95. If you need more additional information, please ask in the comments.\n", "answer": "Here's the dilemma for someone in your situation: Upgrading the camera will have much less of an impact than it could when still using your current lens, yet upgrading the lens when still using your 350D will limit the improvement as well. I think the lens needs the improvement first, and here is why:\n\nLenses with more than a 3X ratio between the shortest and longest focal lengths have many design compromises. In addition to distortion and chromatic aberration, narrow apertures are common with these lenses. For not much more than the EF 24-105mm f/4L IS you are looking at you can buy the Tamron AF 24-70mm f/2.8 Di VC. It is a little sharper than the original Canon 24-70mm f/2.8 L and is both sharper and a stop faster than the 24-105. Your image quality will improve considerably, even with the 350D.\nAlthough there has been a lot of improvement in high ISO/low noise performance in low light, in the Canon realm most of it at the sensor level has been on the full frame side of things. I own a Rebel Xti/400D, a 50D, and a 7D. In terms of low light performance, there isn't a lot of difference between any of those APS-C models when compared to the clearly superior 5D Mark II. This is particularly true when I use the NR tools in today's software versions on RAW files captured with the older bodies instead of comparing the output from the newer models to shots from the older cameras processed by the older versions of the software. Canon traded the improvements in their Digic processors and firmware/software's noise reduction ability for the smaller pixels used for increased resolution. Your 350D has a sensor with pixels that are 6.4\u00b5m wide, comparable to the 6.4\u00b5m pixels on the 5D II. The current crop of APS-C Canon bodies, from the Rebel T2i to T5i, the 60D, and the 7D are all based on the same sensor with 4.3\u00b5m pixels. The biggest difference between those models are the focus systems, handling speed, and video capabilities - not the basic image quality. If you are intent on upgrading to a Canon APS-C body, I would encourage you to wait until Canon releases a new body with the next generation APS-C sensor. It is long overdue!\u00b9\n\nTo get really good low light performance in an environment like your example picture, you're going to have to forget zoom lenses altogether and go with a fast prime lens and a full frame body. Other than the sweet spot around 50mm (due to the way lenses can be designed for cameras with the typical flange distances of DSLRs), wider aperture lenses get expensive very fast. The further away from 50mm you get, the more expensive lenses with comparable apertures become.\nIf you are serious about improving your low light performance capability on the budget of what an EF 24-105mm f/4 L IS costs, I would suggest looking at used FF bodies like the original 5D and fast primes such as the EF 85mm f/1.8 or the EF 100mm f/2. For 50mm I like the f/1.4\u00b2 and feel the faster auto-focus and more usable manual focus as well as the durability are worth the difference. Others feel the f/1.8 is a better value. Be sure to buy from reputable dealers like B&H or through places like amazon marketplace where you have recourse if someone sells you a lemon.\n\u00b9 Canon's release of the 80D in 2016 finally gave Canon shooters an APS-C sensor with better low light performance.\n\u00b2 When Canon introduced EF 50mm f/1.8 STM they corrected most of the shortcomings of the older EF 50mm f/1.8 II. I would even recommend it over the EF 50mm f/1.4 for the vast majority of users (including myself if I were in the market for a new 50mm prime for less than $1,000).\n", "topic": "photo", "url": "https://photo.stackexchange.com/questions/40139"}, {"image": "VQAonline_00051012.png", "question": "Coulomb's law with an $r^3$, not $r^2$, in the denominator", "context": "I am reading an older physics book that my professor gave me. It is going over Coulomb's law and Gauss' theorem. However, the book gives both equations with an $r^3$, not $r^2$, in the denominator. Can somebody please explain why it is given as r^3? An image is attached for reference.\nAlso for equation 1-24, can somebody please explain how the middle side is equal to the right side with the del operator?\n\n", "answer": "It does give \"Coulomb's law\" with $\\frac{1}{r^3}$, it gives it in its proper vectorial form\n$$ \\vec E \\propto \\frac{\\vec r}{r^3}$$\nwhich, when taking the absolute values, yields the form you are probably more familiar with\n$$ E \\propto \\frac{1}{r^2}$$\nsince $\\lvert \\vec r \\rvert = r$.\n", "topic": "physics", "url": "https://physics.stackexchange.com/questions/194894"}, {"image": "VQAonline_00052956.png", "question": "Some Holes formed when water is put on flat plate", "context": "\nWhy such holes are formed when we put water on a plate\nWhat is the relation can be derived between h(height of standing water) and r(radius of hole).(if P\u2080 is atmospheric pressure,\u03c3 is surface tension between water and container,\u03c3\u2080 surface tension between fluid molecules,\u03bc being coefficient of static friction,g acceleration due to gravity and \u03c1\u2080 is fluid density and any other constant if required)?\nWhat all forces we need to consider while solving this problem.\n", "answer": "As a first guess it will be the result of surface tension of the water , and irregularities on the plate allowing bubbles to form when the water is poured.\n", "topic": "physics", "url": "https://physics.stackexchange.com/questions/531027"}, {"image": "VQAonline_00052572.png", "question": "How do two waves after approaching and neutralising each other give birth to two waves the very next moment?", "context": "\nMy question is after the third event(in the picture) how do the waves originate again? \nAn obvious answer might be that two waves simply propagate and just add up when they interfere with each other. But if we take a closer look, in the third position, all the velocities are perfectly cancelled out. Then why should another set of waves originate?\nWouldn't it be mysterious to an observer who starts observing from the third position? \nCan anyone give me a mechanical interpretation? Where am I making the mistake?\n", "answer": "At the third position the different parts of the string still have vertical velocities that aren't shown.\nThe wave equation is second order in time, so you need to specify the vertical velocities of each part. Not just the position.\n", "topic": "physics", "url": "https://physics.stackexchange.com/questions/469434"}, {"image": "VQAonline_00054224.png", "question": "How to resolve \"Circular dependency\" in Microsoft Project", "context": "I am working in Microsoft Project for creating Low level plan in .mpp files. \nI am having the circular dependency error as shown in below snapshot:\n\nI want to know, is there any out-of-box functionality/tool/macro to find the circular dependency in Project(.mpp) files. I am debugging manually right now & plan has many thousand lines.\nAny help is appreciated.\n", "answer": "I finally found a easily approach.\n\nInsert a new column in MSP copy all predeccessor column value in the new custom text field(say Text1_Pred)\nnow bulk delete all values in predeccessor column\nNow copy back all the value from Text1_Pred into Predeccessor Column\nagain\nit would start giving error with \"Task ID No\" as pasting error for\ncircular reference. Note TaskId.\nIt give you option to ignore problem or continue without problem or cancel\nChoose NO (\"continue without problem \") option to remove.\n\nHope it will help someone else with the problem.\nSource:\nhttp://www.domorethanmanage.com/articles/2008/05/29/Resolvingcircularreferenc.html\nhttp://pranavwithyou.wordpress.com/2014/06/02/how-to-resolve-circular-dependency-in-microsoft-project/\nhttps://go4answers.webhost4life.com/Example/resolve-circular-reference-sub-project-160953.aspx\n", "topic": "pm", "url": "https://pm.stackexchange.com/questions/11473"}, {"image": "VQAonline_00054229.png", "question": "How to send email notification when assigning task to resources in project server 2013 web app?", "context": "I use Project server 2013 and I created a Project web app site.\nI created an enterprise project and entered the team data.\nNow I am able to assign tasks to resources.\nWhat I want is that the team member is notified by email if when I assign a task to that team member. \n\nIn the screen shot you can see that I have assigned tasks to users.\n You can see Liza Naeem which is an active directory user and has an email.\nHow can I notify Liza Naeem by email using a Project automatism that this task was assigned to him.\n", "answer": "After searching on the internet and Microsoft site\nI solved my problem\n1- Go to SharePoint Central administration page click on under general application settings\nclick on PWA settings Manage, once the page is opened under operational policies \nclick on alerts and reminders\nset your SMTP mail server and email address it will work.\n", "topic": "pm", "url": "https://pm.stackexchange.com/questions/14842"}, {"image": "VQAonline_00054251.png", "question": "Where to (initially) create a task/bug with the Spotify model", "context": "When using the Spotify model, there are many different squads, who each own their own product. Now as far as I understand, these products could be interdependant (like services). One way to manage this in Jira (or similar) could be to create a project for each squad (so each squad has their own backlog).\nIf a bug occurs during a black box test, it might not be immediately clear, which service (product) is the cause of the bug and therefore which backlog it goes to. This leads me to my question:\n\nWhere do you report a bug, before you know what service (product) it\nrelates to?\n\nExample:\nWhen placing an order in a webshop, there are three different services involved:\n\nOrdering service\nInventory managment service (stock)\nPayment service\n\nA bug occur during a black box test, while trying to place an order. It's not immediately clear, what is causing the bug. In which squads backlog should the bug be (initially) created?\n\n", "answer": "This is a problematic question to answer because the \"Spotify Model\" isn't really a model or a framework or anything that would contain an answer to your question. What people call the \"Spotify Model\" was a white-paper that generalized the approach being taken at Spotify at a specific moment in time. Therefor, there can be no right answer to your question. I can share some insight, but take it for what it is.\nFirst, I went back and looked at the white paper (it's been a few years) and one of the things it contains is a picture of a few of their applications with sections highlighted to show how \"Squads\" have broken up the product. You'll notice that they are broken up in fairly clear value streams. One has the ability to play music. Another has the recommendations. In this scenario, even black-box testing has a pretty clear place to start. The fact that you are having difficulty finding where to go with bugs may indicate that you are building teams around architectural components, not functionality.\nSecond, in an agile environment, I'm unsure how you ended up in this situation. The fact that you need to create a bug in some team's backlog suggests that testing is happening outside of the teams. While it is certainly possible that a bug appears well after release and the root cause is an unexpected odd side-effect of another change, if it happens in testing, the source of the change should be very recent and should have very few teams that could cause it.\nOn that point, to a certain degree, it simply doesn't matter who you ask to look at it. Let's say you have a bug that shows up after a feature has been in production for a year and no change has been made to that feature. Frankly, at that point, anyone is equally qualified to troubleshoot it. The teams should have strong enough connections to each other to rally around the problem, let someone take lead, and they can reach out to anyone they need to in order to fix it.\nAs it says in the link you shared, Spotify created a people-driven autonomous approach to structuring their organization to support agility. If you can't simply spotlight that the bug exists and then let the teams self-organize around it, then you should probably be looking at what was missed when trying to apply lessons from Spotify.\n", "topic": "pm", "url": "https://pm.stackexchange.com/questions/30072"}, {"image": "VQAonline_00054265.png", "question": "Site with beatable PLO rake?", "context": "I've just started getting into PLO but have quickly realized the low-stakes games are next to unbeatable because of rake. Apparently paying 2buyins/1k hands in rake is not uncommon.\nAn example of a site with a huge rake at the smaller stakes is PokerStars.\nHere you can see the players that played the most hands in the last 12 months at the 0.50/1.00$ level, which is a rather soft level with a lot of people that don't really know what they're doing.\nOnly 3 of these 25 regular players are winners, they altogether paid almost $8 million in rake for the 63 million hands they played.\n\nSource\nDoes anyone play at a site that treats PLO differently?\n", "answer": "Everything you want is listed here. According to the table, PokerStars has the lowest rake at PLO games. Not surprising for me, though.\nHope it helps.\nGood luck!\n", "topic": "poker", "url": "https://poker.stackexchange.com/questions/1180"}, {"image": "VQAonline_00054274.png", "question": "Kelly criterion for SnG", "context": "This paper has the correct formula for the Kelly criterion for SNG's where there are multiple outcomes. It stated it is more correct than the one given on sitandgoplanet.\n\nThe problem is I do not know how to do the maths. Can someone give a clear step-by-step calculation on how that x value is derived?\n", "answer": "If you have a little background in calculus, you'll notice that the dg(x)/dx line is the derivative of g(x) and then is set equal to 0 to find the critical point which in this case will be a maximum point and thus the optimal fraction of the bankroll to bet. Here is a little background/refresher on performing derivatives with log functions.\nWhether or not you understand how to arrive at the derivative of log functions yourself, these particular ones boil down to a simple formula:\nthe derivative of a*log(b+cx) = ac/(b+cx), where a, b, and c are constants.\nWith this example, you have three terms with their respective a, b, and c values being: (0.62, 1, -1); (0.24, 1, 1); (0.14, 1, 3). Plugging these into the above formula gets you to the dg(x)/dx line and from there you'll use some algebra to combine terms and solving for x.\n", "topic": "poker", "url": "https://poker.stackexchange.com/questions/9219"}, {"image": "VQAonline_00054283.png", "question": "Counterfeit confusion in Omaha Hi-Lo", "context": "Can anyone help me to wrap my head around an explanation of counterfeiting that I think is incorrect?\n\nI have the hand on the left and I thought I could play the A2 for the low pot. The other players at the table told me that since there was a 2 on the board it counterfeited my 2s and I had no qualifying low hand.\nThis is contrary to my understanding of counterfeiting. Who is correct?\n", "answer": "You would use [A2][543] and the villain would use [A5][432] to make the same Lo. There is no counterfeiting here.\nIf the board was instead K324J, you would have no Lo.\n", "topic": "poker", "url": "https://poker.stackexchange.com/questions/11372"}, {"image": "VQAonline_00054319.png", "question": "Why do some countries recognise Taiwan as the sole legitimate government of China?", "context": "There are some countries that recognise Taiwan (Republic of China) as the sole legitimate government of China instead of the People's Republic of China.\nRecently, just a few days ago, S\u00e3o Tom\u00e9 and Pr\u00edncipe switched its recognition from the ROC to the PRC.\n\nAccording to Wikipedia, there are still 21 entities that has full diplomatic relations with Taiwan:\n\n\n\nSo, why do they recognise the ROC and are there any benefits for doing so?\nNote: Sources or examples are greatly appreciated \n", "answer": "In order to have an official relationship with the People's Republic of China (mainland China), you have to discontinue official relationships with the Republic of China. So if you want any official relationship with the Republic of China, you can't have any official relationship with the PRC. Source: https://en.m.wikipedia.org/wiki/One-China_policy\nThere are some developing countries that probably choose to have relations with the ROC (or the PRC) in return for favours of one sort or another.\nThe Vatican City recognizes the ROC rather than the PRC. Plausible reasons for doing so include China's interference in religious freedom of Christians, and the ROC being a capitalist state while the PRC was, and is still nominally, a communist state.\n", "topic": "politics", "url": "https://politics.stackexchange.com/questions/14142"}, {"image": "VQAonline_00054433.png", "question": "Why is the western portion of the USA moving towards the Democratic Party while the Midwest is moving towards the GOP?", "context": "A lot has been made of the former president Donald Trump's narrow upset wins in three Rust Belt states which won him the presidency. This was part of a long term trend that continued into 2020, as I describe below.\nTrend 2004-2020 president (trend is shift adjusted for average):\n\nYou can see all but one Southwestern state move towards Democrats relative to the nation. (Nevada narrowly did not.) You can also see parts of the Southeast and Northeast trending blue too. (I don't think including DC is fair because of how heavily Democratic it is, but it still barely trended blue!)\nThis trend continued albeit at a slower rate in 2020 with Ohio and Wisconsin trending towards Republicans vs 2016, and Texas, Arizona, and Georgia trending towards Democrats.\nBecause we adjusted for national popular vote shift, the number of people living in a state trending towards Democrats is about the same as the number living in a state trending towards Republicans.\nWhat is causing this long term trend that has been present since at least 2016? (I know there are other areas trending this way, I just couldn't fit them in the question.)\n", "answer": "One contributing pattern that was particularly emphasized in the industrial centers around the great lakes, was reduced Dem turnout due to a series of disappointments in economic policy. Author Thomas Frank in particular has focused heavily on this dynamic.\nThis included\n\nUS manufacturing job losses (in the face of growing overall North American manufacturing employment), which became noticable shortly after NAFTA went into effect\nlackluster or ineffective efforts by the post-Bill-Clinton Democratic party to defend organized labor\nthe response to the 2008 financial-and-mortgage crisis, seen as having too much emphasis on large banks and not enough on middle-class individuals who lost their homes in large numbers.\n\nIn the case of NAFTA in particular, opposition to it comes from many angles. For some voters it is rooted in xenophobia or nationalism. But for a large number, it is more simply a \"kitchen table\" issue, because it affects employment and pay. Into the latter category fits the the segment Frank focuses on: middle-class, non-college educated, formerly unionized (i.e. strongly Democrat leaning, historically).\n", "topic": "politics", "url": "https://politics.stackexchange.com/questions/65521"}, {"image": "VQAonline_00054424.png", "question": "What is the significance of the light blue \"ribbon\" style lapel pins worn by Cuomo and Schumer?", "context": "I saw this photo of NY Gov. Andrew Cuomo and Sen. Chuck Schumer, both of them are wearing the same or similar lapel pins:\n\nThere was no date given for the picture.\nThe other people visible in the photo don't seem to be wearing these also (including Joe Biden).\nUnfortunately searching for \"blue ribbon\" turns up lots of irrelevant results since that is a common term. I haven't found any reference to what these mean with any searches I can think of. There are some articles about \"Cuomo lapel pin\" but they are older and not related.\nWhat is the significance of these pins?\n(source)\n", "answer": "This picture is from September 11th, 2020, when Biden, Cuomo, Schumer, and others met at Ground Zero in New York to commemorate the 19th anniversary of the 9/11 attacks. More pictures can be found on Schumer's Twitter post from that day. In these, you can see many other attendees wearing these blue ribbon pins on their lapels.\nThe blue ribbon represents solidarity with the relatives of the victims of these attacks, and is also worn in memory of the victims themselves. This is supported by this Guardian article from the 2014 anniversary, which notes that many attendees at the annual memorial ceremony wore such pins:\n\nIn New York, grieving relatives, many wearing light blue ribbons and\ncarrying pictures of their loved ones, gathered under overcast skies\nfor the annual 9/11 memorial ceremony.\n[...]\n\u201cI just wanted one year to hear her name said correctly,\u201d said Kirby,\nwho wore the blue ribbon on her lapel. She had lived in New York, but\nmoved to Florida after 9/11. It was her first time visiting the\ngrounds where her sister was killed.\n\nThis Daily Mail article from 2015 mentions that Rudy Giuliani is wearing the ribbon \"in memory of those who perished\".\n", "topic": "politics", "url": "https://politics.stackexchange.com/questions/61540"}, {"image": "VQAonline_00054477.png", "question": "Por que algumas fontes listam o voc\u00e1bulo \"mir\u00edade\" como coletivo para insetos?", "context": "Eu sou um cientista entomologista. Apenas recentemente, estudando a gram\u00e1tica portuguesa novamente, que descobri este fato, que n\u00e3o consigo entender. \nEncontrei a defini\u00e7\u00e3o primeiramente na Nova Gram\u00e1tica da L\u00edngua Portuguesa para Concursos, por Rodrigo Bezerra, 6.\u00aa Ed. Editora M\u00e9todo, pag. 147 (vide na foto). Esta palavra encontra-se em uma tabela listando termos coletivos que se extende entre as pags. 145-148, apresentada com o seguinte enunciado.\n\nColetivos: S\u00e3o os substantivos que, no singular, indicam uma cole\u00e7\u00e3o, um agrupamento, um conjunto de seres da mesma esp\u00e9cie. Observe abaixo uma tabela com os principais coletivos:\n\nO livro tamb\u00e9m pode ser encontrado no seguinte s\u00edtio. \nAdicionalmente, pela internet existem alguns sites (informais) onde o mesmo termo foi assim definido.\n\nPor favor, algu\u00e9m saberia se esta defini\u00e7\u00e3o \u00e9 realmente oficial e qual teria sido sua origem?\n", "answer": "Significado e Uso\nOs doze dicion\u00e1rios\u00b9 dos \u00faltimos trezentos anos que consultei concordam: mir\u00edade significa apenas \u2018dez mil\u2019 ou \u2018n\u00famero muito elevado\u2019; s\u00f3 por si, mir\u00edade n\u00e3o significa \u2018quantidade de insetos\u2019 ou de qualquer outra coisa em particular. Os exemplos da literatura no Google Books, abundantes nos \u00faltimos dois s\u00e9culos, confirmam: encontramos muitos mir\u00edade de insetos, de estrelas, de anjos, de cores, de estilos, de tudo o que se queira; mas mir\u00edade sem dizer de qu\u00ea, n\u00e3o se encontra.\nA ace\u00e7\u00e3o \u2018dez mil\u2019 s\u00f3 se encontra em obras antigas: excluindo edi\u00e7\u00f5es e cita\u00e7\u00f5es posteriores dessas obras, o \u00faltimo exemplo que encontrei \u00e9 de 1622; a partir da\u00ed s\u00f3 encontrei a ace\u00e7\u00e3o \u2018n\u00famero muito elevado\u2019. Isto assemelha-se \u00e0 situa\u00e7\u00e3o no ingl\u00eas. Diz o Etymoline que (tradu\u00e7\u00e3o minha) \u201co uso especificamente num\u00e9rico encontra-se habitualmente em tradu\u00e7\u00f5es do grego ou latim\u201d, l\u00ednguas em que a palavra era realmente usada no sentido \u2018dez mil\u2019.\nClassifica\u00e7\u00e3o: substantivo coletivo ou o qu\u00ea?\nMir\u00edade tem de facto cabimento entre os substantivos coletivos. H\u00e1 coletivos de dois tipos: os dependentes, como pilha, que precisam de um complemento, como de pratos, de livros, de caixas, etc.; e os aut\u00f3nomos, como maquinaria, que n\u00e3o admitem complemento, ou como matilha que admite um complemento opcional e redundante\u2014podemos dizer s\u00f3 matilha ou matilha de c\u00e3es. Mir\u00edade pertence ao dependentes. Quando pensamos em substantivos coletivos, creio que s\u00e3o os aut\u00f3nomos que mais facilmente nos v\u00eam \u00e0 cabe\u00e7a. De modo que para os dependentes eu vou apelar \u00e0 autoridade da Gram\u00e1tica do Portugu\u00eas (Gulbenkian, Lisboa, 2013, tomo I, p. 973-4):\n\nEm contrapartida, os nomes coletivos que n\u00e3o representam lexicalmente o tipo de entidades que formam o grupo chamam-se coletivos dependentes ou agrupadores [\u2026] s\u00e3o exemplos bra\u00e7ada, conjunto, grupo e mont\u00e3o.\nEstes nomes coletivos necessitam de ser complementados por uma express\u00e3o adicional que introduza informa\u00e7\u00e3o sobre a composi\u00e7\u00e3o do grupo [\u2026] cf. bra\u00e7ada {de flores/de lenha}, conjunto {de regras/de pa\u00edses}, grupo {de deputados/de rapazes}, mont\u00e3o {de pedras/de destro\u00e7os}. Nestes casos, \u00e9 a combina\u00e7\u00e3o do nome com o sintagma preposicional que forma uma express\u00e3o coletiva semanticamente completa, equivalente a um nome coletivo aut\u00f3nomo.\n\nEstes nomes coletivos dependentes n\u00e3o s\u00e3o inteiramente pac\u00edficos. A Gram\u00e1tica do Portugu\u00eas explica (p, 974) que:\n\nPor n\u00e3o serem semanticamente completos, algumas gram\u00e1ticas excluem inteiramente da classe dos coletivos os nomes aqui tratados como coletivos dependentes (ver, p.e., Bosque 1999), argumentando que se trata, na realidade, de quantificadores lexicais indefinidos do tipo de alguns, muitos, v\u00e1rios, etc.\n\nNesta perspetiva, mir\u00edade seria um quantificador lexical indefinido, por mir\u00edade de insetos ser mais ou menos o mesmo que muit\u00edssimos insetos.\nA minha primeira rea\u00e7\u00e3o foi, como o ANeves acima, considerar mir\u00edade um numeral coletivo, como d\u00fazia, milhar, milh\u00e3o, usado muito vagamente. O problema \u00e9 que na linguagem corrente mir\u00edade significa apenas numero muito elevado; o sentido dez mil aparece apenas em obras antigas ou coment\u00e1rios sobre essas obras. Deve ser por isto que encontrei mir\u00edade em v\u00e1rias listas de substantivos coletivos, mas n\u00e3o de numerais coletivos. \nNota sobre a tabela\nPosto isto, a tabela \u00e9 no m\u00ednimo confusa e induz facilmente em erro, porque n\u00e3o deixa claro como \u00e9 que o conte\u00fado da coluna da direita deve ser interpretado. O leitor poderia pensar que de insetos est\u00e1 para mir\u00edade tal como, logo na linha acima, de c\u00e3es est\u00e1 para matilha, isto \u00e9 que que mir\u00edade s\u00f3 por si pudesse significar \u2018conjunto de insetos\u2019 tal como matilha significa \u2018conjunto de c\u00e3es\u2019. Tanto mais que logo poucas linhas acima houve o cuidado de juntar um etc. para indicar que de bois e de elefantes s\u00e3o apenas dois dos complementos poss\u00edveis de manada (e o mesmo com nuvem umas linhas abaixo), coisa que n\u00e3o foi feita com o de insetos da mir\u00edade.\nEtimologia\nA palavra vem do grego myrias, myriados no genitivo, que j\u00e1 significava \u2018dez mil\u2019 ou \u00b4n\u00famero incont\u00e1vel\u2019; passou ao latim tardio, myriadis no genitivo; aparece em franc\u00eas como myriade na ace\u00e7\u00e3o \u2018dez mil\u2019 em 1525 e \u2018n\u00famero infinito\u2019 em 1557 (ver CNTRL); e em ingl\u00eas como myriad em 1550-60 (Etymoline). \nEm portugu\u00eas encontrei myriada em 1597\u00b2 e meriada em 1622.\u00b3 Estas s\u00e3o os \u00fanicos exemplos que encontrei no sentido \u2018dez mil\u2019 (tirando cita\u00e7\u00f5es posteriores destas obras). A palavra vem no Vocabulario Portuguez e Latino Raphael Bluteau (1728) que d\u00e1 apenas a ace\u00e7\u00e3o \u2018dez mil\u2019; o [dicion\u00e1rio de Moraes Silva de 1789] faz o mesmo, mas a 4\u00aa edi\u00e7\u00e3o, de 1831, j\u00e1 acrescenta o significado figurado \u2018grande multid\u00e3o\u2019, dando como exemplo \u201cMiriadas infindas de Serafins cerc\u00e3o seu trono\u201d. O dicion\u00e1rio de Domingos Vierira de 1873 d\u00e1 a ace\u00e7\u00e3o \u2018dez mil\u2019 como \u201cTermo de Antiguidade\u201d e diz que na \u201clinguagem ordin\u00e1ria\u201d (isto \u00e9, \u2018linguagem comum\u2019; ordin\u00e1rio naqueles tempos n\u00e3o significava \u2018reles, mau\u2019; era s\u00f3 o oposto de extraordin\u00e1rio), significa \u201cquantidade indefinida e innumeravel\u201d, o que pelo que eu vi no Google Books reflete tanto o uso no s\u00e9culo XIX como o atual.\n\nNotas:\n\u00b9 Doze dicion\u00e1rios (19 edi\u00e7\u00f5es): Raphael Bluteau (1728), Morais Silva (1789, 1813, 1823, 1831, 1858, 1877), Domingos Vieira (1873), C\u00e2ndido de Figueiredo (1899, 1913, 1939), Sociedade de L\u00edngua Portuguesa (1958-71), Lello Universal (1992), Academia das Ci\u00eancias de Lisboa (2001), Houaiss (Lisboa, 2002) e dicion\u00e1rios online, Priberam, Infop\u00e9dia, Aulete, Michaellis.\n\u00b2 \u201c[D]igamos com Nicephoro Calixto, que em todo o Imperio Romano, se contar\u00e3o nesta discrip\u00e7\u00e3o vinte & seis mil, & trinta & sete Myriadas, & vallendo cada Myriada dez mil, vem a somar duzentos & sessenta contos, & trezentas & settenta mil pessoas.\u201d Frei Bernardo de Brito, Monarchia Lusitana, 1\u00aa parte, Alcoba\u00e7a, 1597, p. 415.\n\u00b3 \u201c[O] Ceo empireo tem de comprimento dez mil, trezentas, quatorze meriadas, oitoc\u1ebdtos, cincomil, setenta, & dez milhares de passos, & em largura tres mil, seis centas meriadas.\u201d Padre frey Antonio Rozado, Tratados sobre os quatro nov\u00edssimos conlugares comuns, Porto, 1622, p. 311.\n", "topic": "portuguese", "url": "https://portuguese.stackexchange.com/questions/6024"}, {"image": "VQAonline_00054481.png", "question": "Ao listar t\u00f3picos, eu devo usar alguma pontua\u00e7\u00e3o?", "context": "No meio de um texto, se eu tenho um t\u00f3pico de caracter\u00edsticas de um produto, como o abaixo mostrado, qual a forma correta de pontuar? N\u00e3o sei se h\u00e1 alguma diferen\u00e7a, mas gostaria de saber em rela\u00e7\u00e3o ao portugu\u00eas europeu.\nExemplo:\n\nO que os especialistas dizem\nUm ber\u00e7o \u00e9 um \u00f3timo complemento para carrinhos, mas tem uma janela de uso bastante limitada e \u00e9 outra coisa para guardar quando os dias de rec\u00e9m-nascido acabarem. O Mixx resolve isso com uma combina\u00e7\u00e3o de assento e ber\u00e7o; basta colocar o assento em uma posi\u00e7\u00e3o reclinada completamente plana e dobrar o apoio para os p\u00e9s para criar um ber\u00e7o aconchegante perfeito.\nO que vale a pena considerar\n\u00c9 um pouco volumoso quando dobrado.\nCaracter\u00edsticas:\n\nIdade recomendada: at\u00e9 os 03 anos\nCesto grande\nCapa de chuva inclu\u00edda\nTamanho fechado: 58 x 51 x 48 cm\nPeso: 14 kg\nLimite de peso: 17 kg\n\n\n\n", "answer": "N\u00e3o h\u00e1 norma consolidada, quer em pt-br, quer em pt-pt (ver, e.g., esta resposta do Ciberd\u00favidas) e as tr\u00eas possibilidade mencionadas na pergunta (sem pontua\u00e7\u00e3o, com ponto e v\u00edrgula, com ponto) s\u00e3o aceit\u00e1veis. Assim, talvez o ponto mais importante seja manter coer\u00eancia dentro de uma mesma publica\u00e7\u00e3o/texto.\nMas h\u00e1 regras e tradi\u00e7\u00f5es, al\u00e9m de normas espec\u00edficas de uma dada \u00e1rea ou publica\u00e7\u00e3o.\nPor exemplo, se pode preferir, quando os itens s\u00e3o curtos (e, certamente, quando inline, i.e., incorporados ao longo do texto) iniciais min\u00fasculas e separa\u00e7\u00e3o por v\u00edrgula ou ponto e v\u00edrgula; e, para itens mais extensos, iniciais mai\u00fasculas e separa\u00e7\u00e3o por pontos. Estes e outros crit\u00e9rios s\u00e3o descritos em mais detalhes, por exemplo, no Manual do Editorial do IPEA. Por esse crit\u00e9rio, o exemplo da pergunta faria uso de min\u00fasculas e do ponto e v\u00edrgula.\nNo caso de uma listagem online, se poderia argumentar que o estilo tipogr\u00e1fico dominante atual favorece a listagem \"mais limpa\", sem pontua\u00e7\u00e3o.\n", "topic": "portuguese", "url": "https://portuguese.stackexchange.com/questions/8240"}, {"image": "VQAonline_00054478.png", "question": "Por que \"professora\" pode significar \u2019prostituta que inicia adolescentes\u2019?", "context": "Se voc\u00ea fizer uma pesquisa no Google com a express\u00e3o \"significado de professora\", vai aparecer:\n\nDe acordo com a mat\u00e9ria que pode ser visualizada neste link, \"o mesmo resultado aparece em outros dicion\u00e1rios, como o Houaiss, e no dicio.com.br\". Quando a mesma busca \u00e9 feita por \"professor\", o resultado \u00e9 diferente. Neste caso, o primeiro resultado diz \"aquele que professa uma cren\u00e7a\"; e o segundo, \"aquele que ensina\".\nPor que existe esse sentido de \"prostituta\" para professora?\n", "answer": "Esse significado \u00e9 um regionalismo, uma g\u00edria; por sinal, muito incomum para o significado de \"professora\".\nA palavra \"prostituta\", assim como as palavras que se referem aos \u00f3rg\u00e3os sexuais, possuem uma infinidade de termos com o mesmo significado, sem que se perca ou se descaracterize o significado origin\u00e1rio da palavra.\nAssim, tamb\u00e9m pode significar \"prostituta\" a palavra \"prima\", sem que isso fa\u00e7a que a palavra \"prima\" seja imediatamente identificada como prostituta ou fa\u00e7a com que seu uso seja restrito ou mal visto de qualquer maneira, quando usada na forma original.\nDa mesma forma, poder-se-ia dizer que as prostitutas atuam como professoras na arte do sexo para jovens inexperientes, mas esse uso \u00e9 muito incomum ou mesmo desconhecido entre a maioria dos falantes de portugu\u00eas.\nPode-se citar de forma an\u00e1loga o uso da palavra \"lingui\u00e7a\" para o \u00f3rg\u00e3o sexual masculino; ainda que esse uso seja relativamente comum, isso n\u00e3o faz com que imediatamente se atribua esse significado ao lermos a palavra \"lingui\u00e7a\", que, em 99% das vezes, realmente significa o alimento com o qual se faz churrasco.\n", "topic": "portuguese", "url": "https://portuguese.stackexchange.com/questions/6302"}, {"image": "VQAonline_00054540.png", "question": "The SErial Killer (Part 2)", "context": "This puzzle is part of a series - part 1 is here. Any comments I make on the OP are canonical and out of character (OOC). Any comments I post on answers are for flavour only.\nUpdate: As it turns out, this was a very easy puzzle, and I'm actually having to rewrite the plot of the series thanks to SKV being far too clever for his own bleeding good (I love you really). The glory's gone, but try to solve it without checking out the answers. I enjoyed making it, anyway :-) Part 3 will be tougher.\n\nYou're staring at the list of Stacks when out of nowhere, you hear a voice going rik tika tik tik tik tik and before you wonder where it came from you realise something - the capital letters in the titles of all the dead sites can be re-arranged to spell \"Moderators\"!\nYou cast your eyes over the the list of murdered sites to find the Moderators Stack, and in your best Scottish-detective voice you announce \"Och! There's bin a murrrrrrder, and ah nae where ahm going tae look!\" (OOC: Sorry, Scotland). Tentatively, you click through to the Moderators stack and right at the top, you see a post entitled \"We Will Conquer\". You recognise the title from one of the dead Stacks, so you click on to read it...\n\nYou gasp in disbelief and check back to the Stacks list - sure enough, Ask Ubuntu is dead. You're about to click on it to make sure, but then remember the ominous warning about entering a new Stack, and think better of it. This \"Handel\" character doesn't seem like someone to mess around with.\nYou realise you're suddenly on a countdown, trying to minimise the number of stacks that die, and every false step you make could bring a flood of confused newbies asking inane questions about anything on to your beloved Puzzling.SE... or even worse, Puzzling.SE might get killed off!\nBaffled and starting to panic a little, you sit back to mull things over. What's your next move? Where should you look?\n", "answer": "\n English language & usage. The ampersand in the logo has hex color 90383b, which is the roman numeral in the tag.\n\n", "topic": "puzzling", "url": "https://puzzling.stackexchange.com/questions/4263"}, {"image": "VQAonline_00055847.png", "question": "4x4 number grid with one empty cell", "context": "Can you guess what comes in place of the question mark?\n\nHere's a text-based version of the table:\n\n\n\n\n\n\n\n\n\n\n\n\n1536\n48\n96\n3\n\n\n384\n192\n24\n12\n\n\n768\n96\n48\n6\n\n\n192\n?\n12\n24\n\n\n\n", "answer": "It's:\n\n $384$. Starting from the top-left, go down and do a U'y at the bottom, go up, do a U'y at the top, you get: $1536, 384, 768, 192, ?, 96, 192, 48, 96, \\dots$, which is divide by $4$, multiply by $2$, and repeat.\n\n", "topic": "puzzling", "url": "https://puzzling.stackexchange.com/questions/83605"}, {"image": "VQAonline_00055030.png", "question": "(addition) 7 point star / heptagram", "context": "\nPlace the numbers $1$ through $14$ onto each point, so that:\n\nA = 1\nThe sum of points in every line are equal,\n(A+I+J+C = B+J+K+D = C+K+L+E = D+L+M+F = E+M+N+G = F+N+H+A = G+H+I+B)\nA+B+C+D+E+F+G+1 = H+I+J+K+L+M+N\n\nThere is only one solution (if we rule out mirroring). \n", "answer": "Ok, I started out trying to deduce the numbers, got stuck and decided to brute-force it since there's no no-computers tag. The solution is:\n\n A = 1\n B = 8\n C = 13\n D = 5\n E = 2\n F = 14\n G = 9\n H = 3\n I = 10\n J = 6\n K = 11\n L = 4\n M = 7\n N = 12\n\nStill working on a more mathematical approach...\nPS. Ugliest Python code I've ever written for who's interested:\na = 1\nfor b in range(2,15):\n for c in range(2,15):\n for d in range(2,15):\n for e in range(2,15):\n for f in range(2,15):\n for g in range(2,15):\n for h in range(2,15):\n for i in range(2,15):\n if g + h + i + b == 30:\n for j in range(2,15):\n if a + i + j + c == 30:\n for k in range(2,15):\n if b + j + k + d == 30:\n for l in range(2,15):\n if c + k + l + e == 30:\n for m in range(2,15):\n if d + l + m + f == 30:\n for n in range(2,15):\n if e + m + n + g == 30:\n if f + n + h + a == 30:\n x = [a, b, c, d, e, f, g, h, i, j, k, l, m, n]\n if len(x) > len(set(x)):\n break\n if a + b + c + d + e + f + g + 1 == h + i + j + k + l + m + n:\n print('a = ' + str(a))\n print('b = ' + str(b))\n print('c = ' + str(c))\n print('d = ' + str(d))\n print('e = ' + str(e))\n print('f = ' + str(f))\n print('g = ' + str(g))\n print('h = ' + str(h))\n print('i = ' + str(i))\n print('j = ' + str(j))\n print('k = ' + str(k))\n print('l = ' + str(l))\n print('m = ' + str(m))\n print('n = ' + str(n))\n\n", "topic": "puzzling", "url": "https://puzzling.stackexchange.com/questions/43596"}, {"image": "VQAonline_00057012.png", "question": "Proving Scaled Random Walk Approaches Normal Distribution", "context": "I'm reading Stochastic Calculus for Finance II: Continuous-Time Models by Steven Shreve and I don't understand how he went from the equation on the left to the middle one. If it helps, this section is proving that the distribution of a scaled random walk converges to the normal distribution.\n\n", "answer": "$X_j$ can be either 1 or -1 with 50% probability each. So this step is just applying the expectation to both possible cases.\nSee definition of the Expectation...\n\\begin{align}\n{\\mathbb E}\\bigl[ X \\bigr] = \\sum_i i \\cdot p(x = i)\n\\end{align}\nIt's the sum over all possibilities of the probability of getting that value (both ${\\frac 1 2}$ in your case) multiplied by the value\n", "topic": "quant", "url": "https://quant.stackexchange.com/questions/58746"}, {"image": "VQAonline_00057016.png", "question": "Geometric Brownian Motion simulation in Python: strange results", "context": "I am trying to simulate Geometric Brownian Motion in Python, however the results that I get seem very strange and in my opinion they can't be correct. My goal is to simulate each day of 1 year. Basically, I used two slightly different approaches. Based on my research, it should be either possible to use daily drift and volatility with dt = 1 or annualize drift and volatility and use dt = 1/365. I computed the daily drift and volatility based on daily historical data for the first approach and annualized them for the second. Accordingly also T is changed. Below is the code in Python which I partially got from this answer (Geometric Brownian Motion simulation in Python):\nimport numpy as np\nimport matplotlib.pyplot as plt\n\nnp.random.seed(1000)\n\nquandl.ApiConfig.api_key = \"XXXXXX\"\n\n\ndef gen_paths(S0, r, sigma, T, M, I):\n dt = float(T) / M\n paths = np.zeros((M + 1, I), np.float64)\n paths[0] = S0\n for t in range(1, M + 1):\n rand = np.random.standard_normal(I)\n paths[t] = paths[t - 1] * np.exp((r - 0.5 * sigma ** 2) * dt + sigma * np.sqrt(dt) * rand)\n return paths\n\n\n# Get data\ndf = quandl.get(...)\nvalue = df['Value']\n\n# Compute returns\nreturns = value.pct_change(1)\n\n\n# Daily drift and volatility\nS0 = value[-1] # initial stock price\nr = np.mean(returns) # drift\nsigma = np.std(returns) # volatility\nT = 365 # time\nM = 365 # number of steps\nI = 100000 # number of simulations\n\npaths_1 = gen_paths(S0, r, sigma, T, M, I)\nprint(\"Average: \" + str(np.average(paths_1[-1])))\n\n\n# Yearly drift and volatility\nr = np.mean(returns) * 365\nsigma = np.std(returns) * np.sqrt(365)\nT = 1.0\n\npaths_2 = gen_paths(S0, r, sigma, T, M, I)\nprint(\"Average: \" + str(np.average(paths_2[-1])))\n\n# Visualize 5 simulated paths for each\nfig, axs = plt.subplots(2)\naxs[0].plot(paths_1[:, :5])\naxs[1].plot(paths_2[:, :5])\nplt.show() \n\nHere is the result that I get:\n\nFor the averages I get 117183 for the approach with daily drift and volatility and 76145 for the approach with yearly drift and volatility. Here are my questions:\n\nCan these results be correct or what is wrong with my code?\nShouldnt the averages be very similar for both approaches?\nWhich approach is favorable to use?\n\n", "answer": "Great question!\nAbstract: Your code and math are correct, but you use too high vol and drift to be real world realistic. Your simulations decay to zero due to high vol and LogNormality.\n\n\nBasically, I used two slightly different approaches. Based on my research, it should be either possible to use daily drift and volatility with dt = 1 or annualize drift and volatility and use dt = 1/365\n\nThe solution using yearly quantities $\\{t, \\sigma, r\\}$ is\n\\begin{align}\nS(t+\\Delta t) = S(t) \\exp \\left((r-\\tfrac{1}{2}\\sigma^2)\\Delta t + \\sigma\\sqrt{\\Delta t}\\cdot Z \\right).\n\\end{align}\nCreate new daily quantities $\\{d, s, \\nu\\}$ by subsituting with $\\Delta t := d/365$, $\\; \\sigma := s \\sqrt{365}$, $\\; r:= \\nu \\cdot 365$, we get\n\\begin{align}\nS(d+\\Delta d) = S(d) \\exp \\left((\\nu-\\tfrac{1}{2}s^2)\\Delta d + s\\sqrt{\\Delta d}\\cdot Z \\right).\n\\end{align}\nHence, the \"formula\" looks the same, whether you use annual or daily parameters.\n\n\n\nCan these results be correct or what is wrong with my code?\n\n\nYour code and results are correct. The thing is that your parameters are enormous. What you assume to be daily values should be annual. What you assume to be annual are therefore gigantic. Daily rate should be $\\nu=\\tfrac{0.04}{365}$; vol should be $s=\\tfrac{0.4}{\\sqrt{365}}.$ Annual versions are then $\\sigma=0.4$ and $r=0.04$.\nWhat is confusing, although expected, is that all your simulations decay towards zero. This is because your GBM is a lognormal distribution, and when your variance approaches infinity (your vol was gigantic), almost all of the terminal density lies at zero. There is a small probability that the terminal value is very large, but this has very small probabiltiy, so we don't see this. You can see this by plotting the PDF and CDF with a large variance.\n\n\n\nShouldnt the averages be very similar for both approaches?\n\n\nYou use different random numbers for your two simulations. Enforce the random numbers to be the same by remove your np.random.seed(1000) at the top and add it once before each call to your functions, like this\n...\nnp.random.seed(1000)\npaths_1 = gen_paths(S0, r, sigma, T, M, I)\n...\nnp.random.seed(1000)\npaths_2 = gen_paths(S0, r, sigma, T, M, I)\n\nThis will ensure that they use the same randomness, and you will see that your two plots are identical.\n\n\nWhich approach is favorable to use?\n\nAnnual (almost) always, unless you work on intraday modelling and really high resolution and to avoid numerical issues with underflow. I have never really seen anyone use anything other than annual for standard GBM modelling and pricing.\n\n", "topic": "quant", "url": "https://quant.stackexchange.com/questions/60005"}, {"image": "VQAonline_00056914.png", "question": "Why is this utility function not picking up its penalty?", "context": "I was reading this seminal paper by Infanger. On page 40, Figure 11. was quite interesting. In particular I was interested in the top one, 19 Years and I wanted to reproduce this plot. To give some background: It's about utility maximization which should be solved by DP approach, i.e.\n$$ \\max_{x_t,0\\le t\\le T} E[u(W_T)]$$\nwhere $u$ is a utility function and $W_T$ is the wealth at time $T$. We want therefore to maximize terminal wealth. For the picture he uses the following \"quadratic downside risk\" function\n$$ u(W) = W - \\frac{\\lambda}{2}\\max{(0,W_d-W)^2}$$\nwhere $W_d$ is a target amound and $\\lambda$ a scaling parameter. As wealth evolves via $W_{t+1} = W_t \\cdot\\langle x_t, R\\rangle$ where $x_t$ are the allocation and $R$ the return, he writes down the Bellman equation of this problem:\n$$V_{t}(W_t) = \\max_{x_t}E[V_{t+1}(W_t\\cdot \\langle x_t, R\\rangle)|W_t]$$\nsince I'm just interested in the final step, we have $V_T = u$ and the maximization problem I want to solve is\n$$V(W) = \\max_{x}E[u(W\\cdot \\langle x, R\\rangle)|W] $$\ndropping the time $t$ index. I assumed (as Infanger did if I get him right), that $R$ are normally distributed. I use then Gauss-Hermite quadrature to approximate the expectation (see page 71 in this paper).\n$$V(W) = \\frac{1}{\\sqrt{\\pi}}\\max_{x}\\sum_{i=1}^m w_i u(W(1+\\hat{\\mu}(x)+\\sqrt{2}\\hat{\\sigma}(x)\\cdot q_i)) $$\nwhere $\\hat{\\mu} = \\langle \\mu, x\\rangle$ and $\\hat{\\sigma} = \\sqrt{\\langle x, \\Sigma x\\rangle}$ and $w_i$ are the Gauss-Hermite weights and $q_i$ the corresponding nodes.\nI've coded a very simple and not optimized version to see if I get the desired picture. \nfirst a picture of the utility function:\nutility <- function(w){\n K <- 100000\n temp <- K-w\n temp[temp<=0] <- 0\n return(w - 1000*temp^2)\n}\n\nx <- seq(90000,120000,1000)\ny <- utility(x)\nplot(x,y,type=\"l\")\n\n\nNow I just generated a sequences of $W$ and solved the above problem. The following code junk defines the covariance and expected return vector. The data is from the Infanger paper above.\nwealth <- seq(50000,150000,5000)\nmu <- c(0.108, 0.1037, 0.0949, 0.079, 0.0561)\ncor <- matrix(c(1, 0.601, 0.247, 0.062, 0.094,\n 0.601, 1.0, 0.125, 0.027, 0.006,\n 0.247, 0.125, 1.0, 0.883, 0.194,\n 0.062, 0.027, 0.883, 1.0, 0.27,\n 0.094, 0.006, 0.194, 0.27, 1.0),\n ncol=5, nrow=5,byrow=T)\nstd <- c(0.1572, 0.1675, 0.0657, 0.0489, 0.007)\ntemp <- std%*%t(std)\ncov <- temp*cor\n\nWith this data at hand and a sequence of wealth (see above) I just run an optimization for each given wealth and store the solution (assuming no short selling). To solve the problem I used the Rsolnp package in R. It solves a minimization problem that's why I'm returning a $-1$ in the objective function below:\nlibrary(Rsolnp)\nlibrary(statmod)\nobj <- function(x, currentWealth, mu, cov, r=0, nodes, weights){\n drift <- sum((mu-r)*x)+r\n cor <- sqrt(sum(x*(cov%*%x)))\n term1 <- currentWealth*(1+drift)\n term2 <- currentWealth*sqrt(2)*cor*nodes\n return(-1/sqrt(pi)*sum(weights*utility(term1+term2)))\n}\n\ng_constraints <- function(x,currentWealth, mu, cov, r=0, nodes, weights){\n return(sum(x))\n}\n\nx0 <- rep(0.25,length(mu))\nweights <- gauss.quad(10,\"hermite\")$weights\nnodes <- gauss.quad(10,\"hermite\")$nodes\nsolmat <- matrix(NA, ncol=length(mu),nrow=length(wealth))\nfor(i in 1:length(wealth)){\n sol <- solnp(pars=x0, fun = obj,\n eqfun = g_constraints,\n eqB = 1,\n LB = rep(0, length(mu)),\n UB = rep(1, length(mu)),\n currentWealth = wealth[i], mu = mu, cov = cov,\n r = 0, nodes = nodes, weights = weights)\n solmat[i,] <- sol$pars\n x0 <- sol$pars\n}\ncolnames(solmat) <- c(\"US Stock\", \"Int Stocks\", \"Corp Bonds\", \"Gvnt Bond\", \"Cash\")\nrownames(solmat) <- as.character(wealth)\n\nHowever, I get a constant allocation where all money is invested in US Stocks. What's wrong with this and how do I get this chart from Infanger?\n", "answer": "The problem was a missing $W_t$ in the equation for correlation. I've updated the above code and did a rerun. We have now the following allocation which is much closer to the Infanger paper.\n> solmat\n US Stock Int Stocks Corp Bonds Gvnt Bond Cash\n50000 5.043872e-01 0.089871441 0.40574133 2.745030e-08 1.788550e-09\n55000 4.050341e-01 0.090625580 0.50434024 2.744996e-08 1.788417e-09\n60000 3.222272e-01 0.091347143 0.58642565 2.744972e-08 1.788325e-09\n65000 2.521815e-01 0.091750138 0.65606829 2.744945e-08 1.788218e-09\n70000 1.920722e-01 0.092167629 0.71576010 2.744928e-08 1.788152e-09\n75000 1.401952e-01 0.092551771 0.76725296 2.744917e-08 1.788109e-09\n80000 9.542976e-02 0.092965569 0.81160464 2.744770e-08 1.787578e-09\n85000 5.926248e-02 0.085949047 0.77256462 2.548693e-10 8.222386e-02\n90000 2.556086e-02 0.042435548 0.35690767 2.546768e-10 5.750959e-01\n95000 5.666367e-07 0.007460414 0.02786258 1.724982e-12 9.646764e-01\n1e+05 4.086260e-03 0.018524853 0.14238886 1.318764e-04 8.348682e-01\n105000 4.229705e-03 0.021298108 0.33601246 1.319004e-04 6.383278e-01\n110000 4.261748e-03 0.022020057 0.49978208 1.319047e-04 4.738042e-01\n115000 1.014474e-02 0.042426599 0.62859439 3.219158e-03 3.156151e-01\n120000 1.040348e-02 0.046451692 0.76095787 3.218435e-03 1.789685e-01\n125000 1.308464e-02 0.132081249 0.79793418 3.218537e-03 5.368139e-02\n130000 1.429210e-02 0.239570571 0.72417454 3.105067e-03 1.885772e-02\n135000 1.471366e-02 0.313967841 0.65235180 3.064252e-03 1.590244e-02\n140000 1.492658e-02 0.369961444 0.59727148 3.044485e-03 1.479601e-02\n145000 1.506062e-02 0.416426483 0.55128612 3.032353e-03 1.419442e-02\n150000 1.515564e-02 0.456708072 0.51130955 3.023891e-03 1.380285e-02\n\n", "topic": "quant", "url": "https://quant.stackexchange.com/questions/35067"}, {"image": "VQAonline_00057266.png", "question": "How to get my nitrocellulose paintcoat shiny and prevent clouding?", "context": "I am working on a prop helmet for LARP. It's made from foam and by now I have spent almost one and a half rattle cans of shiny black, high gloss nitrocellulose-based spraypaint on it in about 6 or 7 layers to attempt to remove the dreaded paint blush. However... it always turns a dull gray as the paint blushes/hazes/mists.\n\nBecause the helmet is made from high-density EVA foam and contact cement, I can't use a heat gun on the lacquer to try and dry it faster, and nitrocellulose lacquers are highly flammable anyway. It is sealed with Ponal Express, then primed with 2 layers of an automotive filler-primer and has accumulated quite some black paint layers by now. Nitrocellulose lacquer is not known to react kindly to other paint types getting slapped over it, with results from not adhering at all over bubbling to even noxious gas. As such, a different paint is not in the cards, as stripping the paint isn't possible without destroying the prop.\nIs there a secret I miss during application of nitrocellulose lacquer to prevent the blush, or something I can make to get the dull surface to shine black? Test pieces were blush free and the blush itself isn't everywhere, so it must be a technique error.\nI do NOT need a brand suggestion, the problem seems to be in some other factor like the application technique or the circumstances as I am getting some glossy bits at the edge and then all hazed paint.\n", "answer": "What you're seeing with that nitrocellulose lacquer (equivalent to nitrate airplane dope, guitar lacquer or old style fingernail polish) is blush.\nBlush is caused by the rapid cooling of freshly applied solvent based coatings generating condensation, which produces a matte surface rather than the intended high gloss. The solution is humidity control.\nEither apply your lacquer when the local humidity is quite low (I've had good results with spray enamel at 20%, but lower would be better for a coating that contains ether or acetone, like your nitrocellulose lacquer), or take direct control of your local humidity by painting in an aggressively dehumidified room (while wearing a respirator with a suitable filter cartridge for the solvents you're using -- breathing concentrate ether and acetone is decidedly bad). Gentle heating while drying will do no harm as long as the air is very dry, but likely isn't needed if the air is dry enough to avoid blush in the first place.\nOnce you have humidity under control, you can wet sand the existing coats (600 or finer wet/dry sanding paper and water), dry thoroughly, and then apply at least two additional coats of your lacquer, observing the \"fat over lean\" rule -- that is, either use less reducer in the first coat and increase it with each subsequent coat, or if you're using cans, spray minimal amounts and then work up to heavier coats (observing recoating recommendations the can, often \"recoat within 20 minutes or wait 24 hours\" or similar).\n", "topic": "rpg", "url": "https://rpg.stackexchange.com/questions/190970"}, {"image": "VQAonline_00057084.png", "question": "How do I read the 2[W] and 'at level X' of attack cards?", "context": "Probably a simple question but one I cannot figure out from the books:\nSome power and attack cards have at the HIT statement a small symbol saying 1[W], or 2[W] at level 21. For example:\n\nHow do I read the 2[W]? Is it 2 times the weapon? does someone need to roll 2 d20 checks? I understand this is probably for the higher levels but my players ask questions alot and I just want to be prepared. :)\n", "answer": "This is nothing special for power cards in particular - powers in the books share this notation as well.\nThe [W] means your weapon damage dice. Crack your Player's Handbook open to page 276 \u2014 the Attack Results section \u2014 and look at the right hand side.\nAs in the example it gives: if the weapon you're using for this power is a regular dagger, your damage dice is 1d4. If a power instructs you to roll 2[W], you roll twice that. For a dagger, that means 2d4.\nIf your weapon dice consists of multiple dice already, you still roll twice that for 2[W]. A Scythe's weapon damage dice is 2d4, so 1[W] means 2d4, 2[W] means 4d4, 3[W] means 6d4, etc.\nIf you're still getting used to the combat system and powers (i.e. most things you will use often), there's a few sections you should liberally refer to:\n\nThe Combat chapter, on PHB page 264.\nThe How to read a power section, which is located right near the beginning of the Classes chapter (since that's where all the powers are) and begins at page 54.\nAlso, equipment blocks (or equipment cards) aren't quite the same as power blocks (and cards). They're explained in the Equipment chapter, right at the beginning of the Magic items section, under the heading Reading a Magic Item on page 224.\n\nThe \"at level 21\" bit means your power upgrades at level 21. A lot of at-wills do this (and so do some other powers), because they'll be with you the whole way and had better not get outdated. From levels 1 to 20, you'll be rolling 1[W] for damage, but as of level 21, you'll roll 2[W] instead. (That's the \"increase damage to\" bit.)\n", "topic": "rpg", "url": "https://rpg.stackexchange.com/questions/41431"}, {"image": "VQAonline_00057109.png", "question": "Does Wild Shape actually allow [Monster] feats?", "context": "Background\nI have never had a problem with Druids in my games take things such as, Multiattack, Improved Multiattack, and Improved Natural Attack. However, I reached a 'scratch my head' moment whenever a player, who loves wolves, decided on specializing in wolf forms.\nShe embraced the whole Trip attack thing, and I fully supported her build. However, she wanted the archtypical 'intimidating wolf howl' and saw Mighty Roar, and Greater Mighty Roar in Savage Species.\n\n\nProblem\nMighty Roar [Monstrous]\n\nPrerequisite: Animal or magical beast; Large size.\n\nGreater Mighty Roar [Monstrous]\n\nPrerequisite: Animal or magical beast; Large size, Mighty Roar.\n\nThe prerequisites... Animal or magical beast. I told her, \"But you aren't an animal, and you don't 'turn' into an animal, you retain the humanoid type.\"\nHer reply was, \"Well, you let me have Improved Natural Attack, and as a humanoid I have no natural attack.\"\nShe had me speechless.\n\nQuestion: Does Wild Shape actually allow Monster feats mentioned above? Why or why not?\n", "answer": "[Monster] feats vs. [Monstrous] feats\nYes, these are different things. Which is absurd, and a terrible choice on Wizards part.\n\nMonster Feats\nThese feats apply to abilities most commonly found amongst monsters or are related to monsters.\n\n\nMonstrous Feats\nOnly creatures with a monstrous form or one or more monstrous abilities may select these feats. Monstrous forms are those that are unavailable to normal humanoid and animal creatures, including but not limited to extra appendages, nonstandard appendages (such as tentacles or a tail), supernatural abilities, and spell-like abilities. With your DM\u2019s permission, you may be able to choose monstrous feats for your character if your character acquired unusual abilities through transformation or by advancing in a prestige class.\n\nMonster feats are from the Monster Manual and are so-called just because Wizards assumed that monsters were the most likely ones to want them; they have no special requirements and PCs are allowed to take them. This includes Improved Natural Attack.\nMonstrous feats were introduced by Savage Species, and as you can see, have extremely vague and ill-defined requirements. In particular, monstrous features are \u201cunavailable to normal humanoid creatures\u201d but include both spell-like abilities and supernatural abilities, which are not at all unusual in humanoid creatures. Then there\u2019s mention of being allowed to take them if you transformed or took a prestige class, but no mention of what happens if you had those features without doing so. Savage Species is not a well-written book.\nAnyway, it\u2019s pretty clear that Wild Shape probably qualifies the druid for [Monstrous] feats anyway, so long as you approve it as DM. The final issue is Mighty Roar, which has another requirement on top of being Monstrous: it requires Animal or Magical Beast type. Why the authors felt they needed another requirement on top of those from Monstrous, I couldn\u2019t say, but they did. As you note, Wild Shape does not change type. And you are correct to make a distinction between this and Improved Natural Attack; INA does not require any particular type.\nBut again, Savage Species is a poorly-written book. If the druid is willing to accept that they\u2019ll only get to use Mighty Roar while using Wild Shape, I strongly feel it should absolutely be allowed.\n", "topic": "rpg", "url": "https://rpg.stackexchange.com/questions/63281"}, {"image": "VQAonline_00057355.png", "question": "\u041a\u0430\u043a \u043d\u0430\u0437\u044b\u0432\u0430\u0435\u0442\u0441\u044f \u043e\u0434\u0435\u0436\u0434\u0430 \u0440\u0443\u0441\u0430\u043b\u043e\u043a", "context": "\u0415\u0441\u0442\u044c \u043b\u0438 \u0443 \u044d\u0442\u043e\u0439 \u043e\u0434\u0435\u0436\u0434\u044b \u043d\u0430\u0437\u0432\u0430\u043d\u0438\u0435, \u043a\u043e\u0442\u043e\u0440\u043e\u0435 \u0431\u044b \u043e\u0434\u043d\u043e\u0437\u043d\u0430\u0447\u043d\u043e \u0443\u043a\u0430\u0437\u044b\u0432\u0430\u043b\u043e \u0438\u043c\u0435\u043d\u043d\u043e \u043d\u0430 \u043d\u0435\u0451? \u0427\u0442\u043e \u044d\u0442\u043e: \u043f\u043b\u0430\u0442\u044c\u0435, \u0441\u043e\u0440\u043e\u0447\u043a\u0430?\n\n", "answer": "\u041f\u043e \u0432\u0438\u0434\u0443 \u043f\u043e\u0445\u043e\u0436\u0438 \u043d\u0430 \u0441\u043b\u0430\u0432\u044f\u043d\u0441\u043a\u0438\u0445 \u0440\u0443\u0441\u0430\u043b\u043e\u043a, \u0430 \u0442\u0435 \u0447\u0430\u0449\u0435 \u0432\u0441\u0435\u0433\u043e \u0440\u0438\u0441\u043e\u0432\u0430\u043b\u0438\u0441\u044c \u0432 \u0431\u0435\u043b\u044b\u0445 \u0440\u0443\u0431\u0430\u0445\u0430\u0445, \u0438\u043d\u043e\u0433\u0434\u0430 \u0432 \u0441\u0430\u0440\u0430\u0444\u0430\u043d\u0430\u0445, \u043d\u043e \u0443 \u0412\u0430\u0441 \u043d\u0430 \u0440\u0438\u0441\u0443\u043d\u043a\u0435 \u0438\u043c\u0435\u043d\u043d\u043e \u0440\u0443\u0431\u0430\u0445\u0430 - \u0442\u0430\u043a \u043d\u0430\u0437\u044b\u0432\u0430\u0435\u043c\u043e\u0435 \u043d\u0438\u0436\u043d\u0435\u0435 \u043f\u043b\u0430\u0442\u044c\u0435. https://web-kapiche.ru/177-slavyanskie-rusalki.html\n", "topic": "rus", "url": "https://rus.stackexchange.com/questions/454075"}, {"image": "VQAonline_00057322.png", "question": "\u0421\u0434\u0435\u043b\u0430\u0439\u0442\u0435 \u0441\u0438\u043d\u0442\u0430\u043a\u0441\u0438\u0447\u0435\u0441\u043a\u0438\u0439 \u0440\u0430\u0437\u0431\u043e\u0440 \u0441\u043b\u043e\u0436\u043d\u044b\u0445 \u043f\u0440\u0435\u0434\u043b\u043e\u0436\u0435\u043d\u0438\u0439. \u0421\u043e\u0441\u0442\u0430\u0432\u044c\u0442\u0435 \u0441\u0445\u0435\u043c\u044b \u0438\u0445 \u0441\u0442\u0440\u043e\u0435\u043d\u0438\u044f", "context": "\u0418 \u0435\u0441\u043b\u0438 \u043f\u0440\u043e\u0433\u0440\u0435\u0441\u0441 \u043d\u0430\u0440\u0430\u0432\u043d\u0435 \u0441 \u0443\u043c\u043d\u043e\u0436\u0435\u043d\u0438\u0435\u043c \u0441\u0440\u0435\u0434\u0441\u0442\u0432 \u0431\u043b\u0430\u0433\u043e\u0441\u043e\u0441\u0442\u043e\u044f\u043d\u0438\u044f \u0441\u043e\u0441\u0442\u043e\u0438\u0442 \u0432 \u043e\u0434\u043d\u043e\u0432\u0440\u0435\u043c\u0435\u043d\u043d\u043e\u043c \u043f\u043e\u0432\u044b\u0448\u0435\u043d\u0438\u0438 \u043c\u043e\u0440\u0430\u043b\u044c\u043d\u044b\u0445 \u043e\u0431\u044f\u0437\u0430\u043d\u043d\u043e\u0441\u0442\u0435\u0439, \u043f\u043e\u0442\u043e\u043c\u0443 \u0447\u0442\u043e \u0442\u043e\u043b\u044c\u043a\u043e \u0441\u043e\u0432\u0435\u0440\u0448\u0435\u043d\u043d\u044b\u0439 \u0447\u0435\u043b\u043e\u0432\u0435\u043a \u0441\u043f\u043e\u0441\u043e\u0431\u0435\u043d \u0434\u043e\u0431\u0438\u0442\u044c\u0441\u044f \u0441\u043e\u0432\u0435\u0440\u0448\u0435\u043d\u043d\u043e\u0433\u043e \u0441\u0447\u0430\u0441\u0442\u044c\u044f, \u0434\u043b\u044f \u044d\u0442\u043e\u0433\u043e \u043d\u0430\u0434\u043b\u0435\u0436\u0430\u043b\u043e \u043a\u0430\u0436\u0434\u043e\u043c\u0443 \u0438\u043c\u0435\u0442\u044c \u0438 \u0441\u043e\u0432\u0435\u0440\u0448\u0435\u043d\u043d\u0443\u044e \u0431\u0438\u043e\u0433\u0440\u0430\u0444\u0438\u044e, \u0447\u0442\u043e\u0431 \u043d\u0435 \u0441\u0442\u044b\u0434\u043d\u043e \u0431\u044b\u043b\u043e \u0440\u0430\u0441\u0441\u043a\u0430\u0437\u0430\u0442\u044c \u0435\u0435 \u0432\u0441\u043b\u0443\u0445, \u043f\u0440\u0438 \u0434\u0435\u0442\u044f\u0445, \u0432 \u0441\u043e\u043b\u043d\u0435\u0447\u043d\u044b\u0439 \u043f\u043e\u043b\u0434\u0435\u043d\u044c, \u043d\u0430 \u0441\u0430\u043c\u044b\u0445 \u043b\u044e\u0434\u043d\u044b\u0445 \u043f\u043b\u043e\u0449\u0430\u0434\u044f\u0445 \u043c\u0438\u0440\u0430 (\u041b\u0435\u043e\u043d.).\n\u0412\u043e\u0442 \u043c\u043e\u0438 \u043f\u043e\u043f\u044b\u0442\u043a\u0438\n\u041f\u043e\u0432\u0435\u0441\u0442\u0432\u043e\u0432\u0430\u0442\u0435\u043b\u044c\u043d\u043e\u0435, \u043d\u0435 \u0432\u043e\u0441\u043a\u043b\u0438\u0446\u0430\u0442\u0435\u043b\u044c\u043d\u043e\u0435, \u0441\u043b\u043e\u0436\u043d\u043e\u0435, \u0441\u0432\u044f\u0437\u044c \u0441\u043e\u044e\u0437\u043d\u0430\u044f, \u0441\u0440\u0435\u0434\u0441\u0442\u0432\u043e \u0441\u0432\u044f\u0437\u0438 \u043f\u043e\u0434\u0447\u0438\u043d\u0438\u0442\u0435\u043b\u044c\u043d\u044b\u0435 \u0441\u043e\u044e\u0437\u044b \u043f\u043e\u0442\u043e\u043c\u0443 \u0447\u0442\u043e, \u0435\u0441\u043b\u0438, \u0447\u0442\u043e\u0431, \u0434\u043b\u044f \u044d\u0442\u043e\u0433\u043e, \u0421\u041f\u041f. \n1-\u0435 \u041f\u041f: \u0434\u0432\u0443\u0441\u043e\u0441\u0442\u0430\u0432\u043d\u043e\u0435, \u0433\u0440\u0430\u043c\u043c\u0430\u0442\u0438\u0447\u0435\u0441\u043a\u0430\u044f \u043e\u0441\u043d\u043e\u0432\u0430 \u2013 \u043f\u0440\u043e\u0433\u0440\u0435\u0441\u0441 \u0441\u043e\u0441\u0442\u043e\u0438\u0442, \u0440\u0430\u0441\u043f\u0440\u043e\u0441\u0442\u0440\u0430\u043d\u0435\u043d\u043d\u043e\u0435, \u043d\u0435 \u043e\u0441\u043b\u043e\u0436\u043d\u0435\u043d\u043e. \n2-\u0435 \u041f\u041f: \u0434\u0432\u0443\u0441\u043e\u0441\u0442\u0430\u0432\u043d\u043e\u0435, \u0433\u0440\u0430\u043c\u043c\u0430\u0442\u0438\u0447\u0435\u0441\u043a\u0430\u044f \u043e\u0441\u043d\u043e\u0432\u0430 \u2013 \u043c\u044b \u0441 \u043a\u043b\u0430\u0441\u0441\u043e\u043c \u043f\u043e\u0435\u0445\u0430\u043b\u0438, \u0440\u0430\u0441\u043f\u0440\u043e\u0441\u0442\u0440\u0430\u043d\u0435\u043d\u043d\u043e\u0435, \u043d\u0435 \u043e\u0441\u043b\u043e\u0436\u043d\u0435\u043d\u043e.\n3-\u0435 \u041f\u041f: \u043e\u0441\u043b\u043e\u0436\u043d\u0435\u043d\u043e \u043e\u0434\u043d\u043e\u0440\u043e\u0434\u043d\u044b\u043c\u0438 \u0447\u043b\u0435\u043d\u0430\u043c\u0438 \u043f\u0440\u0435\u0434\u043b\u043e\u0436\u0435\u043d\u0438\u044f.\n\n", "answer": "(1) \u0418 \u0435\u0441\u043b\u0438 \u043f\u0440\u043e\u0433\u0440\u0435\u0441\u0441 \u043d\u0430\u0440\u0430\u0432\u043d\u0435 \u0441 \u0443\u043c\u043d\u043e\u0436\u0435\u043d\u0438\u0435\u043c \u0441\u0440\u0435\u0434\u0441\u0442\u0432 \u0431\u043b\u0430\u0433\u043e\u0441\u043e\u0441\u0442\u043e\u044f\u043d\u0438\u044f \u0441\u043e\u0441\u0442\u043e\u0438\u0442 \u0432 \u043e\u0434\u043d\u043e\u0432\u0440\u0435\u043c\u0435\u043d\u043d\u043e\u043c \u043f\u043e\u0432\u044b\u0448\u0435\u043d\u0438\u0438 \u043c\u043e\u0440\u0430\u043b\u044c\u043d\u044b\u0445 \u043e\u0431\u044f\u0437\u0430\u043d\u043d\u043e\u0441\u0442\u0435\u0439, (2) \u043f\u043e\u0442\u043e\u043c\u0443 \u0447\u0442\u043e \u0442\u043e\u043b\u044c\u043a\u043e \u0441\u043e\u0432\u0435\u0440\u0448\u0435\u043d\u043d\u044b\u0439 \u0447\u0435\u043b\u043e\u0432\u0435\u043a \u0441\u043f\u043e\u0441\u043e\u0431\u0435\u043d \u0434\u043e\u0431\u0438\u0442\u044c\u0441\u044f \u0441\u043e\u0432\u0435\u0440\u0448\u0435\u043d\u043d\u043e\u0433\u043e \u0441\u0447\u0430\u0441\u0442\u044c\u044f, (3) (\u0442\u043e) \u0434\u043b\u044f \u044d\u0442\u043e\u0433\u043e \u043d\u0430\u0434\u043b\u0435\u0436\u0430\u043b\u043e \u043a\u0430\u0436\u0434\u043e\u043c\u0443 \u0438\u043c\u0435\u0442\u044c \u0438 \u0441\u043e\u0432\u0435\u0440\u0448\u0435\u043d\u043d\u0443\u044e \u0431\u0438\u043e\u0433\u0440\u0430\u0444\u0438\u044e, (4)\u0447\u0442\u043e\u0431 \u043d\u0435 \u0441\u0442\u044b\u0434\u043d\u043e \u0431\u044b\u043b\u043e \u0440\u0430\u0441\u0441\u043a\u0430\u0437\u0430\u0442\u044c \u0435\u0435 \u0432\u0441\u043b\u0443\u0445, \u043f\u0440\u0438 \u0434\u0435\u0442\u044f\u0445, \u0432 \u0441\u043e\u043b\u043d\u0435\u0447\u043d\u044b\u0439 \u043f\u043e\u043b\u0434\u0435\u043d\u044c, \u043d\u0430 \u0441\u0430\u043c\u044b\u0445 \u043b\u044e\u0434\u043d\u044b\u0445 \u043f\u043b\u043e\u0449\u0430\u0434\u044f\u0445 \u043c\u0438\u0440\u0430 (\u041b\u0435\u043e\u043d.).\n\u042d\u0442\u043e \u043e\u0434\u043d\u043e \u0441\u043b\u043e\u0436\u043d\u043e\u0435 \u043f\u0440\u0435\u0434\u043b\u043e\u0436\u0435\u043d\u0438\u0435 (\u0441\u043b\u043e\u0436\u043d\u043e\u0435 \u0441\u0438\u043d\u0442\u0430\u043a\u0441\u0438\u0447\u0435\u0441\u043a\u043e\u0435 \u0446\u0435\u043b\u043e\u0435), \u0441\u043e\u0441\u0442\u043e\u0438\u0442 \u0438\u0437 4-\u0445 \u043f\u0440\u0435\u0434\u043b\u043e\u0436\u0435\u043d\u0438\u0439. \u041c\u0435\u0436\u0434\u0443 (1) \u0438 (3) \u043f\u043e\u0434\u0447\u0438\u043d\u0438\u0442\u0435\u043b\u044c\u043d\u0430\u044f \u0441\u0432\u044f\u0437\u044c , \u043f\u0440\u0435\u0434\u043b\u043e\u0436\u0435\u043d\u0438\u0435 (1) \u2013 \u043f\u0440\u0438\u0434\u0430\u0442\u043e\u0447\u043d\u043e\u0435 \u0443\u0441\u043b\u043e\u0432\u0438\u044f.\n\u041f\u0440\u0438\u0434\u0430\u0442\u043e\u0447\u043d\u043e\u0435 \u0443\u0441\u043b\u043e\u0432\u0438\u044f (1) \u044f\u0432\u043b\u044f\u0435\u0442\u0441\u044f \u0433\u043b\u0430\u0432\u043d\u044b\u043c \u043f\u043e \u043e\u0442\u043d\u043e\u0448\u0435\u043d\u0438\u044e \u043a \u043f\u0440\u0438\u0434\u0430\u0442\u043e\u0447\u043d\u043e\u043c\u0443 \u043f\u0440\u0438\u0447\u0438\u043d\u044b (2), \u0430 \u043f\u0440\u0435\u0434\u043b\u043e\u0436\u0435\u043d\u0438\u0435 (3) \u0438\u043c\u0435\u0435\u0442 \u043f\u0440\u0438\u0434\u0430\u0442\u043e\u0447\u043d\u043e\u0435 \u0446\u0435\u043b\u0438 (4). \n\u041f\u0440\u0435\u0434\u043b\u043e\u0436\u0435\u043d\u0438\u044f (1) \u0438 (4) \u2013 \u044d\u0442\u043e \u043d\u0435\u043e\u0434\u043d\u043e\u0440\u043e\u0434\u043d\u044b\u0435 \u043f\u0440\u0438\u0434\u0430\u0442\u043e\u0447\u043d\u044b\u0435, \u043e\u0442\u043d\u0435\u0441\u0435\u043d\u043d\u044b\u0435 \u043a \u043e\u0434\u043d\u043e\u043c\u0443 \u0433\u043b\u0430\u0432\u043d\u043e\u043c\u0443 (3), \u0442\u0430\u043a\u0430\u044f \u0441\u0432\u044f\u0437\u044c \u044f\u0432\u043b\u044f\u0435\u0442\u0441\u044f \u043f\u0430\u0440\u0430\u043b\u043b\u0435\u043b\u044c\u043d\u043e\u0439. \u0410 \u0441\u0432\u044f\u0437\u044c \u043f\u0440\u0435\u0434\u043b\u043e\u0436\u0435\u043d\u0438\u0439 3, 1, 2 \u2013 \u044d\u0442\u043e \u043f\u043e\u0441\u043b\u0435\u0434\u043e\u0432\u0430\u0442\u0435\u043b\u044c\u043d\u043e\u0435 \u043f\u043e\u0434\u0447\u0438\u043d\u0435\u043d\u0438\u0435.\n\u0412\u0441\u0451 \u044d\u0442\u043e \u043d\u0430\u0434\u043e \u0438\u0437\u043e\u0431\u0440\u0430\u0437\u0438\u0442\u044c \u0432 \u0432\u0438\u0434\u0435 \u0441\u0445\u0435\u043c\u044b, \u0433\u0434\u0435 \u0443\u043a\u0430\u0437\u0430\u043d\u044b \u043f\u0440\u0435\u0434\u0438\u043a\u0430\u0442\u0438\u0432\u043d\u044b\u0435 \u043e\u0441\u043d\u043e\u0432\u044b \u0438 \u0441\u043e\u044e\u0437\u044b.\n\u041f\u0440\u0435\u0434\u0438\u043a\u0430\u0442\u0438\u0432\u043d\u044b\u0435 \u043e\u0441\u043d\u043e\u0432\u044b: \u043f\u0440\u043e\u0433\u0440\u0435\u0441\u0441 \u0441\u043e\u0441\u0442\u043e\u0438\u0442, \u0447\u0435\u043b\u043e\u0432\u0435\u043a \u0441\u043f\u043e\u0441\u043e\u0431\u0435\u043d \u0434\u043e\u0431\u0438\u0442\u044c\u0441\u044f, \u043d\u0430\u0434\u043b\u0435\u0436\u0430\u043b\u043e \u0438\u043c\u0435\u0442\u044c, \u043d\u0435 \u0441\u0442\u044b\u0434\u043d\u043e \u0431\u044b\u043b\u043e \u0440\u0430\u0441\u0441\u043a\u0430\u0437\u0430\u0442\u044c.\n\u041f\u0440\u0438\u043c\u0435\u0447\u0430\u043d\u0438\u0435. \u0412 \u043f\u0440\u0435\u0434\u043b\u043e\u0436\u0435\u043d\u0438\u0438 3 \u0445\u043e\u0440\u043e\u0448\u043e \u0431\u044b \u0438\u0441\u043f\u043e\u043b\u044c\u0437\u043e\u0432\u0430\u0442\u044c \u0441\u043e\u0441\u043b\u0430\u0433\u0430\u0442\u0435\u043b\u044c\u043d\u043e\u0435 \u043d\u0430\u043a\u043b\u043e\u043d\u0435\u043d\u0438\u0435 \"\u043d\u0430\u0434\u043b\u0435\u0436\u0430\u043b\u043e \u0431\u044b\".\n", "topic": "rus", "url": "https://rus.stackexchange.com/questions/434276"}, {"image": "VQAonline_00057364.png", "question": "\u041c\u0435\u0442\u0430\u0444\u043e\u0440\u043e\u0439 \u0447\u0435\u0433\u043e \u043c\u043e\u0436\u0435\u0442 \u0431\u044b\u0442\u044c \u043f\u0435\u0440\u0435\u043f\u0451\u043b\u043a\u0430?", "context": "\n\u0422\u0440\u0443\u0434\u043d\u043e \u043f\u0435\u0440\u0435\u0434\u0430\u0442\u044c \u0441\u043b\u043e\u0432\u0430\u043c\u0438, \u043a\u0430\u043a\u043e\u044e \u043f\u0435\u0440\u0435\u043f\u0435\u043b\u043a\u043e\u0439 \u0432\u043b\u0435\u0442\u0435\u043b \u0432 \u043a\u043e\u043c\u043d\u0430\u0442\u0443 \u043c\u043e\u043b\u043e\u0434\u043e\u0439 \u043f\u0440\u043e\u0433\u0440\u0435\u0441\u0441\u0438\u0441\u0442.\n\u0418.\u00a0\u0421.\u00a0\u0422\u0443\u0440\u0433\u0435\u043d\u0435\u0432. \u00ab\u041e\u0442\u0446\u044b \u0438\u00a0\u0434\u0435\u0442\u0438\u00bb.\n\n\u0427\u0442\u043e \u0438\u043c\u0435\u0435\u0442 \u0432 \u0432\u0438\u0434\u0443 \u0430\u0432\u0442\u043e\u0440?\n\u041a\u0430\u043a\u0438\u0435 \u0442\u0430\u043a\u0438\u0435 \u0445\u0430\u0440\u0430\u043a\u0442\u0435\u0440\u043d\u044b\u0435 \u0447\u0435\u0440\u0442\u044b \u0435\u0441\u0442\u044c \u0443 \u043f\u0435\u0440\u0435\u043f\u0451\u043b\u043a\u0438, \u0447\u0442\u043e\u0431 \u043f\u043e\u0432\u0435\u0434\u0435\u043d\u0438\u0435 \u0447\u0435\u043b\u043e\u0432\u0435\u043a\u0430 \u043c\u043e\u0436\u043d\u043e \u0431\u044b\u043b\u043e \u0441\u0440\u0430\u0432\u043d\u0438\u0442\u044c \u0441 \u043f\u0435\u0440\u0435\u043f\u0451\u043b\u043a\u043e\u0439?\n\n\u0418\u0441\u0442\u043e\u0447\u043d\u0438\u043a \u0438\u0437\u043e\u0431\u0440\u0430\u0436\u0435\u043d\u0438\u044f: \u0410\u0432\u0442\u043e\u0440: Luis Miguel Bugallo S\u00e1nchez (Lmbuga Commons)(Lmbuga Galipedia)Publicada por/Publish by: Luis Miguel Bugallo S\u00e1nchez - \u0441\u043e\u0431\u0441\u0442\u0432\u0435\u043d\u043d\u0430\u044f \u0440\u0430\u0431\u043e\u0442\u0430, CC BY-SA 3.0, https://commons.wikimedia.org/w/index.php?curid=353465\n\n\n\u041f\u0435\u0440\u0435\u043f\u0435\u043b\u0430 \u0434\u0435\u0440\u0436\u0430\u0442\u0441\u044f \u043d\u0430 \u0437\u0435\u043c\u043b\u0435, \u0432 \u0433\u0443\u0441\u0442\u043e\u0439 \u0442\u0440\u0430\u0432\u0435 \u0438 \u043d\u0430 \u043f\u043e\u043b\u044f\u0445, \u0431\u043e\u043b\u0435\u0435 \u0438\u043b\u0438 \u043c\u0435\u043d\u0435\u0435\n\u043e\u0431\u0449\u0435\u0441\u0442\u0432\u0435\u043d\u043d\u044b. \u041f\u0440\u0435\u043a\u0440\u0430\u0441\u043d\u043e \u0431\u0435\u0433\u0430\u044e\u0442, \u043b\u0435\u0442\u0430\u044e\u0442 \u0431\u044b\u0441\u0442\u0440\u043e, \u043d\u043e \u0440\u0435\u0434\u043a\u043e \u043f\u0440\u043e\u0434\u043e\u043b\u0436\u0438\u0442\u0435\u043b\u044c\u043d\u043e.\n\u041a\u0440\u0438\u0447\u0430\u0442 \u043f\u0440\u043e\u043d\u0437\u0438\u0442\u0435\u043b\u044c\u043d\u043e, \u043d\u043e \u043c\u043e\u043d\u043e\u0442\u043e\u043d\u043d\u043e. \u041f\u0438\u0442\u0430\u044e\u0442\u0441\u044f \u043c\u0435\u043b\u043a\u0438\u043c\u0438 \u0441\u0435\u043c\u0435\u043d\u0430\u043c\u0438 \u0438\n\u043d\u0430\u0441\u0435\u043a\u043e\u043c\u044b\u043c\u0438.\n[\u0412\u0438\u043a\u0438\u043f\u0435\u0434\u0438\u044f]\n\n\u0427\u0442\u043e \u0438\u0437 \u044d\u0442\u043e\u0433\u043e \u043c\u043e\u0436\u0435\u0442 \u043f\u043e\u0441\u043b\u0443\u0436\u0438\u0442\u044c \u043e\u0441\u043d\u043e\u0432\u043e\u0439 \u0434\u043b\u044f \u043c\u0435\u0442\u0430\u0444\u043e\u0440\u044b?\n\u041d\u0430\u043f\u0440\u0438\u043c\u0435\u0440:\n\n\u00ab[\u041f\u0435\u0440\u0435\u043f\u0435\u043b\u0430] \u043b\u0435\u0442\u0430\u044e\u0442 \u0431\u044b\u0441\u0442\u0440\u043e\u00bb: \u00ab\u043f\u0435\u0440\u0435\u043f\u0435\u043b\u043a\u043e\u0439 \u0432\u043b\u0435\u0442\u0435\u043b\u00bb\u00a0\u2014 \u0431\u044b\u0441\u0442\u0440\u043e \u0432\u043b\u0435\u0442\u0435\u043b?\n\u00ab[\u041f\u0435\u0440\u0435\u043f\u0435\u043b\u0430] \u041a\u0440\u0438\u0447\u0430\u0442 \u043f\u0440\u043e\u043d\u0437\u0438\u0442\u0435\u043b\u044c\u043d\u043e, \u043d\u043e\u00a0\u043c\u043e\u043d\u043e\u0442\u043e\u043d\u043d\u043e\u00bb:\u00a0\u00ab\u043f\u0435\u0440\u0435\u043f\u0435\u043b\u043a\u043e\u0439 \u0432\u043b\u0435\u0442\u0435\u043b\u00bb\u00a0\u2014 \u0432\u043b\u0435\u0442\u0435\u043b, \u043a\u0440\u0438\u0447\u0430/\u0433\u043e\u0432\u043e\u0440\u044f \u0433\u0440\u043e\u043c\u043a\u043e \u043f\u0440\u043e\u043d\u0437\u0438\u0442\u0435\u043b\u044c\u043d\u043e \u0438\u00a0\u043c\u043e\u043d\u043e\u0442\u043e\u043d\u043d\u043e?\n\n\n\u0412\u043e\u0437\u043c\u043e\u0436\u043d\u043e, \u0438\u043c\u0435\u0435\u0442\u0441\u044f \u0432 \u0432\u0438\u0434\u0443 \u043d\u0435 \u043f\u0442\u0438\u0446\u0430, \u0430 \u0442\u0430\u043d\u0435\u0446?\n\n\u041f\u0435\u0440\u0435\u043f\u0451\u043b\u043a\u0430 \u2014 \u0443\u043a\u0440\u0430\u0438\u043d\u0441\u043a\u0438\u0439 \u0438 \u0431\u0435\u043b\u043e\u0440\u0443\u0441\u0441\u043a\u0438\u0439 \u0432\u0435\u0441\u0435\u043d\u043d\u0438\u0439 \u043d\u0430\u0440\u043e\u0434\u043d\u044b\u0439 \u0442\u0430\u043d\u0435\u0446-\u0438\u0433\u0440\u0430. \u0422\u0430\u043d\u0435\u0446 \u0438\u0441\u043f\u043e\u043b\u043d\u044f\u0435\u0442\u0441\u044f \u0432 \u043a\u0440\u0443\u0433\u0435, \u0432 \u0446\u0435\u043d\u0442\u0440\u0435 \u043a\u043e\u0442\u043e\u0440\u043e\u0433\u043e \u043d\u0430\u0445\u043e\u0434\u0438\u0442\u0441\u044f \u0443\u0447\u0430\u0441\u0442\u043d\u0438\u0446\u0430 (\u00ab\u043f\u0435\u0440\u0435\u043f\u0451\u043b\u043a\u0430\u00bb). \u0412\u043e \u0432\u0440\u0435\u043c\u044f \u043f\u0435\u043d\u0438\u044f \u00ab\u043f\u0435\u0440\u0435\u043f\u0451\u043b\u043a\u0430\u00bb \u0432 \u0442\u0430\u043a\u0442 \u0441\u043b\u043e\u0432\u0430\u043c \u043f\u0435\u0441\u043d\u0438 \u043f\u043e\u043a\u0430\u0437\u044b\u0432\u0430\u0435\u0442: \u0442\u043e \u0443 \u043d\u0435\u0451 \u0431\u043e\u043b\u0438\u0442 \u0433\u043e\u043b\u043e\u0432\u043a\u0430, \u0442\u043e \u0431\u0435\u043b\u044b\u0435 \u0440\u0443\u0447\u0435\u043d\u044c\u043a\u0438, \u0442\u043e \u043d\u043e\u0436\u043a\u0438 \u0438 \u0442.\u0434. \u0412\u043e \u0432\u0442\u043e\u0440\u043e\u0439 \u0447\u0430\u0441\u0442\u0438 \u0438\u0433\u0440\u044b \u0438\u0434\u0451\u0442 \u0441\u0442\u0430\u0440\u0438\u043a \u0438 \u043d\u0430\u0433\u0430\u0439\u043a\u0443 \u043d\u0435\u0441\u0451\u0442, \u0438\u043b\u0438 \u043c\u0438\u043b\u044b\u0439-\u0447\u0435\u0440\u043d\u043e\u0431\u0440\u043e\u0432\u044b\u0439 \u043d\u0435\u0441\u0451\u0442 \u0440\u0443\u0431\u0430\u0448\u0435\u0447\u043a\u0443 (\u0432\u0430\u0440\u0438\u0430\u043d\u0442: \u043d\u043e\u0432\u044b\u0435 \u0447\u043e\u0431\u043e\u0442\u044b). \u0412\u043e \u0432\u0441\u0435\u0445 \u0432\u0430\u0440\u0438\u0430\u043d\u0442\u0430\u0445 \u0438\u0433\u0440\u044b \u0432\u043e \u0432\u0442\u043e\u0440\u043e\u0439 \u0447\u0430\u0441\u0442\u0438 \u00ab\u043f\u0435\u0440\u0435\u043f\u0451\u043b\u043e\u0447\u043a\u0430\u00bb \u0440\u0430\u0437\u0440\u044b\u0432\u0430\u0435\u0442 \u043a\u0440\u0443\u0433. \u0417\u0430\u0442\u0435\u043c \u0438\u0433\u0440\u0430 \u043d\u0430\u0447\u0438\u043d\u0430\u0435\u0442\u0441\u044f \u0441\u043d\u043e\u0432\u0430. \u0423\u043a\u0440\u0430\u0438\u043d\u0441\u043a\u0438\u0439 \u0442\u0430\u043d\u0435\u0446 \u0438\u0441\u043f\u043e\u043b\u043d\u044f\u0435\u0442\u0441\u044f \u0432 \u0431\u044b\u0441\u0442\u0440\u043e\u043c \u0442\u0435\u043c\u043f\u0435, \u0431\u0435\u043b\u043e\u0440\u0443\u0441\u0441\u043a\u0438\u0439 \u2014 \u0432 \u0443\u043c\u0435\u0440\u0435\u043d\u043d\u043e\u043c.\n[\u0412\u0438\u043a\u0438\u043f\u0435\u0434\u0438\u044f]\n\n\u041f\u043e\u0432\u0435\u0434\u0435\u043d\u0438\u0435 \u00ab\u043f\u0435\u0440\u0435\u043f\u0451\u043b\u043a\u0438\u00bb \u0432 \u0442\u0430\u043d\u0446\u0435 \u0442\u043e\u0436\u0435 \u0442\u0440\u0443\u0434\u043d\u043e \u0441\u0440\u0430\u0432\u043d\u0438\u0442\u044c \u0441 \u043f\u043e\u0432\u0435\u0434\u0435\u043d\u0438\u0435\u043c \u0433\u0435\u0440\u043e\u044f \u0432 \u043a\u043d\u0438\u0433\u0435.\n\n\u041c\u043e\u0436\u0435\u0442, \u0435\u0441\u0442\u044c \u043a\u0430\u043a\u043e\u0435-\u0442\u043e \u0445\u0430\u0440\u0430\u043a\u0442\u0435\u0440\u043d\u043e\u0435 \u043f\u043e\u0432\u0435\u0434\u0435\u043d\u0438\u0435 \u043f\u0435\u0440\u0435\u043f\u0451\u043b\u043a\u0438, \u0441\u0440\u0430\u0432\u043d\u0435\u043d\u0438\u0435 \u0441 \u043a\u043e\u0442\u043e\u0440\u044b\u043c \u043b\u0435\u0433\u043b\u043e \u0432 \u043e\u0441\u043d\u043e\u0432\u0443 \u043c\u0435\u0442\u0430\u0444\u043e\u0440\u044b \u0432 \u043a\u043d\u0438\u0433\u0435, \u0438 \u0432 \u043e\u0441\u043d\u043e\u0432\u0443 \u0442\u0430\u043d\u0446\u0430?\n", "answer": "\u0423 \u043f\u0435\u0440\u0435\u043f\u0451\u043b\u043e\u043a \u0432 \u043e\u0431\u044b\u0447\u0430\u0435, \u0434\u043e\u0432\u043e\u043b\u044c\u043d\u043e \u0431\u043b\u0438\u0437\u043a\u043e \u043f\u043e\u0434\u043f\u0443\u0441\u0442\u0438\u0432 \u0447\u0435\u043b\u043e\u0432\u0435\u043a\u0430, \u043d\u0435\u043e\u0436\u0438\u0434\u0430\u043d\u043d\u043e \u0432\u0437\u043b\u0435\u0442\u0430\u0442\u044c \u0438\u0437 \u0432\u044b\u0441\u043e\u043a\u043e\u0439 \u0442\u0440\u0430\u0432\u044b \u0438\u043b\u0438 \u043a\u043e\u043b\u043e\u0441\u044c\u0435\u0432, \u2014 \u0442\u0430\u043a \u0447\u0442\u043e \u043c\u043e\u0436\u043d\u043e \u0434\u0430\u0436\u0435 \u0438\u0441\u043f\u0443\u0433\u0430\u0442\u044c\u0441\u044f. \u0422\u0443\u0440\u0433\u0435\u043d\u0435\u0432 \u0431\u044b\u043b \u043e\u0445\u043e\u0442\u043d\u0438\u043a\u043e\u043c \u0438 \u043f\u043e\u0432\u0430\u0434\u043a\u0438 \u0440\u0430\u0437\u043d\u044b\u0445 \u043f\u0442\u0438\u0446 \u0445\u043e\u0440\u043e\u0448\u043e \u0437\u043d\u0430\u043b. \u041a\u0440\u043e\u043c\u0435 \u0442\u043e\u0433\u043e, \u0441\u0447\u0438\u0442\u0430\u0435\u0442\u0441\u044f, \u0447\u0442\u043e \u043f\u0442\u0438\u0446\u044b \u0432 \u043e\u0441\u043d\u043e\u0432\u043d\u043e\u043c \u0433\u043b\u0443\u043f\u044b. \u0421\u043c\u044b\u0441\u043b \u0444\u0440\u0430\u0437\u044b: \u043c\u043e\u043b\u043e\u0434\u043e\u0439 \u043f\u0440\u043e\u0433\u0440\u0435\u0441\u0441\u0438\u0441\u0442 \u0432\u043b\u0435\u0442\u0435\u043b (\u0442. \u0435. \u043d\u0430 \u0441\u0430\u043c\u043e\u043c \u0434\u0435\u043b\u0435 \u043e\u043d, \u043a\u043e\u043d\u0435\u0447\u043d\u043e, \u0432\u0431\u0435\u0436\u0430\u043b) \u0432 \u043a\u043e\u043c\u043d\u0430\u0442\u0443 \u0442\u0430\u043a \u0441\u0442\u0440\u0435\u043c\u0438\u0442\u0435\u043b\u044c\u043d\u043e, \u0447\u0442\u043e \u043c\u043e\u0433 \u0434\u0430\u0436\u0435 \u0438\u0441\u043f\u0443\u0433\u0430\u0442\u044c, \u2014 \u0438 \u043f\u043e\u0441\u0442\u0443\u043f\u0438\u043b \u043e\u043d \u0442\u0430\u043a \u043d\u0435 \u043e\u0442 \u0431\u043e\u043b\u044c\u0448\u043e\u0433\u043e \u0443\u043c\u0430.\n", "topic": "rus", "url": "https://rus.stackexchange.com/questions/467372"}, {"image": "VQAonline_00057416.png", "question": "\u0440\u043e\u0434\u0438\u0442\u0435\u043b\u044c\u043d\u044b\u0439 \u043f\u0430\u0434\u0435\u0436 and \u0434\u0430\u0442\u0435\u043b\u044c\u043d\u044b\u0439 \u043f\u0430\u0434\u0435\u0436 how to choose \u043c\u043d\u0435 or \u043c\u043d\u044f?", "context": "I start to studied Russian one month ago. My teacher show me how to use \u0440\u043e\u0434\u0438\u0442\u0435\u043b\u044c\u043d\u044b\u0439 \u043f\u0430\u0434\u0435\u0436 and \u0434\u0430\u0442\u0435\u043b\u044c\u043d\u044b\u0439 \u043f\u0430\u0434\u0435\u0436 but i really don't understand the differences.\nFor example if I want to say:\nHe give me a rose - \u043e\u043d \u043f\u043e\u0434\u0430\u0440\u0438\u043b \u043c\u043d\u0435 \u0440\u043e\u0437\u0443 \nwhy I use \u043c\u043d\u0435 and not \u043c\u0415\u043d\u044f ? \n\u043a\u043e\u043c\u0443 or \u043a\u043e\u0433\u043e meaning always \"To Whom?\" so how can I understand the difference between \u0440\u043e\u0434\u0438\u0442\u0435\u043b\u044c\u043d\u044b\u0439 \u043f\u0430\u0434\u0435\u0436 and \u0434\u0430\u0442\u0435\u043b\u044c\u043d\u044b\u0439 \u043f\u0430\u0434\u0435\u0436 if the question is always same and also the translation of \u043c\u043d\u0435 / \u043c\u0415\u043d\u044f? \nThanks\n\n", "answer": "The Dative case (\u0434\u0430\u0442\u0435\u043b\u044c\u043d\u044b\u0439 \u043f\u0430\u0434\u0435\u0436) is used for the addressee in 'give':\n\n\u041e\u043d \u0434\u0430\u043b \u043c\u043d\u0435 \u043a\u043d\u0438\u0433\u0443. = \u041e\u043d \u0434\u0430\u043b \u043a\u043d\u0438\u0433\u0443 \u043c\u043d\u0435.\nHe gave me the book. = He gave the book to me.\nMi ha dato il libro. = Lui ha dato il libro a me.\n\nThe Accusative case (\u0432\u0438\u043d\u0438\u0442\u0435\u043b\u044c\u043d\u044b\u0439 \u043f\u0430\u0434\u0435\u0436) is used for the direct object of the action:\n\n\u041e\u043d \u0434\u0430\u043b \u043c\u043d\u0435 \u043a\u043d\u0438\u0433\u0443. (\u043a\u043d\u0438\u0433\u0443 is the Accusative of \u043a\u043d\u0438\u0433\u0430, book)\n\u041e\u043d \u0441\u043f\u0440\u043e\u0441\u0438\u043b \u043c\u0435\u043d\u044f. (\u043c\u0435\u043d\u044f is the Accusative of \u044f, I)\n\nThe Genitive case (\u0440\u043e\u0434\u0438\u0442\u0435\u043b\u044c\u043d\u044b\u0439 \u043f\u0430\u0434\u0435\u0436) often appears after prepositions like \u0431\u0435\u0437, \u0443, \u043e\u0442:\n\n\u0418\u0434\u0438 \u0431\u0435\u0437 \u043c\u0435\u043d\u044f. - Go without me.\n\u0423 \u043c\u0435\u043d\u044f \u0435\u0441\u0442\u044c \u043a\u043d\u0438\u0433\u0430. - I have a book.\n\nWhat complicates matters a bit, \u043c\u0435\u043d\u044f can be either Genitive or Accusative, depending on context. In \"\u041e\u043d \u0441\u043f\u0440\u043e\u0441\u0438\u043b \u043c\u0435\u043d\u044f\" it's Accusative, not Genitive. Your teacher is mistaken.\n", "topic": "russian", "url": "https://russian.stackexchange.com/questions/17486"}, {"image": "VQAonline_00057425.png", "question": "'\u043e\u0437\u043d\u0430\u043a\u043e\u043c\u043b\u044f\u0341\u0439\u0442\u0435\u0441\u044c' or '\u043e\u0437\u043d\u0430\u043a\u0430\u0341\u043c\u043b\u0438\u0432\u0430\u0439\u0442\u0435\u0441\u044c'?", "context": "Is '\u043e\u0437\u043d\u0430\u043a\u0430\u0341\u043c\u043b\u0438\u0432\u0430\u0439\u0442\u0435\u0441\u044c' correct? \nI can't find this word in Zaliznyak's dictionary or in Russian Wiktionary. Instead, I see only '\u043e\u0437\u043d\u0430\u043a\u043e\u043c\u043b\u044f\u0341\u0439\u0442\u0435\u0441\u044c'. However, I keep hearing it all the time:\n\n", "answer": "Despite the known official recommendation (probably, coming from the 1950-s - the argument being: \"\u043e\u0444\u043e\u0440\u043c\u043b\u044f\u0442\u044c\", not \"\u043e\u0444\u0430\u0440\u043c\u043b\u0438\u0432\u0430\u0442\u044c\"), the matter is not as simple as it might seem. It's still arguable if just one word form is correct: \n\u0414\u043e\u043a\u0443\u043c\u0435\u043d\u0442\u043d\u0430\u044f \u043b\u0438\u043d\u0433\u0432\u0438\u0441\u0442\u0438\u043a\u0430, \u041d. \u041c\u0443\u0440\u0430\u0432\u044c\u0435\u0432\u0430, 2016\nhttps://books.google.ru/books/content?id=oikJDgAAQBAJ&pg=PA189&img=1&zoom=3&hl=en&sig=ACfU3U0Cy60xESFBuDAGJJqRskWcYodctg&w=1280\n\n\u041c\u043e\u0440\u0444\u043e\u043b\u043e\u0433\u0438\u0447\u0435\u0441\u043a\u0438\u043c \u0432\u0430\u0440\u0438\u0430\u043d\u0442\u043e\u043c \u0433\u043b\u0430\u0433\u043e\u043b\u0430 \u043d\u0435\u0441\u043e\u0432\u0435\u0440\u0448\u0435\u043d\u043d\u043e\u0433\u043e \u0432\u0438\u0434\u0430 \u043e\u0437\u043d\u0430\u043a\u043e\u043c\u043b\u044f\u0442\u044c/\u0441\u044f \u044f\u0432\u043b\u044f\u0435\u0442\u0441\u044f \u0444\u043e\u0440\u043c\u0430 \u043e\u0437\u043d\u0430\u043a\u0430\u043c\u043b\u0438\u0432\u0430\u0442\u044c/\u0441\u044f \u0441 \u043a\u043e\u0440\u043d\u0435\u0432\u044b\u043c \u0447\u0435\u0440\u0435\u0434\u043e\u0432\u0430\u043d\u0438\u0435\u043c \u043e/\u0430. \u0415\u0449\u0451 \u043d\u0435\n \u0442\u0430\u043a \u0434\u0430\u0432\u043d\u043e \u0444\u043e\u0440\u043c\u0430 \u043e\u0437\u043d\u0430\u043a\u0430\u043c\u043b\u0438\u0432\u0430\u0442\u044c (\u0441\u043c., \u043d\u0430\u043f\u0440\u0438\u043c\u0435\u0440, \u0441\u043b\u043e\u0432\u0430\u0440\u044c \u0414.\u041d. \u0423\u0448\u0430\u043a\u043e\u0432\u0430 \u0438\u043b\u0438\n \u043c\u0430\u043b\u044b\u0439 \u0430\u043a\u0430\u0434\u0435\u043c\u0438\u0447\u0435\u0441\u043a\u0438\u0439 \u0441\u043b\u043e\u0432\u0430\u0440\u044c) \u0441\u0447\u0438\u0442\u0430\u043b\u0430\u0441\u044c \u043d\u0435\u043f\u0440\u0430\u0432\u0438\u043b\u044c\u043d\u043e\u0439. \u041c\u0435\u0436\u0434\u0443 \u0442\u0435\u043c \u0432 \u0441\u043e\u0432\u0440\u0435\u043c\u0435\u043d\u043d\u043e\u0439 \u0440\u0435\u0447\u0435\u0432\u043e\u0439 \u043f\u0440\u0430\u043a\u0442\u0438\u043a\u0435\n \u043e\u043d\u0430 \u0430\u043a\u0442\u0438\u0432\u043d\u043e \u0443\u043f\u043e\u0442\u0440\u0435\u0431\u043b\u044f\u0435\u0442\u0441\u044f.\n\n\u0410.\u042e.\u0418\u0432\u0430\u043d\u043e\u0432\u0430, \u0420\u0443\u0441\u0441\u043a\u0438\u0439 \u044f\u0437\u044b\u043a \u0432 \u0434\u0435\u043b\u043e\u0432\u043e\u0439 \u0434\u043e\u043a\u0443\u043c\u0435\u043d\u0442\u0430\u0446\u0438\u0438, \"\u042e\u0440\u0430\u0439\u0442\", \u043c. 2018\nhttps://books.google.ru/books/content?id=x5N2DwAAQBAJ&pg=PA115&img=1&zoom=3&hl=en&sig=ACfU3U3OS5f7UZSmgJMT_C0M7tZdxv5FwA&w=1280\nhttps://books.google.ru/books/content?id=x5N2DwAAQBAJ&hl=ru&pg=PA116&img=1&zoom=3&sig=ACfU3U39SpnMGk-fPLGGvl-TrXBaQtAK9Q&w=1280\n\n\u0412 \u0440\u0443\u0441\u0441\u043a\u043e\u043c \u044f\u0437\u044b\u043a\u0435 \u0441\u0443\u0449\u0435\u0441\u0442\u0432\u0443\u044e\u0442 \u043f\u0430\u0440\u0430\u043b\u043b\u0435\u043b\u044c\u043d\u044b\u0435 \u0433\u043b\u0430\u0433\u043e\u043b\u044c\u043d\u044b\u0435 \u0441\u0442\u0440\u0443\u043a\u0442\u0443\u0440\u044b, \u043a\u043e\u0442\u043e\u0440\u044b\u0435 \u0440\u0430\u0437\u043b\u0438\u0447\u0430\u044e\u0442\u0441\u044f \u0441\u0442\u0438\u043b\u0438\u0441\u0442\u0438\u0447\u0435\u0441\u043a\u043e\u0439 \u043e\u043a\u0440\u0430\u0441\u043a\u043e\u0439. \u041d\u0430\u043f\u0440\u0438\u043c\u0435\u0440, \u0433\u043b\u0430\u0433\u043e\u043b\u0430\u043c \u0441\u043e\u0432\u0435\u0440\u0448\u0435\u043d\u043d\u043e\u0433\u043e\n \u0432\u0438\u0434\u0430, \u0442\u0438\u043f\u0430 \u0437\u0430\u0433\u043e\u0442\u043e\u0432\u0438\u0442\u044c, \u043e\u0437\u043d\u0430\u043a\u043e\u043c\u0438\u0442\u044c, \u0443\u0437\u0430\u043a\u043e\u043d\u0438\u0442\u044c, \u043c\u043e\u0433\u0443\u0442 \u0441\u043e\u043e\u0442\u0432\u0435\u0442\u0441\u0442\u0432\u043e\u0432\u0430\u0442\u044c\n \u0433\u043b\u0430\u0433\u043e\u043b\u044b \u043d\u0435\u0441\u043e\u0432\u0435\u0440\u0448\u0435\u043d\u043d\u043e\u0433\u043e \u0432\u0438\u0434\u0430: \u0437\u0430\u0433\u043e\u0442\u043e\u0432\u043b\u044f\u0442\u044c \u2014 \u0437\u0430\u0433\u043e\u0442\u0430\u0432\u043b\u0438\u0432\u0430\u0442\u044c; \u043e\u0437\u043d\u0430\u043a\u043e\u043c\u043b\u044f\u0442\u044c\n \u2014 \u043e\u0437\u043d\u0430\u043a\u0430\u043c\u043b\u0438\u0432\u0430\u0442\u044c; \u0443\u0437\u0430\u043a\u043e\u043d\u044f\u0442\u044c \u2014 \u0443\u0437\u0430\u043a\u043e\u043d\u0438\u0432\u0430\u0442\u044c. \u0412 \u044d\u0442\u0438\u0445 \u0433\u043b\u0430\u0433\u043e\u043b\u0430\u0445 \u0444\u043e\u0440\u043c\u044b \u0441\n \u0441\u0443\u0444\u0444\u0438\u043a\u0441\u0430\u043c\u0438 -\u0438\u0432\u0430, -\u044b\u0432\u0430 \u0443\u043f\u043e\u0442\u0440\u0435\u0431\u0438\u0442\u0435\u043b\u044c\u043d\u044b \u0432 \u043e\u0444\u0438\u0446\u0438\u0430\u043b\u044c\u043d\u043e\u0439 \u0440\u0435\u0447\u0438, \u0430 \u0444\u043e\u0440\u043c\u044b \u0431\u0435\u0437\n \u044d\u0442\u043e\u0433\u043e \u0441\u0443\u0444\u0444\u0438\u043a\u0441\u0430 \u0432\u043e\u0441\u043f\u0440\u0438\u043d\u0438\u043c\u0430\u044e\u0442\u0441\u044f \u043a\u0430\u043a \u0440\u0430\u0437\u0433\u043e\u0432\u043e\u0440\u043d\u044b\u0435.\n\u0412 \u0434\u0440\u0443\u0433\u0438\u0445 \u043f\u0430\u0440\u0430\u0445 \u0433\u043b\u0430\u0433\u043e\u043b\u043e\u0432 \u0442\u0438\u043f\u0430: \u043e\u0431\u0443\u0441\u043b\u043e\u0432\u043b\u0438\u0432\u0430\u0442\u044c - \u043e\u0431\u0443\u0441\u043b\u0430\u0432\u043b\u0438\u0432\u0430\u0442\u044c, \u0441\u043e\u0441\u0440\u0435\u0434\u043e\u0442\u043e\u0447\u0438\u0432\u0430\u0442\u044c - \u0441\u043e\u0441\u0440\u0435\u0434\u043e\u0442\u0430\u0447\u0438\u0432\u0430\u0442\u044c \u0444\u043e\u0440\u043c\u044b \u0441 \"\u043e\" \u0443\u043f\u043e\u0442\u0440\u0435\u0431\u0438\u043c\u044b \u0432\n \u043a\u043d\u0438\u0436\u043d\u043e-\u043f\u0438\u0441\u044c\u043c\u0435\u043d\u043d\u044b\u0445 \u0441\u0442\u0438\u043b\u044f\u0445, \u0430 \u0444\u043e\u0440\u043c\u044b \u0441 \"\u0430\" \u0441\u0432\u043e\u0439\u0441\u0442\u0432\u0435\u043d\u043d\u044b \u0440\u0430\u0437\u0433\u043e\u0432\u043e\u0440\u043d\u043e\u0439 \u0440\u0435\u0447\u0438.\n\nFrom an older classic dictionary: \u041c\u044e\u043b\u043b\u0435\u0440 \u0412\u043b\u0430\u0434\u0438\u043c\u0438\u0440 \u041a\u0430\u0440\u043b\u043e\u0432\u0438\u0447 (1880-1941) \u041f\u043e\u043b\u043d\u044b\u0439 \u0420\u0443\u0441\u0441\u043a\u043e-\u0430\u043d\u043b\u0438\u0439\u0441\u043a\u0438\u0439 \u0441\u043b\u043e\u0432\u0430\u0440\u044c - only the form \u043e\u0437\u043d\u0430\u043a\u0430\u043c\u043b\u0438\u0432\u0430\u0442\u044c is present.\nhttps://books.google.ru/books/content?id=cUCFAQAAQBAJ&pg=PA1141&img=1&zoom=3&hl=en&sig=ACfU3U19Ib4MXSVtDt5gsRkz4V3zaOz-FQ&w=1280\n", "topic": "russian", "url": "https://russian.stackexchange.com/questions/20960"}, {"image": "VQAonline_00057417.png", "question": "How would you translate, \"where life is worth living\" into Russian?", "context": "My hometown has a saying or motto on their sign which reads, \"Where life is worth living\" and I'm trying to complete a Russian exercise for a language course in which I give a tour of my hometown in Russian. \nMy best idea of how to say this is, \"\u0422\u0443\u0442 \u0421\u0442\u043e\u0438\u0442 \u0416\u0438\u0442\u044c\"\nHowever, I was wondering if anyone more skilled in the language knows the most natural way to say this? \nTo use this phrase in a sentence, you would say, \"This town is a place where life is worth living\". \nIt's meant to sound positive and catchy. In reality, it is sometimes used ironically when things go poorly in the town, but in general, it is a positive thing to say. \n", "answer": "Imperfective \u0436\u0438\u0442\u044c does not really work as a transitive verb in Russian, so I don't really think you can make the literal translation work the same way the original English sentence does.\nYou can try replacing it with another pun or a word play or something:\n\n\n\u0417\u0434\u0435\u0441\u044c \u043d\u0435 \u043f\u0440\u043e\u0436\u0438\u0432\u0430\u044e\u0442, \u0437\u0434\u0435\u0441\u044c \u0436\u0438\u0432\u0443\u0442\n\n\u0418 \u0436\u0438\u0437\u043d\u044c \u0445\u043e\u0440\u043e\u0448\u0430, \u0438 \u0436\u0438\u0442\u044c \u0445\u043e\u0440\u043e\u0448\u043e\n\n\u0422\u0443\u0442 \u0436\u0438\u0437\u043d\u044c, \u0432 \u043a\u043e\u0442\u043e\u0440\u043e\u0439 \u0441\u0442\u043e\u0438\u0442 \u0436\u0438\u0442\u044c\n\n\n\n, or similar\n", "topic": "russian", "url": "https://russian.stackexchange.com/questions/17679"}, {"image": "VQAonline_00057604.png", "question": "Shallow water equations (SWE): well-posed initial data for single travelling pulse", "context": "This question concerns the 1-dimensional (i.e. only one spatial dimension) shallow water equations (SWE) shown below and how to find initial conditions such that we obtain a travelling pulse/wave instead of the typical water droplet scenario that yields two travelling waves in opposite directions (this corresponds to setting the momentum $m$ to zero everywhere as initial condition). Also, I use the Roe scheme to solve this problem numerically.\nThe SWEs are\n$$\n\\begin{align}\n\\begin{pmatrix} h \\\\ m \\end{pmatrix}_{t} + \\begin{pmatrix} m \\\\\t\\frac{m^{2}}{h} + \\frac{1}{2} gh^{2} \\end{pmatrix}_{x} = \\begin{pmatrix} 0 \\\\ -ghB_{x} \\end{pmatrix},\n\\end{align}\n$$\nwhere $B(x)$ denotes the bottom's elevation (over the $x$-axis), or the bathymetry if you will. I will set $B=0$, so we do not have to worry about reflections. Let's say I set the initial data for the state variable $h$ to $h(x,0)=\\phi(x)$, where $\\phi$ denotes some Gaussian. How do I construct a intitial condition for $m$ such that I obtain a single travelling pulse in my simulation? See figure below of the results I obtain for zero momentum initial data.\n \nWhat I would like is just one \"bump\". I have tried setting the initial momentum as large as possible for the higher water staples and also positive, so that (in my mind) we will get a right-going wave. This led to spurious solutions unfortunately. I read somewhere that you have to consider eigenvectors to obtain such initial data, but I don't understand what they mean. Eigenvectors of the Roe-average-matrix?\nReply to Ketcheson's answer:\nYes, this is what I found in LeVeque's book (right after I posted my question). I am looking at those integral curves in Figure 13.12. Is it the case that $u_{0}(0)$ and $h_{0}(0)$ are always equal to zero, or can be set to any point in the state space, e.g. zero? What I am looking for specifically is a closed expression for $u_{0}(x)$ (or $m_{0}(x)$) given $h_{0}(x)$, so if I understand this correctly I would obtain\n$$\n\\begin{align}\n&\\text{Given:}& h_{0}(x) &= H + a\\exp\\left(-\\frac{(x-L/2)^{2}}{w^{2}}\\right), \\\\[3mm]\n&\\text{simple wave going rightwards}\\implies& u_{0}(x) &= 2\\sqrt{gh_{0}(x)}.\n\\end{align}\n$$\nHere I have just set $h_{0}(0)=0$ and $u_{0}(0)=0$, but this seems unsatisfactory since $h_{0}(0)=H\\equiv1$ with my initial data. Now, according to the integral curves in Fig 13.12, I may choose $h_{*}=h_{0}(0)=1$ and $m_{*}=m_{0}(0)=0$. I then obtain as a closed expression for $u_{0}$ (or $m_{0}(0)$ rather)\n$$\n\\begin{align}\nm_{0}(x) = 2h_{0}(x)\\left(\\sqrt{g}-\\sqrt{gh_{0}(x)}\\right).\n\\end{align}\n$$\nThis is according to equation (13.31) in LeVeque's book. Is this the correct formulation? Thank you for your time.\n", "answer": "What you are looking for is known as a simple wave solution, in which the variation in the solution belongs entirely to one characteristic family. If you had a linear hyperbolic system, this would just mean that your initial data should be proportional to a particular eigenvector of the flux Jacobian. Since you have a nonlinear system, it's a bit more complicated. You want your initial data to all lie along a singe integral curve of the shallow water system.\nWhat this means specifically is that your initial data should satisfy\n$$\nu_0(x) \\pm 2\\sqrt{gh_0(x)} = u_0(0) \\pm 2\\sqrt{gh_0(0)}.\n$$\nHere $h_0(x), u_0(x)$ are the initial depth and velocity. If you take \"+\" the solution will go left, and if you take \"-\" it will go right.\nIf you want to understand this better, I recommend Chapter 13 of LeVeque's finite volumes book and the shallow water equations chapter of this book on Riemann problems written by LeVeque, del Razo, and myself.\n", "topic": "scicomp", "url": "https://scicomp.stackexchange.com/questions/35062"}, {"image": "VQAonline_00057529.png", "question": "How to define a non-square Legendre pseudospectral differentiation matrix?", "context": "I am going to discuss my reasons for wanting this first, as this may in fact not be what I am looking for.\nMy reason for asking this that I have finished writing a piece of code that solves, $-\\nabla \\cdot(a \\nabla u) = f$ where $a$ is discontinuous, this is the result of the code\n\nwe can see oscillations about the line of discontinuity, so I have been told that I have to enrich the approximation. After some manipulation, the meat of the problem is dealing with this integral:\n$$\\int_{\\Omega}a\\nabla u \\cdot \\nabla v dx dy$$\nOn enriching my approximation.\n$$u(x,y) = \\sum_{i = 0}^N\\sum_{j = 0}^Nu_{ij}h_i(x)h_j(y) + \\sum_{k = 1}^N\\sum_{l = 1}^N\\alpha_{kl}\\tilde{h_k(x)}\\tilde{h_l(y)}\\Phi(x,y)$$\nOn substituting this into the integral\n$$\\int_{\\Omega}a\\nabla u \\cdot \\nabla v dx dy $$\n$$\\approx w_s\\sum_{n = 0}^N \\sum_{i = 0}^N w_n h_i'(x_n)h_r'(x_n)u_{is}, \\forall s\\in[0,N]$$\n$$+ w_r\\sum_{m = 0}^N \\sum_{j = 0}^N w_m h_j'(y_m)h_s'(y_m)u_{rj}, \\forall r\\in[0,N]$$\n$$+ \\color{red}{ w_s\\sum_{n = 1}^{2N + 2} a_1(x_n,y_s)\\sum_{k = 1}^{N-1} w_n \\tilde{h_k}'(x_n)h_r'(x_n)\\alpha_{ks}\\Phi_{1,x}(x_n,y_s), \\forall s\\in[1,N]}$$\n$$+ w_r\\sum_{m = 0}^{2N + 2} a_1(x_r,y_m) \\sum_{l = 0}^{N-1} w_m \\tilde{h_l}'(y_m)h_s'(y_m)\\alpha_{rl}\\Phi_{1,y}(x_r,y_m), \\forall r\\in[1,N]$$\n$$+ \\color{red}{w_s\\sum_{n = 1}^{2N + 2} a_2(x_n,y_s)\\sum_{k = 1}^{N-1} w_n \\tilde{h_k}'(x_n)h_r'(x_n)\\alpha_{ks}\\Phi_{2,x}(x_n,y_s), \\forall s\\in[1,N]}$$\n$$+ w_r\\sum_{m = 0}^{2N + 2} a_2(x_r,y_m) \\sum_{l = 0}^{N-1} w_m \\tilde{h_l}'(y_m)h_s'(y_m)\\alpha_{rl}\\Phi_{2,y}(x_r,y_m), \\forall r\\in[1,N] \\textbf{(1)}$$\nHow do I form the Chebyshev differentiation matrix in MATLAB?\nThe first answer in this link deals with the definition of the $h_i'(x_j)$ functions, or the construction of the square Legendre pseudospectral differentiation matrix. The $\\tilde{h_i}'(x_j)$ have a slightly different definition. \n$$h_i'(x) = \\frac{(1-x^2)(L_N'(x))}{N(N+1)L_N'(x_i)(x - x_i)}$$\n$$\\tilde{h_i}'(x) = \\frac{(1-x_i^2)(L_N'(x))}{N(N+1)L_N'(x_i)(x - x_i)}$$\nThe construction of the corresponding square Legendre pseudospectral differentiation matrix for each of the above is given in code as\nN1=N+1;\ncheb=cos(pi*(0:N)/N)';\nunif=linspace(-1,1,N1)';\n\nif N<3\n x=cheb;\nelse\n x=cheb+sin(pi*unif)./(4*N);\nend\n\nP=zeros(N1,N1);\n%N1xN1 zero matrix\nxold=2;\n%eps= epsilon!\nwhile max(abs(x-xold))>eps\n\n xold=x; \n P(:,1)=1;\n P(:,2)=x;\n %set first collumn entries to 1 (P(:,1) = 1);\n %set second collumn entries to x (P(:,2) = x);\n % and so on, the collumns of this matrix are the legendre polynomials\n for k=2:N\n P(:,k+1)=( (2*k-1)*x.*P(:,k)-(k-1)*P(:,k-1) )/k;\n %%Bonnets formula ;)\n end\n % Roots of (1-x^2)L'_N\n x=xold-( x.*P(:,N1)-P(:,N) )./( N1*P(:,N1));\nend\n\n%---Legendre pseduspectral differentiation matrix------------page 124-----\n% of the book \n\nx = -x;\nD=zeros(N+1,N+1);\nfor k=1:N+1;\n for j=1:N+1;\n D(j,k)=P(j,N+1)/(P(k,N+1)*(x(j)-x(k)));\n end\n D(k,k)=0; \nend\nD(1,1)=-N*(N+1)/4 ; \nD(N+1,N+1)=N*(N+1)/4; \n\n%------------------------------------------------------------------------\nC1 = -(1-x(1:N1).^2);\nC2 = N*N1.*P(:,N1);\nC = C1+C2;\n% L1 = P' L2 = P''...\n% LN(i,N) is the Nth derivative of the Nth legendre polynomial at x(i)\n\n\n% The last collumn in each is only really relevant.\n%The coll's before will give a wierd result, this is easily fixed \n%but we dont use them so....\n\n\nL1 = zeros(N1,N1);\nL2 = zeros(N1,N1);\nL3 = zeros(N1,N1);\nL1(:,1) = 0;\nL1(:,2) = 1;\n\nfor m = 2:N;\n for k = 1:N1;\n L1(k,m+1) = ((2*m + 1)*(P(k,m) + x(k)*L1(k,m)) - m*L1(k,m-1))/(m+1); \n end\nend\nL1(1,N1) = N*N1/2;\nL1(N1,N1) = (N*N1/2)*(-1)^(N-1);\n\nL2(:,1) = 0;\nL2(:,2) = 0; \nfor m = 2:N;\n for k = 1:N1;\n L2(k,m+1) = ((2*m + 1).*(2*L1(k,m) + x(k).*L2(k,m)) - m.*L2(k,m-1))/(m+1); \n end\nend\nL2(1,N1) = (N*N1 - 2)*(N*N1)^2/8;\nL2(N1,N1) = (N*N1 - 2)*((N*N1)^2/8)*(-1)^N;\n\nL3(:,1) = 0;\nL3(:,2) = 0;\nfor m = 2:N;\n for k = 1:N1;\n L3(k,m+1) = ((2*m + 1).*(3*L2(k,m) + x(k).*L3(k,m)) - m.*L3(k,m-1))/(m+1); \n end\nend\n\nL3(1,N1) = (N*N1 - 6)*L2(1,N1)/6;\nL3(N1,N1) = ((N*N1 - 6)*L2(1,N1)/6)*(-1)^N;\n%this is the matrix that corresponds to the derivatives of h tilde.\nDT = zeros(N1,N1);\nfor l = 2:N;\n for k = 2:N;\n if(k == l)\n DT(l,k) = C(k)*(L3(k,N1)/2);\n else \n DT(l,k) = C(k)*(L2(l,N1)/(x(l) - x(k)));\n end\n end\nend\nfor k = 2:N;\n DT(1,k) = C(1)*((1 - x(k))*(L2(1,N1) - L1(1,N1))/(1-x(k))^2); \n DT(N1,k) = C(N1)*((1 - x(k))*(L2(N1,N1) - L1(N1,N1))/(1-x(k))^2); \nend\n\nThe reason for the 4 extra terms is that I have split the region into two, either side of the discontinuity. According to my professor I have to use a technique called over-integration. I am going to need to define another D matrix such that is it a $(2N+2) \\times (N+1)$ matrix and similarly for DT(ilde).\nSo my question is\n\nHow do I form the non-square Legendre pseudospectral differentiation matrix?, if such a thing exists. If not, how do I deal with the over integration parts of the expression?\n\n", "answer": "For the purpose of solving a differential equation, the differentiation matrix must be square (and invertible). You have added additional degrees of freedom -- thus increasing the number of columns -- so you also need to add the same number of (independent) conditions by increasing the number of rows. \nIf you have selected your enriching functions properly, this can be done by using these functions as additional test functions, i.e., inserting\n$$v_{N+k,N+l}(x,y) = \\tilde h_k(x)\\tilde h_l(y)\\Phi(x,y)$$\nand integrating.\nSince LGL-pseudospectral methods use nodal basis functions based on quadrature points, this can be seen as adding more quadrature points for each element; in particular, since the enriching functions do not raise the maximal degree, more than you need for exact integration of each basis function. Hence, this is sometimes referred to as over-integration; see, e.g., this reference.\n", "topic": "scicomp", "url": "https://scicomp.stackexchange.com/questions/24940"}, {"image": "VQAonline_00057574.png", "question": "Analytical Solution of Transport Equation", "context": "I'm looking at the analytical solution of the convection-diffusion equation\n$$\\frac{\\partial C}{\\partial t} = D\\frac{\\partial ^2 C}{\\partial x^2}-v\\frac{\\partial C}{\\partial x}$$\nwith initial condition\n$$c(x,0) = 4000$$\nand with Dirichlet boundary condition\n$$C(x=0,t>0) = 4100$$\nNeumann boundary condition\n$$\\frac{\\partial C}{\\partial x}=0\\text{ at } x=L ; t>0.$$\n\nHowever, when the above analytical solution is coded I obtain negative values of C which is unrealistic.\nfunction AnalyticalSoln2()\nformat long\nR = 1;\nD = 900;\nv = 200;\nL = 60; \nco = 4100;\nci = 4000;\nX = linspace(0,60,10);\nt = 0:0.001:2;\nsol=[];\nfor pos = 1:length(X)\n x = X(pos);\nA1 = 0.5*erfc((x.*R-t.*v)./(2*(t.*D*R).^0.5));\nA2 = 0.5*exp(x.*v/D).*erfc((x.*R+t.*v)./(2*(D*t.*R).^0.5));\nA31 = 0.5*(2+v*(2*L-x)/D + (t.*v^2)/(D*R))*exp(v*L/D);\nA32 = erfc((R*(2*L-x)+t.*v)./(2*(D*t.*R).^0.5));\nA41 = -((t.*v^2)./(pi*D*R)).^0.5;\nA42 = exp((v*L/D)-(R./(4*t.*D)).*(2*L -x + t.*v/R).^2);\nA = A1 + A2 + A31.*A32 + A41.*A42;\nif t==0 \nC = ci + (co - ci)*A'\nelse\nC = ci + (co - ci)*A' - co*A';\nend\nsol = horzcat(sol,C);\nend\nsol(1:100:end,:)\nend\n\nWhereas, the numerical solution obtained from the pdepe solver is non-negative.\nHere's the solution obtained using pdepe\nfunction DiffusionConvectionMATLAB\nformat short\nglobal D\nm = 0;\nx = linspace(0,60,10);\nt = 0.1:0.1:2;\nD = 900;\nsol = pdepe(m,@pdefun,@icfun,@bcfun,x,t)\n function [g,f,s] = pdefun(x,t,c,DcDx)\n v = 200;\n g = 1;\n f = D*DcDx;\n s = -v*DcDx;\n end\n\n function c0 = icfun(x)\n c0 = 4000;\n end\n\n function [pl,ql,pr,qr] = bcfun(xl,cl,xr,cr,t)\n pl = cl-4100;\n ql = 0;\n pr = 0;\n qr = 1;\n end\nend\n\nCould someone suggest if there is any issue with the way I'm computing the solution form the analytical expression?\n", "answer": "Here's one way to implement the analytic solution. However, I don't seem to be getting the correct plots if I use t0 = 0 (as you mentioned in the comment). It works if you set t0 to something greater than the final time you have (like t0 = 10 for example).\nI also used function handles so that A(x,t) can be evaluated at any x and t value.\nformat long\nR = 1;\nD = 900;\nv = 200;\nL = 60; \nco = 4100;\nci = 4000;\nx = linspace(0,60,100);\ntvec = 0.0:0.01:2;\nsol=[];\n\n%Use function handles so that A(x,t) can be evaluated for any x and t\nA1 = @(x,t) 0.5*erfc((x.*R-t.*v)./(2*(t.*D*R).^0.5));\nA2 = @(x,t) 0.5*exp(x.*v/D).*erfc((x.*R+t.*v)./(2*(D*t.*R).^0.5));\nA31 = @(x,t) 0.5*(2+v*(2*L-x)/D + (t.*v^2)/(D*R))*exp(v*L/D);\nA32 = @(x,t) erfc((R*(2*L-x)+t.*v)./(2*(D*t.*R).^0.5));\nA41 = @(x,t) -((t.*v^2)./(pi*D*R)).^0.5;\nA42 = @(x,t) exp((v*L/D)-(R./(4*t.*D)).*(2*L -x + t.*v/R).^2);\nA = @(x,t) A1(x,t) + A2(x,t) + A31(x,t).*A32(x,t) + A41(x,t).*A42(x,t);\n\nt0 = 0; %this doesn't seem to give the right solution\n%t0 = 10; %this seems to work\n\nfor t = tvec %loop over time\n if (t <= t0)\n C = ci + (co - ci)*A(x,t);\n else\n C = ci + (co - ci)*A(x,t) - co*A(x,t-t0);\n end\n\n sol = [sol; C];\nend\n\n", "topic": "scicomp", "url": "https://scicomp.stackexchange.com/questions/30818"}, {"image": "VQAonline_00058044.png", "question": "Why didn't Qui-Gon Jinn use his Jedi mind tricks to exchange his Republic credits?", "context": "In Star Wars Episode I, Qui-Gon Jinn tries to use his Jedi mind tricks to convince Watto that Republic credits will do fine as payment for the hyperdrive they need. Conveniently, mind tricks don't work on Watto because he is a Toydarian, so Qui-Gon has to hatch an elaborate plan with Anakin and his secret pod-racer to win the Boonta Eve Classic and gamble with Anakin's life to win the hyperdrive and Anakin's freedom. \nWhy didn't he just go to a money changer who wasn't Toydarian and use his mind tricks to exchange his credits for something that Watto would accept? Then he could have just straight up traded the pod-racer for Anakin, or bought him... and Shmi too for that matter, since Watto ends up selling her anyways.\n\n", "answer": "Although it may sound like a cop-out, the answer is that it was because the Force didn't tell him to. Qui-Gon Jinn was drawn to the only junkyard in town that had a compatible hyperdrive. When faced with an apparent obstacle, he simply waits for the Force to present him with a solution. Presumably when you're a powerful Jedi these kinds of coincidences happen all the time.\nThe novelisation is a little more clear on his motives.\n\n\u201cAll right,\u201d Qui-Gon responded with a frown. \u201cAnother solution will\n present itself. I\u2019ll check back.\u201d He tucked the comlink beneath his\n poncho and signaled to the others. He was moving toward the street\n again when Jar Jar grabbed his arm. \u201cNoah gain, sire,\u201d the Gungan\n pleaded. \u201cDa beings hereabouts crazy nuts. We goen be robbed and\n crunched!\u201d \u201cNot likely,\u201d Qui-Gon replied with a sigh, freeing himself.\n \u201cWe have nothing of value. That\u2019s our problem.\u201d They started back down the street, Qui-Gon trying to think what to do next.\n\nA few seconds later, after having learned that pod-racing is a big deal in these parts, they encounter the same small boy from the junkyard. He offers them both assistance and insight into a way in which they can parlay their one solid asset (the ship) into enough money to buy a hyperdrive. \nThis has the added benefit of saving them from having to wander around town announcing their presence to all and sundry, remembering of course that their enemies may be on the lookout for a Naboobian transport ship or they may encounter people who're just flat-out unfriendly to the Republic:\n\n\u201cThe Republic doesn\u2019t exist out here,\u201d Shmi interrupted quickly, her\n voice hard. \u201cWe must survive on our own. ... And they need my help.\n They\u2019re in trouble. The prize money would more than pay for the parts\n they need\" ... \u201cDo you know of anyone friendly to the Republic who\n might be able to help us?\" Shmi stood silent and unmoving as she\n thought the matter through. She shook her head no.\n\n", "topic": "scifi", "url": "https://scifi.stackexchange.com/questions/72431"}, {"image": "VQAonline_00057793.png", "question": "How was the Golden Condor able to fly?", "context": "In the 80's TV cartoon The Mysterious Cities of Gold there was a Golden Condor:\n\nI remember it was somehow solar-powered, but I don't remember how\nit flew. For instance, it did not flap its wings to take off.\nBy what principle was it able to fly?\n", "answer": "I found this Trailer (sorry, in French only) of the upcoming second season of The Mysterious Cities of Gold. This sequel is pretty official as one of the original writer, Jean Chalopin, is involved.\nIf you look at 3m30s you will see it taking off :\n\n\n\nAs you see, there is air coming from behind it, then the tail lights up as it move forward.\nSo, all we could conclude it the way the Golden Condor flew is intentionally kept mysterious, making it a even more incredible machine.\n", "topic": "scifi", "url": "https://scifi.stackexchange.com/questions/18214"}, {"image": "VQAonline_00058523.png", "question": "Was it reckless to allow Wormtongue to go free?", "context": "When Grima Wormtongue is evicted from Theoden's court in The Two Towers he is given free leave to go wherever he wants. Gandalf, Theoden and the whole company are in no doubt that he is a plant of Saruman's who, once exposed, will presumably be quite keen to return to his true master's side (as indeed he does).\nIn the movie Theoden goes to strike Wormtongue down only to have his hand stayed by Aragorn, who says, \"No, my lord. Let him go. Enough blood has been spilt on his account.\" Wormtongue then goes back to Saruman and tells him about the hidden weakness in Helm's Deep, which in turn leads to half the wall being blown up. Countless men and elves die and Isengard gets to within a whisker of victory.\n\nGreat decision, Aragorn.\nThis seems like a glaring plot hole in the film. But it is hardly less so in the books. True, the weakness in the wall may have been found without Wormtongue anyway. The explosion in the books is attributed by Aragorn only to \"devilry of Saruman\" and isn't tied to Wormtongue himself. Yet, even when you put Helm's Deep to one side, Wormtongue was a known rogue agent. Surely he was capable of causing greater harm to Rohan - and of being a greater aid to Saruman - when let loose? Indeed, he went on to cause a great deal of mischief in the Shire, proving himself to be a dangerous and slippery individual.\nIn the books they discuss a number of different options for Wormtongue. He asks that he be left in charge of Edoras. Understandably, that request is rejected out of hand. Theoden suggests sending him into battle:\n\n\"You have my pity,\" said Theoden. \"And I do not send you from my side. I go myself to war with my men. I bid you come with me and prove your faith.\"\n(The Two Towers, Chapter VI, The King of the Golden Hall).\n\nI can't understand the rationale for this suggestion. Surely Wormtongue already 'proved his faith' (or lack of it) when he tricked and enslaved Theoden's mind for Saruman. They already know he's untrustworthy. If they send him to war, that he would betray them seems inevitable. Gandalf seems to recognise this.\n\n\"See, Theoden, here is a snake! With safety you cannot take it with you, nor can you leave it behind. To slay it would be just. But it was not always as it now is. Once it was a man, and did you service in its fashion. Give him a horse and let him go at once, wherever he chooses. By his choice you shall judge him.\"\n \"Do you hear this, Wormtongue?\" said Theoden. \"This is your choice: to ride with me to war, and let us see in battle whether you are true; or to go now, whither you will. But then, if ever we meet again, I shall not be merciful.\"\n(The Two Towers, Chapter VI, The King of the Golden Hall).\n\nSo the danger of leaving him behind or sending him to war is obvious. Again, there's surely no need to see whether he is \"true\" or not since he's already been revealed as a traitor. Why didn't they consider the obvious course - taking him to Helm's Deep with them as a prisoner and keeping him locked away in some dark corner until the battle was over?\nWhy let him go free where he could cause further damage?\nIdeally, I'd like an answer that references both the book and the film.\n", "answer": "There is always some risk in being merciful\nThe question quotes the discussion between Th\u00e9oden and the others about what to do with Wormtongue. It is clear from that discussion that they are aware that Wormtongue may cause problems if he is allowed to go free. Th\u00e9oden lets him go as an act of mercy in recognition of his earlier service, giving him one last chance to reform.\nImprisoning Wormtongue\nThe last part of the question asks why Wormtongue wasn't simply taken to Helm's Deep as a prisoner. This does at least seem a logical choice, and is not really addressed in the discussion. I suspect that Gandalf would point to the power of Wormtongue's tongue (he may have learned a little from Saruman). We know that his words were enough to poison Th\u00e9oden's mind so that he was enfeebled. In The Houses of Healing, Gandalf tells \u00c9omer that Wormtongue was able to infect \u00c9owen (so that she was ready to throw away her life).\n\n\u2018Think you that Wormtongue had poison only for Th\u00e9oden\u2019s ears? Dotard! What is the house of Eorl but a thatched barn where brigands drink in the reek, and their brats roll on the floor among their dogs? Have you not heard those words before? Saruman spoke them, the teacher of Wormtongue. Though I do not doubt that Wormtongue at home wrapped their meaning in terms more cunning. My lord, if your sister\u2019s love for you, and her will still bent to her duty, had not restrained her lips, you might have heard even such things as these escape them. But who knows what she spoke to the darkness, alone, in the bitter watches of the night, when all her life seemed shrinking, and the walls of her bower closing in about her, a hutch to trammel some wild thing in?\u2019\nThe Lord of the Rings Book Five, Chapter 8: The Houses of Healing\nPage 867 (Houghton Mifflin Harcourt; Single Volume 50th Anniversary Edition)\n\nThis would be part of what Gandalf has in mind when he says of Wormtongue \"With safety you cannot take it with you, nor can you leave it behind.\" So imprisoning Wormtongue would also be dangerous.\nLet's look at the consequences of the decision to free him:\nThe culvert at Helm's Deep\nWormtongue's role in blowing up the culvert in the movie is just an unnecessary detail made up by Jackson (it isn't mentioned in the books). Even if this information came from Wormtongue, there is no reason to assume he told Saruman about it after being released (as mentioned in Edlothiad's answer). I don't consider this a consequence of the decision to free Wormtongue\nThe Palant\u00edr\nDuring the parley at Orthanc, it is Wormtongue who throws the Palant\u00edr at Saruman or Gandalf.\n\nAt that moment a heavy shining thing came hurtling down from above. It glanced off the iron rail, even as Saruman left it, and passing close to Gandalf\u2019s head, it smote the stair on which he stood. The rail rang and snapped. The stair cracked and splintered in glittering sparks. But the ball was unharmed: it rolled on down the steps, a globe of crystal, dark, but glowing with a heart of fire. As it bounded away towards a pool Pippin ran after it and picked it up.\n\u2018The murderous rogue!\u2019 cried \u00c9omer. But Gandalf was unmoved. \u2018No, that was not thrown by Saruman,\u2019 he said; \u2018nor even at his bidding, I think. It came from a window far above. A parting shot from Master Wormtongue, I fancy, but ill aimed.\u2019\n\u2018The aim was poor, maybe, because he could not make up his mind which he hated more, you or Saruman,\u2019 said Aragorn.\nThe Lord of the Rings Book Three, Chapter 10: The Voice of Saruman\nPage 583-4 (Houghton Mifflin Harcourt; Single Volume 50th Anniversary Edition)\n\nUsing the Palant\u00edr, Aragorn is able to learn the state of Sauron's preparation for war and to distract Sauron. This leads him to try the desperate journey through the Paths of the Dead. When he tells Gimli and Legolas about revealing himself to Sauron, we read:\n\n\u2018The hasty stroke goes oft astray,\u2019 said Aragorn. \u2018We must press our Enemy, and no longer wait upon him for the move. See my friends, when I had mastered the Stone, I learned many things. A grave peril I saw coming unlooked-for upon Gondor from the South that will draw off great strength from the defence of Minas Tirith. If it is not countered swiftly, I deem that the City will be lost ere ten days be gone.\u2019\n\u2018Then lost it must be,\u2019 said Gimli. \u2018For what help is there to send thither, and how could it come there in time?\u2019\n\u2018I have no help to send, therefore I must go myself,\u2019 said Aragorn. \u2018But there is only one way through the mountains that will bring me to the coastlands before all is lost. That is the Paths of the Dead.\u2019\nThe Lord of the Rings Book Five, Chapter 2: The Passing of the Grey Company\nPage 780-1 (Houghton Mifflin Harcourt; Single Volume 50th Anniversary Edition)\n\nArguably the result here is that Minas Tirith is saved. I consider this a good consequence of the decision to free Wormtongue.\nDestruction in the Shire\nWormtongue certainly played a part in the destruction in the Shire. The one thing we know that Wormtongue did was to kill Lotho.\n\n\u2018Then I will,\u2019 said Saruman. \u2018Worm killed your Chief, poor little fellow, your nice little Boss. Didn\u2019t you, Worm? Stabbed him in his sleep, I believe. Buried him, I hope; though Worm has been very hungry lately. No, Worm is not really nice. You had better leave him to me.\u2019\nThe Lord of the Rings Book Six, Chapter 8: The Scouring of the Shire\nPage 1020 (Houghton Mifflin Harcourt; Single Volume 50th Anniversary Edition)\n\nSaruman was the architect of the destruction and would presumably have caused as much havoc if Wormtongue had not been there. We can assume that Lotho or some other hobbit would have died. I don't consider this a consequence of the decision to free Wormtongue\nThe death of Saruman\nLeaving aside the metaphysical question of whether Saruman actually \"died\", it was certainly Wormtongue who removed him from Middle-earth. Frodo (who has presumably gained a lot of wisdom in these matters), did not want this to happen. When the hobbits want to kill Saruman after his attempt on Frodo's life, Frodo says:\n\n\u2018No, Sam!\u2019 said Frodo. \u2018Do not kill him even now. For he has not hurt me. And in any case I do not wish him to be slain in this evil mood. He was great once, of a noble kind that we should not dare to raise our hands against. He is fallen, and his cure is beyond us; but I would still spare him, in the hope that he may find it.\u2019\nThe Lord of the Rings Book Six, Chapter 8: The Scouring of the Shire\nPage 1020 (Houghton Mifflin Harcourt; Single Volume 50th Anniversary Edition)\n\nNote that he is using a similar argument to the one Gandalf used about Wormtongue. I consider this neither a good nor bad consequence of the decision to free Wormtongue\nConclusion\nI didn't judge any of those consequences to be bad, and one of them was actually good. So while Gandalf and Th\u00e9oden understood that there might have been bad consequences, they freed Wormtongue as an act of mercy and were \"rewarded\" with a good outcome (as often happens in Tolkien's works when someone makes a moral choice).\n", "topic": "scifi", "url": "https://scifi.stackexchange.com/questions/152449"}, {"image": "VQAonline_00059072.png", "question": "Is mathematical modelling of investment markets a \"science\"?", "context": "A relative of mine works for a big bank (of bailout fame) in the investment division. We had a discussion where he claimed that what he does is actually a science. Me being an engineer called baloney on this. IMHO there is no science that can predict the future without a decent mathematical model, and I think the world of investing lacks said models.\nSome of the arguments he presented have to do with the sophisticated software models that are used for automated trading. The argument I presented is that all models are wrong (see 2008) and they are just delusional in their science.\nTo settle this, I need to know if there is any falsifiable and repeatable propositions that form the basis of the science for big time, multi-billion dollar bank investing.\nUpdate:\nI found an older article here with mathematical modeling for macroeconomics for the Bank of New Zealand, and it shows (below) the forecast (wide gray areas) nowhere near the actual data (dashed lines). \n\n", "answer": "A famous example of the efficacy (and also problems inherent in) mathematical modeling is the concept of Dynamic Hedging \u2190 a transcript of an excellent documentary on the subject. The model was created by Black and Scholes, for which they were awarded the Nobel prize in Economics. Under certain reasonable assumptions, their model is guaranteed to reduce risk in proportion to the amount of investment one has made (which seems a bit counter-intuitive at first). The best analogy up with which I can come is that Dynamic Hedging is a lot like opening a casino. The model ensures that the odds will always be in the house's favor, so in the long run you will, in expectation, make a profit. The problem is that every once in a while someone will likely \"win big\", and your casino will have to make a payout to them. That's fine, as long as you have enough liquid money to make that payout; you'll eventually recoup the loss in the long run. The problem is that, unlike a casino, the instantaneous risks of the market are unbounded. For example, there is unbounded risk associated with shorting a stock. Therefore, as long as you have an unlimited supply of liquid cash (or credit) to deliver those payouts along the way, you are guaranteed to at least break even. Black and Scholes' model was implemented in the form of a number of hedge funds, most notably Long-Term Capital Management, which enjoyed spectacularly consistent returns in the 40% range throughout the 1980s and most of the 1990s. The problem (I'm greatly simplifying things here) was that eventually the fund got so big that it was no longer able to borrow/acquire enough liquid capital to sustain itself.\nSo what's happening these days? The financial concepts of arbitrage and hedging are theoretically guaranteed to be risk-free. The idea is quite simple:\n\nA widget is being sold by party X in London for $5.\nAt the exact same point in time, due to market fluctuations, the same type of widget is being bought by party Y in New York for $6.\nIf we can find out about #1 and #2 quicker than Y can find that same information, then we can buy the widget from X and sell it to Y at a $1 profit.\n\nThat's why we currently see the \"big banks\" investing millions (and some speculate billions) of dollars into high-tech computer centers very close to the big stock exchanges. (Sorry, that video is in Nederlands, but a lot of it is just dubbed English, and many Dutch words are mutually intelligible with English.) Since almost all trading is done electronically these days, the quicker one can get information from and make trades on the markets, the greater chance one has at exploiting arbitrage. In practice, though, stochasticity of the market does induce risk, which has led to widespread use of statistical arbitrage.\nEdit:\nI feel I should elaborate on the \"science\" vs. \"art\" component of this question, and why I brought up the question of their differentiation in the original set of comments. In my mind, the classifications of science and art are not mutually exclusive. The primary defining characteristic of science is a method by which predictions about a previously mysterious process can be made. Art is the display, application, or expression of a method, almost always with the intention of affecting others' emotions or intellect. By these definitions, science and art are not mutually exclusive, e.g., I believe one could make an argument that cooking is both a science and an art.\nAnyway, without getting sidetracked too much, I also want to mention models. I think Newtonian mechanics and Newton's law of universal gravitation are great examples. Newton's models for the way the universe works are extremely good at \"predicting the future\", at least on the scale of ordinary interactions here on Earth. Unfortunately, Newton's models break down if we change the scale. For example, if we look at a planetary scale, they don't accurately predict the perihelion of Mercury; it wasn't until the turn of the 20th century that models were created that could both accurately predict mechanics on the scale of humans and also mechanics on the scale of planets (most notably Einstein's general relativity). Given that we know there are phenomena in the universe that Newton's model is incapable of predicting, does that mean Newton's model isn't science or is \"delusional in [its] science\"? Of course not. In fact, we still use Newton's model for many tasks (e.g., engineering) down on the human scale, because it's a lot easier to use than the more robust models.\n(I apologize for all of the Wikipedia references; I just think they're a good way to get a high-level understanding of some of these concepts.)\n", "topic": "skeptics", "url": "https://skeptics.stackexchange.com/questions/3315"}, {"image": "VQAonline_00059447.png", "question": "Do cars drop 7% in value every month?", "context": "In their email communication car-buying organization wijkopenautos.nl (wkda.de) claims cars drop 7% in value each month. Their exact claim is:\n\nLet op: een auto verliest iedere maand ongeveer 7% aan waarde, ongeacht hoe vaak er mee wordt gereden. Veelgereden auto's, auto's van pendelaars en stadsauto's hebben over het algemeen meer slijtage. Ook auto's die veel in de garage staan, zijn reparatiegevoelig. Door inactiviteit lekt er veelvuldig olie in de motor en ontstaat er sneller roest.\n\nTranslation by me:\n\nNote: a car loses about 7% in value every month, regardless of how often it is driven. Frequently driven cars, cars of commuters and city cars generally show more wear. Even cars that are often in a garage are repair-prone. As a result of inactivity, oil will frequently leak into the engine and it will rust quicker.\n\nThat 7% per month seems excessively high to me. It would mean a car loses 60% of its value over a year, and would only be worth 7% of its original value after 3 years. \nDo cars drop 7% in value every month?\n\n", "answer": "When buying a new car, a car will depreciate around 11% just driving it off the lot. However, after this first trip, the depreciation rate falls drastically.\n\nRates of depreciation on cars are highest when purchasing new, and when driving off the lot. Trusted Choice wrote in an article \"Car Depreciation: How Much Have You Lost?\"\n\nThe newer the car, the faster its rate of depreciation. Are you wondering, \"What will my car be worth?\" Consider this: The moment you drive your new car off the lot, it will depreciate by as much as 11 percent of its value.\n(...)\nThat means that if you purchase a $20,000 vehicle, it will lose as much as $2,200 in value just by the simple act of your driving it home.\n\nHowever, this depreciation is short lived, and can hardly be estimated across all makes and models.\n\nIn general, popular cars are easier to sell as used vehicles. In this case, depreciation rates will be slower than cars that are difficult to sell. Some vehicles just never seem to catch on, though they may be great cars with fantastic features. A fast depreciation rate does not necessarily reflect on the reliability or performance of the car.\nOther factors can affect depreciation. For example, the recent spike in gas prices caused depreciation rates on gas guzzlers to increase at an alarming rate. Some of the hardest hit vehicles were SUVs and Hummers. As a result, crossovers, with their better gas mileage, have been gaining in popularity over the past few years.\n\nThe article lists the five fastest depreciating cars at the time of the article.\n\nFiat 500L \u2013 34.6 percent price different between new and 1-year old vehicle for a total of $8,096 decline.\nLincoln MKS \u2013 34.5 percent price different between new and 1-year old vehicle for a total of $16,039 decline.\nVolvo S60 \u2013 34.4 percent price different between new and 1-year old vehicle for a total of $14,204 decline.\nKia Cadenza \u2013 34.3 percent price different between new and 1-year old vehicle for a total of $12,940 decline.\nMercedes C250 \u2013 34.3 percent price different between new and 1-year old vehicle for a total of $15,247 decline.\n\nNote that even in the worst case in the article (The Fiat 500L) the depreciation rate over the first year seems to be closer to 3.5% than the 7% found in the claim.\n\nGoodCalculators.com has a vehicle depreciation calculator as well, and while you can set custom depreciation rates, the website states\n\nThe average car depreciation rate is 14% per year.\n\n\nBoston.com looked into the depreciation rate of cars just after purchasing (which, again, is the highest rate of depreciation for new cars.)\n\nThe used car research company looked at more than 14 million new and used cars sold between August 1, 2015 and July 31, 2016 and found the average price difference between a new car and a one-year-old used car is 21.2 percent.\n\n\nFurthermore, if the 7% number was correct, a $20,000 car after 6 years (a standard financing period for cars in the US would leave the car at\n\n$20,000 * (0.93 ^ 72) = $107.59\n\nA quick search on iseecars.com for cars available nationwide with value under $350 returns 8 cars total.\n\n1994 Honda Accord - 230,000 miles\n1998 Nissan Altima - 180,000 miles\n1999 Ford Taurus - Mileage Unlisted\n2014 Volkswagen CC Sport - Mileage Unlisted(1)\n2004 Mitsubishi Eclipse GS - 145,000 miles\n2004 Saturn Ion 1 - Mileage Unlisted\n1993 Chrysler New Yorker Fifth Ave - 142,300 miles\n1998 Chevrolet Malibu LS - 264,000 miles\n\n(1) There is very little information on this Volkswagen, and it seems to be an aberration. Searches for similar cars return prices in the $10,000+ range.\n", "topic": "skeptics", "url": "https://skeptics.stackexchange.com/questions/41120"}, {"image": "VQAonline_00059523.png", "question": "Do 50% of Canadians live south of the 45\u00b042\u2032N latitude?", "context": "Brilliant Maps claim that:\n\nWhile the 49th parallel is often thought of as the border between the US and Canada, the vast majority of Canadians (roughly 72%) live below it, with 50% of Canadians living south of 45\u00b042\u2032 (45.7 degrees) north or the red line above.\n\nThey illustrate it with this diagram:\n\n\n\nThey go on to explain that Toronto, Montreal and Ottawa are \"below\" this line.\nThis is a widely-shared claim. AmazingMaps make a similar claim.\nIt was discussed in a Quora question but the answers seem to be contradicting each other and are mostly based on opinions.\nDoes 50% of the Canadian population live south of the 45\u00b042\u2032N latitude?\n", "answer": "Close Enough\nThe claim is from 2015. Canada in 2016 had a population of 35,151,728.\nLooking at the 2016 population for Census Metropolitan Areas in Ontario, Quebec and Nova Scotia in that area with over 100,000 population...\n\nToronto 5,928,040\nMontreal 4,098,927 (some of the Montreal CMA is above the line, someone could dig into the individual Census Divisions)\nOttawa - Gatineau 1,323,783\nHamilton 747,545\nKitchener - Cambridge - Waterloo 523,894\nLondon 494,069\nSt. Catharines \u2013 Niagara 406,074\nHalifax 403,390\nOshawa 379,848\nWindsor 329,144\nSherbrooke 212,105\nBarrie 197,059\nKingston 161,175\nGuelph 151,984\nBrantford 134,203\nPeterborough 121,721\nBelleville 103,472\n\nThey add up to 15,716,433 or 44.7% of the population. It's reasonable to assume the remaining smaller population centers, and people outside of population centers, will bring it near enough to 50% to say this claim is true.\n", "topic": "skeptics", "url": "https://skeptics.stackexchange.com/questions/50122"}, {"image": "VQAonline_00059674.png", "question": "How to translate 'alright' when used as a trait (as in \"being alright at racing\")?", "context": "I'm having a rough time finding the best translation for this J. Taylor tweet made during Fernando Alonso's first stint in the 24 Hours of Daytona race:\n\nThis [Alonso] guy seems alright.\n\n\nOk, so we all know 'alright' means bien: \n\n\"Everything's alright\" \u2192 \"Todo est\u00e1 bien\"\n\"Are you alright?\" \u2192 \"\u00bfEst\u00e1s bien?\", etc. \n\nIn these examples, 'alright' is used to describe the state of something/someone -- the kind that'd force you to use \"estar\" instead of \"ser\" in Spanish. \nIn the above tweet, however, 'alright' is being used as a trait: the kind we use \"ser\" with \u2014 like saying Alonso seems \"OK at racing\" (at least that's how I understood it).\nAnd this is where the rough part begins, because everything I come up with seems slightly off. For staters, Google Translate seems to disagree with my interpretation, and translates it as just \"Este chico Alonso parece estar bien\", but that looks like we're talking about his health. As for me, I've tried:\n\n\"parece bueno\", but I think someone being \"bueno\" at something is a tad over being just 'alright' at something;\n\"parece aceptable\", \"parece decente\"... but those don't give the same informal vibe that 'alright' does.\n\nMaybe I'm just overthinking this, but whatever.\nWhat would be the best translation for 'alright' in this context?\n\nEDIT: Looking at the current answers, it seems like there are more types of 'alright' than I thought!\nPlease notice that I'm asking specifically about being 'alright' at something:\n\n[...] like saying Alonso seems \"OK at racing\" (at least that's how I understood it). [...] I've tried \"parece bueno\", but I think someone being \"bueno\" at something is a tad over being just 'alright' at something.\n\n", "answer": "Maybe you can use:\n\nParece un t\u00edo majo/legal/guay.\n\nI have seen a comment in the Urban Dictionary that says:\n\n[When] you call a person 'alright' you are saying that they are amazing, awesome, brilliant, and you want to love them.\n\nIf this is true (it seems so according to @mdewey's comment) you can even say:\n\nParece un t\u00edo genial/fant\u00e1stico/etc.\n\n\nAfter your edit it seems that we misunderstood your question. You actually wanted to translate \"being alright at something\", so I will stick to @mdewey's comment then and propose the following.\n\nNo se le da mal a Alonso [eso de conducir].\n\nI think this sentence keeps the feeling of saying that he is actually very good at it while stating that he is just \"not bad\".\n", "topic": "spanish", "url": "https://spanish.stackexchange.com/questions/28451"}, {"image": "VQAonline_00059624.png", "question": "\u00bfEs correcto usar \"aperturar\" como sin\u00f3nimo de \"abrir\"?", "context": "Le\u00eda ayer en Twitter:\n\n#16J 07:00am Comienzan a aperturarse los puntos soberanos en todo el pa\u00eds para que ciudadanos manifiesten su voluntad. #HoyElPuebloDecide\n\nLo que la Asamblea Nacional de Venezuela quer\u00eda decir es que comenzaban a abrirse unos puntos de votaci\u00f3n.\nComo hablante de castellano de Espa\u00f1a, este uso me pareci\u00f3 muy raro y as\u00ed me lo confirm\u00f3 que no aparezca en el DLE. Sin embargo, para mi sorpresa el Diccionario de americanismos s\u00ed lo recoge:\n\naperturar.\n I. 1. tr. Ho, Ve, Pe, Bo. Inaugurar alguien algo. pop + cult \u2192 espon.\n\nY veo que efectivamente cada vez se usa m\u00e1s en detrimento de \"inaugurar\", como se ve en Google Trends:\n\nSin embargo, \u00bfse puede usar como sin\u00f3nimo de abrir?\n", "answer": "No, no es correcto.\nSe pronunci\u00f3 Fund\u00e9u en abrir, mejor que aperturar:\n\nAbrir, mejor que aperturar, es el verbo relacionado con el sustantivo apertura.\nEn los medios de comunicaci\u00f3n est\u00e1 muy extendido el verbo aperturar: \u00abFirman el contrato para aperturar la calle que da acceso a la Urbanizaci\u00f3n Garc\u00eda Guill\u00e9n\u00bb, \u00abM\u00e1s de 600 mujeres en este municipio de Tierra Caliente necesitan recursos para aperturar o ampliar un negocio\u00bb o \u00abDesde que el Grupo Dolle comenz\u00f3 a aperturar tiendas de moda en el a\u00f1o 2014, su crecimiento se ha incrementado de forma notoria\u00bb.\nTal como indica el Diccionario panhisp\u00e1nico de dudas, el uso de aperturar, especialmente habitual en el lenguaje bancario, no est\u00e1 justificado. Del mismo modo, el diccionario Clave se\u00f1ala que la utilizaci\u00f3n de aperturar es innecesaria.\nAs\u00ed pues, en los ejemplos iniciales habr\u00eda sido preferible escribir \u00abFirman el contrato para construir la calle que da acceso a la Urbanizaci\u00f3n Garc\u00eda Guill\u00e9n\u00bb, \u00abM\u00e1s de 600 mujeres en este municipio de Tierra Caliente necesitan recursos para abrir o ampliar un negocio\u00bb o \u00abDesde que el Grupo Dolle comenz\u00f3 a abrir/inaugurar tiendas de moda en el a\u00f1o 2014, su crecimiento se ha incrementado de forma notoria\u00bb.\n\nEl enlace interno nos lleva al Diccionario panhisp\u00e1nico de dudas:\n\napertura\n2. A partir del sustantivo apertura (\u2018acci\u00f3n de abrir\u2019), se ha formado el verbo aperturar, que ha empezado a utilizarse en los \u00faltimos a\u00f1os como equivalente de abrir: \u00abOrdeno que esos contenedores sean aperturados y revisados\u00bb (DHoy [Ec.] 8.7.97); \u00abAyer domingo la Cooperativa Agraria de Producci\u00f3n Casa Grande apertur\u00f3 sus Terceros Juegos de Verano\u00bb (Comercio [Per\u00fa] 14.1.75). Es especialmente frecuente en el lenguaje bancario, donde se ha puesto de moda la expresi\u00f3n aperturar una cuenta, en lugar de abrir una cuenta. Su uso no est\u00e1 justificado y debe evitarse.\n\nFinalmente leo en Diccionario de dudas el art\u00edculo Aperturar o abrir:\n\nLa palabra correcta es abrir. No es apropiado emplear el t\u00e9rmino aperturar como equivalente del verbo abrir.\nEl verbo abrir se refiere a la acci\u00f3n de \u2018descubrir o destapar algo que estaba oculto\u2019, \u2018iniciar una actividad\u2019, \u2018hacer algo patente o manifiesto\u2019, \u2018separar lo que estaba junto\u2019, entre otras cosas.\nComo tal, aperturar es un verbo que deriva de la voz sustantiva apertura, que significa \u2018acci\u00f3n de abrir\u2019. Coloquialmente, la palabra ha venido imponi\u00e9ndose en el lenguaje, sobre todo en el \u00e1mbito bancario, para hacer referencia al \u2018acto de hacer apertura de una cuenta bancaria\u2019, y de all\u00ed se ha ido extendiendo a otros \u00e1mbitos.\n\n", "topic": "spanish", "url": "https://spanish.stackexchange.com/questions/21525"}, {"image": "VQAonline_00059722.png", "question": "Transcribing Spanish Civil Guard notes for 1909 (note 3 of 4)", "context": "For the year 1909 in my Great Grandfathers Civil Guard records there are four entries.\nHere is the third one:\n\nThe transcribed text so far:\n\n1909: Por Real orden fecha 10 de agosto (D. O. no 177) se decir las\ngracias a este individuo por haber contendiendo a la xxx de xxx xxx\nxxx a Canf\u00f3rica.\nEl 2\u00ba Jefe\n\nTo be honest, I am not sure if the abbreviate is D.O. I have located D. O. no. 177 but don't know what I am looking for. I did wonder if the abbreciation was R.O. but it does look like D.O. to me.\n\nBulletin Update\nI realise that it must be related to the main article on pages 1 and 2 with the title Real Ordenes. I seeit is dated 10 Agosto. As to what it says ... another matter. :)\n\nTranslation\nUsing the transcribed text provided in the comments by @gparis below we get the following translation:\n\nBy Royal Order dated 10 August (D. O. No. 177) this individual was\nthanked for having contributed to the mobilisation of troops for the\nCampaign (?).\nThe 2nd Chief\n\n\nRelated Questions\n\nTranscribing Spanish Civil Guard notes for 1909 (note 1 of 4)\nTranscribing Spanish Civil Guard notes for 1909 (note 2 of 4)\nTranscribing Spanish Civil Guard notes for 1909 (note 4 of 4)\n\n", "answer": "Based on the suggestions by @gparisin the comments, we have settled on the following transcription:\n\nPor Real orden fecha 10 de agosto (D. O. no 177) se dan las gracias a\neste individuo por haber contribuido a la movilizaci\u00f3n de tropas con\ndestino a Campa\u00f1a (?).\n\nAnd, we have agreed on the following translation:\n\nBy Royal Order dated 10 August (D. O. No. 177) this individual was\nthanked for having contributed to the mobilisation of troops for the\nCampaign (?).\n\n\nHistorial Insights\nIt is worth mentioning the information that @gparis provided in the comments:\n\nCampa\u00f1a may refer to the Second Melillan campaign. As you said, the\nmain article of the D.O. no. 177 is an order to thank all the\nindividuals involved in the organization of this troop movement and\nthe repression of the protests.\nThe same article mentions that the additional troop movement to\nBarcelona involved forces from Mahon, Valencia, Zaragoza, Pamplona,\nBurgos, Madrid and Sevilla.\n\n", "topic": "spanish", "url": "https://spanish.stackexchange.com/questions/39422"}, {"image": "VQAonline_00059746.png", "question": "Why are pitcher's mounds higher than the rest of the baseball field?", "context": "On softball fields, the pitcher's \"mound\" appears flat. However, on baseball fields, the pitcher's mound is a mound. MLB Official Rules state (Rule 1.04, in part):\n\nThe pitcher\u2019s plate shall be 10 inches above the level of home plate.\n\nWhy are pitcher's mounds higher than the rest of the baseball field? What noticeable differences do pitchers (and hitters, if applicable) experience between a flat surface and a mound?\n\n", "answer": "Pitcher mound is a result of trial and error in order to find the perfect distance and height that will be some kind of point of equality so the pitcher and the batter won't be advantage on each other and ensuring the attraction of the game.\nIt is because we don't want a game when a pitcher is always striking out players and on the other hand we don't want the batter to do a home run in every bat.\nFirst a little bit of history..\n\nHow the pitcher's mound was born?\nIn the early days of baseball , pitchers were required to stay within\na pitcher's box that had its front edge 45 feet away from home plate.\nBut pitchers were dominating batters, so in 1881, the front edge of\nthe box was moved back five feet.\nBut this was still not enough, so in 1887, a new rule required a\npitcher to keep his back foot on a line that was 55 and one-half feet\nfrom the plate. Finally in 1893, the pitcher's box was removed, but\nthe pitcher's plate, or rubber, was introduced and moved back an\nadditional five feet. That gave us today's 60 feet 6 inches.\nWith an official diameter of 18 feet, and rising to 10 inches above\nthe level field, the pitcher's mound, or hill, has been a part of\nbaseball since around the turn of the 20th century.\n\nThe elevation on pitcher's mound was made in order to return some advantages to pitchers that was lost due to extending the pitcher position.\n\nBy elevating their delivery point, pitchers can gain momentum as they\nstride down towards the plate. This forward motion adds speed and a\ndownward angle to the ball, making it more difficult to hit.\n\noff course there are another speculations like this one:\n\nThat the mounds evolved as a matter of\ngrounds-keeping practice, for better drainage and water absorption.\nAfter overhand pitching was legislated in 1884, pitchers undoubtedly\nfound the mounds to be an advantage: the downward weight-shift and\nmomentum enable them to generate greater velocity on their pitches.\n\nas you stated in the question\n\nThe pitcher\u2019s plate shall be 10 inches above the level of home plate.\n\nBut this was not always true:\n\nDuring different periods in baseball history, the mound has been much\ntaller. The mound in Philadelphia's Shibe park was rumored to be 20\"\nhigh at one time, and the mound throughout baseball in the late 60s\nwas as high as 16\". It wasn't until 1969 that it was lowered to\ntoday's standard.\n\nAccording to baseball historian Bill Deane:\n\nthe idea for a mound was claimed by an 1880s pitcher, John Montgomery\nWard. From 1903 to 1968, baseball rules limited the height of the\nmound to 15 inches, although several groundskeepers were rumored to\nhave added an inch or two depending on the state of their team's\npitching.\nOver the years, as pitchers grew bigger and stronger, hitting\ndeclined. Major League Baseball knew that most fans would rather see\nhome runs than shutouts, so the mound was lowered to ten inches in\n1969. The adjustment worked \u2013 team batting averages jumped up about 15 points.\n\nSources: 1 2\n", "topic": "sports", "url": "https://sports.stackexchange.com/questions/1549"}, {"image": "VQAonline_00059745.png", "question": "What was the first team to use the 1-3-1 Forecheck", "context": "I'm a big hockey guy, but over the last 2 or so seasons I've seen the Tampa Bay Lightning use the incredibly boring 1-3-1 forecheck. As you see in the picture below, the Tampa Bay Lightning (blue) is using the 1-3-1 forecheck against the Philadelphia Flyers (orange).\nI was wondering, who was the first team / coach that used the 1-3-1 forecheck?\n\n", "answer": "The New Jersey Devils used a similar setup back in the mid 90's. It was so revolutionary that it knocked fans out of their seats -- and into their beds ;)\nThe 1-3-1 is a type of neutral zone trap, which many teams will employ situationally at different points in the game. It is a natural evolution of the single forechecker on a penalty kill, and is often done late in games to preserve comfortable leads. As a result, I am almost certain you can see 1-3-1 variants at various points of games as far back as you can find professional hockey footage; as such, it is hard to figure out who used it first.\nBut as for using it as a general game strategy, probably go back to 1995 NJ Devils, or so.\nEdit, for more detail:\nThe resurgence of the 1-3-1 can be linked to the removal of the two-line pass rule. Prior to the two-line pass removal, teams had to carry the puck at least over their own blue line before attempting to connect with another player near the offensive zone. Two forecheckers could successfully jam the breakout at the blue line.\nHowever, with two-line pass permissible, many teams (cf. Blackhawks 2009) will position a winger just before the offensive blue line. During the breakout, a player will fire a hard pass towards this winger, who will tip it deep, thus avoiding an icing. Then, the remaining winger and center will attack the offensive zone with speed, which gives them an advantage over the defenders, who have to transition to the puck.\nThis causes a lot of \"dump and chase\" hockey that favors the offense -- the fast moving offence can either get to the puck first, or come into the defender hard.\nThe counter to this is to jam the passing lane through the neutral zone by stacking three defenders deep. The back defender can still get back to jam the puck in the corner if necessary.\nConsequently, the 1-3-1 saw little use pre-lockout in the NHL, because it wasn't a useful counter strategy. You saw it in College hockey somewhat, which didn't have a two-line pass rule.\n", "topic": "sports", "url": "https://sports.stackexchange.com/questions/1484"}, {"image": "VQAonline_00059812.png", "question": "What is the importance of the game scoresheet?", "context": "In sports games, what is the main importance of the scoresheet?\nAs in the image below?\n\n", "answer": "In general, the scoresheet is the official record of the result of the game. In most sports I've played, it is signed by the captain (or other representative) of each team at the end of the match to show that they agree with the final score, and then sent to the organising body to show the result of the match (and other secondary things like any disciplinary sanctions applied to players).\nTo quote from the offical volleyball rules (just as that's the sport I'm probably most familiar with):\n\nAT THE END OF THE MATCH, the team captain: thanks the referees and signs the score sheet to ratify the result\n\nI'd imagine similar things exist for other sports.\n", "topic": "sports", "url": "https://sports.stackexchange.com/questions/16738"}, {"image": "VQAonline_00061051.png", "question": "What is the restricted equation of a test including three variables?", "context": "\nWhat is the restricted equation of a test including three variables?\n\nOn gretl i've been try to test manually by f test\n$$ y_t = B_1 + B_2x_2 + B_3x_3 + B_4x_4 + e_i$$\nThat \n$$H_0: B_2 + B_3 = B_4 $$\n$$H_1: B_2 + B_3 \\ne B_4 $$\nSo do i want to make a new variable $ w= B_2 + B_3 - B_4$\nThen regress\n $$y_t = B_1 + B_2x_2 + B_3x_3 + wx_4 + e_i$$\nor maybe $ m= B_2 + B_3$\n $$y_t = B_1 + B_2x_2 + B_3x_3 + mx_4 + e_i$$\nBut i can't seem to we get the restricted model right because i keep ending up with a zero or gretl not letting me regres like with m.\nI'm always way off\n\n", "answer": "We start with the regression equaltion:\n$y = \\beta_0 + \\beta_1 x_1 + \\beta_2 x_2 + \\beta_3 x_3 + \\varepsilon$\nand constraint: \n$\\beta_1 + \\beta_2 = \\beta_3$\nWe replace $\\beta_3$ with the constraint\n$y = \\beta_0 + \\beta_1 x_1 + \\beta_2 x_2 + (\\beta_1 + \\beta_2) x_3 + \\varepsilon$\nWrite it out\n$y = \\beta_0 + \\beta_1 x_1 + \\beta_2 x_2 + \\beta_1 x_3 + \\beta_2 x_3 + \\varepsilon$\nCollect the $x$s that share the same parameter\n$y = \\beta_0 + \\beta_1 (x_1 + x_3) + \\beta_2 (x_2 + x_3) + \\varepsilon$\nTrying this out in my favourite statistics program (Stata) gives me these results:\n. sysuse auto, clear\n(1978 Automobile Data)\n\n. constraint 1 _b[mpg] + _b[rep78] = _b[foreign]\n. cnsreg price mpg rep78 foreign, constraint(1)\n\nConstrained linear regression Number of obs = 69\n F( 2, 66) = 11.58\n Prob > F = 0.0000\n Root MSE = 2543.3814\n\n ( 1) mpg + rep78 - foreign = 0\n ------------------------------------------------------------------------------\n price | Coef. Std. Err. t P>|t| [95% Conf. Interval]\n-------------+----------------------------------------------------------------\n mpg | -282.4264 58.67858 -4.81 0.000 -399.5819 -165.2708\n rep78 | 656.5643 271.0772 2.42 0.018 115.3413 1197.787\n foreign | 374.1379 246.6847 1.52 0.134 -118.384 866.6599\n _cons | 9808.867 1192.953 8.22 0.000 7427.059 12190.68\n------------------------------------------------------------------------------\n\n. gen m1 = mpg + foreign\n. gen m2 = rep78 + foreign\n(5 missing values generated)\n\n. reg price m1 m2\n\n Source | SS df MS Number of obs = 69\n-------------+---------------------------------- F(2, 66) = 11.58\n Model | 149856884 2 74928441.8 Prob > F = 0.0000\n Residual | 426940075 66 6468789.02 R-squared = 0.2598\n-------------+---------------------------------- Adj R-squared = 0.2374\n Total | 576796959 68 8482308.22 Root MSE = 2543.4\n\n------------------------------------------------------------------------------\n price | Coef. Std. Err. t P>|t| [95% Conf. Interval]\n-------------+----------------------------------------------------------------\n m1 | -282.4264 58.67858 -4.81 0.000 -399.5819 -165.2708\n m2 | 656.5643 271.0772 2.42 0.018 115.3413 1197.787\n _cons | 9808.867 1192.953 8.22 0.000 7427.059 12190.68\n------------------------------------------------------------------------------\n\n. lincom m1 + m2\n\n ( 1) m1 + m2 = 0\n\n------------------------------------------------------------------------------\n price | Coef. Std. Err. t P>|t| [95% Conf. Interval]\n-------------+----------------------------------------------------------------\n (1) | 374.1379 246.6847 1.52 0.134 -118.384 866.6599\n------------------------------------------------------------------------------\n\n", "topic": "stats", "url": "https://stats.stackexchange.com/questions/180986"}, {"image": "VQAonline_00061010.png", "question": "GLM vs least squares with Gamma errors?", "context": "To illustrate the usefulness of GLMs in comparison to the least square method I did a simple program in which I add random noise to a straight line (Y=m*x + b; red line in the attached plot). The noise is coming from a Gamma distribution \"centered\" on zero (the mode, i.e., the maximum of the gamma distribution is lying over zero). When I plot this artificial signal (Y + g_noise; black dots in the plot) together with Y, I see Y nicely cutting the noise dots in the region of maximum density. I though this simple model of a straight line and asymmetrical not-normal noise is an \"easy\" task for a GLM and should exemplify its superior performance in comparison to least squares. So, I take Y as input of 2 linear models:\n\nThe classical least squares (Y_ls; green line in the plot). As expected, Y_ls is unable of reproducing the original signal Y but it performs quite good: It makes a straight line parallel to Y, cutting the noise dots not in the region of maximum density but in the \"middle\". Namely, Y_ls has offset in comparison to Y. This is in agreement with the Gauss-Markov theorem, since the noise is not normal, least squares cannot yield an unbiased linear estimator. I was expected this, so far so good.\n\nA GLM with a Gamma canonical link. To my big surprise, Y_glm (magenta curve) is not only unable of reproducing the original signal, but from my point of view it even performs worse than Y_ls: It only fits the original signal in the middle of the distribution, which is OK, but at the beginning and end of the record Y_glm is a considerable poorer estimation in comparison to Y_ls, to the naked eye. A GLM with an identity link but Gamma variance ($\\mu^2$) should, from my point of view, also performs well, but it simply reproduces Y_ls, meaning no improvement over least squares.\n\n\nSo, my questions are:\n\nAm I misunderstanding the sense of a GLM? Am I doing something wrong? Is this really the best we can get of a GLM in comparison to least squares?\n\nIs there another model (perhaps a GAM?) which would really be able of seeing through the data, identifying the Gamma noise and correctly reproducing the original line Y (the red line)?\n\n\n", "answer": "Your main problem is that the link is a non-linear transformation. The default (canonical) link function for the Gamma distribution is the reciprocal (the log is also common). You should do better with a Gamma response but an identity link, given the way you generated the data. \nIn addition, all these models are for the mean of your conditional distribution. It seems you centered your data using the mode. That's why the linear model is shifted vertically relative to the red line. \n", "topic": "stats", "url": "https://stats.stackexchange.com/questions/174358"}, {"image": "VQAonline_00060880.png", "question": "What is the best function to fit onto a \"flat top gaussian\"?", "context": "I am studying some signal and I am trying to make an automated algorithm that can extract the parameters for my signal.\nFirst let me describe a bit, I have a light emiting object crossing a window of observation with a given speed. That object is an elipse, thus when I measure the signal I find something that looks like a flat top gaussian. (see scheme)\nI have been using a Gaussian Mixture Model to describe it, but because the top is flatten, it finds many more distribution than what is actually there !\nThus I am looking for a mathematical function that could describe that kind of signal, so that I can implement a least square regression or something along those line.\nIdealy that function would have a \"variance\" and a \"mean\" parameter such that I can make analysis about those two later on.\nThanks a lot !\n\n", "answer": "You have probably solved this problem, since this post is from a year ago. But just in case: is there a reason why you can't directly fit a flat-top Gaussian functional representation to your distribution? The general Gauss function is $g(x) = \\exp[-(x/a)^n]$, \"$a$\" being the half-width at $1/e$. $n=2$ is a standard Gaussian, $n>2$ a flat-top Gaussian, and $n<2$ a peaked Gaussian.\n", "topic": "stats", "url": "https://stats.stackexchange.com/questions/154526"}, {"image": "VQAonline_00063858.png", "question": "How much land would be covered by the plants that turned into the fossil fuels humanity has burned?", "context": "Many climate deniers suggest that humanity is too insignificant to impact the global climate. As a visualization to counter this claim, I thought of the following exercise.\nHow much coal, oil, and gas humanity has burned in our history?\nThis part should be straightforward -- I found this chart of global fossil fuel consumption as a starting point: \n\n\n\nHow much plant life does that represent?\nBasically, how many trees/bushes/grasses does it take to form a ton of coal or a barrel of oil? At some point in history, the coal, oil, and gas we're digging up were plants in a forest, prairie, or swamp. \nThis Quora answer suggests that 10 tons of plant matter is required to form 1 kg of fossil fuel, but the link to the source is broken, and I'm not able to find any others.\nHow much land would all those plants cover?\nOnce we know how many tons of plant matter went in to all the fossil fuels we've burned, we could figure out how many acres of forest, prairie, or swamp that would equate to. This part probably requires a lot of assumptions (density of various ecosystems, etc.), but that's OK.\nThe result\nI'm imagining that this is going to be a vast quantity of land. At least the size of Africa. Perhaps several continents. Once we've got that number, we can visualize it -- imagine burning down a forest the size of [Africa/Asia/some other recognizable land mass] -- now we can understand how humanity's consumption of fossil fuels might impact the climate.\n", "answer": "How coal was formed\nRegarding coal, from trees, it's not simply a question of the area whether it was a continent-size landmass, but more of the temporal side of things -- how long a time period the land was covered.\nCoal was laid down from plant matter during the Carboniferous period, which lasted 60 million (60,000,000) years. For most of that time the micro-organisms that existed were unable to break down the lignin which made up 70% of the plant matter. Effectively the trees didn't rot, they fell down, and more trees grew on top, then died and fell down and more trees grew on top, for millions of years. The carboniferous period ended when the microorganisms evolved the ability to consume the lignin and the coal laying stopped.\nThree million generations of trees\nSo supposing it takes twenty years for 'a tree' to reach maturity, the carboniferous period is three million generations of trees. 'Tree' is a little inaccurate compared to what we currently think of as a tree. The plants were effectively a really tall moss, and it was 80% 'bark' compared to modern trees that are about 10% bark.\nAll land on earth\nDuring the carboniferous period most of the land on planet was covered with trees, Africa, Asia, the Americas. But to somehow recreate all the coal, you need at least twenty years for a tree to grow, and because there were three million generations, you need three million times the amount of land available on earth's land masses.\nSuppose we drained the oceans to create more land for more trees, then you'd only need one million earth's worth of land.\nHow many trees, how many earths?\nCurrently there are around three trillion tress on earth -- it's not unreasonable to think that during the carboniferous period when there was nothing else happening, there would be maybe double that, six trillion trees at any moment. Over a 60,000,000 year period, that would be a total of eighteen quintillion trees (eighteen with eighteen zeros after it).\nMy point is, to recreate the coal that we've consumed alone, ignoring the oil and gas, it's not a matter of planting a few trees, its more a case of planting quintillions of trees on hundreds of thousands of planets. 18 quintillion trees at a rate of 3 trillion per planet requires 6 million planets.\nEven if my sums are a few zeros out, its a mind-boggling number of trees and amount of land.\nWhat about the coal we've already used?\nIn terms of coal that we have already burnt, the World Coal Association figures estimate that the proven coal reserves left in the ground are enough to last another 150 years at current rates -- this is the coal that is easy to dig up with current mining technology. There might be vast quantities of coal located in inaccessible locations, like the bottom of the sea, or spread out too thinly to be cost-effective to mine. This isn't included in the proven reserves figure.\nLooking at the Global Fossil Fuel consumption graph in the question, we could estimate that historic coal usage is about 75 year's worth at current consumption rate. At current usage rates, the sixty million years of laying down coal gave us around 225 years of coal that is economically accessible. We're roughly a third of the way through that.\nFrom the earlier calculations, that's three quintillion trees spread out over only two million earth-sized landmasses.\n", "topic": "sustainability", "url": "https://sustainability.stackexchange.com/questions/9518"}, {"image": "VQAonline_00063844.png", "question": "Energy requirements for sustainable food production system", "context": "Overview\nConsider the following system for sustainable food production:\n\nThe purpose of this system is to provide an environmentally-friendly, semi- (or fully-) automated means to produce food for lots of people. Similar in nature to the Earthship question, but delving into a few more details.\nProblem\nI need help performing calculations on multi-storey indoor farms, using vertical plan production. I'm trying to reconcile the following variables:\n\nPeople to feed.\nBuilding dimensions and storeys.\nTotal wattage necessary for lighting.\n\nFor example:\n\n1,000 people\n64m x 64m x 6 storeys x 4m per storey\n7.71MW\n\nCalculations\nThe following sections serve as an example calculation.\nEnergy\nFor the purposes of an example, assume a population of 350 people (but it could be 1,000 or any other number). The minimum global daily intake as of 2006 is 1850 calories per person, which is 7740kJ per person.\n\ndaily = 7740kJ\ninput = 350 x 7740kJ x 365 = 988785MJ\n\nFor a given population of 350 people, they require a minimum of 988785MJ of energy per year.\nPlant Growth\nSince we know the energy per year needed by the population, it is possible to determine the size and harvests of a food that will yield that energy amount.\nA giant large red round heirloom tomato stores 85kJ of energy per 100g, and reaches an average of 375g in 65 days, which equals 318kJ. (Round the number of annual harvests down to 5 to err on caution.)\n\nenergy = 318kJ (per tomato)\nyield = 50 (per plant)\nharvests = 5 (per year)\n\nThe number of plants is calculated as:\n\nplants = input / (harvests x yield x energy)\nplants = 988785MJ / (5 x 50 x 318kJ)\nplants ~= 12450\n\nThe space required for each tomato plant:\n\ndiameter = 2m\nheight = 1.5m\n\nBuilding Area\nThe minimum building area is calculated using:\n\narea = (pi x diameter/2) x plants\narea = (pi x 2m^2/2) x 12,450\narea = 3.14159265m2 x 12,450 = 39,113m2\n\nSince indoor farms use vertical space, we can pick an arbitrary number of storeys for the building. In this example, we'll choose 10. At 10 storeys, each storey must occupy 3,911.3m2. Therefore, a square building would be approximately 63m per side.\nUsing plants 1.5m tall, means a storey 3m tall fits two levels of plants per storey. This halves the building length and width to 31.5m per side. We still need to account for interstitial space for infrastructure (e.g., ducts, plumbing, wiring). This adds 10.5m per side, bringing the total length and width to ~42m per side. Thus:\n\nstoreys = 10\nplant floor space = 31.5m x 31.5m\ninterstitial = 1/3 (10.5m)\ntotal floor space = 42m x 42m\n\nLighting\nUsing high-efficiency LED grow lamps:\n\nracks = 2\ncoverage = 1.2192m2\nlamps1 = plant floor space / coverage x racks x storeys\nlamps1 = 992.25m2 / 1.2192m2 x 2 x 10\nlamps1 ~= 16277\n\nUsing light movers increases coverage by 35 per cent, therefore:\n\nlamps2 = lamps1 - (0.35 x lamps1) ~= 10,581\nmovers = lamps1 - lamps2 = 5,696\n\nEnergy Requirements\nThe electrical needs are calculated as:\n\npower = 250W (per lamp)\ntotal1 = lamps2 x power ~= 2.65MW\ntotal2 = total1 + movers x 5W = 2.68MW\n\nMy primary concern is whether total1 is correct.\nPlasma lighting seems unbelievably efficient, which could reduce substantially reduce power requirements even further.\nAdditional Calculations\nNate Storey calculated the following:\n\nUsing current LED efficiencies.\nSustain (calorie + nutrition) 1,000 people on a variety of foods.\nThe building is 64m x 64m (approx. 1 acre) by 6 storeys.\nEach storey must be 4m (approx. 13 feet) high.\nVertical plane production within the vertical building.\nRequires 45 watts/sq. ft. per hour, at 18 hours per day for ~3.5 square feet of production.\n\nI think this implies:\n\narea = 64m x 64m x 6 = 24576m2 area\npower = 45W/ft2 = 484.376W/m2\nenergy = area x power = 24576m2 x 484.376W/m2 = 11.9MW\n\nHis calculations show ~11.9MW to feed 1,000 people; my calculations show ~2.68MW to feed 350 people, which is ~7.66MW to feed 1,000.\nQuestions\nWhat assumptions made for total1 are incorrect, if any, and how can they be corrected?\n", "answer": "Given the uncertainties in many of your numbers, your answer is consistent with Nate Storey's.\nYour assumption of 303 lm/W for LEDs doesn't look plausible to me - that's very different to the sort of efficiencies I see on the market. Half that would be much more plausible. So no, your total1 calculation doesn't look right to me.\nAnd you do need to account for uncertainties in the calculation.\nAll in all, with these sorts of complex systems, the sort of back-of-envelope calculation you've done is all very well as a first step, but only as a first step. The next step is to build a very small-scale prototype system and start looking at real data, for each of the system components, and for the system as a whole. And look at what other people have done: real-world experiments with empirical data.\nModel, experiment, reconcile, repeat.\nOnly then will you find out if your calculation is correct and meaningful.\n", "topic": "sustainability", "url": "https://sustainability.stackexchange.com/questions/4821"}, {"image": "VQAonline_00063850.png", "question": "How different are small fruits and vegetables grocery stores from around the corner from big supermarkets?", "context": "When I buy fruits and vegetables in a small grocery store at the corner, I cannot help myself but think that\u2014coming from the idea that \"Small is Beautiful\"\u2014if I do my shoppings in these kinds of stores, then I do good for sustainable development.\nThese are my thoughts:\n\n\"Look at this small shop owner\"\n\"Look at these imperfect fruits and vegetables. They look different than those from the big supermarket\"\n\"Hmmm, but why are they so cheap?\"\n\"Ultimately, where do these fruits and vegetables come from and how are they being produced? Are they actually the same as those found in the supermarket?\"\n\n\n", "answer": "I understand the approach of helping smaller stores, and it is already a good idea to stop supporting these big supermarkets that already own a lot of shares.\nNow it depends on the location of your store, but on your picture are a lot of different fruits. All these fruits have to be transported in some way, some more than others. \nIf the store is focused on bio and fair trade products it is of course a good idea. But if it is a normal store without any focus on sustainability, you're again supporting some bad conditions in different places of the world with your purchase. \nSo talking about the same fruits just from a smaller store has only one difference:\nThe smaller store can't buy quantities of product like a bigger supermarket, and this leads to worse price conditions. Items in a smaller store are more expensive than in a big grocery. It's great, to support a smaller store with people from your place, but imho, nowadays a food store needs more love and focus on the nutrition itself and not a very very big selection. \nI am sure, a smaller market owner is also working differently with food and maybe wouldn't throw away in amounts like big supermarkets. So regarding this fact supporting a local store is a very nice idea.\n", "topic": "sustainability", "url": "https://sustainability.stackexchange.com/questions/5635"}, {"image": "VQAonline_00064014.png", "question": "Do the Hong Kong metro lines have numbers?", "context": "Before I travelled to Hong Kong, I looked at the MTR Map, which described all the lines by colour and by name, but not by number. Generally, the signs up around the MTR did the same.\nHowever, a few times, and especially when changing trains, I spotted signs with name + colour + number. This would've been easier for me, as in a jetlagged state remembering \"I want the line that starts with a T\" turned out to not be nearly so unique as planned...\nAre there actually numbers for lines, or do the numbers mean something else? I didn't get a chance to take a photo, but below is a cc-by photo from Flickr which shows the kind of sign I spotted:\n\n", "answer": "These are platform numbers. The colour indicates the line but the number is specific to the station. (Online confirmation)\nYou can note the platform numbers (I think trains of different lines always leave from different platforms in Hong Kong, ), but they vary from station to station.\n", "topic": "travel", "url": "https://travel.stackexchange.com/questions/22605"}, {"image": "VQAonline_00064306.png", "question": "How bad are the transit queues at Doha airport?", "context": "How bad are the queues when transiting? In other words, how long a transit time would be a safe bet? \nIf makes a difference, both flights are Qatar Airways, transiting between AKL and HEL.\nI'm asking because I see a lot of negative comments about this. A few examples: \nairlinequality.com Doha reviews:\n\nSecurity screening area for transit far too small and queues are huge.\n\nFoursquare, Aug 2015:\n\nExpect big queues at the security check when transferring flights.\n\n\nImage by Vincent Tan\nFoursquare, Jan 2015: \n\nAtrocious immigration and transit queues.\n\n", "answer": "Hamad International Airport in Doha has, in common with the other ME3 hubs, several \"waves\" of arrivals and departures. These are periods with lots of arriving and departing flights, with lots of transit passengers, aimed to give good connections. If memory serves (and you can check on the airport departures board to be sure!), there's one in the morning and another in the evening.\nFor inbound flights, the origins are divided into two groups, \"safe\" and \"might not always be safe in all cases\". That distinction is based not only on the country they're coming from, but also on whether or not the origin airport allows certain kinds of transfers or if all transfers require full re-screening.\nIf your flight arrives from a \"safe\" airport, and you don't have a bus-gate (so jetty disembarking), you may (not always) be permitted as a transit passenger to just go straight out into the departures area. (In this situation, transit is straight on, arrivals follow the normal signs up the stairs). If you have such an arrival, you can get from your seat on the plane to the terminal in a couple of minutes, and hence from one seat to another in under 10 minutes if everything lines up!\nOtherwise, you need to head upstairs, then walk along to the central transit re-screening areas in the middle of the terminal. If you're unlucky with gates, or have to be bussed to the terminal first, this can take a little while. If you get there at a quiet time (so not during a big arrivals wave), you could be through in a minute or two. If you're last off an A380 when a few other widebodys have just landed, in the middle of a big arrivals wave, you could be in for a long wait... (Hence photos you've seen)\nI've had a 90 minute connection, with lucky gates and a \"safe\" origin airport, where I've had time to have a shower (in the business class lounge) plus breakfast before boarding. I've also had a ~2 hour connection, with bus gates both ends, and long security queues, where I didn't have time to visit the lounge at all, just run to the gate to board. It varies!\nHowever, if you hold a through ticket, I wouldn't worry too much. For very tight connections, they should give you a pass (envelope?) to go through an (often) shorted tight connections queue. Otherwise, I've been on a few long-haul flights which were held for 10 minutes to await late-running connecting passengers. Very worst case, Qatar Airways would rebook you for free on a later flight, and provide a hotel if it was a very long wait. Not much fun if it's only a one-flight-a-day route though!\nFor a frequent route, booking the shortest connection the website will allow should be fine if you're an experienced traveller who walks fast and doesn't get flustered if things go wrong. Otherwise, 90-120 minutes might be a good minimum, especially if you want a chance to visit duty free / lounge / etc. You probably don't want much more than 2.5 hours there, unless you have lounge access or like shopping. Well, unless you can get closer to 6-7 hours, when the free tours of the city or the (hard-to-qualify-for) free stopover hotels start being an option!\n", "topic": "travel", "url": "https://travel.stackexchange.com/questions/88012"}, {"image": "VQAonline_00063879.png", "question": "Getting to Terelj National Park from Ulaanbaatar, Mongolia", "context": "I have one day left in Mongolia, and would love to get to Gorkhi-Terelj / Terelj National Park for the day. However, the only info I can find on getting there is on:\nhttp://wikitravel.org/en/Gorkhi-Terelj_National_Park\n\nWhich states there is a bus at 4pm there, and two buses back (8am and 7pm).\nThis seems odd as there MUST be a demand for day-trippers. Can anyone help out with info on alternate buses for getting there?\n", "answer": "There is a bus going from UB to Terelj.\nIf I remember correctly, it runs once every 2 hours? (at least several times a day). We got our info in one of the bars/cafes that is run by an expat. (I don't remember the name , it is a cafe that kinda stands out. lots of wood, it somewhat resembles a mountian hut. They organise tracks and stuff.) the place is called Chez Bernard Caf\u00e9\nThe bus leaves somewhere from the main street, and drops you off at the village at the south of Terelj Nation Park.\n", "topic": "travel", "url": "https://travel.stackexchange.com/questions/1559"}, {"image": "VQAonline_00064752.png", "question": "\u041d\u0435 \u0440\u043e\u0437\u043c\u043e\u0432\u043d\u0438\u0439 \u0432\u0456\u0434\u043f\u043e\u0432\u0456\u0434\u043d\u0438\u043a \u0434\u043e \u0441\u043b\u043e\u0432\u043e\u0441\u043f\u043e\u043b\u0443\u0447\u0435\u043d\u043d\u044f \"\u0432\u043f\u0430\u0434\u0430\u0442\u0438 \u0432 \u0434\u0438\u0442\u0438\u043d\u0441\u0442\u0432\u043e\"", "context": "\u041c\u043e\u0432\u0430 - \u0414\u041d\u041a \u043d\u0430\u0446\u0456\u0457 \n\n\u0422\u0430\u043a\u043e\u0436 \u043d\u0430 OnlineCorrector \u0432\u0438\u0440\u0430\u0437 \"\u0432\u043f\u0430\u0434\u0430\u0442\u0438 \u0432 \u0434\u0438\u0442\u0438\u043d\u0441\u0442\u0432\u043e\" \u0437\u0430\u043c\u0456\u043d\u0438\u0442\u0438 \u043d\u0430 \"\u0434\u0438\u0442\u0438\u043d\u0456\u0442\u0438\". \n\u0412 \u0421\u0423\u041c-20 \u0441\u043b\u043e\u0432\u043e \"\u0434\u0438\u0442\u0438\u043d\u0456\u0442\u0438\" \u0437 \u043f\u043e\u0437\u043d\u0430\u0447\u043a\u043e\u044e \u0440\u043e\u0437\u043c\u043e\u0432\u043d\u0435. \u042f\u043a\u0438\u0439 \u043d\u0435 \u0440\u043e\u0437\u043c\u043e\u0432\u043d\u0438\u0439 \u0432\u0456\u0434\u043f\u043e\u0432\u0456\u0434\u043d\u0438\u043a \u0434\u043e \u0441\u043b\u043e\u0432\u043e\u0441\u043f\u043e\u043b\u0443\u0447\u0435\u043d\u043d\u044f \"\u0432\u043f\u0430\u0434\u0430\u0442\u0438 \u0432 \u0434\u0438\u0442\u0438\u043d\u0441\u0442\u0432\u043e\"? \n", "answer": "\u041d\u0430 \u0442\u043e\u043c\u0443 \u0436 \u0441\u0430\u043c\u043e\u043c\u0443 \u0441\u0430\u0439\u0442\u0456 \u041c\u043e\u0432\u0430 \u0414\u041d\u041a \u043d\u0430\u0446\u0456\u0457 \u0434\u0430\u044e\u0442\u044c\u0441\u044f \u0432\u0430\u0440\u0456\u0430\u043d\u0442\u0438:\n\n\u0437\u0434\u0438\u0442\u0438\u043d\u044e\u0432\u0430\u0442\u0438\u0441\u044f, \u0437\u0434\u0438\u0442\u0438\u043d\u0438\u0442\u0438\u0441\u044f; \u043d\u0430 \u0434\u0438\u0442\u044f\u0447\u0438\u0439 \u0440\u043e\u0437\u0443\u043c \u0437\u0456\u0439\u0442\u0438 (\u043f\u0435\u0440\u0435\u0439\u0442\u0438);\n \u0437\u043c\u0430\u043b\u0438\u0442\u0438\u0441\u044f \u0434\u043e \u0434\u0438\u0442\u0438\u043d\u0438, (\u0456\u043d\u043e\u0434\u0456) \u0432\u0438\u0441\u0442\u0430\u0440\u0456\u0442\u0438 \u0440\u043e\u0437\u0443\u043c.\n\n\u0422\u0430\u043a\u0456 \u0436 \u0432\u0430\u0440\u0456\u0430\u043d\u0442\u0438 \u043f\u0435\u0440\u0435\u043a\u043b\u0430\u0434\u0443 \u043c\u043e\u0436\u043d\u0430 \u0437\u043d\u0430\u0439\u0442\u0438 \u0456 \u0432 \u0420\u043e\u0441\u0456\u0439\u0441\u044c\u043a\u043e-\u0443\u043a\u0440\u0430\u0457\u043d\u0441\u044c\u043a\u043e\u043c\u0443 \u0441\u043b\u043e\u0432\u043d\u0438\u043a\u0443 \u0441\u0442\u0430\u043b\u0438\u0445 \u0432\u0438\u0440\u0430\u0437\u0456\u0432.\n\u041a\u0440\u0456\u043c \u0442\u043e\u0433\u043e, \u0441\u043b\u043e\u0432\u043e \"\u0434\u0438\u0442\u0438\u043d\u0438\u0442\u0438\u0441\u044f\" (\u0442\u0435 \u0441\u0430\u043c\u0435, \u0449\u043e \u0439 \"\u0434\u0438\u0442\u0438\u043d\u0456\u0442\u0438\") \u043c\u043e\u0436\u0435 \u043c\u0430\u0442\u0438 \u0437\u043d\u0430\u0447\u0435\u043d\u043d\u044f: \"\u041f\u043e\u0432\u043e\u0434\u0438\u0442\u0438\u0441\u044f \u044f\u043a \u0434\u0438\u0442\u0438\u043d\u0430; \u043f\u0443\u0441\u0442\u0443\u0432\u0430\u0442\u0438\".\n\u0412 \u0446\u044c\u043e\u043c\u0443 \u0432\u0438\u043f\u0430\u0434\u043a\u0443 \u043d\u0430\u043c \u0434\u043e\u043f\u043e\u043c\u043e\u0436\u0435 \u0421\u043b\u043e\u0432\u043d\u0438\u043a \u0441\u0438\u043d\u043e\u043d\u0456\u043c\u0456\u0432, \u044f\u043a\u0438\u0439 \u043f\u0440\u043e\u043f\u043e\u043d\u0443\u0454 \u0442\u0430\u043a\u0456 \u0432\u0430\u0440\u0456\u0430\u043d\u0442\u0438:\n\n\u041f\u0423\u0421\u0422\u0423\u0412\u0410\u0301\u0422\u0418 (\u043f\u0435\u0440\u0435\u0432. \u043f\u0440\u043e \u0434\u0456\u0442\u0435\u0439 - \u0440\u043e\u0437\u0432\u0430\u0436\u0430\u044e\u0447\u0438\u0441\u044c, \u0431\u0430\u0432\u0438\u0442\u0438\u0441\u044f, \u0433\u0440\u0430\u0442\u0438\u0441\u044f), \u0411\u0410\u0301\u041b\u0423\u0412\u0410\u0422\u0418\u0421\u042f, \u0417\u0410\u0411\u0410\u0412\u041b\u042f\u0301\u0422\u0418\u0421\u042f, \u0411\u0415\u0428\u041a\u0415\u0422\u0423\u0412\u0410\u0301\u0422\u0418, \u0416\u0418\u0420\u0423\u0412\u0410\u0301\u0422\u0418 + \u0449\u0435 \u0440\u043e\u0437\u043c\u043e\u0432\u043d\u0456 \u0442\u0430 \u0434\u0456\u0430\u043b\u0435\u043a\u0442\u043d\u0456 \u0432\u0430\u0440\u0456\u0430\u043d\u0442\u0438 -\n \u0414\u0418\u0422\u0418\u0301\u041d\u0418\u0422\u0418\u0421\u042f, \u0414\u0418\u0422\u0418\u041d\u0406\u0301\u0422\u0418, \u041a\u0410\u0417\u0418\u0301\u0422\u0418\u0421\u042f, \u0414\u0423\u0420\u0406\u0422\u0418, \u0428\u0410\u041b\u0406\u0301\u0422\u0418, \u0413\u0410\u0420\u042e\u0412\u0410\u0301\u0422\u0418, \u0422\u041b\u0423\u041c\u0418\u0301\u0422\u0418\u0421\u042f,\n \u0417\u0411\u0418\u0422\u041a\u0423\u0412\u0410\u0301\u0422\u0418, \u0413\u0410\u0419\u0414\u0410\u0411\u0423\u0301\u0420\u0418\u0422\u0418, \u0412\u0415\u0421\u0415\u041b\u0418\u0422\u0418\u0421\u042f, \u0413\u0420\u0410\u0422\u0418\u0421\u042f.\n\n\u0422\u0430\u043a\u043e\u0436, \u0433\u0430\u0434\u0430\u044e, \u0449\u043e \u043c\u043e\u0436\u043d\u0430 \u0432\u0438\u043a\u043e\u0440\u0438\u0441\u0442\u0430\u0442\u0438 \u0442\u0430\u043a\u043e\u0436 \u0442\u043e\u0439 \u0436\u0435 \u0432\u0438\u0441\u043b\u0456\u0432 \u0456\u0437 \u0421\u0423\u041c\u0443 \"\u043f\u043e\u0432\u043e\u0434\u0438\u0442\u0438\u0441\u044f \u044f\u043a \u0434\u0438\u0442\u0438\u043d\u0430\".\n", "topic": "ukrainian", "url": "https://ukrainian.stackexchange.com/questions/5211"}, {"image": "VQAonline_00064751.png", "question": "\u0420\u043e\u0437\u0431\u0438\u0432\u0430\u0442\u0438 \u043f\u0430\u0440\u043a \u0447\u0438 \u0441\u0430\u0434\u0438\u0442\u0438 \u043f\u0430\u0440\u043a?", "context": "\u0412 \u0443\u043d\u0456\u0432\u0435\u0440\u0441\u0438\u0442\u0435\u0442\u0456 \u044f \u0432\u0447\u0438\u043b\u0430, \u0449\u043e \u043f\u0430\u0440\u043a \u0440\u043e\u0437\u0431\u0438\u0432\u0430\u044e\u0442\u044c \u0437\u0433\u0456\u0434\u043d\u043e \u0440\u043e\u0437\u0431\u0438\u0432\u043e\u0447\u043d\u043e\u0433\u043e \u043a\u0440\u0435\u0441\u043b\u0435\u043d\u043d\u044f.\n\u041f\u0440\u043e\u0442\u0435 \u043d\u0430 \u0441\u0430\u0439\u0442\u0456 \u041c\u043e\u0432\u0430 - \u0414\u041d\u041a \u043d\u0430\u0446\u0456\u0457 \u0432\u043a\u0430\u0437\u0430\u043d\u043e, \u0449\u043e \u043f\u0430\u0440\u043a \u043c\u043e\u0436\u043d\u0430 \u043b\u0438\u0448\u0435 \u043f\u043e\u0441\u0430\u0434\u0438\u0442\u0438.\n\n\u0404 \u0434\u0435\u043a\u0456\u043b\u044c\u043a\u0430 \u043a\u043e\u043c\u0435\u043d\u0442\u0430\u0440\u0456\u0432 \u043d\u0430 \u0437\u0430\u0445\u0438\u0441\u0442 \u0441\u043b\u043e\u0432\u043e\u0441\u043f\u043e\u043b\u0443\u0447\u0435\u043d\u043d\u044f \"\u0440\u043e\u0437\u0431\u0438\u0432\u0430\u0442\u0438 \u043f\u0430\u0440\u043a\".\n\u0412 \u0421\u0423\u041c-11 \u0437\u0430\u0437\u043d\u0430\u0447\u0435\u043d\u043e:\n\n\u0420\u043e\u0437\u0431\u0438\u0432\u0430\u0442\u0438 12. \u0420\u043e\u0437\u043c\u0456\u0440\u044f\u0442\u0438, \u0440\u043e\u0437\u043f\u043b\u0430\u043d\u043e\u0432\u0443\u0432\u0430\u0442\u0438 \u0434\u0456\u043b\u044f\u043d\u043a\u0438 \u0437\u0435\u043c\u043b\u0456. \u2014 \u041d\u0430 \u043d\u0430\u0448\u0443\n\u0434\u0443\u043c\u043a\u0443, \u043c\u043e\u0436\u043d\u0430 \u0431\u0443\u043b\u043e \u0431 \u0437\u0440\u043e\u0431\u0438\u0442\u0438 \u0442\u0430\u043a: \u0440\u043e\u0437\u0431\u0438\u0442\u0438 \u043e\u0446\u044e \u043f\u043b\u043e\u0449\u0443 \u043f\u0435\u0440\u0435\u0434 \u0420\u0430\u043a\u0456\u0432\u043a\u043e\u044e ..\u0437\u0430\n\u043f\u0440\u0438\u043d\u0446\u0438\u043f\u043e\u043c \u0448\u0430\u0445\u0456\u0432\u043d\u0438\u0446\u0456 (\u041c\u0438\u043a\u043e\u043b\u0430 \u0422\u0440\u0443\u0431\u043b\u0430\u0457\u043d\u0456, I, 1955, 67)\n// \u0421\u0430\u0434\u0438\u0442\u0438 \u0449\u043e-\u043d\u0435\u0431\u0443\u0434\u044c \u043d\u0430 \u043f\u0435\u0432\u043d\u0456\u0439 \u0434\u0456\u043b\u044f\u043d\u0446\u0456 \u0437\u0435\u043c\u043b\u0456 \u043f\u0456\u0441\u043b\u044f \u0440\u043e\u0437\u043f\u043b\u0430\u043d\u0443\u0432\u0430\u043d\u043d\u044f. \u041d\u0430\n\u043c\u0430\u0439\u0434\u0430\u043d\u0447\u0438\u043a\u0443 \u0434\u043b\u044f \u0432\u0456\u0434\u043f\u043e\u0447\u0438\u043d\u043a\u0443 \u0440\u043e\u0437\u0431\u0438\u0432\u0430\u044e\u0442\u044c \u043a\u0432\u0456\u0442\u043d\u0438\u043a\u0438 \u0437 \u043e\u0434\u043d\u043e\u0440\u0456\u0447\u043d\u0438\u0445 \u0456\n\u0431\u0430\u0433\u0430\u0442\u043e\u0440\u0456\u0447\u043d\u0438\u0445 \u043a\u0432\u0456\u0442\u0456\u0432 \u0442\u0430 \u043a\u0432\u0456\u0442\u0443\u0447\u0438\u0445 \u043a\u0443\u0449\u0456\u0432 \u2014 \u0441\u043f\u0456\u0440\u0435\u0457, \u0436\u0430\u0441\u043c\u0456\u043d\u0443 \u0442\u043e\u0449\u043e (\u0425\u043b\u0456\u0431\u043e\u0440\u043e\u0431\n\u0423\u043a\u0440\u0430\u0457\u043d\u0438, 9, 1964, 24); \u2014 \u0412\u0438\u0440\u0456\u0437\u0443\u0454\u043c\u043e \u0437 \u043f\u043e\u043b\u044f \u0448\u043c\u0430\u0442\u043e\u043a \u0437\u0435\u043c\u043b\u0456 \u0437\u0430\u0434\u043b\u044f \u0434\u0432\u043e\u0440\u0443,\n\u0433\u0440\u043e\u043c\u0430\u0434\u0441\u044c\u043a\u0438\u043c \u0437\u0430\u0445\u043e\u0434\u043e\u043c \u0437\u0431\u0443\u0434\u0443\u0454\u043c\u043e \u0431\u0443\u0434\u0438\u043d\u043e\u0447\u043e\u043a, \u0440\u043e\u0437\u0456\u0431'\u0454\u043c\u043e \u0441\u0430\u0434\u043e\u043a... (\u041f\u0430\u043d\u0430\u0441\n\u041c\u0438\u0440\u043d\u0438\u0439, IV, 1955, 327).\n\n\u041e\u0442\u0436\u0435, \u044f\u043a \u043f\u0440\u0430\u0432\u0438\u043b\u044c\u043d\u043e \u0433\u043e\u0432\u043e\u0440\u0438\u0442\u0438 \u0442\u0430 \u043f\u0438\u0441\u0430\u0442\u0438: \"\u0440\u043e\u0437\u0431\u0438\u0432\u0430\u0442\u0438\" \u0447\u0438 \"\u0441\u0430\u0434\u0438\u0442\u0438\" \u043f\u0430\u0440\u043a?\n", "answer": "\u0417\u0430\u0433\u043b\u044f\u043d\u0435\u043c\u043e \u043d\u0430 \u0443\u0440\u043e\u043a\u0438 \u0434\u0435\u0440\u0436\u0430\u0432\u043d\u043e\u0457 \u043c\u043e\u0432\u0438 \u043f\u0440\u043e\u0444\u0435\u0441\u043e\u0440\u0430 \u041f\u043e\u043d\u043e\u043c\u0430\u0440\u0435\u0432\u0430:\n\n\u0412 \u0443\u0441\u043d\u043e\u043c\u0443 \u0439 \u043f\u0438\u0441\u0435\u043c\u043d\u043e\u043c\u0443 \u043c\u043e\u0432\u043b\u0435\u043d\u043d\u0456 \u043d\u0435\u0440\u0456\u0434\u043a\u043e \u0447\u0443\u0454\u043c\u043e \u0439 \u0447\u0438\u0442\u0430\u0454\u043c\u043e \u043a\u0443\u043c\u0435\u0434\u043d\u0456 \u0437\n \u043f\u043e\u0433\u043b\u044f\u0434\u0443 \u043b\u0435\u043a\u0441\u0438\u043a\u043e-\u0444\u0440\u0430\u0437\u0435\u043e\u043b\u043e\u0433\u0456\u0447\u043d\u0438\u0445 \u043d\u043e\u0440\u043c \u0443\u043a\u0440\u0430\u0457\u043d\u0441\u044c\u043a\u043e\u0457 \u043c\u043e\u0432\u0438 \u0432\u0438\u0441\u043b\u043e\u0432\u0438: \u00ab\u041d\u0430\n \u043c\u0456\u0441\u0446\u0456 \u0437\u0432\u0430\u043b\u0438\u0449\u0430 \u043c\u0435\u0448\u043a\u0430\u043d\u0446\u0456 \u043c\u043e\u043b\u043e\u0434\u043e\u0433\u043e \u043c\u0456\u0441\u0442\u0430 \u0440\u043e\u0437\u0431\u0438\u043b\u0438 \u043f\u0430\u0440\u043a; \u0421\u0442\u0440\u0430\u0439\u043a\u0430\u0440\u0456 \u0440\u043e\u0437\u0431\u0438\u043b\u0438\n \u043d\u0430\u043c\u0435\u0442\u0438 \u043f\u0435\u0440\u0435\u0434 \u043c\u0456\u0441\u044c\u043a\u0440\u0430\u0434\u043e\u044e\u00bb \u0442\u0430 \u0456\u043d. \u0414\u0456\u0454\u0441\u043b\u043e\u0432\u043e \u0440\u043e\u0437\u0431\u0438\u0442\u0438 \u0454 \u0441\u0438\u043d\u043e\u043d\u0456\u043c\u043e\u043c \u0441\u043b\u0456\u0432\n \u0437\u043d\u0438\u0449\u0438\u0442\u0438, \u0440\u043e\u0437\u0442\u0440\u043e\u0449\u0438\u0442\u0438, \u0440\u043e\u0437\u2019\u044e\u0448\u0438\u0442\u0438 \u0442\u043e\u0449\u043e. \u0412\u043e\u043d\u043e \u0432\u0445\u043e\u0434\u0438\u0442\u044c \u0434\u043e \u0442\u0430\u043a\u0438\u0445\n \u0441\u043b\u043e\u0432\u043e\u0441\u043f\u043e\u043b\u0443\u0447\u0435\u043d\u044c, \u044f\u043a \u0440\u043e\u0437\u0431\u0438\u0442\u0438 \u043b\u043e\u0431\u0430, \u0440\u043e\u0437\u0431\u0438\u0442\u0438 (\u0440\u043e\u0437\u0456\u0440\u0432\u0430\u0442\u0438) \u043a\u0430\u0439\u0434\u0430\u043d\u0438, \u0440\u043e\u0437\u0431\u0438\u0442\u0438\n \u0441\u0435\u0440\u0446\u0435 \u0456 \u0442. \u0456\u043d. \u0429\u043e\u0434\u043e \u043f\u0430\u0440\u043a\u0443 \u0430\u0431\u043e \u0441\u0430\u0434\u0443 \u0442\u0440\u0435\u0431\u0430 \u0432\u0438\u043a\u043e\u0440\u0438\u0441\u0442\u043e\u0432\u0443\u0432\u0430\u0442\u0438 \u0431\u0456\u043b\u044c\u0448 \u043f\u0456\u0434\u0445\u043e\u0436\u0456\n \u043b\u0435\u043a\u0441\u0435\u043c\u0438 \u2014 \u043f\u043e\u0441\u0430\u0434\u0438\u0442\u0438, \u043d\u0430\u0441\u0430\u0434\u0438\u0442\u0438, \u0432\u0438\u0440\u043e\u0441\u0442\u0438\u0442\u0438: \u00ab\u042f \u0433\u043e\u0441\u0442\u0440\u043e \u0432\u0456\u0434\u0447\u0443\u0432\u0430\u044e \u0437\u0430\u043f\u0430\u0445\n \u0441\u0430\u0434\u0443, \u0449\u043e \u044f \u043f\u043e\u0441\u0430\u0434\u0438\u0432\u00bb (\u042e. \u042f\u043d\u043e\u0432\u0441\u044c\u043a\u0438\u0439). \u041d\u0430\u043c\u0435\u0442\u0438 \u0442\u0435\u0436 \u043d\u0435 \u0440\u043e\u0437\u0431\u0438\u0432\u0430\u044e\u0442\u044c, \u0430\n \u0441\u0442\u0430\u0432\u043b\u044f\u0442\u044c \u0430\u0431\u043e \u043d\u0430\u043f\u0438\u043d\u0430\u044e\u0442\u044c (\u044f\u043a \u0443 \u043f\u0440\u044f\u043c\u043e\u043c\u0443, \u0442\u0430\u043a \u0456 \u0432 \u043f\u0435\u0440\u0435\u043d\u043e\u0441\u043d\u043e\u043c\u0443 \u0437\u043d\u0430\u0447\u0435\u043d\u043d\u0456):\n \u00ab\u041d\u0430\u0434 \u043f\u043e\u043a\u0440\u0456\u0432\u043b\u044f\u043c\u0438 \u0431\u0443\u0434\u0438\u043d\u043a\u0456\u0432 \u043e\u0441\u0456\u043d\u043d\u0456 \u0445\u043c\u0430\u0440\u0438 \u043d\u0430\u043f\u2019\u044f\u043b\u0438 \u0441\u0438\u0437\u0438\u0439 \u043d\u0430\u043c\u0435\u0442\u00bb (\u041e.\n \u041a\u043e\u043f\u0438\u043b\u0435\u043d\u043a\u043e).\n\n\u041f\u0440\u043e \u0446\u0435 \u043f\u0438\u0448\u0435 \u0456 \u043d\u0430 \u0446\u044c\u043e\u043c\u0443 \u0441\u0430\u0439\u0442\u0456 \u0456 \u043d\u0430 \u0421\u043b\u043e\u0432\u043f\u0435\u0434\u0456\u0457 (\u0456\u043d\u0444\u043e\u0440\u043c\u0430\u0446\u0456\u044f \u0437 \u0433\u0430\u0437\u0435\u0442\u0438 \u0425\u0440\u0435\u0449\u0430\u0442\u0438\u043a). \u0423 \u0442\u0456\u0439 \u0436\u0435 \u0433\u0430\u0437\u0435\u0442\u0456 \u0425\u0440\u0435\u0449\u0430\u0442\u0438\u043a \u0447\u0438\u0442\u0430\u0454\u043c\u043e \u043f\u043e\u044f\u0441\u043d\u0435\u043d\u043d\u044f:\n\n\u0412\u0435\u043b\u0438\u0447\u0435\u0437\u043d\u0430 \u043a\u0456\u043b\u044c\u043a\u0456\u0441\u0442\u044c \u0442\u0430\u043a\u0438\u0445 \u043f\u043e\u043c\u0438\u043b\u043e\u043a \u043f\u043e\u044f\u0441\u043d\u044e\u0454\u0442\u044c\u0441\u044f \u0442\u0438\u043c, \u0449\u043e \u0442\u0440\u0438\u0432\u0430\u043b\u0438\u0439 \u0447\u0430\u0441\n \u0441\u0435\u0440\u0435\u0434 \u0431\u0430\u0433\u0430\u0442\u044c\u043e\u0445 \u0443\u043a\u0440\u0430\u0457\u043d\u0446\u0456\u0432 \u0443\u043a\u0440\u0430\u0457\u043d\u0441\u044c\u043a\u0430 \u043c\u043e\u0432\u0430 \u043d\u0435 \u0432\u0438\u043a\u043e\u043d\u0443\u0432\u0430\u043b\u0430 \u0444\u0443\u043d\u043a\u0446\u0456\u0457 \u0440\u0456\u0434\u043d\u043e\u0457\n \u043c\u043e\u0432\u0438, \u044f\u043a\u043e\u044e \u0434\u0443\u043c\u0430\u044e\u0442\u044c, \u0441\u043f\u0456\u043b\u043a\u0443\u044e\u0442\u044c\u0441\u044f \u0432 \u0443\u0441\u0456\u0445 \u0441\u0444\u0435\u0440\u0430\u0445 \u0436\u0438\u0442\u0442\u044f, \u043c\u0440\u0456\u044e\u0442\u044c \u0456 \u0441\u043d\u044f\u0442\u044c,\n \u0440\u0430\u0434\u0456\u044e\u0442\u044c \u0456 \u0441\u0443\u043c\u0443\u044e\u0442\u044c. \u0412\u043e\u043d\u0430 \u0431\u0443\u043b\u0430 \u043d\u0435\u043f\u043e\u0442\u0440\u0456\u0431\u043d\u0438\u043c \u0434\u043e\u0434\u0430\u0442\u043a\u043e\u043c \u0434\u043e \u0456\u043d\u0448\u043e\u0457,\n \u043f\u0440\u0435\u0441\u0442\u0438\u0436\u043d\u0456\u0448\u043e\u0457 \u043c\u043e\u0432\u0438. \u0423 \u0440\u0430\u043c\u043a\u0430\u0445 \u043f\u043e\u043b\u0456\u0442\u0438\u043a\u0438 \u0437\u0431\u043b\u0438\u0436\u0435\u043d\u043d\u044f \u043d\u0430\u0446\u0456\u0439 \u0442\u0430 \u0437\u043b\u0438\u0442\u0442\u044f \u043c\u043e\u0432, \u0449\u043e\n \u0434\u043e\u0441\u044f\u0433\u043b\u0430 \u0430\u043f\u043e\u0433\u0435\u044e \u043f\u0456\u0434 \u0447\u0430\u0441 \u0437\u0430\u0441\u0442\u043e\u044e 70\u201380-\u0445, \u0441\u043b\u043e\u0432\u0430, \u043d\u0435 \u0441\u0445\u043e\u0436\u0456 \u043d\u0430 \u0440\u043e\u0441\u0456\u0439\u0441\u044c\u043a\u0456,\n \u0432\u0438\u043a\u0438\u0434\u0430\u043b\u0438 \u0437 \u043d\u0430\u0448\u043e\u0457 \u043c\u043e\u0432\u0438 \u0430\u0431\u043e \u043e\u0433\u043e\u043b\u043e\u0448\u0443\u0432\u0430\u043b\u0438 \u0434\u0456\u0430\u043b\u0435\u043a\u0442\u043d\u0438\u043c\u0438, \u0437\u0430\u0441\u0442\u0430\u0440\u0456\u043b\u0438\u043c\u0438,\n \u0448\u0442\u0443\u0447\u043d\u0438\u043c\u0438, \u043d\u0435 \u0437\u0440\u043e\u0437\u0443\u043c\u0456\u043b\u0438\u043c\u0438 \u043d\u0430\u0440\u043e\u0434\u043e\u0432\u0456. \u0417 \u0430\u043a\u0430\u0434\u0435\u043c\u0456\u0447\u043d\u043e\u0433\u043e \u0406\u043d\u0441\u0442\u0438\u0442\u0443\u0442\u0443\n \u043c\u043e\u0432\u043e\u0437\u043d\u0430\u0432\u0441\u0442\u0432\u0430 \u0434\u043e \u0432\u0438\u0434\u0430\u0432\u043d\u0438\u0446\u0442\u0432 \u0456 \u0440\u0435\u0434\u0430\u043a\u0446\u0456\u0439 \u043d\u0430\u0434\u0445\u043e\u0434\u0438\u043b\u0438 \u0441\u043f\u0438\u0441\u043a\u0438 \u0437\u0430\u0431\u043e\u0440\u043e\u043d\u0435\u043d\u0438\u0445\n \u0441\u043b\u0456\u0432 \u0442\u0430 \u0432\u0438\u0441\u043b\u043e\u0432\u0456\u0432 (\u043d\u0430 \u0437\u0440\u0430\u0437\u043e\u043a \u043b\u0438\u0431\u043e\u043d\u044c, \u0440\u043e\u0431\u0456\u0442\u043d\u044f, \u043a\u0440\u0430\u0441\u043d\u0435 \u043f\u0438\u0441\u044c\u043c\u0435\u043d\u0441\u0442\u0432\u043e), \u044f\u043a\u0456\n \u043f\u0440\u0438 \u0440\u0435\u0434\u0430\u0433\u0443\u0432\u0430\u043d\u043d\u0456 \u043d\u0430\u043a\u0430\u0437\u0443\u0432\u0430\u043b\u043e\u0441\u044f \u0432\u0438\u043b\u0443\u0447\u0430\u0442\u0438 \u0437 \u043e\u0440\u0438\u0433\u0456\u043d\u0430\u043b\u044c\u043d\u0438\u0445 \u0442\u0430 \u043f\u0435\u0440\u0435\u043a\u043b\u0430\u0434\u043d\u0438\u0445\n \u0442\u0432\u043e\u0440\u0456\u0432.\n\u041c\u0430\u0454 \u0440\u0430\u0446\u0456\u044e \u041e\u043b\u0435\u043a\u0441\u0430\u043d\u0434\u0440 \u041f\u043e\u043d\u043e\u043c\u0430\u0440\u0456\u0432: \u0442\u0440\u0438\u0432\u0430\u043b\u0435 \u0437\u0440\u043e\u0441\u0456\u0439\u0449\u0435\u043d\u043d\u044f \u0443\u043a\u0440\u0430\u0457\u043d\u0446\u0456\u0432 \u043f\u0440\u0438\u0437\u0432\u0435\u043b\u043e\n \u0434\u043e \u0432\u0442\u0440\u0430\u0442\u0438 \u0434\u043e\u0440\u043e\u0433\u043e\u0432\u043a\u0430\u0437\u0456\u0432 \u0443 \u043a\u043e\u0440\u0438\u0441\u0442\u0443\u0432\u0430\u043d\u043d\u0456 \u0440\u0456\u0434\u043d\u043e\u044e \u043c\u043e\u0432\u043e\u044e. \u0414\u043e\u0441\u0456 \u0431\u0430\u0433\u0430\u0442\u043e \u0445\u0442\u043e \u0437\n \u043c\u043e\u0432\u0446\u0456\u0432 \u0437\u0430\u043b\u044e\u0431\u043a\u0438 \u0432\u0436\u0438\u0432\u0430\u0454 \u0441\u043b\u043e\u0432\u0430 \u0431\u043b\u0430\u0433\u043e\u043f\u043e\u043b\u0443\u0447\u0447\u044f, \u0432\u0456\u0434\u043f\u0440\u0430\u0432\u043b\u044f\u0442\u0438\u0441\u044f, \u0434\u0456\u044e\u0447\u0438\u0439,\n \u0434\u043e\u0431\u0440\u043e\u0441\u043e\u0432\u0456\u0441\u043d\u0438\u0439, \u0435\u043a\u043e\u043d\u043e\u043c\u0438\u0442\u0438, \u0437\u043d\u0430\u0447\u0438\u043c\u0438\u0439, \u043a\u043e\u0440\u0437\u0438\u043d\u0430, \u043d\u0430\u0441\u0442\u0456\u0439\u043b\u0438\u0432\u0438\u0439, \u043f\u0440\u0438\u0439\u043e\u043c\u043d\u0430,\n \u043f\u0440\u043e\u0442\u0438\u0440\u0456\u0447\u0447\u044f, \u043f\u043e\u0447\u0430\u0442\u043a\u0443\u044e\u0447\u0438\u0439 \u0456 \u043c\u0430\u0439\u0436\u0435 \u0446\u0456\u043b\u043a\u043e\u0432\u0438\u0442\u043e \u0456\u0433\u043d\u043e\u0440\u0443\u0454 \u0434\u0430\u043b\u0435\u043a\u043e \u043a\u0440\u0430\u0449\u0456\n \u0441\u0438\u043d\u043e\u043d\u0456\u043c\u0456\u0447\u043d\u0456 \u0432\u0456\u0434\u043f\u043e\u0432\u0456\u0434\u043d\u0438\u043a\u0438 \u0434\u043e\u0431\u0440\u043e\u0431\u0443\u0442, \u0432\u0438\u0440\u0443\u0448\u0430\u0442\u0438, \u0434\u0456\u0439\u043e\u0432\u0438\u0439, \u0441\u0443\u043c\u043b\u0456\u043d\u043d\u0438\u0439,\n \u0437\u0430\u043e\u0449\u0430\u0434\u0436\u0443\u0432\u0430\u0442\u0438, \u0437\u043d\u0430\u0447\u0443\u0449\u0438\u0439, \u043a\u043e\u0448\u0438\u043a, \u043d\u0430\u043f\u043e\u043b\u0435\u0433\u043b\u0438\u0432\u0438\u0439, \u043f\u0440\u0438\u0439\u043c\u0430\u043b\u044c\u043d\u044f, \u0441\u0443\u043f\u0435\u0440\u0435\u0447\u043d\u0456\u0441\u0442\u044c,\n \u043f\u043e\u0447\u0430\u0442\u043a\u0456\u0432\u0435\u0446\u044c.\n\u0412\u0456\u0434\u043e\u043c\u043e, \u0449\u043e \u043a\u0430\u043b\u044c\u043a\u0438 \u043c\u0430\u044e\u0442\u044c \u0432\u0438\u043a\u043e\u0440\u0438\u0441\u0442\u043e\u0432\u0443\u0432\u0430\u0442\u0438\u0441\u044f \u043b\u0438\u0448\u0435 \u0442\u043e\u0434\u0456, \u043a\u043e\u043b\u0438 \u0443 \u0441\u0432\u043e\u0457\u0439 \u043c\u043e\u0432\u0456\n \u043d\u0435\u043c\u0430\u0454 \u043f\u043e\u0442\u0440\u0456\u0431\u043d\u0438\u0445 \u043b\u0435\u043a\u0441\u0435\u043c \u0434\u043b\u044f \u043f\u043e\u0437\u043d\u0430\u0447\u0435\u043d\u043d\u044f \u0442\u043e\u0433\u043e \u0430\u0431\u043e \u0442\u043e\u0433\u043e \u043f\u0440\u0435\u0434\u043c\u0435\u0442\u0430, \u0456\u0441\u0442\u043e\u0442\u0438,\n \u044f\u0432\u0438\u0449\u0430. \u0410\u043b\u0435 \u0446\u0456 \u043a\u0430\u043b\u044c\u043a\u0438, \u044f\u043a\u0438\u0445 \u201c\u043f\u043e\u0431\u0443\u0442\u0443\u0454\u201d \u0443 \u043d\u0430\u0441 \u0441\u0438\u043b\u0430-\u0441\u0438\u043b\u0435\u043d\u043d\u0430 (\u0456 \u044f\u0432\u043d\u0438\u0445, \u0456\n \u043f\u0440\u0438\u043c\u0430\u0441\u043a\u043e\u0432\u0430\u043d\u0438\u0445 \u0440\u043e\u0441\u0456\u044f\u043d\u0456\u0437\u043c\u0456\u0432, \u0437 \u043d\u0435 \u0432\u043b\u0430\u0441\u0442\u0438\u0432\u0438\u043c\u0438 \u0434\u043b\u044f \u0443\u043a\u0440\u0430\u0457\u043d\u0441\u044c\u043a\u043e\u0457 \u043c\u043e\u0432\u0438\n \u0447\u0443\u0436\u043e\u043c\u043e\u0432\u043d\u0438\u043c\u0438 \u0437\u043d\u0430\u0447\u0435\u043d\u043d\u044f\u043c\u0438), \u044f\u043a \u0437\u0432\u0438\u0447\u0430\u0439\u043d\u0456\u0441\u0456\u043d\u044c\u043a\u0456 \u0434\u0443\u0431\u043b\u0435\u0442\u0438 \u043d\u0430\u0448\u0438\u0445 \u043f\u0438\u0442\u043e\u043c\u0438\u0445 \u0441\u043b\u0456\u0432\n \u043d\u0435 \u0442\u0456\u043b\u044c\u043a\u0438 \u0432\u0456\u0434\u0442\u0456\u0441\u043d\u044f\u044e\u0442\u044c \u0457\u0445 \u043d\u0430 \u0434\u0440\u0443\u0433\u0438\u0439 \u043f\u043b\u0430\u043d, \u0430 \u0439 \u0443\u0437\u0430\u0433\u0430\u043b\u0456 \u0432\u0438\u0442\u0438\u0441\u043a\u0443\u044e\u0442\u044c \u0456\u0437\n \u0437\u0430\u0433\u0430\u043b\u044c\u043d\u043e\u0433\u043e \u0432\u0436\u0438\u0442\u043a\u0443\n\n\u0417\u043d\u043e\u0432\u0443 \u043f\u043e\u0432\u0435\u0440\u0442\u0430\u0454\u043c\u043e\u0441\u044f \u0434\u043e \u0456\u0441\u0442\u043e\u0440\u0456\u0457. \u0421\u0423\u041c-11 \u0431\u0443\u043b\u043e \u0432\u0438\u0434\u0430\u043d\u043e \u0443 1970\u20131980, \u0442\u043e\u043c\u0443 \u043c\u043e\u0436\u043b\u0438\u0432\u043e, \u044f\u043a\u0456\u0441\u044c \u0441\u043a\u0430\u043b\u044c\u043a\u043e\u0432\u0430\u043d\u0456 \u0432\u0438\u0441\u043b\u043e\u0432\u0438 \u0442\u0443\u0434\u0438 \u0456 \u043f\u043e\u0442\u0440\u0430\u043f\u0438\u043b\u0438 (\u043a\u0440\u0456\u043c \u0442\u043e\u0433\u043e, \u044f\u043a\u0449\u043e \u0434\u0438\u0432\u0438\u0442\u0438\u0441\u044f \u043d\u0430 \u043f\u0440\u0438\u043a\u043b\u0430\u0434\u0438, \u044f\u043a\u0456 \u043f\u043e\u0434\u0430\u044e\u0442\u044c\u0441\u044f \u0432 \u0421\u0423\u041c\u0456, \u0442\u043e \u043c\u043e\u0436\u043d\u0430 \u043f\u043e\u0431\u0430\u0447\u0438\u0442\u0438, \u0449\u043e \u0432\u0441\u0456 \u0432\u043e\u043d\u0438 \u0432\u0437\u044f\u0442\u0456 \u0456\u0437 \u0442\u0432\u043e\u0440\u0456\u0432, \u044f\u043a\u0456 \u0432\u0438\u0445\u043e\u0434\u0438\u043b\u0438 \u043a\u043e\u043b\u0438 \u0423\u043a\u0440\u0430\u0457\u043d\u0430 \u0437\u043d\u0430\u0445\u043e\u0434\u0438\u043b\u0430\u0441\u044f \u0443 \u0441\u043a\u043b\u0430\u0434\u0456 \u0421\u0420\u0421\u0420, \u0430 \u0440\u043e\u0441\u0456\u0439\u0441\u044c\u043a\u0430 \u043c\u043e\u0432\u0430 \u0431\u0443\u043b\u0430 \u0434\u043e\u043c\u0456\u043d\u0430\u043d\u0442\u043d\u043e\u044e, \u0442\u043e\u043c\u0443 \u043c\u043e\u0436\u043b\u0438\u0432\u043e \u0434\u0435\u044f\u043a\u0456 \u043f\u0438\u0441\u044c\u043c\u0435\u043d\u043d\u0438\u043a\u0438 \u0456 \u0432\u0438\u043a\u043e\u0440\u0438\u0441\u0442\u043e\u0432\u0443\u0432\u0430\u043b\u0438 \u043a\u0430\u043b\u044c\u043a\u0443).\n\u0422\u0430\u043a\u043e\u0436 \u043d\u0430 \u0446\u044c\u043e\u043c\u0443 \u0444\u043e\u0440\u0443\u043c\u0456 \u0437\u0430\u043f\u0435\u0440\u0435\u0447\u0443\u044e\u0442\u044c \u043f\u0440\u0430\u0432\u0438\u043b\u044c\u043d\u0456\u0441\u0442\u044c \u0432\u0438\u0441\u043b\u043e\u0432\u0443 \"\u0440\u043e\u0437\u0431\u0438\u0442\u0438 \u043f\u0430\u0440\u043a\".\n\u042f\u043a\u0438\u0439 \u0432\u0438\u0441\u043d\u043e\u0432\u043e\u043a \u043c\u043e\u0436\u0435\u043c\u043e \u0437\u0440\u043e\u0431\u0438\u0442\u0438?\n\u0413\u0430\u0434\u0430\u044e, \u0449\u043e \u043a\u0430\u0437\u0430\u0442\u0438 \u0447\u0438 \u043f\u0438\u0441\u0430\u0442\u0438 \"\u0440\u043e\u0437\u0431\u0438\u0442\u0438 \u043f\u0430\u0440\u043a\" \u043c\u043e\u0436\u043d\u0430, \u0431\u043e \u0432\u0441\u0435 \u0442\u0430\u043a\u0438 \u043c\u0438 \u0437\u043d\u0430\u0445\u043e\u0434\u0438\u043c\u043e \u0442\u0430\u043a\u0438\u0439 \u0432\u0430\u0440\u0456\u0430\u043d\u0442 \u0432 \u0421\u0423\u041c11, \u0430 \u0446\u0435 \u0454 \u043e\u0444\u0456\u0446\u0456\u0439\u043d\u0435 \u0434\u0436\u0435\u0440\u0435\u043b\u043e. \u041f\u0440\u043e\u0442\u0435, \u043f\u043e\u0442\u0440\u0456\u0431\u043d\u043e \u0432\u0440\u0430\u0445\u0443\u0432\u0430\u0442\u0438, \u0449\u043e \u043d\u0430\u0432\u0456\u0442\u044c \u0442\u0430\u043c \u043c\u043e\u0436\u0443\u0442\u044c \u0431\u0443\u0442\u0438 \u043f\u043e\u043c\u0438\u043b\u043a\u0438 (\u0442\u0430\u043a \u0441\u0430\u043c\u043e \u044f\u043a \u0456 \u043c\u043e\u0436\u0435 \u043f\u043e\u043c\u0438\u043b\u044f\u0442\u0438\u0441\u044f \u043f\u0440\u043e\u0444\u0435\u0441\u043e\u0440 \u041f\u043e\u043d\u043e\u043c\u0430\u0440\u0456\u0432), \u0430 \u0442\u043e\u043c\u0443 \u0449\u043e\u0431 \u0442\u043e\u0447\u043d\u043e \u0443\u043d\u0438\u043a\u043d\u0443\u0442\u0438 \u043a\u0430\u043b\u044c\u043a\u0438 \u043c\u043e\u0436\u043d\u0430 \u0432\u0438\u043a\u043e\u0440\u0438\u0441\u0442\u043e\u0432\u0443\u0432\u0430\u0442\u0438 \"\u043f\u043e\u0441\u0430\u0434\u0438\u0442\u0438/\u043d\u0430\u0441\u0430\u0434\u0438\u0442\u0438/\u0432\u0438\u0440\u043e\u0441\u0442\u0438\u0442\u0438 \u043f\u0430\u0440\u043a\", \u0430\u0431\u043e \u0436 \u043c\u043e\u0436\u0435 \u043d\u0430\u0432\u0456\u0442\u044c \"\u0437\u0430\u043a\u043b\u0430\u0441\u0442\u0438 \u043f\u0430\u0440\u043a\".\n", "topic": "ukrainian", "url": "https://ukrainian.stackexchange.com/questions/5169"}, {"image": "VQAonline_00064757.png", "question": "\u0424\u0456\u043d\u0430\u043d\u0441\u043e\u0432\u0430 \u0437\u0432\u0456\u0442\u043d\u0456\u0441\u0442\u044c \u0430\u0443\u0434\u0438\u0442\u043e\u0432\u0430\u043d\u0430 \u0447\u0438 \u0430\u0443\u0434\u0438(\u0456)\u0439\u043e\u0432\u0430\u043d\u0430?", "context": "\u041d\u0456 \u0417\u0430\u043a\u043e\u043d \u0423\u043a\u0440\u0430\u0457\u043d\u0438 \"\u041f\u0440\u043e \u0430\u0443\u0434\u0438\u0442 \u0444\u0456\u043d\u0430\u043d\u0441\u043e\u0432\u043e\u0457 \u0437\u0432\u0456\u0442\u043d\u043e\u0441\u0442\u0456 \u0442\u0430 \u0430\u0443\u0434\u0438\u0442\u043e\u0440\u0441\u044c\u043a\u0443 \u0434\u0456\u044f\u043b\u044c\u043d\u0456\u0441\u0442\u044c\", \u043d\u0456 \u0439\u043e\u0433\u043e \u043f\u043e\u043f\u0435\u0440\u0435\u0434\u043d\u0438\u043a (\u0417\u0423 \"\u041f\u0440\u043e \u0430\u0443\u0434\u0438\u0442\u043e\u0440\u0441\u044c\u043a\u0443 \u0434\u0456\u044f\u043b\u044c\u043d\u0456\u0441\u0442\u044c\") \u043d\u0435 \u043c\u0456\u0441\u0442\u044f\u0442\u044c \u0434\u0456\u0454\u043f\u0440\u0438\u043a\u043c\u0435\u0442\u043d\u0438\u043a\u0456\u0432, \u043f\u043e\u0445\u0456\u0434\u043d\u0438\u0445 \u0432\u0456\u0434 \u0442\u0435\u0440\u043c\u0456\u043d\u0443 \u0430\u0443\u0434\u0438\u0442.\n\u0421\u043c\u0456\u0448\u043d\u043e, \u0430\u043b\u0435 \u0432\u0456\u0434\u0441\u0443\u0442\u043d\u0456\u0439 \u043d\u0430\u0432\u0456\u0442\u044c \u0441\u0430\u043c \u0446\u0435\u0439 \u0456\u043c\u0435\u043d\u043d\u0438\u043a \u0456 \u0432 \u0421\u0423\u041c-20, \u0456 \u0432 \u0421\u0423\u041c, \u044f\u043a\u0438\u0439 \u043c\u0456\u0441\u0442\u0438\u0442\u044c \u0442\u0456\u043b\u044c\u043a\u0438 \u0456\u043c\u0435\u043d\u043d\u0438\u043a \u0430\u0443\u0434\u0438\u0442\u043e\u0440, \u0442\u0430 \u0439 \u0442\u043e \u0434\u0430\u043b\u0435\u043a\u043e \u043d\u0435 \u0437\u0456 \u0441\u0443\u0447\u0430\u0441\u043d\u0438\u043c \u0437\u043d\u0430\u0447\u0435\u043d\u043d\u044f\u043c:\n\n\u0410\u0423\u0414\u0418\u0301\u0422\u041e\u0420, \u0410\u0412\u0414\u0418\u0301\u0422\u041e\u0420, \u0430, \u0447\u043e\u043b., \u0437\u0430\u0441\u0442.\n\n\u0423 \u0434\u043e\u0440\u0435\u0444\u043e\u0440\u043c\u0435\u043d\u0438\u0445 \u0441\u0435\u043c\u0456\u043d\u0430\u0440\u0456\u044f\u0445 \u2014 \u0443\u0447\u0435\u043d\u044c, \u043f\u0440\u0438\u0437\u043d\u0430\u0447\u0435\u043d\u0438\u0439 \u0443\u0447\u0438\u0442\u0435\u043b\u0435\u043c \u043f\u0435\u0440\u0435\u0432\u0456\u0440\u044f\u0442\u0438, \u044f\u043a \u0432\u0438\u0432\u0447\u0438\u043b\u0438 \u0443\u0440\u043e\u043a \u0439\u043e\u0433\u043e \u0442\u043e\u0432\u0430\u0440\u0438\u0448\u0456. \u041f\u0435\u0440\u0435\u0432\u0435\u043b\u0438 \u043c\u0435\u043d\u0435 \u0432 \u0441\u0435\u043c\u0456\u043d\u0430\u0440\u0456\u044e, \u0437\u0430\u0447\u0443\u0432 \u044f\n \u0432\u043e\u043b\u044e. \u0414\u0438\u0432\u043b\u044e\u0441\u044c, \u0430\u0432\u0434\u0438\u0442\u043e\u0440\u0456\u0432 \u043d\u0435\u043c\u0430\u0454, \u0433\u0440\u0435\u043a\u0438 \u0442\u0430 \u043b\u0430\u0442\u0438\u043d\u0438 \u0432\u0430\u043b\u044f\u044e\u0442\u044c\u0441\u044f \u0432 \u043d\u0430\u0441 \u043f\u0456\u0434\n \u043b\u0456\u0436\u043a\u0430\u043c\u0438 (\u041d\u0435\u0447\u0443\u0439-\u041b\u0435\u0432\u0438\u0446\u044c\u043a\u0438\u0439, I, 1956, 127).\n\u0434\u043e\u0440\u0435\u0432. \u041f\u0440\u043e\u043a\u0443\u0440\u043e\u0440.\n\n\u0421\u043b\u043e\u0432\u043d\u0438\u043a \u0443\u043a\u0440\u0430\u0457\u043d\u0441\u044c\u043a\u043e\u0457 \u043c\u043e\u0432\u0438: \u0432 11 \u0442\u043e\u043c\u0430\u0445. \u2014 \u0422\u043e\u043c 1, 1970. \u2014 \u0421\u0442\u043e\u0440. 72.\n\n\u0414\u0456\u0454\u043f\u0440\u0438\u043a\u043c\u0435\u043d\u0442\u043d\u0438\u043a \u0430\u0443\u0434\u0438\u0442\u043e\u0432\u0430\u043d\u0438\u0439 \u0437\u043d\u0430\u0445\u043e\u0434\u0438\u043c\u043e \u0432 \u0415\u043a\u043e\u043d\u043e\u043c\u0456\u0447\u043d\u043e\u043c\u0443 \u0441\u043b\u043e\u0432\u043d\u0438\u043a\u0443: \u0431\u0430\u043d\u043a\u0456\u0432\u0441\u044c\u043a\u0430 \u0441\u043f\u0440\u0430\u0432\u0430, \u0444\u043e\u043d\u0434\u043e\u0432\u0438\u0439 \u0440\u0438\u043d\u043e\u043a (\u0443\u043a\u0440\u0430\u0457\u043d\u0441\u044c\u043a\u043e-\u0430\u043d\u0433\u043b\u0456\u0439\u0441\u044c\u043a\u043e-\u0440\u043e\u0441\u0456\u0439\u0441\u044c\u043a\u0438\u0439 \u0442\u043b\u0443\u043c\u0430\u0447\u043d\u0438\u0439 \u0441\u043b\u043e\u0432\u043d\u0438\u043a) \u043f\u0456\u0434 \u0440\u0435\u0434. \u0410\u043b\u0435\u043a\u0441\u0435\u0454\u043d\u043a\u043e \u041b.\u041c., \u041e\u043b\u0435\u043a\u0441\u0456\u0454\u043d\u043a\u043e \u0412.\u041c., \u042e\u0440\u043a\u0435\u0432\u0438\u0447 \u0410.\u0406. \u2014 \u041a.: \u0412\u0438\u0434\u0430\u0432\u043d\u0438\u0447\u0438\u0439 \u0431\u0443\u0434\u0438\u043d\u043e\u043a \u00ab\u041c\u0430\u043a\u0441\u0438\u043c\u0443\u043c\u00bb; \u0422\u0435\u0440\u043d\u043e\u043f\u0456\u043b\u044c: \u00ab\u0415\u043a\u043e\u043d\u043e\u043c\u0456\u0447\u043d\u0430 \u0434\u0443\u043c\u043a\u0430\u00bb, 2000. \u2014 592 \u0441.\n\n\u0423\u0442\u0456\u043c, \u0432 \u043c\u0435\u0440\u0435\u0436\u0456 \u043f\u043e\u0440\u044f\u0434 \u0437 \u0432\u0430\u0440\u0456\u0430\u043d\u0442\u043e\u043c \u0430\u0443\u0434\u0438\u0442\u043e\u0432\u0430\u043d\u0438\u0439:\n\n\u041d\u0430\u044f\u0432\u043d\u0456\u0441\u0442\u044c \u0430\u0443\u0434\u0438\u0442\u043e\u0432\u0430\u043d\u043e\u0457 \u043f\u0443\u0431\u043b\u0456\u0447\u043d\u043e\u0457 \u0444\u0456\u043d\u0430\u043d\u0441\u043e\u0432\u043e\u0457 \u0437\u0432\u0456\u0442\u043d\u043e\u0441\u0442\u0456 \u0432 \u043a\u043e\u043c\u043f\u0430\u043d\u0456\u0457\n \u0434\u043e\u043f\u043e\u043c\u0430\u0433\u0430\u0454 \u0446\u0456\u043b\u043e\u043c\u0443 \u043a\u043e\u043b\u0443 \u0457\u0457 \u043a\u043e\u0440\u0438\u0441\u0442\u0443\u0432\u0430\u0447\u0456\u0432 \u0435\u0444\u0435\u043a\u0442\u0438\u0432\u043d\u043e \u0432\u0437\u0430\u0454\u043c\u043e\u0434\u0456\u044f\u0442\u0438 \u0437\n \u043a\u043e\u043c\u043f\u0430\u043d\u0456\u0454\u044e, \u043f\u0456\u0434\u043a\u0440\u0456\u043f\u043b\u044e\u0454 \u0457\u0457 \u0441\u0442\u0430\u0442\u0443\u0441 \u0456 \u0437\u0430\u0431\u0435\u0437\u043f\u0435\u0447\u0443\u0454 \u0432\u0438\u0441\u043e\u043a\u0438\u0439 \u0440\u0456\u0432\u0435\u043d\u044c\n \u043a\u043e\u0440\u043f\u043e\u0440\u0430\u0442\u0438\u0432\u043d\u043e\u0433\u043e \u0443\u043f\u0440\u0430\u0432\u043b\u0456\u043d\u043d\u044f. \u041a\u0440\u0456\u043c \u0442\u043e\u0433\u043e, \u0430\u0443\u0434\u0438\u0442\u043e\u0432\u0430\u043d\u0430 \u0444\u0456\u043d\u0430\u043d\u0441\u043e\u0432\u0430 \u0437\u0432\u0456\u0442\u043d\u0456\u0441\u0442\u044c\n \u0434\u043e\u043f\u043e\u043c\u0430\u0433\u0430\u0454 \u043a\u043e\u043c\u043f\u0430\u043d\u0456\u044f\u043c \u043e\u0442\u0440\u0438\u043c\u0430\u0442\u0438 \u0434\u043e\u0441\u0442\u0443\u043f \u0434\u043e \u0440\u0438\u043d\u043a\u0456\u0432 \u0444\u0456\u043d\u0430\u043d\u0441\u043e\u0432\u043e\u0433\u043e \u043a\u0430\u043f\u0456\u0442\u0430\u043b\u0443 \u0456\n \u0441\u0442\u0438\u043c\u0443\u043b\u044e\u0454 \u0456\u043d\u0432\u0435\u0441\u0442\u0438\u0446\u0456\u0457, \u044f\u043a \u0432\u043d\u0443\u0442\u0440\u0456\u0448\u043d\u0456, \u0442\u0430\u043a \u0456 \u0437\u043e\u0432\u043d\u0456\u0448\u043d\u0456.\n\n\n\u0421\u0442\u0430\u0442\u0442\u044f \"\u042f\u043a\u0456\u0441\u0442\u044c \u0430\u0443\u0434\u0438\u0442\u043e\u0440\u0441\u044c\u043a\u0438\u0445 \u043f\u043e\u0441\u043b\u0443\u0433: \u043f\u0440\u043e\u0431\u043b\u0435\u043c\u0438 \u0442\u0430 \u043f\u0435\u0440\u0441\u043f\u0435\u043a\u0442\u0438\u0432\u0438\" (\u041e.\u041e. \u0406\u043b\u044c\u0447\u0435\u043d\u043a\u043e, \u041c.\u0412. \u041a\u0443\u0437\u0443\u0431)\n\n\n\u041e\u0434\u043d\u0456\u0454\u044e \u0437 \u043e\u0441\u043d\u043e\u0432\u043d\u0438\u0445 \u0432\u0438\u043c\u043e\u0433, \u0449\u043e \u0441\u0442\u0430\u0432\u043b\u044f\u0442\u044c\u0441\u044f \u0434\u043e \u043a\u043e\u043c\u043f\u0430\u043d\u0456\u0457-\u0435\u043c\u0456\u0442\u0435\u043d\u0442\u0430 \u043d\u0435\u0437\u0430\u043b\u0435\u0436\u043d\u043e\n \u0432\u0456\u0434 \u043e\u0431\u0440\u0430\u043d\u043e\u0433\u043e \u043c\u0430\u0439\u0434\u0430\u043d\u0447\u0438\u043a\u0430 \u0454 \u0430\u0443\u0434\u0438\u0442\u043e\u0432\u0430\u043d\u0430 \u0444\u0456\u043d\u0430\u043d\u0441\u043e\u0432\u0430 \u0437\u0432\u0456\u0442\u043d\u0456\u0441\u0442\u044c, \u0441\u043a\u043b\u0430\u0434\u0435\u043d\u0430\n \u0437\u0433\u0456\u0434\u043d\u043e \u0432\u0438\u043c\u043e\u0433 \u041c\u0456\u0436\u043d\u0430\u0440\u043e\u0434\u043d\u0438\u0445 \u0441\u0442\u0430\u043d\u0434\u0430\u0440\u0442\u0456\u0432 \u0444i\u043d\u0430\u043d\u0441\u043e\u0432\u043e\u0457 \u0437\u0432i\u0442\u043d\u043e\u0441\u0442i ( \u041c\u0421\u0424\u0417) \u0437\u0430\n 1-3 \u043e\u0441\u0442\u0430\u043d\u043d\u0456\u0445 \u0440\u043e\u043a\u0438 \u0434\u0456\u044f\u043b\u044c\u043d\u043e\u0441\u0442\u0456.\n\n- \u0421\u0442\u0430\u0442\u0442\u044f \"\u041e\u0446\u0456\u043d\u043a\u0430 \u043c\u043e\u0436\u043b\u0438\u0432\u043e\u0441\u0442\u0456 \u0437\u0434\u0456\u0439\u0441\u043d\u0435\u043d\u043d\u044f \u0443\u0441\u043f\u0456\u0448\u043d\u043e\u0433\u043e \u0406\u0420\u041e \u0430\u043a\u0446\u0456\u043e\u043d\u0435\u0440\u043d\u0438\u043c\u0438 \u0442\u043e\u0432\u0430\u0440\u0438\u0441\u0442\u0432\u0430\u043c\u0438 \u0423\u043a\u0440\u0430\u0457\u043d\u0438 \u0432 \u0443\u043c\u043e\u0432\u0430\u0445 \u0454\u0432\u0440\u043e\u0456\u043d\u0442\u0435\u0433\u0440\u0430\u0446\u0456\u0457\" (\u041d.\u041e. \u0427\u0435\u0445)\n\u0437\u0443\u0441\u0442\u0440\u0456\u0447\u0430\u0454\u0442\u044c\u0441\u044f \u0439 \u0430\u0443\u0434\u0438\u0439\u043e\u0432\u0430\u043d\u0438\u0439:\n\n\u0410\u0443\u0434\u0438\u0439\u043e\u0432\u0430\u043d\u0438\u0439 \u0447\u0438\u0441\u0442\u0438\u0439 \u043f\u0440\u0438\u0431\u0443\u0442\u043e\u043a RosUkrEnergo \u0432 2006 \u0440. \u0432\u0438\u0440\u0456\u0441 \u043d\u0430 4% \u0443\n \u043f\u043e\u0440\u0456\u0432\u043d\u044f\u043d\u043d\u0456 \u0437 2005 \u0440. - \u0437 755,4 \u043c\u043b\u043d \u0434\u043e\u043b. \u0434\u043e 785 \u043c\u043b\u043d \u0434\u043e\u043b., \u0432\u0438\u0440\u0443\u0447\u043a\u0430\n \u0442\u0440\u0435\u0439\u0434\u0435\u0440\u0430 \u0432\u0456\u0434 \u0435\u043a\u0441\u043f\u043e\u0440\u0442\u0443 \u0433\u0430\u0437\u0443 \u0432 2006 \u0440. \u0441\u043a\u043b\u0430\u043b\u0430 7 \u043c\u043b\u0440\u0434 135,9 \u043c\u043b\u043d \u0434\u043e\u043b.\n\n- \u0406\u043d\u0442\u0435\u0440\u043d\u0435\u0442-\u0440\u0435\u0441\u0443\u0440\u0441 \u0420\u0411\u041a-\u0423\u043a\u0440\u0430\u0457\u043d\u0430,\n\u0430\u043b\u0435, \u0449\u043e \u0431\u0456\u043b\u044c\u0448 \u0434\u0438\u0432\u043d\u043e, \u0447\u0430\u0441\u0442\u0456\u0448\u0435 - \u0430\u0443\u0434\u0456\u0439\u043e\u0432\u0430\u043d\u0438\u0439:\n\n\u041d\u0430\u0439\u0431\u0456\u043b\u044c\u0448\u0430 \u043d\u0430\u0444\u0442\u043e\u0433\u0430\u0437\u043e\u0432\u0438\u0434\u043e\u0431\u0443\u0432\u043d\u0430 \u043a\u043e\u043c\u043f\u0430\u043d\u0456\u044f \u0423\u043a\u0440\u0430\u0457\u043d\u0438 \u041f\u0410\u0422 \u00ab\u0423\u043a\u0440\u043d\u0430\u0444\u0442\u0430\u00bb\n \u043e\u043f\u0443\u0431\u043b\u0456\u043a\u0443\u0432\u0430\u043b\u0430 \u0430\u0443\u0434\u0456\u0439\u043e\u0432\u0430\u043d\u0443 \u0444\u0456\u043d\u0430\u043d\u0441\u043e\u0432\u0443 \u0437\u0432\u0456\u0442\u043d\u0456\u0441\u0442\u044c \u0437\u0430 2016 \u0440\u0456\u043a, \u043f\u0456\u0434\u0442\u0432\u0435\u0440\u0434\u0436\u0435\u043d\u0443\n \u043d\u0435\u0437\u0430\u043b\u0435\u0436\u043d\u0438\u043c \u0437\u043e\u0432\u043d\u0456\u0448\u043d\u0456\u043c \u0430\u0443\u0434\u0438\u0442\u043e\u043c PWC.\n\n- \u0441\u0430\u0439\u0442 \u041f\u0410\u0422 \"\u0423\u043a\u0440\u043d\u0430\u0444\u0442\u0430\"\n\n\u041f\u0440\u043e \u0446\u0435 \u0439\u0434\u0435\u0442\u044c\u0441\u044f \u0432 \u0430\u0443\u0434\u0456\u0439\u043e\u0432\u0430\u043d\u0456\u0439 \u043a\u043e\u043d\u0441\u043e\u043b\u0456\u0434\u043e\u0432\u0430\u043d\u0456\u0439 \u0437\u0432\u0456\u0442\u043d\u043e\u0441\u0442\u0456 \u041d\u0410\u041a \u00ab\u041d\u0430\u0444\u0442\u043e\u0433\u0430\u0437\n \u0423\u043a\u0440\u0430\u0457\u043d\u0438\u00bb \u0437\u0430 \u043c\u0456\u0436\u043d\u0430\u0440\u043e\u0434\u043d\u0438\u043c\u0438 \u0441\u0442\u0430\u043d\u0434\u0430\u0440\u0442\u0430\u043c\u0438 \u0444\u0456\u043d\u0430\u043d\u0441\u043e\u0432\u043e\u0457 \u0437\u0432\u0456\u0442\u043d\u043e\u0441\u0442\u0456 (\u041c\u0421\u0424\u0417),\n \u043e\u0431\u043d\u0430\u0440\u043e\u0434\u0443\u0432\u0430\u043d\u0456\u0439 \u043d\u0430 \u043e\u0444\u0456\u0446\u0456\u0439\u043d\u043e\u043c\u0443 \u0441\u0430\u0439\u0442\u0456 \u043a\u043e\u043c\u043f\u0430\u043d\u0456\u0457.\n\n- \u0406\u043d\u0444\u043e\u0440\u043c\u0430\u0446\u0456\u0439\u043d\u0435 \u0430\u0433\u0435\u043d\u0442\u0441\u0442\u0432\u043e \"\u0423\u043d\u0456\u0430\u043d\",\n\u0449\u043e, \u043d\u0430 \u043c\u043e\u044e \u0434\u0443\u043c\u043a\u0443, \u043d\u0435 \u0437\u043e\u0432\u0441\u0456\u043c \u043f\u0440\u0430\u0432\u0438\u043b\u044c\u043d\u043e, \u043e\u0441\u043a\u0456\u043b\u044c\u043a\u0438 \u043c\u0430\u0454\u043c\u043e \u043e\u043c\u043e\u043d\u0456\u043c \u0437\u0456 \u0437\u043d\u0430\u0447\u0435\u043d\u043d\u044f\u043c \"\u0443 \u0444\u043e\u0440\u043c\u0430\u0442\u0456 \u0430\u0443\u0434\u0456\u043e\":\n\n\u0424\u043e\u0440\u043c\u0430\u0442 \u0434\u0435\u0439\u0437\u0456 \u0434\u043e\u0437\u0432\u043e\u043b\u044f\u0454 \u043d\u0435\u0437\u0440\u044f\u0447\u043e\u043c\u0443 \u043a\u043e\u0440\u0438\u0441\u0442\u0443\u0432\u0430\u0447\u0435\u0432\u0456 \u043f\u0440\u043e\u0442\u044f\u0433\u043e\u043c \u043b\u0456\u0447\u0435\u043d\u0438\u0445 \u0441\u0435\u043a\u0443\u043d\u0434\n \u0437\u043d\u0430\u0439\u0442\u0438 \u0432 \u0430\u0443\u0434\u0456\u0439\u043e\u0432\u0430\u043d\u0456\u0439 \u043a\u043d\u0438\u0437\u0456 \u043f\u043e\u0442\u0440\u0456\u0431\u043d\u0438\u0439 \u0440\u043e\u0437\u0434\u0456\u043b, \u043f\u0430\u0440\u0430\u0433\u0440\u0430\u0444, \u0441\u0442\u043e\u0440\u0456\u043d\u043a\u0443,\n \u0430\u0431\u0437\u0430\u0446, \u0444\u0440\u0430\u0437\u0443.\n\n\n\u0406\u043d\u0442\u0435\u0440\u043d\u0435\u0442-\u0440\u0435\u0441\u0443\u0440\u0441 lviv.vgorode.ua,\n\n\u044f\u043a\u0438\u0439 \u0432\u0438\u0434\u0430\u0454\u0442\u044c\u0441\u044f \u043b\u043e\u0433\u0456\u0447\u043d\u0456\u0448\u0438\u043c \u0432\u0430\u0440\u0456\u0430\u043d\u0442\u043e\u043c (\u0430\u0443\u0434\u0456\u043e, \u0430\u0443\u0434\u0456\u044e\u0432\u0430\u043d\u043d\u044f, \u0430\u0443\u0434\u0456\u0439\u043e\u0432\u0430\u043d\u0438\u0439), \u0430\u043d\u0456\u0436 \u043f\u043e\u0445\u0456\u0434\u043d\u0438\u0439 \u0432\u0456\u0434 \u0430\u0443\u0434\u0438\u0442\u0443.\nP.S. \u041f\u043e\u0448\u0443\u043a Google \u0449\u0435 \u0432\u0438\u0434\u0430\u0454 \u0440\u0435\u0437\u0443\u043b\u044c\u0442\u0430\u0442\u0438 \u0437 \u0430\u0443\u0434\u043e\u0432\u0430\u043d\u0438\u0439, \u043b\u043e\u0433\u0456\u043a\u0430 \u0442\u0432\u043e\u0440\u0435\u043d\u043d\u044f \u044f\u043a\u043e\u0433\u043e \u043c\u0435\u043d\u0456 \u0442\u0435\u0436 \u043d\u0435 \u0437\u0440\u043e\u0437\u0443\u043c\u0456\u043b\u0430.\n\u0410 \u044f\u043a\u0438\u0439 \u0456\u0437 \u043d\u0430\u0432\u0435\u0434\u0435\u043d\u0438\u0445 \u0432\u0430\u0440\u0456\u0430\u043d\u0442\u0456\u0432 \u0432\u0456\u0434\u043f\u043e\u0432\u0456\u0434\u0430\u0454 \u043d\u043e\u0440\u043c\u0430\u043c \u0441\u043b\u043e\u0432\u043e\u0442\u0432\u043e\u0440\u0435\u043d\u043d\u044f \u0434\u0456\u0454\u043f\u0440\u0438\u043a\u043c\u0435\u0442\u043d\u0438\u043a\u0456\u0432 \u0432 \u0443\u043a\u0440\u0430\u0457\u043d\u0441\u044c\u043a\u0456\u0439 \u043c\u043e\u0432\u0456?\n", "answer": "\u041a\u043e\u0440\u043e\u0442\u043a\u043e:\n\u041d\u0430\u0439\u043a\u0440\u0430\u0449\u0435 \u0432\u0438\u043a\u043e\u0440\u0438\u0441\u0442\u043e\u0432\u0443\u0432\u0430\u0442\u0438 \u0441\u043b\u043e\u0432\u043e \"\u0430\u0443\u0434\u0438\u0442\u043e\u0432\u0430\u043d\u0438\u0439\", \u0430\u0434\u0436\u0435, \u044f\u043a \u0432\u0436\u0435 \u0437\u0430\u0437\u043d\u0430\u0447\u0430\u043b\u043e\u0441\u044f \u0443 \u043f\u0438\u0442\u0430\u043d\u043d\u0456 \u0439 \u043a\u043e\u043c\u0435\u043d\u0442\u0430\u0440\u044f\u0445, \u0432\u043e\u043d\u043e \u0434\u043e\u0437\u0432\u043e\u043b\u044f\u0454 \u0443\u043d\u0438\u043a\u043d\u0443\u0442\u0438 \u0440\u0456\u0437\u043d\u043e\u0447\u0438\u0442\u0430\u043d\u043d\u044f, \u0430 \u0442\u0430\u043a\u043e\u0436 \u0448\u0438\u0440\u043e\u043a\u043e \u0432\u0436\u0438\u0432\u0430\u0454\u0442\u044c\u0441\u044f \u0443 \u043d\u0430\u0443\u043a\u043e\u0432\u0456\u0439 \u0456 \u043e\u0444\u0456\u0446\u0456\u0439\u043d\u0456\u0439 \u043b\u0456\u0442\u0435\u0440\u0430\u0442\u0443\u0440\u0456.\n\u0412\u0456\u0434\u043d\u043e\u0441\u043d\u043e \u0434\u043e\u0432\u0433\u0435 \u0456\u043d\u0442\u0435\u0440\u043d\u0435\u0442-\u0434\u043e\u0441\u043b\u0456\u0434\u0436\u0435\u043d\u043d\u044f:\n1. \u0410\u0443\u0434\u0438\u0442\u043e\u0432\u0430\u043d\u0438\u0439.\n45 \u0440\u0435\u0437\u0443\u043b\u044c\u0442\u0430\u0442\u0456\u0432 \u0491\u0443\u0491\u043b\u0430 \u043d\u0430 \u0441\u043b\u043e\u0432\u043e \"\u0430\u0443\u0434\u0438\u0442\u043e\u0432\u0430\u043d\u0438\u0439\" \u0432 \u043b\u0430\u043f\u043a\u0430\u0445 \u043b\u0438\u0448\u0435 \u0443\u043a\u0440\u0430\u0457\u043d\u0441\u044c\u043a\u043e\u044e \u043c\u043e\u0432\u043e\u044e.\n\u041f\u0435\u0440\u0448\u0456 \u0442\u0440\u0438 \u043f\u043e\u0441\u0438\u043b\u0430\u043d\u043d\u044f \u2014 \u043d\u0430 \u0440\u043e\u0441\u0456\u0439\u0441\u044c\u043a\u0438\u0439 \u0441\u0430\u0439\u0442.\n\u0414\u0430\u043b\u0456 \u043d\u0430 \u0441\u0430\u0439\u0442\u0456 http://cyclop.com.ua/ \u0454 \"\u0424\u0456\u043d\u0430\u043d\u0441\u043e\u0432\u043e-\u0435\u043a\u043e\u043d\u043e\u043c\u0456\u0447\u043d\u0438\u0439 \u0441\u043b\u043e\u0432\u043d\u0438\u043a\" - \u0417\u0430\u0433\u043e\u0440\u043e\u0434\u043d\u0456\u0439 \u0410.\u0413.:\n\n\u0410\u0423\u0414\u0418\u0422\u041e\u0412\u0410\u041d\u0418\u0419 \u0417\u0412\u0406\u0422 - \u0434\u0438\u0432. \u0417\u0432\u0456\u0442 \u0430\u0443\u0434\u0438\u0442\u043e\u0432\u0430\u043d\u0438\u0439.\n\u0417\u0412\u0406\u0422 \u0410\u0423\u0414\u0418\u0422\u041e\u0412\u0410\u041d\u0418\u0419 - \u0437\u0432\u0456\u0442 \u043f\u0456\u0434\u043f\u0440\u0438\u0454\u043c\u0441\u0442\u0432\u0430, \u0449\u043e \u043f\u0440\u043e\u0439\u0448\u043e\u0432 \u0430\u0443\u0434\u0438\u0442\u043e\u0440\u0441\u044c\u043a\u0443 \u043f\u0435\u0440\u0435\u0432\u0456\u0440\u043a\u0443.\n\n\u0414\u0430\u043b\u0456 \u041c\u0456\u0436\u043d\u0430\u0440\u043e\u0434\u043d\u0430 \u043e\u0440\u0491\u0430\u043d\u0456\u0437\u0430\u0446\u0456\u044f \u0437 \u043c\u0456\u0491\u0440\u0430\u0446\u0456\u0457 \u043c\u0430\u0454 \u0444\u043e\u0440\u043c\u0443 \u0437\u0430\u044f\u0432\u043a\u0438 \u043d\u0430 \u043f\u0435\u0440\u0435\u043a\u0432\u0430\u043b\u0456\u0444\u0456\u043a\u0430\u0446\u0456\u044e (\u043e\u0431\u0435\u0440\u0435\u0436\u043d\u043e, \u0437\u0430\u0432\u0430\u043d\u0442\u0430\u0436\u0435\u043d\u043d\u044f Excel-\u0444\u0430\u0439\u043b\u0443), \u0434\u0435 \u0431\u0430\u0447\u0438\u043c\u043e:\n\n\u0410\u0443\u0434\u0438\u0442\u043e\u0432\u0430\u043d\u0438\u0439 \u0444\u0456\u043d\u0430\u043d\u0441\u043e\u0432\u0438\u0439 \u0437\u0432\u0456\u0442 \u0437\u0430 2016 \u0444\u0456\u043d\u0430\u043d\u0441\u043e\u0432\u0438\u0439 \u0440\u0456\u043a.\n\n\u0429\u0435 \u043e\u0434\u043d\u0435 \u043f\u043e\u0441\u0438\u043b\u0430\u043d\u043d\u044f \u043d\u0430 \"\u0420\u0456\u0447\u043d\u0443 \u0456\u043d\u0444\u043e\u0440\u043c\u0430\u0446\u0456\u044e \u0435\u043c\u0456\u0442\u0435\u043d\u0442\u0430 \u0446\u0456\u043d\u043d\u0438\u0445 \u043f\u0430\u043f\u0435\u0440\u0456\u0432 \u0437\u0430 2017 \u0440\u0456\u043a\" \u043c\u0456\u0436\u043d\u0430\u0440\u043e\u0434\u043d\u043e\u0457 \u0431\u0443\u0434\u0456\u0432\u0435\u043b\u044c\u043d\u043e\u0457 \u043a\u043e\u043c\u043f\u0430\u043d\u0456\u0457 \"konecranes\":\n\n\u041f\u043e \u0448\u043e\u0441\u0442\u043e\u043c\u0443 \u043f\u0438\u0442\u0430\u043d\u043d\u044e \u0441\u043b\u0443\u0445\u0430\u043b\u0438 \u0422\u0430\u0440\u0430\u0441\u043e\u0432\u0430 \u0421\u0435\u0440\u0433i\u044f \u041e\u043b\u0435\u043a\u0441\u0430\u043d\u0434\u0440\u043e\u0432\u0438\u0447\u0430 \u0442\u0430 \u043f\u0440\u0438\u0439\u043d\u044f\u043b\u0438 \u0440i\u0448\u0435\u043d\u043d\u044f \u0437\u0430\u0442\u0432\u0435\u0440\u0434\u0438\u0442\u0438 \u0440i\u0447\u043d\u0438\u0439 \u0430\u0443\u0434\u0438\u0442\u043e\u0432\u0430\u043d\u0438\u0439 \u0437\u0432i\u0442 \u041f\u0440\u0438\u0432\u0430\u0442\u043d\u043e\u0433\u043e \u0430\u043a\u0446i\u043e\u043d\u0435\u0440\u043d\u043e\u0433\u043e \u0442\u043e\u0432\u0430\u0440\u0438\u0441\u0442\u0432\u0430 \"\u041a\u043e\u043d\u0435\u043a\u0440\u0435\u0439\u043d\u0441 \u0423\u043a\u0440\u0430\u0457\u043d\u0430\" \u0437\u0430 2016 \u0440i\u043a.\n\n\u0414\u0430\u043b\u0456 \u0442\u0430 \u0441\u0430\u043c\u0430 \u0444\u0456\u0440\u043c\u0430 \u043e\u0442\u0440\u0438\u043c\u0430\u043b\u0430 \u0437\u0432\u0456\u0442 \u0432\u0456\u0434 openprocurement.org (\u043e\u0431\u0435\u0440\u0435\u0436\u043d\u043e, \u0437\u0430 \u043f\u043e\u0441\u0438\u043b\u0430\u043d\u043d\u044f\u043c \u0437\u0430\u0432\u0430\u043d\u0442\u0430\u0436\u0435\u043d\u043d\u044f \u043f\u0434\u0444-\u0444\u0430\u0439\u043b\u0430):\n\n\u0417\u0430\u0442\u0432\u0435\u0440\u0434\u0438\u0442\u0438 \u0440\u0456\u0447\u043d\u0438\u0439 \u0430\u0443\u0434\u0438\u0442\u043e\u0432\u0430\u043d\u0438\u0439 \u0437\u0432\u0456\u0442 \u041f\u0440\u0438\u0432\u0430\u0442\u043d\u043e\u0433\u043e \u0430\u043a\u0446\u0456\u043e\u043d\u0435\u0440\u043d\u043e\u0433\u043e \u0442\u043e\u0432\u0430\u0440\u0438\u0441\u0442\u0432\u0430 \u201e\u041a\u043e\u043d\u0435\u043a\u0440\u0435\u0439\u043d\u0441. \u0423\u043a\u0440\u0430\u0457\u043d\u0430\u201d \u0437\u0430 2010 \u0440\u0456\u043a.\n\n\u0414\u0430\u043b\u0456 \u0439\u0434\u0435 \u0432\u0438\u0434\u0430\u043d\u0438\u0439 \u043f\u0456\u0434 \u043c\u0430\u0440\u043a\u043e\u044e \u041a\u041f\u0406 \u0442\u0435\u0440\u043c\u0456\u043d\u043e\u043b\u043e\u0491\u0456\u0447\u043d\u0438\u0439 \u0441\u043b\u043e\u0432\u043d\u0438\u043a (\u043e\u0431\u0435\u0440\u0435\u0436\u043d\u043e, \u0437\u0430 \u043f\u043e\u0441\u0438\u043b\u0430\u043d\u043d\u044f\u043c \u0437\u0430\u0432\u0430\u043d\u0442\u0430\u0436\u0435\u043d\u043d\u044f \u043f\u0434\u0444-\u0444\u0430\u0439\u043b\u0430) \u2014 \u0428\u0438\u0448\u043a\u0456\u043d \u0412.\u041c., \u0427\u0456\u043a\u0456\u043d \u0421.\u0412.:\n\n\u0410\u0423\u0414\u0418\u0422\u041e\u0412\u0410\u041d\u0418\u0419 \u2013 AUDITEE \u2013 \u041e\u0441\u043e\u0431\u0430 \u0430\u0431\u043e \u043e\u0440\u0433\u0430\u043d\u0456\u0437\u0430\u0446\u0456\u044f, \u044f\u043a\u0430 \u043f\u0456\u0434\u0434\u0430\u0454\u0442\u044c\u0441\u044f \u0430\u0443\u0434\u0438\u0442\u0443\n\n\u0414\u0430\u043b\u0456: \u041c\u0430\u0440\u0442\u0438\u043d\u044e\u043a \u041d.\u042e. (\u0430\u0441\u0438\u0441\u0442\u0435\u043d\u0442 \u043a\u0430\u0444\u0435\u0434\u0440\u0438 \u043e\u0431\u043b\u0456\u043a\u0443 \u0442\u0430 \u0430\u0443\u0434\u0438\u0442\u0443 \u041b\u0443\u0446\u044c\u043a\u043e\u0433\u043e \u043d\u0430\u0446\u0456\u043e\u043d\u0430\u043b\u044c\u043d\u043e\u0433\u043e \u0442\u0435\u0445\u043d\u0456\u0447\u043d\u043e\u0433\u043e \u0443\u043d\u0456\u0432\u0435\u0440\u0441\u0438\u0442\u0435\u0442\u0443) \"\u041e\u0411\u041b\u0406\u041a\u041e\u0412\u0406 \u0410\u0421\u041f\u0415\u041a\u0422\u0418 \u0410\u0423\u0414\u0418\u0422\u0423 \u042f\u041a\u041e\u0421\u0422\u0406\" (\u043e\u0431\u0435\u0440\u0435\u0436\u043d\u043e, \u0437\u0430 \u043f\u043e\u0441\u0438\u043b\u0430\u043d\u043d\u044f\u043c \u0437\u0430\u0432\u0430\u043d\u0442\u0430\u0436\u0435\u043d\u043d\u044f \u043f\u0434\u0444-\u0444\u0430\u0439\u043b\u0430) \u0432 \u0435\u043b\u0435\u043a\u0442\u0440\u043e\u043d\u043d\u043e\u043c\u0443 \u0432\u0438\u0434\u0430\u043d\u043d\u0456 \"\u0413\u043b\u043e\u0431\u0430\u043b\u044c\u043d\u0456 \u0442\u0430 \u043d\u0430\u0446\u0456\u043e\u043d\u0430\u043b\u044c\u043d\u0456 \u043f\u0440\u043e\u0431\u043b\u0435\u043c\u0438 \u0435\u043a\u043e\u043d\u043e\u043c\u0456\u043a\u0438\":\n\n\u043f\u0440\u0435\u0430\u043c\u0431\u0443\u043b\u0430: \u043d\u0430\u0437\u0432\u0430 \u043f\u0456\u0434\u043f\u0440\u0438\u0454\u043c\u0441\u0442\u0432\u0430, \u0456\u043c\u0435\u043d\u0430 \u0441\u043f\u0456\u0432\u0440\u043e\u0431\u0456\u0442\u043d\u0438\u043a\u0456\u0432, \u0437 \u044f\u043a\u0438\u043c\u0438 \u043f\u0440\u043e\u0432\u043e\u0434\u0438\u043b\u0438\u0441\u044c \u0431\u0435\u0441\u0456\u0434\u0438, \u0434\u0430\u0442\u0438, \u043f\u0440\u043e\u0433\u0440\u0430\u043c\u0438, \u043f\u043e\u0441\u0438\u043b\u0430\u043d\u043d\u044f \u043d\u0430 \u0441\u0442\u0430\u043d\u0434\u0430\u0440\u0442 \u0447\u0438 \u0456\u043d\u0448\u0456 \u0434\u043e\u043a\u0443\u043c\u0435\u043d\u0442\u0438, \u0430\u0443\u0434\u0438\u0442\u043e\u0432\u0430\u043d\u0438\u0439 (\u043f\u0456\u0434\u0440\u043e\u0437\u0434\u0456\u043b \u0447\u0438 \u0432\u0456\u0434\u0434\u0456\u043b);\n\n\u0414\u0430\u043b\u0456 \u0449\u0435 \u043d\u0435 \u043e\u0434\u043d\u0435 \u043d\u0430\u0443\u043a\u043e\u0432\u0435 \u0432\u0438\u0434\u0430\u043d\u043d\u044f \u0432\u0438\u043a\u043e\u0440\u0438\u0441\u0442\u043e\u0432\u0443\u0454 \u0446\u0435 \u0441\u043b\u043e\u0432\u043e: 1, 2, 3, 4 \u0442\u043e\u0449\u043e.\n2. \u0410\u0443\u0434\u0438\u0439\u043e\u0432\u0430\u043d\u0438\u0439.\n15 \u0440\u0435\u0437\u0443\u043b\u044c\u0442\u0430\u0442\u0456\u0432 \u0491\u0443\u0491\u043b\u0430 \u043d\u0430 \u0441\u043b\u043e\u0432\u043e \"\u0430\u0443\u0434\u0438\u0439\u043e\u0432\u0430\u043d\u0438\u0439\", \u0437 \u043d\u0438\u0445 \u0436\u043e\u0434\u043d\u043e\u0433\u043e \u043d\u0430\u0443\u043a\u043e\u0432\u043e\u0433\u043e \u0456 \u0434\u0435\u043a\u0456\u043b\u044c\u043a\u0430 \u0440\u043e\u0441\u0456\u0439\u0441\u044c\u043a\u0438\u0445.\n3. \u0410\u0443\u0434\u0456\u0439\u043e\u0432\u0430\u043d\u0438\u0439.\n27 \u0440\u0435\u0437\u0443\u043b\u044c\u0442\u0430\u0442\u0456\u0432 \u0491\u0443\u0491\u043b\u0430 \u043d\u0430 \u0446\u0435 \u0441\u043b\u043e\u0432\u043e. \u0417 \u043d\u0438\u0445 \u043f\u043e \u0434\u0435\u043a\u0456\u043b\u044c\u043a\u0430 \u0440\u0430\u0437\u0456\u0432 \u043f\u043e\u0432\u0442\u043e\u0440\u044e\u044e\u0442\u044c\u0441\u044f \u043d\u043e\u0432\u0438\u043d\u0438 \u043f\u0440\u043e \"\u0430\u0443\u0434\u0456\u0439\u043e\u0432\u0430\u043d\u0438\u0439 \u043f\u0440\u0438\u0431\u0443\u0442\u043e\u043a \u0430\u0435\u0440\u043e\u043f\u043e\u0440\u0442\u0443 \"\u0411\u043e\u0440\u0438\u0441\u043f\u0456\u043b\u044c\"\" (1, 2, 3 \u0442\u043e\u0449\u043e), \u043f\u043e\u0442\u0456\u043c \u043f\u0440\u043e \"\u043f\u0440\u0435\u043a\u0440\u0430\u0441\u043d\u0438\u0439 \u0430\u0443\u0434\u0456\u0439\u043e\u0432\u0430\u043d\u0438\u0439 \u0431\u0430\u043d\u043a\", \u043f\u043e\u0442\u0456\u043c \u0434\u0435\u043a\u0456\u043b\u044c\u043a\u0430 \u0440\u0430\u0437\u0456\u0432 \u0439\u0434\u0435\u0442\u044c\u0441\u044f \u043d\u0435 \u043f\u0440\u043e \u0430\u0443\u0434\u0456\u0439\u043e\u0432\u0430\u043d\u0438\u0439 \u0437\u0432\u0456\u0442, \u0430 \u043f\u0440\u043e \u0430\u0443\u0434\u0456\u0439\u043e\u0432\u0430\u043d\u0438\u0439 \u0441\u0442\u0430\u043d\u0434\u0430\u0440\u0442 (1, 2, 3 (\u043e\u0431\u0435\u0440\u0435\u0436\u043d\u043e, \u043e\u0441\u0442\u0430\u043d\u043d\u0456 \u0434\u0432\u0430 \u043f\u043e\u0441\u0438\u043b\u0430\u043d\u043d\u044f - \u0437\u0430\u0432\u0430\u043d\u0442\u0430\u0436\u0435\u043d\u043d\u044f \u0444\u0430\u0439\u043b\u0456\u0432) \u0442\u043e\u0449\u043e), \u0434\u0435\u043a\u0456\u043b\u044c\u043a\u0430 \u043f\u043e\u0441\u0438\u043b\u0430\u043d\u044c \u043d\u0430 \"\u0430\u0443\u0434\u0456\u0439\u043e\u0432\u0430\u043d\u0438\u0439 \u0437\u0432\u0443\u043a\", \"\u0430\u0443\u0434\u0456\u0439\u043e\u0432\u0430\u043d\u0438\u0439 \u043f\u0435\u0440\u0456\u043e\u0434\", \"\u0430\u0443\u0434\u0456\u0439\u043e\u0432\u0430\u043d\u0438\u0439 \u0445\u0430\u0440\u0430\u043a\u0442\u0435\u0440 \u043d\u0430\u0432\u0447\u0430\u043b\u044c\u043d\u043e\u0433\u043e \u0441\u0435\u0440\u0435\u0434\u043e\u0432\u0438\u0449\u0430\".\n\u041e\u0442\u0436\u0435, \u043c\u0430\u0454\u043c\u043e \u0434\u0443\u0436\u0435 \u0431\u0430\u0433\u0430\u0442\u043e \u0440\u0456\u0437\u043d\u043e\u0447\u0438\u0442\u0430\u043d\u044c, \u043d\u0435\u0437\u0440\u043e\u0437\u0443\u043c\u0456\u043b\u043e, \u0434\u0435 \u0441\u0430\u043c\u0435 \u0432\u0438\u043a\u043e\u0440\u0438\u0441\u0442\u043e\u0432\u0443\u0432\u0430\u0442\u0438 \u0446\u0435 \u0441\u043b\u043e\u0432\u043e, \u043c\u0430\u0439\u0436\u0435 \u043d\u0435\u043c\u0430\u0454 \u0434\u043e\u0441\u0442\u0430\u0442\u043d\u044c\u043e \u043f\u043e\u0432\u0430\u0436\u043d\u0438\u0445 \u0434\u0436\u0435\u0440\u0435\u043b \u0437\u0456 \u0441\u043b\u043e\u0432\u043e\u0441\u043f\u043e\u043b\u0443\u0447\u0435\u043d\u043d\u044f\u043c \"\u0430\u0443\u0434\u0456\u0439\u043e\u0432\u0430\u043d\u0438\u0439 \u0437\u0432\u0456\u0442\" - \u0491\u0443\u0491\u043b \u0432\u0438\u0434\u0430\u0454 \u0434\u0432\u0430 \u0440\u0435\u0437\u0443\u043b\u044c\u0442\u0430\u0442\u0438.\n\u0421\u043b\u043e\u0432\u043e\u0442\u0432\u043e\u0440\u0435\u043d\u043d\u044f \u0434\u0456\u0454\u043f\u0440\u0438\u043a\u043c\u0435\u0442\u043d\u0438\u043a\u0456\u0432 \u0432 \u0443\u043a\u0440\u0430\u0457\u043d\u0441\u044c\u043a\u0456\u0439 \u043c\u043e\u0432\u0456:\n\u0414\u0456\u0454\u043f\u0440\u0438\u043a\u043c\u0435\u0442\u043d\u0438\u043a \"\u0430\u0443\u0434\u0438\u0442\u043e\u0432\u0430\u043d\u0438\u0439\" \u0442\u0432\u043e\u0440\u0438\u0442\u044c\u0441\u044f \u0432\u0456\u0434 \u0434\u043e\u043a\u043e\u043d\u0430\u043d\u043e\u0433\u043e \u0434\u0456\u0454\u0441\u043b\u043e\u0432\u0430 \"\u0430\u0443\u0434\u0438\u0442\u0443\u0432\u0430\u0442\u0438\":\n\n\u0410\u0423\u0414\u0418\u0422\u0423\u0412\u0410\u0301\u0422\u0418, \u0443\u0301\u044e, \u0443\u0301\u0454\u0448, \u043d\u0435\u0434\u043e\u043a. \u0456 \u0434\u043e\u043a., \u0449\u043e. \u0417\u0434\u0456\u0439\u0441\u043d\u044e\u0432\u0430\u0442\u0438 \u0430\u0443\u0434\u0438\u0442. \u0410\u0443\u0434\u0438\u0442\u0443\u0432\u0430\u0442\u0438 \u043f\u0440\u0438\u0432\u0430\u0442\u043d\u0435 \u043f\u0456\u0434\u043f\u0440\u0438\u0454\u043c\u0441\u0442\u0432\u043e.\n\n\u0427\u0438\u0442\u0430\u0454\u043c\u043e \u043f\u0440\u0430\u0432\u0438\u043b\u043e:\n\n\u042f\u043a\u0449\u043e \u043e\u0441\u043d\u043e\u0432\u0430 \u0434\u0456\u0454\u0441\u043b\u043e\u0432\u0430 \u0432 \u043d\u0435\u043e\u0437\u043d\u0430\u0447\u0435\u043d\u0456\u0439 \u0444\u043e\u0440\u043c\u0456 \u0437\u0430\u043a\u0456\u043d\u0447\u0443\u0454\u0442\u044c\u0441\u044f \u043d\u0430 -\u0430, \u0442\u043e \u0434\u043e \u043d\u0435\u0457 \u0434\u043e\u0434\u0430\u0454\u043c\u043e \u0441\u0443\u0444\u0456\u043a\u0441 -\u043d(\u0438\u0439): \u043e\u0431\u043b\u0430\u0434\u043d\u0430(\u0442\u0438) \u2014 \u043e\u0431\u043b\u0430\u0434\u043d\u0430\u043d\u0438\u0439, \u043e\u0431\u0433\u043e\u0432\u043e\u0440\u044e\u0432\u0430(\u0442\u0438) \u2014 \u043e\u0431\u0433\u043e\u0432\u043e\u0440\u044e\u0432\u0430\u043d\u0438\u0439, \u0441\u043f\u0438\u043b\u044f(\u0442\u0438) \u2014 \u0441\u043f\u0438\u043b\u044f\u043d\u0438\u0439.\n\n\u0422\u043e\u0434\u0456 \u0430\u0443\u0434\u0438\u0442\u0443\u0432\u0430\u0442\u0438 - \u0430\u0443\u0434\u0438\u0442\u043e\u0432\u0430\u043d\u0438\u0439.\n\u0412\u0438\u0441\u043d\u043e\u0432\u043e\u043a:\n\u0412\u0440\u0430\u0445\u043e\u0432\u0443\u044e\u0447\u0438 \u0432\u0438\u0449\u0435\u043d\u0430\u0432\u0435\u0434\u0435\u043d\u0443 \u0456\u043d\u0444\u043e\u0440\u043c\u0430\u0446\u0456\u044e, \u0434\u043e\u043a\u043b\u0430\u0434\u0430\u044e\u0447\u0438 \u0434\u043e \u043d\u0435\u0457 \u0456\u043d\u0444\u043e\u0440\u043c\u0430\u0446\u0456\u044e \u0437 \u0441\u0430\u043c\u043e\u0433\u043e \u0437\u0430\u043f\u0438\u0442\u0430\u043d\u043d\u044f \u0456 \u043a\u043e\u043c\u0435\u043d\u0442\u0430\u0440\u0456\u0432, \u043c\u043e\u0436\u0435\u043c\u043e \u0432\u0438\u0441\u043d\u0443\u0432\u0430\u0442\u0438, \u0449\u043e \u043d\u0430\u0439\u043f\u043e\u0448\u0438\u0440\u0435\u043d\u0456\u0448\u0435 \u0439 \u043d\u0430\u0439\u0434\u043e\u0446\u0456\u043b\u044c\u043d\u0456\u0448\u0435 \u0437 \u0442\u0440\u044c\u043e\u0445 \u0441\u043b\u043e\u0432\u043e\u0441\u043f\u043e\u043b\u0443\u0447\u0435\u043d\u044c - \u0441\u043b\u043e\u0432\u043e\u0441\u043f\u043e\u043b\u0443\u0447\u0435\u043d\u043d\u044f \"\u0430\u0443\u0434\u0438\u0442\u043e\u0432\u0430\u043d\u0438\u0439 \u0437\u0432\u0456\u0442\".\n", "topic": "ukrainian", "url": "https://ukrainian.stackexchange.com/questions/5469"}, {"image": "VQAonline_00064872.png", "question": "What is the minimum length I can thickness plane?", "context": "The thickness planers I've seen have a cutter head between two rollers. The rollers grab the stock and feed it past the cutter head, counteracting the force applied by the blades. If the rollers do not have a secure grip on the stock, the cutter may kick the piece backwards if a blade catches.\n\n(Source: https://en.wikipedia.org/wiki/File:Thicknesser.gif)\nWhat safety risks does this present when planing short stock?\nHow can I determine the minimum length which can be safely planed?\n", "answer": "It varies by model, check the manual online for the model you're looking at. Generally the shortest recommended length is also the distance between the center of the rollers.\nBasically, you never want the possibility of a piece of wood being 'stuck' under the cutter head but not being held down by a roller.\nYou can counteract this limitation with some variation of a sled that holds short stock securely to the bottom with sacrificial 'rails' that ride the rollers and get planed down with the short stock itself.\n", "topic": "woodworking", "url": "https://woodworking.stackexchange.com/questions/703"}, {"image": "VQAonline_00064966.png", "question": "What is the best way to slice CHUNKS of wood?", "context": "\nHere is a photo of the floor I plan to install in my barn house. I have a great source for the wood, but it is all in \"chunks\" that are approximately 9\"x11\"x14\". My (old) table saw blade doesn't raise high enough to easily cut even halfway through, my 12\" miter saw won't accommodate them, I don't have a band saw (but might have to buy or borrow one), and I can't cut consistently enough with a chain saw. Is there an option I'm overlooking?\n", "answer": "Here are several options that come to mind:\n\nCut smaller pieces\nGet a bandsaw with resaw capacity of at least 9\" (many 14\" or larger bandsaws either come stock with 11-12\" resaw capacity, or can be upgraded with a riser block)\nUse a saw with a significantly larger circular blade\nCut as deep as you can on both sides, then finish the job with a hand saw\nUse a chainsaw mill (aka Alaska mill)--basically a guide for your chainsaw. It will require a fair amount of cleanup but it will be much less than if you make the cuts freehand\nHave someone else with larger or more appropriate equipment cut the pieces for you--either hire a sawmill or make a new friend, perhaps someone in a local woodworking club.\n\nAs Ashlar semi-jokingly noted in a comment, you could use a handsaw exclusively, but that would not be a fast or easy task, especially if you want to crosscut the pieces as illustrated in the picture.\n", "topic": "woodworking", "url": "https://woodworking.stackexchange.com/questions/4336"}, {"image": "VQAonline_00064894.png", "question": "What is a try square used for?", "context": "I know there are many types of squares, and ultimately all of them are used check an angle for square-ness, i.e., to make sure it is a 90\u00b0 angle. But different squares are good at different jobs. For example, a Swanson speed square has a lip on one side that makes it really good for quickly drawing a perpendicular line on a work piece (among many other things).\nWhat does a try square excel at? If my understanding is correct, a try square has one arm that is thinner than the other arm, like this one:\n\nImage Source: Wikipedia - Image license: CC BY-SA 2.0 ca, Wikimedia commons\nMost sources from around the web just say that \"try square\" is another name for \"square\", and describe what a square does. But I'm thinking there must be something that a square with one arm thinner than the other is better at than, say, a Swanson square or a framing square.\n", "answer": "\na Swanson speed square has a lip on one side that makes it really good for quickly drawing a perpendicular line on a work piece\n\nA try square is similar in that respect: the handle is thicker than the blade, so it's easy to push the inside of the handle up against the edge of a board and mark (or verify) a line perpendicular (square) to the edge. And you can easily stand a try square on the outside edge of the handle to check that your table saw blade is at 90\u00b0 to the table. You can check inside corners with a try square or a speed square, but due to its triangular shape a speed square can't be used to check outside corners the way a try square can.\nA speed square is a rugged, rough tool that is built to be tossed in a bucket of tools and which you can always count on to get you close enough to square for construction purposes. A try square is less rugged and should be treated more like a precision instrument \u2014 take care not to drop it, give it a safe home in your toolbox, etc.\n", "topic": "woodworking", "url": "https://woodworking.stackexchange.com/questions/1492"}, {"image": "VQAonline_00065186.png", "question": "How to start a discussion in a startup in order to start using a more useful workflow tool", "context": "My boss, for the entire company's workflow, uses Google Sheets as a sort of feature-tracker/TODO-list. The company employs 7 people, aside from the CEO (my boss), who all work remotely. There are 2 software developers (myself included), 2 linguists, 2 marketing personnel and the CEO's son.\nI did once suggest to my boss that we at least move to a tool like Trello that functions similarly, but with much more flexibility. This was when he said that Trello would be a new tool that everyone has to learn, and he doesn't want to do that as everyone is already used to the \"Google Sheet flow\", and that it would be difficult for everyone to migrate.\nThe problem is that if things are pushed into the backlog, as in \"not important enough for this next release/campaign\", they are often forgotten about. We will continue to add things to fix/add at the top of the list, and things that were low priority before become essentially non-existent. It's also hard to tell why we backlogged something because of the limited flexibility of Google Sheets, and having progress tracking is out the window. Here's an example from my Sheet (feature tracker) for a high-priority Android app fix that got mixed in with other work:\n\nThis sheet is now about 75 lines long for the next version release of the app, with some duplicated information and no visual organization except \"what's on top is most important\".\nI've been testing out some free workflow/task management tools for the past few hours (on my own time) and I am convinced that it would be beneficial for all the employees of the company to use such tools.\nWhat benefits about employees and management can I use to convince him that it's actually a good idea? I've come up with the following:\n\nTransparency - all employees that could/should see what the others are doing, can\n\n\ne.g. the marketing team would know how close developers are to finishing the next release, so they can start making the appropriate materials/announcements\n\nOrganization - tasks can have custom priorities, making them easier to organize\n\n\nthis is better than the 'top-down' approach that we currently use\n\nDue dates\n\npretty important - due dates can be assigned and on the calendar instead of hidden in a bunch of comments in a spreadsheet cell\n\nLabels and subtasks\n\nget even more specific with labeling a task or dividing it into specific subtasks with completion steps\n\nAutomatic update tracking\n\nwhen an task/subtask's progress is updated, it already has user and timestamps, so much less noise overall (we are currently marking who commented and when)\n\n\nThere are probably more benefits... How can I start a discussion within the company to migrate to a better workflow management tool?\n", "answer": "Business case, business case, business case. The best way to frame nearly any change to management is some combination of:\n\nWhat we are doing now is costing us money.\nThe new way of doing things would save/make money.\n\nIn this case, the biggest thing I would highlight is important work getting lost in your current system. If this is happening it is causing some combination of:\n\nReleases getting delayed.\nA lower quality product being released.\nYour team working at less than optimal efficiency.\n\nAll of this has a direct impact on the company's bottom line. That is the case I would try to make.\nA few other points:\n\nFocus on the positive benefit to the extent possible. If your boss is invested in Google Sheets (and perhaps created the current system), \"this new tool would be beneficial\" is better than \"our current system is bad\". I know this somewhat contradicts what I said above. If there are current problems they do need to be highlighted. But still, make the case as positive as possible.\nAlways aim for incremental change. A completely new way of managing the team's work is a high-risk change. There is a reason management would be reluctant to do that. Try to get permission to use a new tool in a much more limited context instead, for a small project or part of one project. Or you could even use it just for yourself to manage your own tasks, if you are having trouble keep track of them. Once you can show how it works and the benefit, you will have a much easier case to make.\n\n", "topic": "workplace", "url": "https://workplace.stackexchange.com/questions/73920"}, {"image": "VQAonline_00065172.png", "question": "Can it be seen as unprofessional to have an avatar as your LinkedIn profile picture?", "context": "I have created an identity of myself on the internet, with the following image:\n\nI'm using this image on almost every site I use: Twitter, Gravatar, Stack Overflow, blog, ...\nMy question is the following: As a CS engineer, can this image be seen as unprofessional by recruiters and shouldn't be used on a site like LinkedIn?\nMost of the friends I have have a photo of themselves in a suit, that's why I'm asking this question.\nSo what can or should I do to avoid an unprofessional image when using an avatar as my profile picture on social networking sites?\n", "answer": "An avatar is like a signature or a seal, and you are \"signing\" your profile with an avatar. Especially since you are using the same \"signature\" or seal everywhere. That's totally legit, according to me. If your avatar functions as your recognition device, so be it. I use my picture for LinkedIn but my avatar for all other activity on the Internet. \nBack in the Middle Ages, your avatar would have been your heraldric emblem, which you would put on your shield so that others would recognize you, as you using your uncovered face as a recognition device would have been a very bad idea on a battlefield where every single sharp object would have an affinity for your face :)\nThe advantage of using an avatar is that your avatar doesn't need to change over the years as time catches up to you :) Whereas those who use a photo of their faces would have to update it every five to ten years - on every profile that they put out. Look at it this way, if your face got rearranged in an auto accident, you'd have to update your photos from a \"before the event\" photo to an \"after the event\" photo :)\nOn the other hand, if you are young, good looking and physically fit, why not flaunt it? You are always free to switch to an avatar as you feel the years adding up:)\n", "topic": "workplace", "url": "https://workplace.stackexchange.com/questions/33737"}, {"image": "VQAonline_00065176.png", "question": "What can companies do to uncover fake diplomas such as those purportedly sold by Axact?", "context": "So I was reading this article in New York Times about Axact, a Pakistan-based Company that seems to be in the business of selling fake diplomas.\nThe case of Axact is also not some small operation involving diplomas with just one or a couple of (perhaps) easily recognizable names of educational institutes.\nAbove article states that:\n\nTheir websites, glossy and assured, offer online degrees in dozens of disciplines, like nursing and civil engineering. There are glowing endorsements on the CNN iReport website, enthusiastic video testimonials, and State Department authentication certificates bearing the signature of Secretary of State John Kerry.\n\u201cWe host one of the most renowned faculty in the world,\u201d boasts a woman introduced in one promotional video as the head of a law school. \u201cCome be a part of Newford University to soar the sky of excellence.\u201d\nYet on closer examination, this picture shimmers like a mirage. The news reports are fabricated. The professors are paid actors. The university campuses exist only as stock photos on computer servers. The degrees have no true accreditation.\nIn fact, very little in this virtual academic realm, appearing to span at least 370 websites, is real\n\nThis other article at New York Times, titled \"Tracking Axact\u2019s Websites\" lists 132 fake universities and 16 different accreditation bodies that are referenced by Axact.\nNames of some of the accreditation bodies that they use according to above article:\n\nAccreditation Bureau of Online Education and Training\nAssociation for Accreditation of Business Schools and Programs\nEuropean Accreditation Board for Online Education\nEuropean Accreditation Council for Online Learning\nGlobal Accreditation Board for Distance Learning\nGlobal Doctorate Council\nGulf Accreditation Council\nGulf Bureau of Higher Education\n\nSome of the names of their \"universities\" are below:\n\nAshbery University\nBarkley University\nBaycity University\nBranton University\nBrooksville University\nCambell State University\nCamp Lake University\nChapel University\nJohnstown University\nKennedy University\nKingsbridge University\nPittsford University\nPort Jefferson University\nQueen City University\nQueens Bay University\nWestern Valley Central University\nWiley University\nWilford University\n\nAbove universities sport what looks like professionally made websites, such as the one below for Barkley University:\n\nConsidering the scale of this operation, what can a company reasonably do to determine if a candidate has a fake diploma?\n", "answer": "I haven't had a prospective employer ask me to produce a diploma in several decades. Companies will usually check your diploma claims by directly querying the university that you claim that you got a degree from. I am a Columbia alumn and if someone produced a Columbia diploma, I couldn't tell the genuine article from a fake by looking at it. That some Columbia diploma doesn't look like mine, that's meaningless - It's not as if universities don't change the format of their diplomas from time to time. It's a lot more straightforward and almost totally foolproof to just check with the university.\nChecking that a post-secondary institution is legitimate is not hard, at least in the US. The Higher Learning Commission, which accredits Columbia College of Columbia University, is one of the six regional institutional accreditors in the US. You simply find what state the alleged institution of higher learning is, you go on the appropriate institutional accreditor's website and you check the name of the alleged institution against their list of accredited institutions. Having said that, HRs of small companies and of startups probably don't know how to check - It took me one hour to work out how to check, but not everyone is like me. \nThis does not mean that these small companies and startups are defenseless. They are sensitive to poor performance and if they don't catch up to someone's bogus education credentials, they will catch up to that someone's poor performance either at the interview or on the job. And job candidates should know better than to play mind games with a fully staffed HR's like Walmart's or Goldman Sachs'\n", "topic": "workplace", "url": "https://workplace.stackexchange.com/questions/46812"}, {"image": "VQAonline_00065824.png", "question": "Bones with iron like beaver teeth. Would it work?", "context": "Beavers, asides from being the largest rodent in North America and making rather fetching hats, also have iron compounds within their teeth enamel which allows said tooth handle higher amounts of mechanical strength.\nIt also makes their teeth orange.\n\nNow, if humans had iron within their bones they would become stronger, right?\nWhat I'm wondering is that would introducing iron to our skeleton make our bones stronger? Or is this a bad idea with unforeseen consequences, asides from an expanded appetite.\n", "answer": "No.\nThe Iron compounds in a beaver's teeth are not there to make its teeth stronger, they make the front layer of its teeth harder. This means that the beaver can gnaw tough, abrasive material such as wood without its teeth wearing down so fast. Additionally, the differential in hardness between the front and the back of the teeth means that the higher rate of wear at the back of the teeth makes them self-sharpening.\nThis has little or nothing to do with strength.\nBones wouldn't be made stronger by adding these iron compounds... if anything, it would make the bones more brittle.\n", "topic": "worldbuilding", "url": "https://worldbuilding.stackexchange.com/questions/221639"}, {"image": "VQAonline_00065702.png", "question": "Cylindrical radiators on spacecraft?", "context": "Having your radiators tactically protected from enemy fire by the armor shape of your space warship is nice, but having a regular one or double-sided radiators is still a vulnerability. Other concepts such as liquid droplet and electrostatic radiators seem to be too complex and require much more power. I was thinking of radiators similar to that of the graphite spikes that are seen on the fusion torch cages in Attack Vector: Tactical. They would stick out of the body of the warship at an angle that allows them to be blocked from oncoming fire from by the frontal armor. They would have a radiating surface wrap around the entire circumference with working fluid pipes running through the interior. How feasible is this? What are your ideas for their design (approximates for length, diameter, and materials)? How would I calculate how much heat they would dissipate? Assume it is about 1-2GW of waste heat from a 200MW nuclear reactor.\n\n\nStrategically drawn red circle pointing out the radiator spikes on a warship from AV: T\n\n", "answer": "The equation for radiant power is $P = \\epsilon \\sigma A T^4$\nGraphite starts to sublimate at 3,700 degrees Kelvin. Let's assume there's no factor of safety, but a typical one for aerospace is 1.5, which would bring the design operating temperature down to 2,500 Kelvin.\nUnfortunately, as others pointed out, your heat pump is the limiting factor at a much lower radiator temperature of 600 degrees Kelvin.\n$ \\epsilon $ is our emissivity from 0 to 1. Let's assume it's an ideal radiator at 1.\n$ \\sigma $ is a constant $5.67 \\times 10^{-8}$. \nAssuming you had some futuristic better-than-perfect heat pump, and could utilize the radiators fully to their material limit, to radiate 2 GW, then, you need 188 square meters of radiator at failure (the rods start vaporizing) or 903 square meters at the factor of safety.\nMore realistically, with a maximum temperature of 600 degrees Kelvin at the radiators, you need 56 thousand square meters of radiator to radiate the same 2 gigawatts.\nFor those rods to be effective, they can't be radiating back into your hull. So, about half the radiating arc of cylinders shielded from fire by the front of the hull is unavailable. You also don't want the hive pictured because the radiators would be interfering with each other. If you have two sets of 4 offset from one another vertically down the hull, they'd each need to be 225 square meters. Easily doable with 100 meter spikes about 2 meters in diameter.\nFor a more realistic forest with a 600 K limit at the radiator, the same configuration would need to 13.4 meters in diameter.\n", "topic": "worldbuilding", "url": "https://worldbuilding.stackexchange.com/questions/174503"}, {"image": "VQAonline_00065695.png", "question": "How big does a pelican type creature need to be to carry a grown up human inside its mouth and still fly?", "context": "as the title, my question is how big a pelican like creature with its pouch mouth need to be in order to carry grown up human and still fly.\nsince as far as i know hollow bone is to fragile for human to ride big flying creature on their back, and i want to know the possibility by carrying them using pelican type of mouth, beside i am not good with math and only know stone age physic at best.\nif its not clear, i dont mean the pelican eat them or swallow them, but just carry them inside the pouch as a flying transportation, and i do at least think a way to make sure they dont accidentally get swallowed by using cormorant fishing method by tying their neck to make sure they dont swallow thing as big as human size while maybe i can incorporate it into a leash too to control the flying.\n\nthe world is pretty much like earth\nits fine if the answer end up says its impossible in reality, but at least provide the math or physic of it to show of how absurd it will be. \nfeel free to include or suggest alternative way if my method is impossible, (still related to the mouth) like for example carrying them like stork carrying baby either using cloth or basket like in this image. \n\nfrom:https://en.wikipedia.org/wiki/White_stork\n\nand feel free to edit my grammar and the tag to the appropriate one, (though i appreciate it if this is not removed when you edit it, give other a chance to know that they are welcome to fix or edit this, because sometimes the edit is not perfect or still has some flaw) \n", "answer": "The largest extinct flying birds include:\nAgentavis magnificens:\n\nThe heaviest bird ever capable of flight was Argentavis magnificens, the largest member of the now extinct family Teratornithidae, found in Miocene-aged fossil beds of Argentina, with a wingspan up to 5\u20136 m (16\u201320 ft), a length of up to 1.26 m (4.1 ft), a height on the ground of up to 1.5\u20132 m (4.9\u20136.6 ft) and a body weight of at least 71 kg (157 lb).3\n\nHarpagornis moorei:\n\nRivaling Argentavis in wingspan if not in bulk and mass, another contender for the largest known flying bird ever is Harpagornis moorei, which had a wingspan of up to 7.3 m (24 ft).7\n\nhttps://en.wikipedia.org/wiki/List_of_largest_birds[2]\nPelagornis sandersi:\n\nThe new species, Pelagornis sandersi, had an estimated wingspan of 20 to 24 feet (6.1 to 7.3 meters) when its feathers are included. This is up to more than twice as big as that of the royal albatross, the largest living flying bird, which has a wingspan of about 11.4 feet (3.5 meters).\nTo find out how P. sandersi could take off and stay aloft despite its giant size, Ksepka fed data about the bird's mass, wingspan and wing shape into a computer program designed to predict flight performance. The researchers estimated the bird weighed from 48.2 to 88.4 lbs. (21.9 to 40.1 kilograms).\n\nhttps://www.livescience.com/46679-largest-flying-bird-discovered.html[3]\nThe largest flying reptiles were even larger, real \"terror dactyls\".\nI found a list of 18 species of flying reptiles with estimated wingspans greater than 5 meters or 16 feet. 11 of those species had estimated wingspans of 6 meters (20 feet) or wider, and 5 of them had estimated wingspans of 9.1 meters (30 feet) or wider.\nThe estimated wingspans of extinct birds include estimates of the length of their feathers at the wingtips, while the estimates for flying reptiles go to the tips of their wing bones.\nhttps://en.wikipedia.org/wiki/Pterosaur_size#Pterosaurs_with_largest_wingspan 4\n\nSome species of pterosaurs grew to very large sizes and this has implications for their capacity for flight. Many pterosaurs were small but the largest had wingspans which exceeded 9 m (30 ft). The largest of these are estimated to have weighed 250 kilograms (550 lb). For comparison, the wandering albatross has the largest wingspan of living birds at up to 3.5 m (11 ft) but usually weighs less than 12 kilograms (26 lb). This indicates that the largest pterosaurs may have had higher wing loadings than modern birds (depending on wing profile) and this has implications for the manner in which pterosaur flight might differ from that of modern birds.\n\n\nFactors such as the warmer climate of the Mesozoic or higher levels of atmospheric oxygen have been proposed but it is now generally agreed that even the largest pterosaurs could have flown in today's skies.[17] Partly, this is due to the presence of air sacs in their wing membranes,[18] and that pterosaurs launched into flight using their front limbs in a quadrupedal stance similar to that of modern bats, a method faster and less energy taxing that the bipedal launching of modern birds.19\n\nhttps://en.wikipedia.org/wiki/Pterosaur_size#Speculation_about_pterosaur_size_and_flight 6\nThe heaviest flying creature ever might possibly have been Quetzalcoatlus northropi:\n\nWeight estimates for giant azhdarchids are extremely problematic because no existing species share a similar size or body plan, and in consequence, published results vary widely.5 Generalized weight, based on some studies that have historically found extremely low weight estimates for Quetzalcoatlus, was as low as 70 kg (150 lb) for a 10 m (32 ft 10 in) individual. A majority of estimates published since the 2000s have been substantially higher, around 200\u2013250 kg (440\u2013550 lb).5\n\nhttps://en.wikipedia.org/wiki/Quetzalcoatlus#Size[1]\nAt the present time, Asia is the continent with the lightest average adult human weight, at 57.7 kilograms (127.2 pounds), and North America is the continent with the heaviest average adult human wight, at 80.7 kilograms (177.9 pounds).\nhttps://en.wikipedia.org/wiki/Human_body_weight#By_region[7]\nThus the largest known flying creatures on Earth weighed a few times as much as an average adult human. Would that make humans weight a small enough fraction for them to be able to take off carrying the extra weight? What is the greatest fraction of its own weight that any bird has ever carried?\nWould any flying creature have a long and wide enough lower jaw for an adult human to fit through the jaw and go into any pouch the flying creature might have had? Would such a pouch be strong enough to hold the weight of an adult human, or would the human tear through the pouch and plummet to their death?\nWould the neck bones and muscles of the flying creature be strong enough to carry their head with the addition of the weight of an adult human?\nIf your setting is another planet, you might want to make it a planet with lower gravity and a denser atmosphere.\nAs a general rule, the higher the gravity of the planet the denser the atmosphere will be, but there are exceptions due to planetary conditions. For example, Titan has much lower gravity than Earth but a somewhat denser atmosphere, and Venus has slightly less gravity than Earth and many times as dense an atmosphere.\nIn a fantasy story, the giant flying creatures might be tamed by giants who train them for various tasks. At fairs the giants might offer rides on their giant flying creatures, or in their pouches, to dwarfs or halfings.\n", "topic": "worldbuilding", "url": "https://worldbuilding.stackexchange.com/questions/172056"}, {"image": "VQAonline_00065926.png", "question": "If your Introduction can function as a Conclusion, isn't it redundant to write anything in the Conclusion beside \"refer to the Introduction\"?", "context": "\nIn the example below, I would state the Conclusion just once, at the beginning. I pick the sentence under Section 5 because it's more detailed than the sentence under Section 1. I find the second conclusion in CREAC superfluous. Why not just do CREA?\n\nEven if you need CREAC, and the second conclusion, to signal the reader that you're concluding, why not just write a curt sentence asking the reader to refer to the conclusion at the beginning (\"Please refer to the beginning or page whatever)? Isn't it pointless and stupid to paraphrase your Introduction?\n\n\n\n\u00a0 \u00a0 \u00a0 This chapter will explain one organizational paradigm, CREAC, and how it is used to express different forms of legal analysis, including analogical reasoning and rule-based reasoning. CREAC has five component parts, each\nbuilding on the other. Each letter in CREAC represents a specific component part of the written expression of legal analysis: Conclusion, Rule, Explanation of the law, Application of the Law, and Conclusion. When drafted\neffectively, the parts combine in a cohesive, logical, and comprehensive expression of legal analysis. CREAC is a flexible paradigm that can be manipulated or translated to fit many different types of legal analyses or documents.\nThe key is to understand how each component part of CREAC fits together.\n\n\n\n\nRomantz. Legal Analysis: The Fundamental Skill (2009). p 120.\n", "answer": "Both the introduction and conclusion are, in part, summarising the paper \u2014 but they\u2019re doing in different contexts, and in different purposes. So there\u2019s some overlap, but they\u2019re not the same.\nThe introduction is written for readers who haven\u2019t read the rest of the paper yet, and may never do so. So it needs to say what the main results are \u2014 to motivate some readers to read the whole paper, and to stick in the memory of others in case they have reason to come back to the paper in future. But it needs to do this in an accessible and self-contained way, not relying on technical background beyond what\u2019s standard in the field. It also needs to orient readers for the rest of the paper \u2014 at least in my field (pure maths), it\u2019s common for the introduction to include a quick outline of the other sections.\nThe conclusion is written for readers who have already read the rest of the paper, or at least skimmed it, and who are already motivated to care about the results, and may be intending to apply them in their own work. So it should summarise the results again, but can include more technical details (introduced and backed up by the body of the paper), and can look back and discuss what has been said in the body of the paper.\n", "topic": "writers", "url": "https://writers.stackexchange.com/questions/53577"}, {"image": "VQAonline_00065918.png", "question": "How to use 2 footnotes in the same word?", "context": "I have this Paragraph and I was wondering if it's possible to do what is underlined in the figure (RED), or if you know how to do this in another way, let me know. Basically the \"DOM.Element\" is a child of WEB API. So there are 2 things there. The \"DOM\" & \"Element\". \nHow to use multiple footnotes in this case? Is that a good way of doing it? TBH I haven't seen anything like it before.\n\nFor the LaTex folks:\nAs mentioned above, the design is created due to the reference of Cascade Style Sheets (CSS3) in a HyperText Markup Language (HTML5). This will assign different design commands for every \n\\textit{\n DOM\\footnote{ Document Object Model (DOM) - \\url{https://developer.mozilla.org/en-US/docs/Web/API/Document_Object_Model}}\n .Element\\footnote{ Element - \\url{https://developer.mozilla.org/en-US/docs/Web/API/Element}}\n}\nindividually, or in a form of a group using \\textit{Element.id} and/or \\textit{Element.className}.The benefit of using CSS at this stage is that it loads the styles of the elements before the page loads on the user's screen.\n\n", "answer": "Just use \"DOM element\" instead.\n\"DOM.Element\" is not actually a word. It is way used to represent a DOM element in a programming context. In that context it would indeed behave like word and your usage might be correct.\nHowever the sentence or the text is not in that kind of context. If it was you'd have no need to footnote either \"DOM\" or \"Element\". This is a general explanation using normal English so you should use normal words and expressions.\nSame is true of element id and class. I understand what you are trying here but the text is talking about the general concepts on a very general level not about the specific types in a programming context.\nI am guessing that later in the text it gets more technical and that is where the usage comes from?\nCSS is from \"Cascading StyleSheets\". Not \"Cascade\".\n", "topic": "writers", "url": "https://writers.stackexchange.com/questions/43247"}, {"image": "VQAonline_00065919.png", "question": "What type of pen is being used here?", "context": "Can someone please tell me what's the pen used here in the below URL called? It looks like a Gel pen, but the text written with it is so sharp:\n\nhttps://www.wikihow.com/Improve-Your-Handwriting\n", "answer": "Does not look like a Pilot G1 to me, not the point anyways.\nThe pen in the picture can be a Technical pen\nBingo: https://www.jetpens.com/Pilot-G-Tec-C-Gel-Pen-0.4-mm-Black/pd/4558\n", "topic": "writers", "url": "https://writers.stackexchange.com/questions/43804"}]